Vous êtes sur la page 1sur 315

GUIDE DE L'ÉTUDIANT

Parkin o Bade

NTRODUCTIONAia

2e édition

GUIDE DE L'ETUDIANT

Avi J. Cohen
York University

Harvey B. King
University of Régina

Pierre Chapleau
Université du Québec à Montréal

5757, RUE CYPIHOT (514) 334-2690


EÆi
TÉLÉPHONE
SAINT-LAURENT (QUÉBEC) TÉLÉCOPIEUR: (514) 334-4720
ÉDITIONS DU RENOUVEAU PÉDAGOGIQUE INC, H4S1R3 courriel: erpidlm@erpi.com
Supervision éditoriale :
Sylvie Chapleau et Jacqueline Leroux

Révision linguistique :
Véra Pollak

Correction d’épreuves :
Suzanne Marquis

Traduction :
Nicole André

Couverture :
E
Édition électronique :
Caractéra inc.

Dans cet ouvrage, le générique masculin est utilisé sans aucune discrimination
et uniquement pour alléger le texte.

Cet ouvrage est une version française de la 3e édition de STUDY GUIDE d’Avi J.
Cohen et Harvey B. King, accompagnant le manuel MICROECONOMICS - Canada in
the global environment, de Parkin et Bade, publiée et vendue à travers le monde
avec l’autorisation d’Addison-Wesley Publishers Limited.

© 1997, ADDISON-WESLEY PUBLISHERS LIMITED.


Tous droits réservés
© 1999, ÉDITIONS DU RENOUVEAU PÉDAGOGIQUE INC.
Tous droits réservés

DANGER On ne peut reproduire aucun extrait de ce livre sous quelque forme


ou par quelque procédé que ce soit - sur machine électronique,
mécanique, à photocopier ou à enregistrer, ou autrement - sans avoir
PHOTOCOPILLAGE
obtenu au préalable la permission écrite des Éditions du Renouveau
TUE LE LIVRE Pédagogique Inc.

Dépôt légal : 4e trimestre 1999


Bibliothèque nationale du Québec
Bibliothèque nationale du Canada

Imprimé au Canada

ISBN 2-7613-1105-1 23456789 II 0987654321


20108 ABCD EQ85
PRÉSENTATION

Avant de commencer... enseignants exigent des connaissances beaucoup plus


approfondies de ces concepts et vous demanderont
Notre expérience nous a enseigné que les étudiants
de les appliquer dans des circonstances nouvelles.
de première année en économie veulent avant tout
avoir un guide qui les aide à maîtriser le contenu Étudiez chaque semaine et sans prendre de retard.
de leur cours, afin d’obtenir de meilleures notes aux Lisez le chapitre correspondant du manuel avant que
examens. C’est dans cette optique que nous avons votre professeur ne l’aborde. Pour cette première lecture,
élaboré ce Guide de l’étudiant. L’étude du Guide, ne vous souciez pas des détails ou des arguments que
toutefois, ne saurait à elle seule vous garantir de vous ne comprenez pas bien - essayez simplement
bonnes notes. Pour vous aider à surmonter les de saisir l’essentiel des sujets et les notions de base. Vous
problèmes et les difficultés auxquels se heurtent serez surpris de constater à quel point les explications
la plupart des étudiants de première année, nous de votre professeur sont limpides si vous arrivez bien
vous proposons ci-dessous quelques conseils d’ordre préparé au cours. Quand votre professeur a traité
général sur la méthode de travail idéale, ainsi que des d’un chapitre, attelez-vous aux exercices proposés
conseils plus précis sur l’utilisation optimale du Guide. dans le Guide de l’étudiant. Il faut éviter à tout prix
de commencer à étudier la veille de l’examen ou même
Quelques conseils d’ami la semaine avant. L’étude de l’économie doit se faire
avec méthode et discipline. En essayant de sauter des
L’économie ne s’étudie pas comme les autres matières.
étapes, on court à l’échec.
Les économistes font souvent appel aux hypothèses
pour fractionner des problèmes complexes en éléments Prenez de bonnes notes de cours. Les notes de cours
plus abordables et plus faciles à analyser. Ce style constituent le volet central de votre travail. Votre
analytique, qui n’est pas plus compliqué que le type professeur sélectionnera les principaux thèmes du
de raisonnement qui s’applique dans les autres matières, chapitre pour les expliquer en classe. Les sujets qu’il
est cependant nouveau pour la plupart des étudiants abordera sont ceux que vous devriez étudier en priorité.
et demande que l’on s’y habitue. Par conséquent, Accordez également la priorité aux figures et aux
pour réussir en économie, les étudiants ne peuvent tableaux expliqués pendant les cours.
pas seulement compter sur leur intelligence et sur Certains professeurs privilégient les notes de cours,
les connaissances acquises au cégep, comme c’est d’autres le manuel. Demandez dès le début du cours
le cas pour la plupart des autres disciplines de première qu’on vous explique clairement lequel de ces deux
année. La plupart des étudiants qui viennent nous outils est le plus important pour les examens. Si votre
consulter sont souvent découragés et déçus car, bien professeur vous répond que les deux sont cruciaux,
qu’ils obtiennent des A et des B dans les autres matières, posez-lui cette question typiquement économique :
ils n’ont, au mieux, que des C en économie. Cet « Quel est celui qui présente l’avantage marginal le plus
écart s’explique simplement par le fait qu ils n ont élevé ? » En d’autres mots, s’il vous reste une heure
pas compris que l’étude de l’économie est differente pour étudier, vaut-il mieux la passer à revoir les notes
de celle des autres disciplines et qu elle nécessite de cours ou à relire le manuel ? Mais, avant d’en arriver
un certain entraînement. Pour vous éviter ce genre là, vous devez lire chaque chapitre du manuel deux fois
de désagréments, nous vous conseillons la marche (une fois avant le cours, pour avoir un aperçu général, et
à suivre ci-dessous. une fois après le cours, pour bien assimiler la matière),
prendre de bonnes notes de cours et faire tous les
Ne comptez pas uniquement sur les connaissances exercices des chapitres correspondants du Guide
acquises au cégep. Si vous avez étudie 1 économie de l’étudiant. En appliquant cette méthode de travail,
au cégep, vous connaissez déjà les notions liees à 1 offre vous réalisez une allocation optimale de votre temps
et à la demande que votre professeur abordera au cours d’étude et vous commencez déjà à raisonner
des premières semaines. N’en concluez pas pour autant en économiste!
que ce cours sera facile. Les connaissances sur les
concepts d’économie que vous avez acquises au cegep N’hésitez pas à demander de l’aide à votre professeur
vous seront certes très utiles, mais elles ne seront pas ou à son assistant. Allez voir les enseignants pendant
suffisantes pour vous permettre, à elles seules, d obtenir leurs heures de consultation, et demandez-leur tous les
de bonnes notes aux examens. À 1 université, les éclaircissements dont vous avez besoin. Vous payez vos
vi PRÉSENTATION

cours, et les enseignants sont là pour vous aider à Concepts clés Cette première rubrique est un
apprendre, ne l’oubliez pas. Nous sommes toujours résumé en style télégraphique, d’une page ou deux,
surpris de constater que rares sont les étudiants qui des définitions, des concepts et du matériel contenus
viennent nous consulter en dehors des cours. N’ayez pas dans le chapitre correspondant du manuel. Ce résumé
peur! Le contact personnel qui s’établit lors de ces reprend les titres du chapitre. Les termes clés utilisés
rencontres est aussi gratifiant pour les enseignants que dans le manuel sont imprimés en caractères gras. Cette
pour vous. rubrique vous permet de vous concentrer rapidement
et de manière précise sur le matériel que vous devez
Formez un groupe de travail. Le travail d’équipe
maîtriser. C’est un outil pédagogique précieux à repasser
est une excellente façon de vous motiver et d’apprendre.
à la veille de l’examen. Considérez-le comme un aide-
En répondant aux questions à voix haute, vous pourrez
mémoire qui vous permet d’effectuer une vérification
mieux vérifier si vous les avez bien comprises. Si vous
finale des concepts clés que vous avez étudiés.
ne répondez que mentalement aux questions posées,
vous sautez souvent, sans vous en rendre compte, des Rappels De nombreux étudiants butent
étapes dans le processus de raisonnement. Lorsque sur certains concepts difficiles à comprendre
vous êtes obligé d’expliquer votre raisonnement et commettent les mêmes erreurs. Ces erreurs sont
à haute voix, les lacunes et les failles ressortent sans suffisamment fréquentes pour que nous sachions
tarder et vous (et les autres membres de votre groupe) maintenant comment aider les étudiants à les éviter.
pouvez aussitôt corriger vos erreurs. Les questions Les conseils contenus sous cette rubrique portent
du type Vrai/Faux/Incertain du Guide de l’étudiant sur ce genre d’erreurs et vous montrent comment
ainsi que les questions de révision qui figurent à les éviter. Ils portent notamment sur les concepts, les
la fin des chapitres du manuel sont d’excellents outils équations et les techniques de résolution de problème
de travail de groupe. Vous pouvez également vous les plus importants du chapitre. Vous y trouverez
réunir après avoir résolu les problèmes et répondu également des conseils sur la façon d’interpréter les
aux questions du Guide de l’étudiant, mais avant graphiques essentiels, que l’on retrouve dans tous les
de regarder les solutions, et vous aider ainsi les examens. Nous espérons que cette rubrique vous aidera
uns les autres à trouver la bonne réponse. à répondre aux questions-pièges que les professeurs
glissent souvent dans les examens pour vérifier
Travaillez sur d’anciens examens. Un des moyens les
la précision de vos connaissances.
plus efficaces d’étudier est de travailler sur des examens
Cette rubrique peut également comporter du
que votre professeur a donnés les années précédentes.
matériel que votre professeur ajoute au cours, mais
Ils vous aideront à prévoir un peu le genre de questions
qui ne figure pas dans les chapitres du manuel. Il peut
qu’il peut poser. Si, de plus, vous vous astreignez
s’agir, par exemple, de la solution à l’équation portant
à les faire dans le temps imparti, ils vous habituent
sur l’offre et la demande de la Note mathématique
à travailler dans les conditions de l’examen, c’est-à-dire
du chapitre 4 du manuel. Ce matériel est indiqué
vite et bien. Travailler sur d’anciens examens n’est pas
par le symbole ® et vous pouvez le sauter si votre
synonyme de tricher, si vous vous les êtes procurés
professeur ne l’aborde pas.
en toute légalité. Certains établissements conservent
Le symbole ® est utilisé dans le Guide de
les anciens examens à la bibliothèque, d’autres les
l’étudiant pour vous aider à reconnaître les questions
laissent au département ou dans les locaux de
et les réponses qui se rapportent au matériel
l’association des étudiants. Vous pourrez également
supplémentaire.
vous en procurer auprès des étudiants qui ont déjà
suivi le cours. Rappelez-vous, cependant, que les Autoévaluation Cette rubrique est
anciens examens ne sont utiles que si vous les employez incontestablement l’une des plus utiles du Guide
pour comprendre le raisonnement qui sous-tend de l’étudiant. Elle se compose de questions et de
chaque question. Si vous mémorisez simplement problèmes dont l’objectif est de vous aider à vérifier
les réponses dans l’espoir que votre professeur posera si vous maîtrisez ou non les notions et les techniques
exactement les mêmes questions, vous courez à l’échec. essentielles pour réussir aux examens. Cette rubrique
D’une année à l’autre, les professeurs changent leurs propose un grand nombre de questions à choix
questions ou du moins modifient les valeurs multiple (25 par chapitre) similaires à celles qu’on
numériques. retrouve dans les examens. Vous y trouverez aussi
d’autres questions, décrites ci-dessous, ayant chacune un
Utilisation du Guide de l’étudiant but pédagogique précis. Les questions (et les réponses)
Vous devez lire le chapitre et suivre attentivement basées sur le matériel supplémentaire qui apparaît
le cours avant de vous attaquer aux exercices du Guide sous la rubrique Rappels portent le symbole ® .
de l’étudiant. Chaque chapitre du Guide de l’étudiant Il vous sera ainsi facile de les sauter si votre professeur
contient les rubriques suivantes :
PRÉSENTATION vil

n’aborde pas ce matériel. Avant de passer à l’analyse L’Autoévaluation comporte trois rubriques.
des trois parties de la rubrique Autoévaluation, voici
quelques remarques générales. Vrai/Faux/Incertain (Justifiez votre réponse.)
Inscrivez vos réponses au crayon dans le Guide Ces questions évaluent vos connaissances des concepts
de l’étudiant. Vous pourrez ainsi effacer vos erreurs lors contenus dans le chapitre correspondant et votre
des corrections et vous aurez des pages propres et bien capacité de les appliquer. Certaines questions permettent
remplies sur lesquelles travailler. Tracez un graphique de tester votre compréhension et de voir si vous pouvez

chaque fois que cela est utile. Parfois, on vous demandera déceler les erreurs dans les énoncés qui portent sur des

explicitement d’illustrer vos propos graphiquement. concepts de base. Vous pourrez ainsi reconnaître vos

D’autres fois, même si on ne vous demande pas lacunes sans tarder. Il est préférable de répondre à ces

de le faire, vous serez appelé à vous engager dans questions à haute voix, lors de votre travail en groupe.

un processus de raisonnement qui implique des Lorsque vous donnez votre réponse, précisez

déplacements de courbes. Tracez toujours un graphique. si l’énoncé est vrai, faux ou incertain, car, parfois,

N’essayez pas de faire tout votre raisonnement celui-ci peut être vrai ou faux selon les circonstances
ou les hypothèses. Expliquez votre réponse en une
mentalement, car vous risqueriez fort de vous tromper.
seule phrase. L’espace prévu sous chaque question est
Chaque fois que vous tracez un graphique en marge
du Guide, fut-ce rapidement, nommez vos axes. Les suffisant pour l’y inscrire.

étudiants croient souvent pouvoir mémoriser les noms


Questions à choix multiple Ces questions, plus
des axes et se dispensent de les écrire, mais ils oublient
difficiles, visent à évaluer vos capacités d’analyse. Pour
qu’ils auront à tracer de nombreux graphiques dont les
y répondre, vous allez devoir appliquer les concepts
axes mesurent des variables très diverses. Évitez les erreurs, dans des conditions nouvelles, traiter l’information
nommez les axes ! Rappelez-vous enfin que, aux examens, et résoudre des problèmes chiffrés et graphiques.
les professeurs retirent des points aux étudiants qui Il s’agit du type de questions qui sont le plus
ne nomment pas les axes. souvent posées lors des tests et des examens. Dans
Répondez aux questions de l’Autoévaluation comme l’Autoévaluation, qui en contient 25, elles sont
s’il s’agissait d’un véritable examen, c’est-à-dire sans présentées dans le désordre, pour vous mettre
regarder les réponses. C’est le conseil le plus important en situation d’examen réel.
que nous puissions vous donner sur le bon usage du Lisez attentivement chaque question et les cinq
Guide de l’étudiant. Souvenez-vous qu’on n’a jamais possibilités avant de donner votre réponse. Plusieurs
rien sans mal. On apprend beaucoup des bonnes de ces possibilités seront plausibles et ne différeront
réponses obtenues à force de patience et de réflexion, que peu les unes des autres. Vous devrez choisir
mais aussi des erreurs et des mauvaises réponses. la réponse la plus appropriée. Nous vous conseillons
Quand vous regarderez les réponses, essayez d’éliminer tout d’abord les réponses qui, de toute
de comprendre pourquoi vous vous êtes trompé évidence, sont fausses et de vous concentrer, ensuite,
et pourquoi les réponses données sont les bonnes. sur les possibilités restantes. Notez que, parfois,
Si vous voulez vous imposer un rythme d examen la bonne réponse sera «Aucune de ces réponses».
dans votre travail de révision, prévoyez deux minutes par Ne vous découragez pas et ne pensez pas que vous
question du type Vrai/Faux/Incertain ou par question n’êtes pas bon si vous n’arrivez pas à déterminer
à choix multiple. Les problèmes à court développement d’emblée la bonne réponse. Les questions sont
appellent des réponses très diverses auxquelles il conçues pour vous faire réfléchir.
est difficile d’attribuer une durée moyenne. Sachez
cependant que nous ne recommandons pas de limite Problèmes à court développement La meilleure
de temps pour répondre aux questions contenues dans façon d’étudier en économie est de résoudre des
le Guide. Il vaut mieux faire ces exercices avec sérieux problèmes. Les problèmes figurent aussi au deuxième
et concentration, mais calmement. Si vous voulez vous rang des questions le plus souvent posées lors des tests
entraîner à répondre dans un laps de temps donne, et des examens — alors faites-en le plus souvent
utilisez plutôt les «examens de mi-étape» (voir la possible ! À la fin de chaque Autoévaluation, vous
description à la page viii) ou d anciens examens. trouverez dix problèmes à court développement
Chaque chapitre comporte de nombreuses questions chiffrés ou graphiques, qui s’inspirent souvent de sujets
et il vous faudra de deux à cinq heures pour y repondre. de politique économique. Pour de nombreux chapitres,
Si vous êtes fatigué (ou que vous commencez à vous c’est la partie la plus ardue de l’Autoévaluation. C’est
ennuyer), ne vous astreignez pas à répondre à toutes également la plus utile, car elle vous permettra de

les questions en une seule fois. Divisez votre travail consolider votre compréhension du matériel étudié
d’autoévaluation en plusieurs séances (deux au moins). dans ce s chapitres. Il s’agit par ailleurs d’un outil
viii PRÉSENTATION -

didactique plus que d’un outil d’évaluation. Nous Révision des parties et examens
présentons chaque problème en plusieurs parties, de mi-étape Au bout d’un certain nombre
allant des plus faciles aux plus difficiles, pour vous de chapitres, à la fin de chacune des parties
accompagner dans votre travail de façon à vous du manuel, vous trouverez un problème spécial
permettre d’avancer étape par étape. (et sa réponse), qui porte sur le matériel abordé
Les problèmes difficiles sont accompagnés dans tous les chapitres de la partie en question.
du symbole <*> qui indique que vous aurez besoin Ce problème met en lumière des questions de politique
de plus de temps et d’efforts pour les résoudre. Le et d’actualité (par exemple, les effets de la contrebande
symbole Ô vous donne le même genre d’indications que de cigarettes sur les recettes fiscales). Ces problèmes
vous donne, lors d’un test ou d’un examen, le nombre vous aideront à intégrer les notions étudiées dans les
de points ou le nombre de minutes alloué à une question. différents chapitres et, contrairement aux apparences,
ils sont étroitement reliés à la matière étudiée. Les
Réponses L’Autoévaluation est suivie des réponses
questions d’examen ressemblent souvent à ces problèmes.
à toutes les questions. Ne regardez pas les réponses
Chaque révision comporte également un examen
avant d’avoir essayé de répondre aux questions. Lorsque
de mi-étape composé de questions à choix multiple,
vous les regarderez, cherchez à comprendre pourquoi
tirées de chacun des chapitres de la partie correspondante
vous vous êtes trompé et pourquoi la réponse donnée
du manuel. Cet examen est présenté comme un véritable
est la bonne.
examen ou test, qui vous limite dans le temps. Comme
Chaque réponse aux questions du type Vrai/Faux/
pour les autres questions à choix multiple du Guide de
Incertain et aux questions à choix multiple comprend
l’étudiant, vous y trouverez des réponses accompagnées
une brève explication. Vous pourrez ainsi voir où vous
d’explications succinctes.
vous êtes trompé et comprendre le raisonnement
Si vous utilisez adéquatement le manuel et le
économique qui sous-tend la réponse. Les réponses
Guide de l’étudiant, vous serez bien préparé pour vos
aux questions difficiles sont accompagnées du symbole
examens, mais, surtout, vous aurez acquis des aptitudes
Ô pour vous indiquer la difficulté de la question.
analytiques et une capacité de raisonnement qui vous
(Pour vous mettre le plus possible en situation de
serviront tout au long de votre vie, quelle que soit
test ou d’examen, nous n’avons pas indiqué quelles
la carrière que vous aurez choisie.
questions du type Vrai/Faux/Incertain ou quelles
questions à choix multiple sont difficiles. Dans un
Avez-vous des conseils à nous donner?
examen, on accorde à toutes les questions à choix
multiple le même nombre de points et vous ne savez Nous avons tout mis en œuvre pour rendre ce Guide
pas lesquelles sont difficiles.) de l’étudiant aussi clair et aussi précis que possible. Mais,
Les réponses détaillées aux problèmes à court malheureusement, il n’est pas exclu que quelques erreurs
développement seront particulièrement utiles pour s’y soient glissées. Vous êtes le mieux placé pour juger
clarifier et illustrer le processus de raisonnement de la qualité de ce recueil et pour évaluer notre travail.
typique de l’analyse économique. Si les réponses ne Si vous décelez des erreurs ou si vous avez des suggestions
réussissent pas à vous éclairer, relisez les rubriques pouvant améliorer le Guide de l’étudiant, écrivez-nous.
appropriées du manuel. Si cela ne vous suffit pas,
demandez des éclaircissements à votre professeur Avi. J. Cohen
ou à son assistant ou, encore, consultez les membres Fbarvey B. King
de votre groupe de travail. Pierre Chapleau
TABLE DES MATIÈRES
| rc
1 PARTIE Introduction
Chapitre 1 Qu’est-ce que la science économique ? 1
Chapitre 2 Les graphiques — construction et utilisation 11
Chapitre 3 La production, la croissance et l’échange 22
Chapitre 4 L’offre et la demande 37
Chapitre 5 L’élasticité 54
Chapitre 6 Les marchés en action 67

namm -ire
1 PARTIE Chapitres 1 à 6 81

4)e
L PARTIE Les marchés de biens et services
Chapitre 7 L’utilité et la demande 87
Chapitre 8 Contraintes budgétaire, préférences
et choix de consommation 100

Chapitre 9 L’organisation de la production 115


Chapitre 10 La production et les coûts 125
Chapitre 11 La concurrence 138

Chapitre 12 Le monopole 154

Chapitre 13 La concurrence monopolistique et l’oligopole 173

imnm p
U PARTIE Chapitres 7 à 13 186

3 Les marchés des facteurs de production


Chapitre 14 La détermination du prix et l’allocation
des facteurs de production 193

Chapitre 15 Le marché du travail 206

Chapitre 16 Les marchés financiers et les marchés


des ressources naturelles 217

Chapitre 17 L’incertitude et l’information 227

3
Révision

PARTIE Chapitres 14 à 17 238

4 Les marchés et le gouvernement


243
Chapitre 18 Les lacunes du marché et les choix publics

Chapitre 19 L’inégalité et la redistribution 254

Chapitre 20 La politique de concurrence 265

Chapitre 21 Les effets externes, l’environnement et le savoir 275

4 PARTIE
Révision

Chapitres 18 à 21 286

e
5 PARTIE L’économie mondiale
Le commerce international 290
Chapitre 22
'
c fl a

Qu’ est-ce que la science


économique ?

♦ Les variations des coûts marginaux (coûts


WÊÊÊÊÊHÊÊKÊmmÊmÊÊmÊmÊmÊmmmKKmÊÊÊÊÊÊÊÊÊKÊÊÊÊÊmÊÊiÊÊÊKÊKÊÊmmmmmÊmKÊm

L P ° INTS CLÉS d’opportunité) et des avantages marginaux sont


des incitatifs qui amènent les gens à reconsidérer
leurs choix.
Les économistes: comment ils raisonnent
♦ La concurrence a des répercussions sur la chaîne des
Le problème économique fondamental est celui de substitutions - des effets ultérieurs - qui dominent
la rareté. les effets initiaux ou immédiats.

♦ Parce que les ressources disponibles pour satisfaire


nos besoins et nos désirs sont insuffisantes, nous Les économistes: ce qu’ils font
ne pouvons obtenir tout ce que nous voulons
et nous devons faire des choix, ce qui nous force Les économistes s’intéressent à :
à prendre des décisions optimales - à choisir
la meilleure utilisation, ou utilisation optimale, ♦ la microéconomie - l’étude des décisions prises
par les ménages et les entreprises ;
des ressources disponibles.

♦ La science économique est la science qui explique ♦ la macroéconomie — l’étude de l’économie


les choix que font les gens pour résoudre le nationale et de l’économie mondiale ainsi que
problème de la rareté. de la croissance et des fluctuations des agrégats
économiques.
Le coût d’opportunité est le concept le plus important
pour faire le meilleur choix parmi les différentes La science économique est une science sociale qui
possibilités offertes. Le coût d’opportunité de tout établit la distinction entre :
choix est la valeur de l’option la plus avantageuse
♦ les énoncés positifs — énoncés qui se rapportent à ce
qui n’a pas été retenue.
qui est -, que l’on peut vérifier en les confrontant
♦ Le coût d’opportunité réel d’un choix se mesure aux faits et
en biens et services auxquels on a renonce et non
♦ les énoncés normatifs — énoncés relatifs à ce qui
en unités monétaires.
devrait être -, qui reposent sur des valeurs et
♦ Le coût d’opportunité inclut la valeur du temps qui sont invérifiables.
consacré à obtenir un bien ou service ainsi que
La science économique tente de comprendre le
le coût externe.
fonctionnement du monde économique et elle ne
Les individus font des choix économiques à la marge, s’intéresse qu’aux énoncés positifs. Les économistes
en comparant le coût additionnel — le coût marginal — essaient de découvrir les énoncés positifs qui rendent
et Xavantage additionnel — l’avantage marginal — compte des phénomènes observés. Cela se fait en
d’une légère augmentation de leur activité. Lorsque trois étapes :
l’avantage marginal est supérieur au coût marginal,
♦ l’observation et la mesure ;
ils choisissent d’augmenter cette activité.

Selon le principe de substitution, lorsque le coût ♦ la construction de modèles économiques —


d’opportunité d’une activité augmente, les gens représentations schématiques du monde réel
remplacent celle-ci par d’autres activités. comprenant deux éléments :
2 CHAPITRE I

• hypothèses sur ce qui est essentiel et ce qui ne • les marchés des produits, où s’échangent les biens
l’est pas ; et services ;
• conséquences ou prédictions pouvant être vérifiées • les marchés des facteurs, où s’échangent les
en les comparant aux faits ; facteurs de production (travail, terre, capital,
esprit d’entreprise).
♦ l’élaboration de théories économiques -
généralisations permettant de comprendre les choix L’économie canadienne est:
économiques à partir de la construction et de la
♦ une économie mixte, car elle a principalement
vérification de modèles économiques (figure 1.1
recours à la coordination par le marché mais aussi
du manuel).
à la coordination par directives ;
Les modèles et théories économiques, qui nous sont ♦ une économie ouverte, puisqu’elle entretient des liens
fort utiles, permettent d’isoler les principales forces en avec d’autres économies par ses exportations et ses
jeu et de départager cause et effet. Pour ce faire, il faut:
importations, ainsi que par des prêts et emprunts
♦ recourir à l’hypothèse ceteris paribus, qui signifie internationaux. Une économie fermée n’entretient
que l’on maintient tous les autres facteurs constants aucun lien économique avec d’autres économies.
afin d’isoler les effets d’un facteur à la fois;

♦ éviter les erreurs de raisonnement, en particulier :

• l’erreur de composition - énoncé faux qui


RAPPELS
stipule que ce qui est vrai pour une partie d’un
1 La définition de l’économique (comment les gens
tout l’est également pour le tout et vice versa;
utilisent les ressources limitées pour satisfaire
• l’erreur du post hoc ergo propter hoc - erreur de
des besoins illimités) nous amène directement
raisonnement qui consiste à prétendre que
à trois concepts économiques importants :
l’événement A a provoqué l’événement B
le choix, le coût d’opportunité et la concurrence.
simplement parce qu’il a eu lieu avant celui-ci.
Lorsque les ressources disponibles sont
La politique économique est la discipline qui tente insuffisantes, nous ne pouvons satisfaire tous
d’améliorer la performance économique en visant nos besoins et devons donc faire des choix entre
les objectifs suivants: plusieurs possibilités. En faisant un choix, nous
renonçons à d’autres possibilités, et le coût
♦ l’efficience économique, soit l’efficience dans la
d’opportunité de tout choix représente la valeur
production, dans la consommation et dans les
de la meilleure possibilité que nous avons écartée.
échanges ;
De plus, lorsque les ressources disponibles
♦ l’équité, soit la justice ou l’égalité économique; sont insuffisantes pour satisfaire l’ensemble
des besoins, ceux-ci doivent être hiérarchisés
♦ la croissance économique, soit l’augmentation
et les individus doivent se faire concurrence
du revenu et de la production par personne ;
pour obtenir les ressources limitées.
♦ la stabilité économique, soit l’atténuation de
variations importantes du taux de croissance 2 L’analyse marginale est une notion fondamentale
économique, du niveau d’emploi et des prix à laquelle se réfèrent les économistes pour
moyens. prédire quels seront les choix des individus.
Pour comprendre ce qu’est l’analyse marginale,
il importe de se concentrer sur les coûts et les
L’économie: vue d’ensemble
avantages additionnels plutôt que sur les coûts et
L’économie est un mécanisme qui permet de répartir les avantages totaux. Par exemple, pour prédire
les ressources limitées entre les diverses utilisations si Taejong va manger un quatrième hamburger,
potentielles. Ce mécanisme répond aux cinq questions l’économiste compare l’avantage additionnel -
essentielles: Quoi? Commenté Quand?Où? Pour qui? soit la satisfaction que retirera Taejong de la
Le modèle économique (figure 1.2 du manuel) inclut consommation d’un quatrième hamburger - à
deux composantes : son coût additionnel. Les avantages et coûts
totaux des quatre hamburgers n’ont ici aucune
♦ les décideurs, soit les ménages, les entreprises et
pertinence. Ce n’est que si l’avantage marginal
les gouvernements ;
est supérieur au coût marginal que Taejong
♦ les marchés, soit les lieux où se coordonnent consommera un quatrième hamburger.
les décisions d’achat et de vente au moyen
d’ajüstements de prix. Il existe deux types 3 Pour tenter de comprendre comment et pourquoi
de marchés: une chose fonctionne (par exemple un avion,
QU'EST-CE QUE LA SCIENCE ÉCONOMIQUE? 3

un objet en chute libre, une économie), nous dans les livres les faits et les descriptions relatifs
pouvons avoir recours à une description ou à une au monde économique. La valeur des études en
théorie. Une description est une liste d’éléments ce domaine repose sur la capacité de réfléchir
qui se rapportent à une chose. Toutefois, elle ne de manière critique aux problèmes économiques
nous dit pas quels sont ceux qui sont essentiels et de comprendre comment fonctionne le monde
pour comprendre comment fonctionne un avion économique. Cette compréhension des forces
(la forme des ailes) ni quels sont ceux qui sont essentielles qui en régissent le fonctionnement ne
moins importants (la couleur de la peinture). peut s’acquérir que par la maîtrise de la théorie
Les scientifiques ont recours à des théories économique et la construction de modèles.
qui font abstraction des éléments descriptifs
complexes du monde réel et qui leur permettent
de ne se concentrer que sur les éléments essentiels
à la compréhension. Ceux-ci sont incorporés AUTOÉVALUATION
dans des modèles - des représentations très
simplifiées du monde réel.
En physique, comme dans d’autres sciences
Vrai/Faux/1 ncertain
naturelles, si nous voulons comprendre la force
(Justifiez votre réponse.)
essentielle d’un phénomène, par exemple ce qui
explique la chute des objets (la gravité), nous I La science économique étudie comment utiliser
utilisons une théorie pour construire un modèle des ressources illimitées pour satisfaire des
simple, que nous vérifions par une expérience besoins limités.
contrôlée. Dans le cas de la chute des objets,
nous créons un vide pour éliminer les forces
moins importantes comme la résistance de l’air.
Les modèles économiques permettent de se 2 Si François dormait au lieu d’assister à un cours
concentrer sur les forces essentielles (concurrence, d’économique ou de faire de la course, le cours
intérêt personnel) à l’œuvre dans une économie, manqué représenterait le coût d’opportunité de
tout en éliminant les forces moins importantes ses heures de sommeil.
(fantaisie, publicité, altruisme). À la différence
des physiciens, les économistes ne peuvent avoir
recours à des expériences contrôlées pour tester
leurs modèles. Ainsi, il est difficile de confirmer 3 Lorsque le coût d’opportunité d’une activité
ou d’infirmer de manière concluante une théorie augmente, les gens remplacent les autres
et ses modèles. activités par celle-ci.

4 Les modèles ressemblent aux cartes routières,


qui sont utiles précisément parce qu’elles ne
reproduisent pas tous les détails du monde reel. 4 Dans une économie coordonnée par directives,
Une carte qui reproduirait tous ces détails les décisions concernant quoi, comment, quand,
(lampadaires, bornes-fontaines, fils électriques) où et pour qui sont le résultat des ajustements
aurait peu d’intérêt. Une carte utile offre une de prix.
vue simplifiée, soigneusement choisie en
fonction de son objectif. N oublions pas que les
modèles économiques ne prétendent pas que le
monde réel est aussi simple que le modèle. Les 5 Une économie fermée est une économie où
modèles prétendent capter 1 effet simplifié des la liberté économique est très limitée.
forces réelles à l’œuvre dans 1 économie. Avant
de tirer des conclusions s’appliquant à 1 économie
réelle à partir d’un modèle, nous devons nous
assurer que, lorsque nous réinsérerons toutes 6 L’économie n’est pas une science puisqu’elle
les complexités du monde reel abstraites par le se rapporte à l’étude d’êtres humains doués
modèle, les conclusions seront les mêmes que de volonté et non à celle d’objets inanimés.
celles fournies par celui-ci.

5 Le but le plus important de l’étude économique


n’est pas d’apprendre ce qu’il faut penser mais
plutôt comment penser. On peut toujours trouver
4 CHAPITRE I

7 Un énoncé positif se rapporte à ce qui est, alors 4 Un énoncé positif


qu’un énoncé normatif se rapporte à ce qui sera. a) se rapporte à ce qui devrait être.
b) se rapporte à ce qui est.
c) est toujours vrai.
d) peut être évalué comme étant faux ou vrai
8 La macroéconomie inclut l’étude des causes par l’observation et la mesure.
de l’inflation. e) b et d.

5 Une économie mixte possède à la fois


a) un commerce interne et international.
9 Pour vérifier un modèle économique, il faut b) des industries ouvertes et fermées.
comparer les prédictions qu’il permet de formuler c) des mécanismes positifs et normatifs.
et les faits. d) des mécanismes de coordination par le
marché et par directives.
e) la richesse et la pauvreté.

10 Lorsque les prédictions d’un modèle sont


6 Renée a la possibilité d’assister à un cours
d’économique ou de jouer au tennis. Si elle
contredites par les faits pertinents, il faut rejeter
choisit d’assister au cours, la valeur d’une partie
la théorie ou la modifier.
de tennis est
a) supérieure à celle du cours d’économique.
b) non comparable à celle du cours
d’économique.
Questions à choix multiple c) égale à celle du cours d’économique.
d) le coût d’opportunité de sa présence au cours
1 La situation caractérisant l’incapacité de satisfaire d’économique.
pleinement tous ses besoins compte tenu des e) nulle.
ressources limitées dont on dispose s’explique par
a) le coût d’opportunité. 7 Pour un client, le coût d’opportunité d’une
b) le problème de la rareté. coupe de cheveux à 10 $ correspond à
c) l’économie normative. a) la valeur de la meilleure possibilité à laquelle
d) la difficulté à déterminer quoi produire. il a dû renoncer pour dépenser ses 10$.
e) la difficulté à déterminer qui consommera b) la valeur de la meilleure possibilité à laquelle
la production. il a dû renoncer pour prendre le temps de se
faire couper les cheveux.
2 Le problème de la rareté c) la valeur de la meilleure possibilité à laquelle
a) n’existe que dans les économies coordonnées il a dû renoncer pour dépenser ses 10 $
par le marché. et prendre le temps de se faire couper
b) n’existe que dans les économies coordonnées les cheveux.
par directives. d) une valeur de 10 $ pour le coiffeur.
c) existe dans toutes les économies. e) la valeur que représente pour le coiffeur
d) n’existe que lorsque les gens n’ont pas le temps nécessaire pour faire une coupe
optimisé leurs choix. de cheveux.
e) existe, mais sera éliminé par la croissance
économique.
8 Tous les éléments suivants composent les
objectifs d’une politique économique sauf
3 Lorsque le gouvernement consacre des ressources
a) l’équité.
à la construction d’un barrage, ces ressources ne
b) l’efficience.
sont plus disponibles pour construire une
c) l’esprit d’entreprise.
autoroute. Cet exemple illustre la notion
d) la stabilité.
a) de mécanisme de marché.
e) la croissance.
b) de macroéconomie.
c) de coût d’opportunité.
d) d’économie fermée.
e) de coopération.
QU’EST-CE QUE LA SCIENCE ÉCONOMIQUE? 5

9 Lequel des éléments suivants est un exemple I5 On peut décrire l’économie canadienne comme
du capital comme facteur de production ? étant une économie
a) La monnaie détenue par General Motors a) fermée.
b) Une obligation émise par General Motors b) de marché.
c) Une usine de fabrication d’automobiles de c) coordonnée par directives.
General Motors d) mixte.
d) Toutes ces réponses. e) prospère et ne connaissant pas le problème
e) Aucune de ces réponses. de la rareté.

10 Comment vérifie-t-on un modèle économique ? 16 Un énoncé normatif se rapporte


a) En examinant le réalisme de ses hypothèses a) à ce qui est habituellement le cas.
b) En comparant ses prédictions aux faits b) aux hypothèses d’un modèle économique.
c) En comparant ses descriptions aux faits c) à ce qui devrait être.
d) En ayant recours au comité de vérification d) aux prédictions d’un modèle économique.
de l’Association économique canadienne e) à ce qui est.
e) Toutes ces réponses.
I7 « Les riches ont des taux d’imposition sur
I I Les éléments suivants sont des facteurs le revenu plus élevés que les pauvres. »
de production, sauf Cet énoncé est
a) les ressources naturelles. a) normatif.
b) les outils. b) positif.
c) l’esprit d’entreprise.
c) descriptif.
d) le gouvernement. d) théorique.
e) la terre.
e) b et c.

18 Toutes autres choses étant égales par ailleurs,


12 Une économie fermée
lequel des énoncés suivants est vrai ?
a) exporte plus qu’elle n’importe.
1. Si le chômage augmente, le coût
b) importe plus quelle n’exporte.
d’opportunité des études universitaires
c) a une production strictement contrôlée par
diminue.
le gouvernement.
2. Si les hommes gagnent généralement plus que
d) n’entretient aucun lien économique avec les
les femmes sur le marché du travail, le coût
ménages et le gouvernement.
d’opportunité des études universitaires est plus
e) n’entretient aucun lien économique avec
élevé pour les hommes que pour les femmes.
d’autres économies.
a) 1
b) 2
I3 Le domaine de l’économique qui étudie les
c) 1 et 2
décisions des ménages et des entreprises s’appelle
d) Ni 1 ni 2
a) la macroéconomie.
e) Impossible de juger sans informations
b) la microéconomie.
supplémentaires
c) l’économie positive.
d) l’économie normative.
I9 Lequel des éléments suivants «estpas une
e) l’économie familiale.
ressource en capital ?
a) La monnaie
14 La rareté peut être éliminée grâce
b) Un marteau de menuisier
a) à la coopération. c) Une fabrique de chaussures
b) à la concurrence. d) Une machine à trancher le pain
c) aux mécanismes de coordination par
e) Le Stade olympique
le marché.
d) aux mécanismes de coordination par directives.
e) Aucune de ces réponses.
6 CHAPITRE I

20 Lequel des énoncés suivants est positif? 25 Lequel des sujets suivants ne doit pas être
a) Les faibles loyers limiteront l’offre de considéré comme relevant de la macroéconomie ?
logements. a) Les causes d’une baisse du prix du jus d’orange
b) Les taux d’intérêt élevés sont néfastes pour b) Les causes d’une baisse des prix moyens
l’économie. c) La cause des récessions
c) Les loyers sont trop élevés. d) L’effet du déficit budgétaire du gouvernement
d) Les propriétaires d’immeubles résidentiels sur l’inflation
devraient pouvoir exiger le loyer qu’ils e) La détermination du revenu global
désirent.
e) Le gouvernement devrait contrôler les loyers
Problèmes à court développement
qu’imposent les propriétaires de logements.

21 Toutes les questions suivantes relèvent de 1 Qu’entend-on par rareté et pourquoi l’existence
la microéconomique sauf de la rareté signifie-t-elle que nous devons faire
a) le progrès technologique. des choix ?
b) les salaires et autres revenus.
c) les disparités nationales en matière 2 Si tous les gens optimisaient, cela résoudrait
de richesse. le problème de la rareté. Dites si vous êtes
d) la production. d’accord ou non avec cette affirmation et
e) la consommation. expliquez pourquoi.

22 Lequel des éléments suivants ne permet pas 3 Élisabeth, Doug et Mei-Lin prévoient se
d’évaluer le coût d’opportunité des études déplacer d’Halifax à Sydney. Le voyage est
universitaires ? d’une heure par avion et de cinq heures par
a) Les frais de scolarité train. Le prix du billet d’avion est de 100 $,
b) Le coût des manuels scolaires et celui du billet de train de 60 $. Les trois
c) Le coût des repas personnes devront s’absenter du travail pour
d) Le revenu qui aurait été gagné en travaillant faire ce voyage. Élisabeth gagne 5 $ l’heure,
e) Toutes ces réponses. Doug 10 $ l’heure, et Mei-Lin 12 $ l’heure.
Calculez le coût d’opportunité du voyage
23 Le coût d’opportunité ne comprend pas par avion et par train pour chacune des personnes.
a) le coût externe. En supposant quelles recherchent toutes trois
b) la meilleure possibilité à laquelle nous avons l’optimisation des coûts, par quel moyen de
renoncé. transport chaque personne devrait-elle se rendre
c) toutes les possibilités auxquelles nous avons à Sydney?
renoncé.
d) le coût du temps. 4 Expliquez l’interdépendance qui existe entre
e) le coût réel. les ménages et les entreprises à la figure 1.2 de
la page 17 du manuel. (Ne pas tenir compte
24 Lequel ou lesquels des énoncés suivants sont de la présence des gouvernements pour répondre
normatifs ? à cette question.)
a) Les scientifiques ne devraient pas faire
d’énoncés normatifs. 5 Supposons que le gouvernement fasse
b) Les verrues sont causées par le contact avec construire un hôpital et le dote en personnel
des crapauds. dans le but de dispenser des services médicaux
c) Lorsque le prix des disques compacts «gratuits».
diminue, les gens en achètent davantage. a) Quel est le coût d’opportunité des services
d) Lorsque le revenu augmente, la vente de biens médicaux gratuits ?
de luxe baisse. b) La société, comme entité, voit-elle ces services
e) c et d. comme étant gratuits ?
QU’EST-CE QUE LA SCIENCE ÉCONOMIQUE? 7

6 Indiquez si chacun des énoncés énumérés


ci-après est de type normatif ou positif. RÉPONSES
S’il s’agit d’un énoncé normatif (positif),
récrivez-le de manière à ce qu’il devienne
positif (normatif).
Vrai/Faux/1 ncertain
(Justifiez votre réponse.)
a) Le gouvernement devrait réduire la taille
du déficit afin d’abaisser les taux d’intérêt.
b) L’imposition par le gouvernement d’une 1 F Ressources limitées et besoins illimités.
taxe sur les produits du tabac entraînera la 2 1 Le coût d’opportunité est la meilleure
réduction de la consommation de ceux-ci. possibilité à laquelle nous renonçons ; dans ce
cas-ci, nous ne savons pas si François préfère
7 Supposons que nous examinions un modèle de le cours d’économique ou la course.
croissance de plantes qui prédit que, compte tenu 3 F Les gens remplacent l’activité devenue plus
de la quantité d’eau et de lumière, l’application coûteuse par d’autres activités.
d’un engrais stimule la croissance des plantes. 4 F Les ajustements de prix sont une des
a) Comment pourriez-vous vérifier ce modèle ? caractéristiques principales de la coordination
b) En quoi cette vérification diffère-t-elle de par le marché.
celle qu’un économiste pourrait faire pour 5 F Une économie fermée n’entretient aucun lien
vérifier un modèle économique ? avec d’autres économies.
6 F La science n’est pas définie par le sujet, mais
8 Lors d’ une entrevue télévisée, lorsque l’interviewer par la méthode d’observation, la mesure et la
a demandé à une rameuse ce qui lui manquait vérification des modèles théoriques.
le plus en raison du temps passé à s’entraîner 7 F Les énoncés normatifs se rapportent à ce qui
pour les Jeux olympiques, elle a répondu: devrait être.
« Une vie sociale normale. » Elle a aussi révélé 8 V L’inflation traite du niveau général des prix.
quelle avait renoncé à un emploi de 20 000 $ 9 V Vérifier les prédictions, et non les
par an pour pouvoir s’entraîner à plein temps. hypothèses.
Elle a reçu une bourse de 10 000 $ de Sport 10 V Les prédictions d’un modèle doivent être
Canada, mais cette bourse n’a pas suffi à couvrir vérifiées par les faits pour faire partie d’une
toutes ses dépenses. Ses frais de nourriture théorie acceptée.
et de logement étaient de 5 000 $ par an, et
ses frais d’entraînement (coût des services de
Questions à choix multiple
l’entraîneur, frais d’équipement, etc.) de 16 000 $
par an.
a) Quel est, pour cette rameuse, le coût 1 b Par définition.
d’opportunité annuel pour tenter de 2 c Avec des besoins illimités et des ressources
remporter la médaille d’or? limitées, la rareté ne sera jamais éliminée.
b) Quel est, pour le Canada, le coût 3 c L’autoroute est la possibilité qui a été
d’opportunité annuel de l’entraînement de abandonnée.
cette rameuse ? 4 e Par définition.
c) En général, quel est, pour le Canada, le coût 5 d Par définition.
d’opportunité annuel de la participation de 6 d En choisissant le cours d’économique, on
cette rameuse et des autres athlètes canadiens indique que la valeur du cours est supérieure
aux Jeux olympiques ? à celle du tennis. Le tennis est la (meilleure)
possibilité qui a été abandonnée pour le cours
9 Pourquoi l’économie du Canada est-elle d’économique.
considérée comme étant une économie mixte ? 7 c Le coût d’opportunité comprend les biens
réels qui pourraient être achetés avec les 10 $
10 Supposons que votre ami, qui se spécialise en ainsi que le coût du temps.
histoire, affirme que les théories économiques 8 c L’esprit d’entreprise est un facteur de
sont inutiles parce qu’elles reposent sur des production.
modèles irréalistes. Il déclare que, puisque ces 9 c Le capital est défini en tant que bien
modèles ne tiennent pas compte des nombreux manufacturé utilisé dans la production.
détails descriptifs du monde reel, ils ne peuvent 10 b Les hypothèses ne sont pas des descriptions
permettre de comprendre le fonctionnement réalistes ; ce sont des représentations
de l’économie. Comment defendriez-vous votre simplifiées du monde.

décision d’étudier la théorie économique? I I d Le gouvernement est un décideur.


8 CHAPITRE I

I2 e Par définition. concerne la durée du voyage et le prix du billet


I3 b Par définition. d’avion ou de train. Les coûts totaux du voyage
14e La rareté est une réalité inévitable. par train ou par avion pour Élisabeth, Doug et
I5 d L’économie mixte repose sur la coordination Mei-Lin sont présentés au tableau 1.1.
par le marché et la coordination par
directives.
TABLEAU l.l
16c Le mot clé de l’énoncé normatif est devrait.
17 e Les énoncés positifs décrivent les faits Voyageur Train Avion
concernant ce qui est.
18c Le chômage réduit le revenu moyen attendu Élisabeth

du travail. Un coût d’opportunité plus élevé (a) Prix du billet 60$ 100$
pour les hommes n’est peut-être pas équitable, (b) Coût d’opportunité
mais c’est un fait. du temps de voyage
19a La monnaie n’est pas un bien manufacturé à 5 $ l’heure 25$ 5$
utilisé dans la production. Coût total 85$ 105 $
20 a Bien que a puisse être évalué comme étant Doug
vrai ou faux, d’autres énoncés sont une (a) Prix du billet 60$ 100$
question de point de vue. (b) Coût d’opportunité
21c c relève de la macroéconomie. du temps de voyage
22 c On doit payer pour nos repas, que l’on aille à 10 $ l’heure 50$ 10$
ou non à l’université ; les repas ne font donc Coût total 110$ 1 10$
pas partie des coûts d’opportunité.
23 c Vous ne renoncez pas à toutes les possibilités. Mei-Lin
Si vous aviez assisté à la conférence, vous (a) Prix du billet 60$ 100$
n’auriez pas pu dormir plus longtemps et faire (b) Coût d'opportunité
de la course à pied. du temps de voyage
24 a Le mot clé est devrait. Même l’énoncé b est à 12 $ l'heure 60$ 12$
positif. Coût total 120$ 112$
25 a Le prix d’un bien individuel relève de la
microéconomie. D’après le calcul des coûts du tableau 1.1,
Élisabeth aurait intérêt à prendre le train,
Mei-Lin l’avion, et Doug pourrait prendre
Problèmes à court développement
indifféremment l’un ou l’autre.

D’une part, les entreprises dépendent des


1 La rareté est le phénomène universel qui repose
ménages en ce qui a trait à l’offre des facteurs de
sur le fait que les êtres humains ont des désirs et
production. D’autre part, les ménages dépendent
besoins illimités et que les ressources nécessaires
des entreprises pour gagner leur revenu. Les
pour produire les biens et services sont limitées.
ménages utilisent ce revenu pour acheter des
Le fait que les biens et services soient limités
biens et services aux entreprises, alors que ces
signifie que les individus ne peuvent satisfaire
dernières comptent sur l’argent qu’elles reçoivent
tous leurs besoins. Ils doivent donc faire des choix.
des ménages pour acheter plus de facteurs de
production au cours de la période subséquente
2 Pas d’accord. Si tout le monde optimisait, nous
et renouveler ainsi le flux circulaire.
ferions la meilleure utilisation possible de nos
ressources et obtiendrions les plus grands a) Même lorsque les services médicaux sont
avantages ou satisfactions possibles étant donné offerts sans frais (gratuitement), il y a
la quantité de ressources limitées. Toutefois, cela toujours des coûts d’opportunité. Le coût
ne veut pas dire que nous satisferions tous nos d’opportunité de la prestation de ces services
besoins illimités. Le problème de la rareté ne représente la meilleure utilisation possible
pourra être «résolu» tant et aussi longtemps que des ressources consacrées à la construction
les gens auront des besoins illimités et que les de l’hôpital, et la meilleure utilisation possible
ressources disponibles pour satisfaire ces besoins des ressources (comprenant les ressources
seront limitées. humaines) consacrées au fonctionnement
de celui-ci.
3 L’élément déterminant est que le coût b) Ces ressources ne sont plus disponibles pour
d opportunité total du voyage comprend la d’autres activités ; elles représentent donc un
valeur de la meilleure possibilité en ce qui coût pour la société.
QU'ES T-CE QUE LA SCIENCE ÉCONOMIQUE? 9

6 a) Il s’agit d’un énoncé normatif. L’énoncé Deuxièmement, parce quelle s’entraîne pour
suivant est positif: «Si le gouvernement les Jeux olympiques, cette rameuse a renoncé à
réduit la taille du déficit, les taux d’intérêt un salaire annuel de 20 000 $. Troisièmement,
vont diminuer. » ses dépenses d’entraînement totalisent 16 000 $
b) Il s’agit d’un énoncé positif. L’énoncé suivant et sont partiellement couvertes par une
est normatif: «Le gouvernement devrait subvention gouvernementale de 10 000 $. En
imposer une taxe sur les produits du tabac s’entraînant, elle renonce donc à une quantité
afin de réduire la consommation de ceux-ci. » de biens et services d’une valeur de 6 000 $.
Quatrièmement, ses dépenses courantes
7 a) Il est possible de vérifier la prédiction du habituelles ne font pas partie du coût
modèle par l’expérience contrôlée suivante d’opportunité de l’entraînement du fait quelle
et par l’observation attentive des résultats. devrait les supporter, quelle s’entraîne ou non.
Choisissez un nombre de parcelles de terrain Le coût d’opportunité total annuel de
de même dimension, aux caractéristiques l’entraînement de cette rameuse est de
semblables, et nécessitant la même quantité 26 000 $ plus la valeur que représente pour
d’eau et de lumière. Plantez des quantités elle une vie sociale normale (un coût subjectif
égales de graines dans les parcelles. Ajoutez qu’il est difficile de mesurer pour les autres).
de l’engrais dans certaines (en quantités b) Le coût d’opportunité annuel pour le Canada
différentes), et pas dans d’autres. Lorsque de l’entraînement de cette rameuse est de
les plantes auront poussé, mesurez-en la 20 000 $ (coût d’opportunité du temps de
croissance, puis comparez la croissance des la rameuse), plus 16 000 $ de dépenses
plantes des parcelles fertilisées à celle des d’entraînement (probablement une estimation
plantes des parcelles non fertilisées. Si la raisonnable du coût d’opportunité des services
croissance des plantes est plus rapide dans de l’entraîneur et de l’équipement), plus la
les parcelles fertilisées, le modèle et la théorie valeur que représente pour la rameuse la vie
sur laquelle il repose sont acceptés à titre sociale à laquelle elle a dû renoncer - un coût
provisoire. Si la croissance des plantes n’est total de 36 000 $, plus la valeur de la vie
pas plus rapide dans les parcelles fertilisées, sociale à laquelle elle a renoncé. Si cette
la théorie (le modèle) est rejetée ou les rameuse ne s’était pas entraînée pour les Jeux
hypothèses en sont modifiées. Pour que olympiques, l’économie canadienne aurait
l’engrais soit efficace, il peut être nécessaire pu produire une valeur de remplacement
d’ajouter de l’eau. de 36 000 $ de biens et services.
Construisez ensuite un nouveau modèle c) Le coût d’opportunité annuel pour le Canada
qui prédit que, avec plus d’eau dans les deux de la participation d’une équipe d’athlètes
cas (et la même quantité de lumière), les aux Jeux Olympiques correspond à la valeur
plantes des parcelles fertilisées pousseront annuelle des biens et services qui auraient
davantage que les plantes des parcelles non pu être produits avec les ressources consacrées
fertilisées. Vérifiez ce modèle et continuez à l’entraînement des athlètes.
de modifier les hypothèses jusqu’à ce que
les prédictions soient conformes aux faits. 9 L’économie canadienne est une économie mixte
b) Les économistes ne peuvent effectuer des car elle repose à la fois sur la coordination par
expériences aussi contrôlées ; ils doivent plutôt le marché et sur la coordination par directives.
modifier une à une les hypothèses des autres La majeure partie de la coordination se fait
modèles envisageables pour ensuite comparer par le marché, mais les gouvernements et
les résultats. Les différents résultats obtenus les grandes entreprises ont également recours
ne peuvent par la suite être comparés qu aux à la coordination par directives.
variations des données réelles de 1 économie.
Il s’agit de procédures de construction et de 10 Une réponse brève au défi que vous lance votre
vérification de modèles plus difficiles et ami pourrait être celle qui apparaît au n° 4
moins précises que celles de 1 expérience de la section « Rappels ». Les modèles sont
contrôlée avec les parcelles fertilisées. comme les cartes routières, qui sont utiles
précisément parce qu’elles ne tiennent pas
8 a) Il s’agit ici de savoir quels sont exactement compte de tous les détails du monde réel.
les coûts attribuables à l’entraînement pour Une carte utile offre une vue simplifiée,
les Jeux olympiques. Premièrement, il y a attentivement choisie en fonction de l’objectif
la valeur que représente pour 1 athlète la «vie visé. Aucun cartographe n’affirmera que le
sociale normale» à laquelle elle a renoncé. monde est aussi simple que celui qui apparaît
10 CHAPITRE I

sur sa carre, et les économistes ne prétendent pas les conditions sous-jacentes de l’offre et de la
que le monde économique réel est aussi simple demande de blé, mais également le type de
que celui de leurs modèles. En revanche, ils monnaie ou d’instrument de crédit utilisé dans
prétendent que ceux-ci isolent l’effet simplifié les échanges ; les caractéristiques personnelles
de certaines forces réelles (le comportement des marchands de blé comme la couleur des
rationnel) à l’œuvre dans le monde économique et yeux et des cheveux de chaque marchand, [...]
qu’ils permettent de faire des prédictions qui sont le nombre de membres de sa famille, leurs
vérifiables par une comparaison avec les faits. caractéristiques, [...] le type de sol sur lequel le
Un autre moyen de répondre à votre ami blé a été cultivé, [...] le temps qu’il a fait durant
serait de le mettre au défi de décrire un autre la saison de croissance, [...] et ainsi de suite
modèle ou une autre théorie plus réaliste. indéfiniment. Tout effort fait pour obtenir
Vous feriez bien à cet égard de citer Milton ce type de «réalisme» rendra certainement une
Friedman (un lauréat du prix Nobel d’économie) : théorie absolument inutile. »
«Une théorie, ou ses «hypothèses», ne peut Extrait de Milton Friedman, «The
être entièrement «réaliste» au sens descriptif Methodology of Positive Economies », dans
immédiat [...] Une théorie entièrement «réaliste» Essays in Positive Economies, Chicago, University
du marché du blé devrait inclure non seulement of Chicago Press, 1953, p. 32.
Les graphiques —
construction et utilisation

♦ Relation positive (directe) — les variables évoluent


CONCEPTS CLÉS ensemble dans le même sens ; la pente est positive.

♦ Relation négative (inverse) — les variables évoluent


La représentation graphique des données en sens opposé; la pente est négative.

Les graphiques permettent de visualiser des quantités ♦ Relations qui ont un maximum et un minimum :
qui sont représentées sous la forme de distances.
• la relation est d’abord positive, atteint un
Sur un graphique à deux variables :
maximum (pente nulle), puis devient négative;
♦ la ligne horizontale est l'axe des abscisses (axe des x) ; • la relation est d’abord négative, atteint un
minimum (pente nulle), puis devient positive.
♦ la ligne verticale est l'axe des ordonnées (axe des y) ;
♦ Les variables n’ayant aucune relation entre elles
♦ l’intersection des deux axes (0) est l'origine.
(indépendantes) — une variable change alors que
Les principaux types de graphiques rencontrés en l’autre reste constante; le graphique est une ligne
économie sont les suivants. droite verticale ou horizontale.

♦ Le diagramme de dispersion — il montre la


relation entre deux variables, l’une étant mesurée La pente d’une relation
sur l'axe des abscisses, l’autre sur l'axe des ordonnées.
La pente d’une relation correspond à la variation de la
♦ Le graphique de série chronologique — il montre
quantité mesurée sur l’axe des ordonnées divisée par la
la relation entre le temps (mesuré sur l’axe des
variation de la quantité mesurée sur l’axe des abscisses.
abscisses) et une ou plusieurs autres variables
(mesurées sur l’axe des ordonnées). Il indique le ♦ A signifie «variation de».
niveau de la variable, la direction du changement,
♦ La formule de la pente est Aj//Ax = Distance
la vitesse du changement et la tendance (tendance
verticale/Distance horizontale parcourue.
générale à la hausse ou à la baisse).
♦ Une ligne droite (relation linéaire) a une pente
♦ Le graphique d’échantillon représentatif — il
constante.
indique la valeur d’une variable pour differents
• La pente d’une relation positive (qui monte de
groupes de population à un moment donne.
la gauche vers la droite) est positive.
Les graphiques trompeurs omettent souvent 1 origine • La pente d’une relation négative (qui descend
ou étirent/compressent l’échelle de mesure pour de la gauche vers la droite) est négative.
accentuer ou amoindrir la variation. Prenez 1 habitude
♦ La pente d’une courbe n’est pas constante et on la
de bien regarder les chiffres et les données places sur
calcule de la manière suivante :
les axes avant d’interpréter un graphique.
• en un point sur la courbe — en traçant une ligne
droite tangente à la courbe à ce point et en
Les graphiques utilisés dans les modèles calculant la pente de la droite;
économiques • le long d’un arc - en traçant une ligne droite
Les graphiques qui montrent les relations entre les entre deux points sur la courbe et en calculant
variables appartiennent aux quatre catégories suivantes. la pente de la droite.
12 CHAPITRE 2

Représentation graphique de relations À l’aide de cette équation, vous pouvez tracer


entre plus de deux variables la ligne en trouvant l’ordonnée à l’origine (là où
la ligne coupe l’axe vertical desjy), en trouvant
On peut représenter graphiquement les relations entre l’abscisse à l’origine (là où la ligne coupe l’axe
plusieurs variables en gardant constantes les valeurs horizontal des x) et en traçant une ligne droite
de toutes les variables sauf deux. On a recours pour entre ces deux points.
cela à l’hypothèse ceteris paribus — « toutes autres choses
Pour trouver l’ordonnée à l’origine, supposez
étant égales».
que x = 0.

y = a + b (0)
y—a
RAPPELS Pour trouver l’abscisse à l’origine, supposez
que y = 0.
1 Dans le manuel, les relations entre les variables
économiques sont presque invariablement 0 — a + bx
représentées et analysées graphiquement. En x = —a /b
acquérant dès le début de bonnes connaissances
En réunissant ces deux points (x = 0, y — a)
sur les graphiques, vous pourrez comprendre
et (x = —a /b, y — 0) ou (0, a) et (—a /b, 0)
plus facilement les analyses économiques
on obtient la ligne montrée à la figure 2.1
présentées aux chapitres suivants. Évitez l’erreur
courante de croire qu’une compréhension FIGURE 2.1
superficielle des graphiques est suffisante.

2 Si votre expérience en analyse graphique est


limitée, étudiez attentivement ce chapitre,
car il vous prépare aux analyses économiques
ultérieures. Il est probable que vous devrez le
consulter de temps en temps. Si vous êtes déjà
familiarisé avec la construction et l’utilisation
des graphiques, ce chapitre peut vous paraître
redondant. Dans ce cas, parcourez-le rapidement
et effectuez le test d’autoévaluation proposé
plus loin.

3 La pente d’une droite est un concept linéaire Pour toute droite d’équation y = a + bx, la
étant donné qu’il s’agit d’une propriété d’une pente est b. Pour voir comment appliquer cette
ligne droite. C’est pourquoi la pente est équation générale, prenez l’exemple suivant :
constante le long d’une ligne droite, mais
y = 6 — 2x
différente en divers points d’une courbe (non
linéaire). Pour connaître la pente d’une courbe, Pour trouver l’ordonnée à l’origine, supposez
nous calculons en fait la pente d’une droite. que x = 0
Dans le manuel, deux possibilités de calcul de la
y = 6 - 2(0)
pente d’une courbe nous sont offertes: 1) la
y— 6
pente en un point donné et 2) la pente le long
d’un arc. Dans le premier cas, on calcule la Pour trouver l’abscisse à l’origine, supposez
pente de la ligne droite au point où elle touche la que y — 0
courbe (lui est tangente). Dans le second, on
0 = 6 — 2x
calcule la pente de la ligne droite le long de l’arc
x= 3
entre deux points.
En réunissant ces deux points, (0, 6) et (3, 0),
® 4 On peut également décrire une ligne droite on obtient la ligne de la figure 2.2.
d un graphique au moyen d’une simple équation.
La forme générale de l’équation d’une ligne
droite est :

y — a + bx
LES GRAPHIQUES: CONSTRUCTION ET UTILISATION 13

FIGURE 2.2 FIGURE 2.3

La pente de cette droite est -2. Étant donné


quelle est négative, la relation entre les variables 6 Dans la figure 2.3, la pente de la courbe
x et y est négative (inverse). augmente à mesure que l’on se déplace
du point b au point c.

AUTOÉVALUATION
7 Dans la figure 2.3, la pente de la courbe se
rapproche de zéro à mesure que l’on déplace
Vrai/Faux/1 ncertain du point a au point b.
(Justifiez votre réponse.)

I Un graphique qui omet l’origine est un


graphique trompeur. 8 Dans la figure 2.3, la valeur de x atteint un
minimum au point b.

2 Si un graphique illustrant la relation entre deux


variables est ascendant (se déplace vers la droite), 9 Pour une ligne droite, lorsqu’une faible variation
de y est associée à une forte variation de x, la
sa pente est positive.
pente est abrupte.

3 La relation entre deux variables indépendantes


10 Pour une ligne droite, lorsqu’une forte variation
est illustrée par une droite verticale.
de y est associée à une faible variation de x, la
courbe a une pente accentuée.

4 On calcule la pente d’une ligne droite en


divisant la variation de la valeur de la variable
inscrite sur l’axe des abscisses par la variation Questions à choix multiple
de la valeur de la variable inscrite sur 1 axe
des ordonnées.
I La figure 2.4 est
a) un graphique de série chronologique à une
variable.
b) un graphique de série chronologique à deux
5 Dans la figure 2.3, la relation entre y et x est
variables.
d’abord négative, atteint un minimum, puis
c) un diagramme de dispersion.
devient positive à mesure que x augmente.
d) b et c.
e) Aucune de ces réponses.
14 CHAPITRE 2

5 La relation entre deux variables qui évoluent


FIGURE 2.4 en sens opposé se traduit graphiquement par
une courbe qui
a) a une pente positive.
b) a une pente relativement abrupte.
c) a une pente relativement faible.
d) a une pente négative.
e) est non linéaire.

6 Pour représenter graphiquement une relation


entre plus de deux variables, sur quelle
hypothèse doit-on se fonder ?
Année a) Un énoncé normatif
b) Un énoncé positif
c) Une relation linéaire

La ligne pointillée de la figure 2.4 représente la d) Des variables indépendantes

variable y. Lequel de ces énoncés décrit le mieux e) Le ceteris paribus

la relation entre x et y ?
a) x et y ont tendance à évoluer en sens opposé 7 Comment appelle-t-on l’évolution d’une variable

au fil du temps. à la hausse ou à la baisse au fil du temps ?

b) y a. tendance à évoluer à l’opposé de x, mais a) La pente

une année plus tard. b) La tendance

c) y a tendance à évoluer dans le même sens que c) L’axe des coordonnées

x au fil du temps. d) Le niveau

d) x a tendance à évoluer dans le même sens que e) La corrélation

y, mais une année plus tard.


e) y a tendance à évoluer dans le même sens que 8 Dans la figure 2.5, lorsque x augmente,

x, mais une année plus tard. la relation entre x et y est


a) positive et la pente décroissante.

D’après les données du tableau 2.1, il semble b) négative et la pente décroissante.

que c) négative et la pente croissante.

a) x et y aient une relation négative. d) positive et la pente croissante.

b) x et y aient une relation positive. e) positive et la pente d’abord croissante

c) x et y n’aient aucune relation. puis décroissante.

d) x et y aient d’abord une relation négative,


puis une relation positive. FIGURE 2.5
e) x et y aient d’abord une relation positive,
puis une relation négative.

TABLEAU 2.1

Année X Y

1990 6,2 143


1991 5,7 156
1992 5,3 162

Lorsque les variables x et y se déplacent


ensemble dans le même sens, on dit quelles
a) ont une relation positive.
b) ont une relation négative.
c) ont une relation inverse.
d) n’ont aucune relation.
e) . suivent une tendance.
LES GRAPHIQUES: CONSTRUCTION ET UTILISATION 15

9 Quelle est la pente le long de l’arc entre b et c 13 En utilisant les données du tableau 2.2 et
dans la figure 2.5 ? en supposant que les prix soient maintenus
a) 1/2 constants, on représente la relation entre
b) 2/3 le revenu familial (axe vertical) et le nombre
c) 1 de casseaux achetés (axe horizontal) par
d) 2 a) une ligne verticale.
e) 3 b) une ligne horizontale.
c) une ligne à pente positive.
10 Dans la figure 2.5, observez les pentes de l’arc d) une ligne à pente négative.
ab et de l’arc bc. Il est difficile de déterminer e) une ligne à pente positive ou négative, selon
exactement la pente au point b, mais elle doit le prix qui est maintenu constant.
être
a) supérieure à 5/2. 14 Dans la figure 2.6, la variable x a
b) d’environ 5/2. a) une relation positive avec y et une relation
c) entre 5/2 et 1. positive avec z.
d) d’environ 1. b) une relation positive avec y et z.

e) inférieure à 1. c) une relation négative avec y et une relation


positive avec z.
I I En utilisant les données du tableau 2.2 et en d) une relation négative avec y et z.
supposant que le revenu soit maintenu constant, e) une valeur supérieure à celle de z.
on représentera la relation entre le prix des
fraises (axe vertical) et le nombre de casseaux FIGURE 2.6
achetés (axe horizontal) par
a) une ligne verticale.
b) une ligne horizontale.
c) une ligne à pente positive.
d) une ligne à pente négative.
e) une ligne qui atteint un minimum.

TABLEAU 2.2

Revenu Prix Nombre


familial par casseau de casseaux
hebdomadaire de fraises achetés
(en dollars) (en dollars) par semaine

300 1,00 5 I5 Dans la figure 2.6, toutes choses étant égales par
300 1,25 3 ailleurs, une baisse de la valeur de 2; entraînera,
1,50 2 a) une diminution de la valeur de x.
300
b) une augmentation de la valeur de x.
400 1,00 7
c) une augmentation de la valeur de y.
400 1,25 5
d) aucun changement de la valeur de y.
400 1,50 4
e) a et d.

I6 Dans le tableau 2.3, supposons que w est


I2 En utilisant les données du tableau 2.2, supposez la variable indépendante mesurée le long de
que le revenu familial diminue, passant de 400 $ l’axe horizontal. La ligne reliant w et u est
à 300 $ par semaine. Dans ce cas, le graphique a) positive avec une pente décroissante.
b) négative avec une pente décroissante.
reliant le prix des fraises (axe vertical) au nombre
c) positive avec une pente croissante.
de casseaux achetés (axe horizontal)
d) négative avec une pente constante.
a) aura une pente négative.
e) positive avec une pente constante.
b) aura une pente positive.
c) se déplacera vers la droite.
TABLEAU 2.3
d) se déplacera vers la gauche.
e) a et d.
w 2 4 6 8 10
u 15 12 9 6 3
16 CHAPITRE 2

I7 En vous reportant au tableau 2.3, supposez que 21 En tous points le long d’une ligne droite,
w soit la variable indépendante mesurée le long la pente est
de l’axe horizontal. La pente de la ligne reliant a) positive.
w et u est b) négative.
a) +3. c) constante.
b) -3. d) égale à zéro.
c) -2/3. e) Aucune de ces réponses.
d) +3/2.
e) -3/2. 22 Quelle est la pente de la ligne de la figure 2.8 ?
a) 2
18 Dans la figure 2.7, lorsque le revenu des b) 1/2
ménages augmente de 1 000 $, les dépenses c) 3
des ménages d) 1/3
a) augmentent de 1 333 $. e) -3
b) diminuent de 1 333 $.
c) restent inchangées. ® 23 Si la ligne de la figure 2.8 se poursuivait jusque
d) augmentent de 1 000 $. sur l’axe des abscisses, quelle serait la valeur
e) augmentent de 750 $. de x lorsque y est égal à zéro ?
a) 0
FIGURE 2.7 b) 2
c) 2/3
d) -2/3
e) -3/2

FIGURE 2.8

Revenu des ménages


(en milliers de dollars par an)

19 Dans la figure 2.7, lorsque le revenu des ménages


est de 0, les dépenses des ménages sont
a) égales à 0.
b) égales à -4 000 $.
c) égales à 4 000 $. ® 24 Si l’équation d’une droite est y ~ 6 + 3x, alors
d) égales à 8 000 $. la pente est égale à
e) impossibles à déterminer à partir a) —3 et l’ordonnée à l’origine à 6.
du graphique. b) —3 et l’ordonnée à l’origine à —2.
c) 3 et l’ordonnée à l’origine à 6.
20 Dans la figure 2.7, lorsque les dépenses d) 3 et l’ordonnée à l’origine à —2.
des ménages sont de 28 000 $, le revenu des e) 3 et l’ordonnée à l’origine à —6.
ménages est
a) de 36 000 $. ® 25 Si l’équation d’une droite est y = 8 - 2x, alors
b) de 32 000 $. la pente est égale à
c) de 28 000 $. a) —2 et l’abscisse à l’origine à —4.
d) de 25 000 $. b) —2 et l’abscisse à l’origine à 4.
e) Aucune de ces réponses. c) —2 et l’abscisse à l’origine à 8.
d) 2 et l’abscisse à l’origine à —4.
e) 2 et l’abscisse à l’origine à 4.
LES GRAPHIQUES: CONSTRUCTION ET UTILISATION 17

Problèmes à court développement


FIGURE 2.9

1 Tracez un graphique illustrant chacune des


relations suivantes entre les variables x et jy:
a) Les variables x et y évoluent dans le même
sens.
b) Les variables x et y évoluent en sens opposé.
c) Lorsque x augmente, y atteint un maximum.
d) Lorsque x augmente, y atteint un minimum.
e) Les variables x et y évoluent en sens opposé
mais, chaque fois que x augmente, y diminue
dans une proportion toujours plus forte.
f) y est indépendante de la valeur de x.
g) x est indépendante de la valeur de y.

2 Que veut-on dire lorsque l’on affirme que la


pente d’une droite est égale à —2/3?

3 Indiquez deux manières de mesurer la pente


d’une courbe.

4 Comment tracer le graphique d’une relation


entre plus de deux variables en utilisant un
graphique à deux dimensions ?

5 En utilisant les données du tableau 2.4,


7 Tracez
a) tracez un graphique de série chronologique
a) une ligne droite ayant une pente égale à —10
pour le taux d’intérêt.
et passant par le point (2, 80).
b) tracez un graphique de série chronologique
b) une ligne droite ayant une pente égale à 2 et
à deux variables pour le taux d’inflation et
passant par le point (6, 10).
pour le taux d’intérêt.
c) tracez un diagramme de dispersion pour le
8 L’équation d’une droite est jy = 4 — 2x.
taux d’inflation (axe horizontal) et le taux
a) Calculez l’ordonnée à l’origine, l’abscisse
d’intérêt (axe vertical).
à l’origine et la pente.
d) diriez-vous que la relation générale entre
b) Tracez le graphique de cette droite.
le taux d’inflation et le taux d’intérêt est
positive, négative ou nulle ?
9 Utilisez le graphique de la figure 2.10 pour
calculer la pente
TABLEAU 2.4
a) le long de l’arc entre les points a et b.
b) au point b.
Taux d’inflation Taux d’intérêt
c) au point c, et expliquez votre réponse.
Année (%) (%)

1970 5,4 6,4 FIGURE 2.10


1971 3,2 4,3
1972 3,4 4,1
1973 8,3 7,0
1974 1 1,8 7,9
1975 6,7 5,8
1976 4,9 5,0
1977 6,5 5,3
1978 8,6 7,2
1979 12,3 10,0

6 Calculez les pentes des droites de la figure 2.9


(a) et (b).
18 CHAPITRE 2

10 Dans le tableau 2.5, la variable x représente 3 I Le graphique des variables indépendantes


le nombre de parapluies vendus par mois, la peut être vertical ou horizontal.
variable y, le prix d’un parapluie et la variable z, 4 F Pente = A variable sur l’axe vertical (y) I A
le nombre moyen de jours de pluie par mois. variable sur l’axe horizontal (x).
a) sur le même graphique, tracez la courbe 5 V L’arc ab aurait une pente négative, l’arc bc,
illustrant la relation entre x (axe horizontal) une pente positive.
et y (axe vertical) lorsque z = 4, z = 5 et 6 V La courbe s’accentue, ce qui signifie que Ay
2: = 6. En moyenne, il pleut 6 jours par mois. augmente plus vite que Ax; la pente est donc
Cela implique une certaine relation moyenne positive et croissante.
entre la vente mensuelle de parapluies et le 7 V Au point b, la pente de la tangente est égale à
prix des parapluies. Supposons que « l’effet 0, étant donné que Ay = 0 le long de la ligne
de serre » réduise le nombre moyen de jours horizontale jusqu’à b.
de pluie à 4 jours par mois. Qu’advient-il de 8 F La valeur de y est à son minimum au point b.
la courbe illustrant la relation entre la vente 9 F Une pente accentuée signifie une Ay élevée
de parapluies et le prix des parapluies ? associée à une faible Ax.
b) Sur un graphique, tracez la courbe illustrant 10 V Une ligne abrupte a une pente forte, ce qui
la relation entre x (axe horizontal) et z (axe signifie qu’une grande Ay est associée à une
vertical) lorsque y = 10 $ et lorsque y = 12 $. petite Ax.
Cette relation entre x et z est-elle positive
ou négative ? Questions à choix multiple
c) Sur un graphique, tracez la courbe illustrant 1 b Les deux variables sont x et y. Dans les
la relation entre y (axe horizontal) et z (axe diagrammes de dispersion, le temps n’est
vertical) lorsque x = 120 et x = 140. La relation pas inscrit sur un axe.
entre y et z est-elle positive ou négative ? 2 e Par exemple, x commence à diminuer en 1991,
y commence à diminuer en 1992.
TABLEAU 2.5 3 a Des valeurs plus élevées de x (6,2) sont
associées à des valeurs plus faibles dey (143).
Nombre 4 a Définition.
Nombre de moyen 5 d La ligne peut être abrupte, plate ou non
parapluies de jours linéaire, mais sa pente doit être négative.
vendus Prix par de pluie 6 e Les autres variables doivent être maintenues
par mois parapluie par mois constantes afin que l’on puisse dégager la
(*) (y) « relation entre deux variables.
7 b Définition.
120 10$ 4
8 a Pente de l’arc ab = +2,5. Pente de l’arc
140 10$ 5
bc = +1.
160 10$ 6
9 c Ay = 3 (8-5); Ax=3 (6-3).
100 12$ 4
10 c 5/2 est la pente de ab, alors que 1 est la pente
120 12$ 5
de bc.
140 12$ 6
I I d Utilisez soit les données des trois premières
80 14$ 4
lignes (revenu = 300), soit les données des
100 14$ 5
trois dernières lignes (revenu = 400). Le prix
120 14$ 6
le plus élevé est associé au plus petit nombre
de casseaux achetés.
©12 e Pour chaque prix, il y aura moins de casseaux
achetés (relation négative qui se déplace vers
l’origine).
RÉPONSES ©13 c Pour P = 1, les deux points sur la ligne
seront : (5 casseaux, 300 $) et (7 casseaux,
Vrai/Faux/Incertain 400 $). Même relation pour les autres prix.
(Justifiez votre réponse.) 14 c îy —> -lx en maintenant z constant. îz —» Tx
en maintenant y constant.
1 I C’est parfois trompeur, mais il arrive aussi © I5 a ■l'Z —y ■lot en maintenant y constant. 4^z —y 4^y
que le graphique fasse alors mieux ressortir en maintenant x constant.
' le phénomène étudié. 16 d À mesure que u>\, u-l. Au I Aw est constant.
2 V Les lignes ou courbes à pente ascendante ont I 7 e Entre deux points quelconques, Au — 3,
des pentes positives. Au) — —2.
LES GRAPHIQUES : CONSTRUCTION ET UTILISATION 19

18 e Pente (Ay/Ax) — 3/4. Si Ax (À revenu des 3 On peut mesurer la pente d’une courbe en un
ménages) = 1 000 $, alors Ay (A dépenses point ou le long d’un arc. On mesure la pente
des ménages) = 750 $. en un point en calculant la pente de la droite
19 c La où la ligne coupe l’axe (y) des dépenses qui est tangente à la courbe (touche la courbe)
des ménages. au point donné. On mesure la pente le long
20 b À partir de 28 000 $ sur l’axe (des dépenses) d’un arc en calculant la pente de la droite qui
vertical, tracez une ligne parallèle à x, puis forme l’arc.
descendez vers 32 000 $ sur l’axe (des revenus)
horizontal.
4 Pour faire le graphique d’une relation entre plus
21 c Le long d’une ligne droite, la pente peut être
de deux variables, nous supposons que toutes les
ou ne pas être a, b ou d.
variables sauf deux sont constantes, et traçons le
22 c Entre deux points quelconques, Ay = 3 et
graphique de la relation entre les deux variables
Ax = 1.
restantes. Nous pouvons ainsi tracer le graphique
Ô@ 23 d L’équation de la droite est y = 2 + 3x. Trouvez
de la relation entre n’importe quelle paire de
la solution pour l’abscisse à l’origine (lorsque
variables, étant donné que les valeurs des autres
y- 0). variables restent constantes.
@24 c Utilisez la formule y — a + bx. Pente = b,
ordonnée à l’origine = a.
5 a) La figure 2.12(a) présente un graphique de
@25 b Utilisez la formuley-a-vbx. Pente = b,
série chronologique pour le taux d’intérêt.
abscisse à l’origine = —a !b.
b) La figure 2.12(b) présente un graphique de
série chronologique à deux variables pour le
Problèmes à court développement taux d’inflation et le taux d’intérêt. Le taux
d’inflation est représenté par la ligne en
1 La figure 2.11 (a) jusqu’à (g) présente les
pointillé et le taux d’intérêt par la ligne
graphiques désirés.
continue.
2 Le signe négatif de la pente égale à -2/3 signifie c) La figure 2.12(c) présente le diagramme de
qu’il existe une relation négative entre les deux dispersion pour le taux d’inflation et le taux
variables. La valeur de 2/3 signifie que lorsque d’intérêt.
la variable mesurée sur l’axe vertical diminue d) En utilisant les graphiques (b) et (c) de la
de 2 unités (la distance verticale ou Ay), la figure 2.12, nous voyons que la relation
variable mesurée sur l’axe horizontal augmente de entre le taux d’inflation et le taux d’intérêt
3 unités (la distance horizontale parcourue ou Ax). est généralement positive.

FIGURE 2.1 I
20 CH-A PITRE 2

prenez un second point dont la coordonnée


FIGURE 2.12
en y diminue de 10 chaque fois que la
(a) coordonnée en x augmente d’une unité,
par exemple (5, 50). La pente entre les deux
points est égale à —30/3 = —10.
b) La ligne droite requise est présentée à la
figure 2.13(b). Tracez d’abord le point
(6, 10), puis prenez un second point dont
la coordonnée en y diminue de 2 chaque fois
que la coordonnée en x baisse d’une unité,
par exemple (5, 8). La pente entre ces deux
points est égale à —2/—1 = 2.

FIGURE 2.13

(b)

(c)
gll
<a>
10
c

^
Z)
9
£
8

5 -

4 -

8 a) Pour trouver l’ordonnée à l’origine, prenez


0 2,5 5,0 7,5 10,0 12,5
x = 0.
Taux d'inflation (%)
y = 4 - 2 (0)
y—4
6 Pour trouver la valeur de la pente, on peut Pour trouver l’abscisse à l’origine, utilisez
prendre deux points quelconques sur une ligne y= 0.
et calculer Ay/Ax. La pente de la ligne de la
0 = 4 - 2x
figure 2.9(a) est égale à -2/3, et la pente de la
ligne de la figure 2.9(b), à 1/2.
x—2
La pente de la droite est égale à -2, la valeur
7 a) La ligne droite requise est présentée à la figure du coefficient « b » devant x.
2.13(a). Tracez d’abord le point (2, 80), puis b) Le graphique est représenté à la figure 2.14.
LES GRAPHIQUES: CONSTRUCTION ET UTILISATION 21

FIGURE 2.14 FIGURE 2.15

9 a) La pente le long de l’arc entre les points a et b


est égale à —3/2.
b) La pente au point b est égale à -3/4.
c) La pente au point c est égale à zéro étant
donné qu’il s’agit d’un point minimum.
Près d’un point minimum, la pente négative
devient positive et doit passer par zéro, ou
devient nulle.

I0 a) Les relations entre x et y pour z = 4, 5 et 6


sont indiquées à la figure 2.15(a). Lorsque
le nombre moyen de jours de pluie (z) passe
de 6 à 4, la courbe représentant la relation
entre les ventes de parapluies (x) et le prix des
parapluies (y) se déplacera de la courbe z = 6
à la courbe z = 4. La relation entre x et y est
négative.
b) La relation entre x et z lorsque y est égal à
10 $ et lorsque y est égal à 12 $ est indiquée
au graphique (b) de la figure 2.15. La relation
entre x et z est positive.
c) La relation entre y et z lorsque x est égal à
120 et à 140 est indiquée au graphique (c )
de la figure 2.15. La relation entre y et z
est positive.

o
La production,
la croissance et l’échange

♦ Il faut préférer les points situés sur la CPP à ceux


CONCEPTS CLÉS situés à l’intérieur de cette courbe, compte tenu du
fait qu’il vaut mieux avoir plus de biens et services
que moins.
La courbe des possibilités de production
♦ Les points situés à l’extérieur de la CPP désignent
La production transforme les facteurs de production des niveaux de production irréalisables, compte
en biens et en services. tenu des ressources et de la technologie dont
on dispose.
♦ Les quatre facteurs de production sont le travail,
la terre, le capital (y compris le capital humain - En faisant un choix parmi les points efficaces situés sur
les aptitudes et les connaissances acquises au la CPP, on doit accepter un coût d’opportunité et faire
moyen de l’éducation et de la formation), ainsi un compromis. Lorsque la CPP est linéaire (droite) :
que l’esprit d’entreprise. ♦ les ressources sont homogènes : également

♦ Les biens sont tangibles/matériels et les services, productives dans toutes les activités ;

intangibles/immatériels. ♦ les coûts d’opportunité sont constants dans le cas


de tous les compromis situés n’importe où le long
♦ Les biens sont classés en biens de production
de la CPP.
(utilisés pour produire d’autres biens) et en biens
de consommation.
L’augmentation du coût d’opportunité
La courbe des possibilités de production {CPP)
Les CPP sont généralement arquées vers l’extérieur
♦ trace la frontière entre les combinaisons de biens
(concaves), et reflètent des coûts d’opportunité
et services qu’il est possible de produire et celles
croissants à mesure que la production d’un bien
qui sont irréalisables ;
augmente.
♦ indique le maximum de combinaisons de biens
♦ La CPP est arquée vers l’extérieur parce que les
et de services qui peuvent être produites avec
ressources ne sont pas homogènes — les ressources
les ressources et la technologie disponibles.
ne sont pas également productives dans toutes
Caractéristiques de la CPP les activités. Les ressources qui conviennent
le mieux à une activité donnée sent les premières
♦ Les points situés sur la CPP indiquent l’efficience
qui seront utilisées.
dans la production lorsqu’il est impossible de
produire des unités supplémentaires d’un bien ♦ La CCP concave représente un coût d’opportunité
sans devoir réduire la production d’un autre bien. croissant - le coût d’opportunité d’un bien
augmente avec la quantité produite de ce bien.
♦ Les points situés à l’intérieur de la CPP désignent
des volumes de production inefficaces ; il est ♦ En se déplaçant entre deux points qui se trouvent
possible de les produire, mais ils ne forment pas sur la CPP, on ne peut obtenir plus de biens X
le maximum de combinaisons de production ; qu’en obtenant moins de biens Y. Sur la CPP,
les ressources sont gaspillées ou mal affectées. le coût d’opportunité de biens X supplémentaires
correspond à la quantité de biens Y à laquelle
on doit renoncer.
LA PRODUCTION, LA CROISSANCE ET L’ÉCHANGE 23

♦ Le coût d’opportunité est nul lorsqu’on se déplace Les conventions sociales et l’échange
d’un point situé à l’intérieur de la CPP à un point
situé sur la CPP. Les conventions sociales qui ont été adoptées pour
faciliter les échanges sont :

La croissance économique ♦ les marchés ;

La croissance économique est l’expansion des ♦ les droits de propriété — régissant la possession,
l’utilisation et la cession de facteurs de production,
possibilités de production, c’est-à-dire un déplacement
vers l’extérieur de la CPP. de biens et de services ;

♦ la monnaie — l’échange monétaire permet de


♦ La CPP se déplace en raison de changements au
résoudre le problème de la double coïncidence
niveau des ressources ou des techniques.
des besoins inhérent au troc.
♦ L’accumulation de capital et le progrès technique
entraînent le déplacement de la CPP vers
l’extérieur — croissance économique. RAPPELS
♦ Le coût d’opportunité d’un plus grand nombre
de biens et de services dans l’avenir (croissance I Ce chapitre porte sur le concept extrêmement

économique grâce à l’accumulation de capital important de coût d’opportunité - la meilleure


possibilité que l’on écarte — qui a été présenté
ou au progrès technique, ou aux deux) correspond
au chapitre 1. Pour calculer le coût d’opportunité,
à la réduction de la consommation des biens et
voici une formule très utile qui permet d’éliminer
services courants.
certaines difficultés, notamment celles qui sont
liées au déplacement vers le haut ou le bas
Les gains de l’échange le long d’une courbe des possibilités de
production (CPP ) :
La production augmente lorsque les gens se spécialisent
dans une activité où ils jouissent d’un avantage . Renoncer Sacrifice
Coût d opportunité =-;— =-7-
comparatif. Obtenir Gain
♦ Un producteur d’un bien ou d’un service détient Le coût d’opportunité est égal à la quantité
un avantage comparatif lorsqu’il est en mesure de biens à laquelle on doit renoncer, divisée
de produire un bien à un coût d’opportunité par la quantité de biens que l’on obtiendra.
inférieur à celui de ses concurrents. Cette formule s’applique à toutes les CPP,
♦ Lorsque chaque producteur se spécialise dans quelles soient linéaires, comme dans la figure 3.1

la production d’un bien pour lequel il possède du manuel, ou concaves, comme dans la
un avantage comparatif et qu’il l’échange contre figure 3.2 du manuel. Pour mieux le comprendre,

d’autres biens, il retire des gains de cet échange. examinons de nouveau la CPP concave.

♦ La spécialisation et l’échange permettent la FIGURE 3.1


consommation (pas la production) à des points
situés à l’extérieur de la CPP.

♦ Un producteur jouit d’un avantage absolu dans


la production de tous les biens lorsque, en utilisant
la même quantité de facteurs de production,
il peut produire un plus grand nombre des memes
biens que tout autre producteur.

• L’avantage absolu n’a aucun rapport avec


la spécialisation et les gains de 1 échange.
• Même les individus qui détiennent un avantage
absolu peuvent tirer profit de 1 échange en se
spécialisant dans les activités qui leur confèrent
un avantage comparatif

♦ L’avantage comparatif dynamique résulte de la


spécialisation dans une activité, de 1 apprentissage
par la pratique et, au fil du temps, de la
Jeux vidéo (en milliers par année)
production d’un bien ou d’un service au coût
d’opportunité le plus bas.
24 CHAPITRE 3

Considérons d’abord un exemple de augmente avec l’augmentation de la production


déplacement vers le bas. En nous déplaçant de l’un ou l’autre bien. Avec le déplacement
de c à d, quel est le coût d’opportunité d’un de la possibilité c vers la possibilité d, le coût
jeu vidéo supplémentaire ? On doit renoncer d’opportunité par jeu vidéo est de 0,5 de missile.
à 500 missiles (3 000 - 3 500) pour obtenir Toutefois, en augmentant la production
1 000 jeux vidéo (3 000 - 2 000). En appliquant de jeux vidéo de d à e, on constate que le
la formule de la pente vue au chapitre 2 (Ay/Ax), coût d’opportunité par jeu vidéo augmente,
on obtient le coût d’opportunité suivant: devenant de 1 missile. En produisant les
premiers 1 000 jeux vidéo, nous utilisons les
—500 missiles _ , ... . .
-= —0,5 missile par jeu video ressources qui conviennent le mieux à leur
+1 000 jeux vidéo production. Cependant, lorsque nous
Prenons ensuite un exemple de déplacement augmentons la production de jeux vidéo, nous
vers le haut, soit de d à c. Quel est le coût devons utiliser des ressources qui conviennent
d’opportunité d’un missile supplémentaire ? moins bien à leur production - augmentant
Nous devons renoncer à 1 000 jeux vidéo donc le coût d’opportunité. On peut avancer
(2 000 - 3 000) pour obtenir 500 missiles un argument parallèle pour expliquer
(3 500 - 3 000). En appliquant la formule, l’augmentation du coût d’opportunité due à
on obtient le coût d’opportunité suivant : l’augmentation de la production de missiles.
—1 000 jeux vidéo ... Il est également possible de construire
--—--= —2 jeux video par missile un modèle de CPP encore plus simple et qui
+500 missiles
suppose que les ressources sont homogènes,
On mesure toujours le coût d’opportunité c’est-à-dire également productives dans toutes
en unités des biens auxquels on a renoncé. les activités. Compte tenu de cette hypothèse,
le coût d’opportunité est constant à mesure
©2 Le coût d’opportunité peut aussi être relié à qu’augmente la production de n’importe lequel
la pente de la CPP. En nous déplaçant entre des biens. Un coût d’opportunité constant
deux points quelconques de la CPP, nous signifie que la CPP sera une ligne droite (plutôt
obtenons un coût d’opportunité d’une unité qu’arquée). Comme certains des exercices
supplémentaire du bien sur l’axe horizontal de : suivants vous le montreront, un modèle aussi
simple illustre bien le principe de l’avantage
| pente de CPP | comparatif, sans qu’on ait à se préoccuper
du coût d’opportunité croissant.
La pente de la CPP est négative, mais
les économistes décrivent le coût d’opportunité © 4 On explique dans le manuel qu’une personne
en termes de quantité positive de biens auxquels qui est plus productive qu’une autre dans la
on a renoncé. Nous devons donc utiliser la valeur production de tous les biens jouit d’un avantage
absolue (symbole : | |) de la pente pour obtenir absolu. Nous pouvons aussi définir 1 \'avantage
le nombre positif souhaité. absolu dans la production d’un seul bien. En
En remontant entre deux points quelconques comparant la productivité de deux personnes,
de la CPP, on obtient un coût d’opportunité on peut définir ce concept plus étroit d’avantage
d’une unité supplémentaire sur l’axe vertical de : absolu en fonction d’une plus grande production
de ce bien par unité de facteurs de production,
1
ou d’une moins grande quantité de facteurs
pente de CPP de production par unité de production. Il est
Il s’agit de la relation réciproque que utile de comprendre ces définitions de l’avantage
nous avons constatée entre les possibilités absolu seulement pour démontrer que ce
c et d. Le coût d’opportunité d’un jeu vidéo concept ne sert pas à expliquer la spécialisation
supplémentaire (sur l’axe horizontal) entre c et et l’échange. Les gains de l’échange ne dépendent
d est de 0,5 de missile. Le coût d’opportunité que de la différence entre les avantages
d’un missile supplémentaire (sur l’axe vertical) comparatifs. Un producteur détient un avantage
entre d et c est de 2 jeux vidéo. comparatif dans la production d’un bien s’il
peut produire ce bien à un coût d’opportunité
3 La courbe des possibilités de production des jeux inférieur à celui de ses concurrents.
vidéo/missiles suppose que les ressources ne sont
pas homogènes ; il s’agit donc de ressources qui 5 Les concepts présentés dans ce chapitre nous
ne sont pas également productives dans toutes les donnent l’occasion d’élaborer et d’utiliser des
activités. Par conséquent, le coût d’opportunité modèles économiques pour la première fois.
LA PRODUCTION, LA CROISSANCE ET L'ÉCHANGE 25

Il est utile d’envisager la nature de ces modèles I Le point a est impossible à atteindre.
dans le contexte de la discussion générale
des modèles du chapitre 1. Ainsi, un des modèles
présentés dans ce chapitre est une représentation
des possibilités de production de deux individus, 2 Le coût d’opportunité de l’augmentation
Marc et Mireille, et de leurs deux types de biens. de la production de 7 à 8 unités du bien 2
Ce modèle fait abstraction des complexités est de 4 unités du bien 1.
du monde réel, où s’activent des milliards
de gens et où les types de biens et de services
sont extrêmement différents. Par ce modèle, nous
pouvons expliquer un nombre de phénomènes 3 Le point c est impossible à atteindre.
que nous observons dans le monde réel,
comme la spécialisation et l’échange. Ce modèle
a également certaines implications et nous
permet de faire plusieurs prédictions. Les pays ©4 En se déplaçant du point b vers le point d,
qui consacrent, par exemple, une plus grande le coût d’opportunité de l’augmentation
proportion de leurs ressources à l’accumulation de la production du bien 2 est égal à la valeur
de capital augmenteront plus rapidement leurs absolue de la pente de la CPP entre b et d.
possibilités de production. On peut «vérifier»
ce modèle en comparant ces prédictions au
monde réel.
5 La croissance économique, en déplaçant la CPP
vers l’extérieur, élimine le problème de la rareté.

AUTOÉVALUATION

6 Dans un modèle où les ressources en capital


Vrai/Faux/Incertain peuvent croître, les points situés sur la CPP, qui
(Justifiez votre réponse.) correspondent à une plus grande consommation
de biens, entraînent une croissance plus rapide.
Veuillez vous reporter à la courbe des possibilités
de production {CPP) de la figure 3.2 pour répondre
aux questions 1 à 4.
7 Les incitations à la spécialisation et à l’échange
FIGURE 3.2 ne sont pas liées aux droits de propriété mais
seulement aux différents coûts d’opportunité.

8 Grâce à la spécialisation et aux échanges, un


pays peut produire à un point situé à l’extérieur
de sa CPP.

9 Le Canada n’a pas intérêt à nouer des relations


commerciales avec un pays comme le Mexique,
où la main-d’œuvre est bon marché.
Bien 2

10 Un système d’échange monétaire présuppose


une double coïncidence des besoins.
26 CHAPITRE 3

Questions à choix multiple 4 À la figure 3.3, l’économie est située au point b


de la CPP. Le coût d’opportunité découlant de
1 Si Henri peut augmenter la production du bien X l’augmentation de la production du bien F à 50
sans réduire la production de tout autre bien, alors unités est de
a) sa production se situe sur sa CPP. a) 2 unités de X.
b) sa production se situe à l’extérieur de sa CPP. b) 6 unités de X.
c) sa production se situe à l’intérieur de sa CPP. c) 8 unités de X.
d) sa CPP doit être linéaire. d) 20 unités de Y.
e) il doit préférer le bien X à tout autre bien. e) 30 unités de Y.

2 La forme arquée vers l’extérieur (concave) 5 Reportez-vous à la CPP de la figure 3.3. Quel
d’une CPP énoncé, parmi les suivants, est faux ?
a) est due au fait que les ressources sont a) Les ressources ne sont pas homogènes.
également productives dans toutes les b) Les points situés à l’intérieur de la courbe
activités. représentent les ressources non utilisées.
b) est due à l’accumulation du capital. c) En partant du point a, une augmentation
c) est due aux progrès techniques. de la production du bien Y entraînerait
d) reflète l’existence d’un coût d’opportunité le déplacement de la courbe vers l’extérieur.
croissant. d) Le coût d’opportunité de la production
e) reflète l’existence d’un coût d’opportunité du bien Y augmente avec l’augmentation
décroissant. de la production du bien Y.
e) On peut préférer le bien X ou le bien Y,
3 L’économie est située au point b de la CPP sans que la courbe se déplace.
de la figure 3.3. Le coût d’opportunité de
la production d’une unité supplémentaire 6 Étant donné que les ressources productives sont
du bien X est de rares, nous devons renoncer à une certaine
a) 1 unité de Y quantité d’un bien pour acquérir une plus grande
b) 20 unités de Y. quantité d'un autre. Il s’agit, en essence,
c) 1 unité de X. du concept
d) 8 unités de X. a) de la spécialisation.
e) 20 unités de X. b) de l’échange monétaire.
c) de l’avantage comparatif.
FIGURE 3.3 d) de l’avantage absolu.
e) du coût d’opportunité.

7 Un déplacement le long d’une CPP donnée


sera dû
a) à un progrès technique.
b) à une variation du stock de capital.
c) à un changement sur le plan de la main-
d’œuvre.
d) Toutes ces réponses.
e) Aucune de ces réponses.

8 Le coût d’opportunité du déplacement de la


CPP vers l’extérieur est déterminé par
a) l’accumulation de capital.
b) le progrès technique.
c) la réduction de la consommation
d’aujourd’hui.
d) l’augmentation de la consommation future.
e) Toutes ces réponses.
LA PRODUCTION, LA CROISSANCE ET L'ÉCHANGE 27

Dans une journée de huit heures, André peut produire I3 Tout ce qui est généralement acceptable
24 pains ou 8 kilogrammes de beurre. Dans une journée en échange de biens et de services est
de huit heures, Raymond peut produire 8 pains ou a) une marchandise.
8 kilogrammes de beurre. A l’aide de cette information, b) un moyen d’échange.
répondez aux questions 9 et 10. c) une propriété privée.
d) un produit de troc.
® 9 Quel énoncé, parmi les suivants, est vrai ? e) appelé une ressource d’échange.
a) André détient un avantage absolu dans
la production de beurre. 14 Le Canada et le Mexique produisent tous deux
b) Raymond détient un avantage absolu dans du pétrole et des pommes par le biais du travail
la production de beurre. seulement. Un baril de pétrole peut être produit
c) André détient un avantage absolu dans en 4 heures de travail au Mexique et en 8 heures
la production de pain. de travail au Canada. Un boisseau de pommes
d) André détient un avantage comparatif dans peut être produit en 8 heures de travail au
la production de beurre. Mexique et en 12 heures de travail au Canada.
e) Raymond détient un avantage comparatif Le Canada détient
dans la production de pain. a) un avantage absolu dans la production
de pétrole.
b) un avantage absolu dans la production
10 André et Raymond
de pommes.
a) peuvent tirer profit de l’échange si André
c) un avantage comparatif dans la production
se spécialise dans la production de beurre
de pétrole.
et Raymond, dans la production de pain.
d) un avantage comparatif dans la production
b) peuvent tirer profit de l’échange si André
de pommes.
se spécialise dans la production de pain
e) Aucune de ces réponses.
et Raymond, dans la production de beurre.
c) ne peuvent obtenir de gains de l’échange.
I5 Au Portugal, le coût d’opportunité de 1 balle
d) peuvent échanger, mais seul Raymond
de laine est de 3 bouteilles de vin. En Angleterre,
y gagnera.
le coût d’opportunité de 1 bouteille de vin est de
e) peuvent échanger, mais seul André y gagnera.
3 balles de laine. À l’aide de cette information,
déterminez si
I I Considérons deux biens, X et Y. Si le coût a) l’Angleterre détient un avantage absolu dans
d’opportunité de la production du bien X la production de vin.
de Paule est inférieur à celui de Gilbert, alors b) l’Angleterre détient un avantage absolu dans
a) Paule détient un avantage absolu dans la production de laine.
la production de X. c) le Portugal détient un avantage comparatif
b) Gilbert détient un avantage absolu dans dans la production de vin.
la production de Y. d) le Portugal détient un avantage comparatif
c) Paule détient un avantage comparatif dans dans la production de laine.
la production de X. e) on peut affirmer qu’aucun échange n’aura lieu.
d) Gilbert détient un avantage comparatif dans
la production de Y 16 La rareté des ressources implique que la CPP
e) c et d sont les bonnes réponses. a) est arquée vers l’intérieur (convexe).
b) est arquée vers l’extérieur (concave).
12 En général, plus la proportion des ressources c) a une pente positive.
consacrées à la recherche technologique est d) a une pente négative.
e) est linéaire.
importante
a) plus la consommation courante sera élevée.
b) plus le déplacement de la CPP vers 1 extérieur 17 Si des unités supplémentaires d’un bien
quelconque peuvent être produites à un coût
sera rapide.
c) plus le déplacement de la CPP vers 1 intérieur d’opportunité constant, la CPP
a) est arquée vers l intérieur (convexe).
sera rapide.
d) plus on aurait des chances de détenir un b) est arquée vers l’extérieur (concave).

avantage comparatif dans la production c) a une pente positive.


d) est parfaitement horizontale.
de tous les biens.
e) est linéaire.
e) plus la CPP sera arquée vers 1 extérieur.
28 CHAPITRE 3

I8 Reportez-vous à la figure 3.4 qui présente la CPP 20 Selon cette CPP


d’une économie non discriminatoire fonctionnant a) les ressources sont homogènes.
à plein rendement. Si cette économie pratiquait b) une combinaison de 3 bâtons de hockey
la discrimination contre les femmes, et de 9 feuilles d’érable est réalisable.
l’élimination de la discrimination entraînerait c) une combinaison de 3 bâtons de hockey
a) un déplacement de a vers b. et de 9 feuilles d’érable n’emploierait pas
b) un déplacement de b vers c. toutes les ressources.
c) un déplacement de a vers c. d) le coût d’opportunité de la production
d) un déplacement de la CPP vers l’extérieur. de bâtons de hockey augmente à mesure que
e) un déplacement de la CPP vers l’intérieur. la production de bâtons de hockey s’accroît.
e) le coût d’opportunité de la production
FIGURE 3.4 de bâtons de hockey diminue à mesure que
la production de bâtons de hockey s’accroît.

21 La CPP pour le vin et pour la laine se déplace


s’il y a variation
a) du prix des ressources.
b) du taux de chômage.
c) de la quantité de ressources.
d) des préférences pour le vin ou pour la laine.
e) Toutes ces réponses.

22 Reportez-vous à la CPP de la figure 3.5.


Un politicien qui affirme que « si nous voulons
que nos enfants aient de meilleures conditions
de vie, nous devons investir maintenant dans
l’avenir», recommande une combinaison actuelle

Supposons qu’une société ne produise que deux correspondant au point

biens - des bâtons de hockey et des feuilles d’érable. a) a.


Le tableau 3.1 indique trois combinaisons possibles b) b.

sur sa CPP. À l’aide des indications du tableau 3.1, c) c.

répondez aux questions 19 et 20. d) d.


e) e.

TABLEAU 3.1 POSSIBILITÉS


DE PRODUCTION FIGURE 3.5

Bâtons Feuilles
de hockey d’érable
Possibilité (unités/temps) (unités/temps)

a 3 0
b 2 3
c 0 9

19 En se déplaçant de la combinaison c vers


la combinaison b, le coût d’opportunité
de la production d’un bâton de hockey
supplémentaire est de
a) 2 feuilles d’érable. 23 Reportez-vous à la CPP de la figure 3.5. L’énoncé
b) 1/2 feuille d’érable. « le chômage est un terrible gaspillage des
c) 6 feuilles d’érable. ressources humaines» fait référence à un point
d) 1/6 de feuille d’érable. tel que
e) 3 feuilles d’érable. a) a.
b) b.
c) c.
d) d.
e) e.
LA PRODUCTION, LA CROISSANCE ET L'ÉCHANGE 29

24 Quel énoncé, parmi les suivants, donne Supposez qu’une entreprise dont les biens de
un avantage à un système d’échange monétaire production sont constants ait la CPP indiquée
par rapport au troc ? au tableau 3.2
a) Un système d’échange monétaire élimine
la base de l’avantage comparatif. TABLEAU 3.2 POSSIBILITÉS
b) Un système d’échange monétaire ne nécessite DE PRODUCTION
pas de moyen d’échange.
c) Seul un système d’échange monétaire permet Maximum Maximum
de retirer des gains de l’échange. d’unités d’unités
d) Un système d’échange monétaire ne requiert de beurre de canons
pas de double coïncidence des besoins. Possibilité par semaine par semaine
e) Tout ce qui précède fait partie des avantages
a 200 0
d’un système d’échange monétaire par rapport
b 180 60
au troc.
c 160 100
d 100 160
25 L’apprentissage par la pratique est la base
e 40 200
a) de l’avantage comparatif absolu.
f 0 220
b) de l’avantage comparatif dynamique.
c) des droits de propriété intellectuelle.
a) Représentez, sur du papier graphique,
d) de l’échange monétaire.
ces possibilités : indiquez les points et tracez
e) Aucune de ces réponses.
la CPP. (Portez les canons sur l’axe des x).
b) Si la production se déplace de la possibilité c
Problèmes à court développement vers la possibilité d, combien d’unités de
beurre correspondent au coût d’opportunité
1 Pourquoi une CPP a-t-elle une pente négative ? par unité de canons ?
Pourquoi est-elle arquée vers l’extérieur? c) Si la production se déplace de la possibilité d
vers la possibilité e, combien d’unités de beurre
2 Considérons deux économies, l’une sans correspondent au coût d’opportunité par
possibilité de fabrication d’outils et l’autre unité de canons ?
ayant cette possibilité. d) En général, qu’arrive-t-il au coût d’opportunité
a) Dans une économie sans possibilité des canons lorsque la production de canons
de fabrication d’outils (biens de production augmente ?
constants), quel est le coût d opportunité e) En général, qu’arrive-t-il au coût d’opportunité
du déplacement d’un point situé à 1 intérieur du beurre lorsque la production de beurre
de la CPP vers un point situé sur la CPP ? augmente? Qu’impliquent les résultats
Expliquez votre réponse. des points d et e sur le plan des ressources ?
b) Dans une économie avec possibilité de f) Si (au lieu des possibilités données) la CPP
fabrication d’outils, quel est le coût était une ligne droite joignant les points
d’opportunité de la consommation courante ? a et f, qu’arriverait-il aux coûts d’opportunité
et aux ressources ?
3 Les avocats gagnent 200 $ l’heure alors que g) En utilisant la première CPP que vous
les secrétaires en gagnent 15- En utilisant avez tracée, est-il possible d’obtenir une
les concepts d avantage absolu et d avantage combinaison de 140 unités de beurre et
comparatif, expliquez pourquoi une avocate de 130 unités de canons? Cette combinaison
qui est meilleure dactylographe que sa secrétaire serait-elle efficace ? Expliquez votre réponse.
continuera de se spécialiser uniquement dans h) En utilisant cette CPP, est-il possible d’obtenir

le travail juridique et fera un échangé avec une combinaison de 70 unités de beurre et

sa secrétaire pour obtenir des services de 170 unités de canons par semaine? Est-ce
une combinaison efficace ? Expliquez.
de dactylographie.

4 Expliquez, en utilisant un exemple précis


d’échange, pourquoi un système d échangé
monétaire est plus efficace que le troc.
30 CHAPITRE 3

Si les événements suivants se produisaient 8 Supposons que le pays de Quark consacre


(chacun d’eux étant indépendant de l’autre), habituellement 10% de ses ressources à la
qu’arriverait-il à la CPP du problème 5 à production de nouveaux biens de production.
court développement ? À l’aide de diagrammes de CPP, comme ceux
a) Une nouvelle source d’énergie, facile de la figure 3.5 de la page 52 du manuel,
à exploiter, est découverte. comparez les conséquences (coûts et avantages)
b) Un grand nombre de travailleurs qualifiés dans chaque cas suivant :
immigrent dans ce pays. a) Quark continue de consacrer 10 %
c) La production de beurre augmente. de ses ressources à la production de biens
d) Une nouvelle invention fait augmenter la de production.
production par personne dans l’industrie du b) Quark commence à consacrer en permanence
beurre, mais non dans l’industrie des canons. 20 % de ses ressources à la production de biens
e) Une nouvelle loi est adoptée pour forcer de production.
les travailleurs à prendre leur retraite à 60 ans,
alors qu’auparavant ils pouvaient travailler & 9 Victoria et Robert sont les deux seuls habitants
aussi longtemps qu’ils le voulaient. survivants de la planète Melmac. Ils passent
30 heures par jour à produire des gadgets
La France et l’Allemagne produisent chacune et des bidules, les deux seuls biens nécessaires
du vin et de la bière en utilisant un seul pour assurer le bonheur des habitants de Melmac.
facteur de production homogène, le travail. Il faut 1 heure à Victoria pour produire
Leurs possibilités de production sont un gadget et 2 heures pour produire un bidule,
les suivantes : alors qu’il faut 3 heures à Robert pour produire
La France dispose de 100 unités de travail un gadget et 3 heures pour produire un bidule.
et peut produire au maximum 200 bouteilles a) Pour une journée de 30 heures, tracez la CPP
de vin ou 400 bouteilles de bière. de Victoria et celle de Robert.
L’Allemagne dispose de 50 unités de travail b) Que nous apprend la forme des CPP sur les
et peut produire au maximum 250 bouteilles de coûts d’opportunité ? sur les ressources ?
vin ou 200 bouteilles de bière. c) Supposons que Victoria et Robert soient
a) Inscrivez les données au tableau 3.3 autosufifisants. Définissez ce qu’est
l’autosuffisance. Expliquez quelles sont
TABLEAU 3.3 les possibilités de consommation individuelles
de Victoria et de Robert ?
Bouteilles d) Qui détient un avantage absolu dans
produites Coût d’opportunité la production de gadgets et de bidules ?
par unité d’une bouteille e) Qui détient un avantage comparatif dans
de travail supplémentaire la production de gadgets et de bidules ?
f) Supposons que Victoria et Robert se
Vin Bière Vin Bière
spécialisent chacun dans la production
France d’un seul bien où il ou elle détient un avantage
comparatif (l’un passe 30 heures à produire
des gadgets et l’autre 30 heures à produire
Allemagne des bidules). Quelle sera la production totale
de gadgets et de bidules ?
Utilisez les données de la partie a pour répondre g) Supposons que Victoria et Robert échangent
aux questions suivantes. 7 gadgets contre 5 bidules. Sur vos diagrammes
® b) Quel pays détient un avantage absolu dans de CPP, indiquez le nouveau point de
la production de vin ? consommation de Victoria et celui de Robert.
® c) Quel pays détient un avantage absolu dans Expliquez comment ces points illustrent
la production de bière ? les gains de l’échange.
d) Quel pays détient un avantage comparatif
dans la production de vin ? I0 Les familles Netsilik et Oonark vivent dans
e) Quel pays détient un avantage comparatif la région côtière de F Arctique, à l’ouest
dans la production de bière ? de la Baie d’Hudson. Elles vont souvent ensemble
f) Si le commerce est permis, décrivez quelle à la pêche et à la chasse au caribou. Durant
serait la spécialisation, le cas échéant. une journée de travail moyenne, la famille
LA PRODUCTION, LA CROISSANCE ET L'ÉCHANGE 31

Nestsilik peut, dans le meilleur des cas, attraper Questions à choix multiple
6 kilogrammes de poisson ou tuer 6 caribous.
1 c Pour avoir un coût d’opportunité nul,
La famille Oonark peut attraper 4 kilogrammes
les ressources doivent être inutilisées ou
de poisson ou tuer 4 caribous.
sous-utilisées.
a) En supposant que les CPP sont des lignes
2 d a serait exact si les ressources étaient inégales ;
droites, tracez la CPP de chaque famille
b et c déplacent la CPP.
sur le même diagramme. Portez le poisson
3 b Pour augmenter la quantité X et la faire passer
en abscisse et le caribou en ordonnée.
à 9, il faut ramener la quantité Y de 20 à 0.
b) Inscrivez les données au tableau 3.4
4 a Pour qu’il y ait déplacement de b à a, la
quantité X doit baisser de 8 à 6.
TABLEAU 3.4 <*> 5 c La production Y augmente le long de la CPP.
6 e Définition.
Coût d’opportunité & 7 e a et b et c déplacent la CPP.
d’une unité
8 c a et b déplacent la CPP vers l’extérieur, mais
supplémentaire non le coût d’opportunité; d déplace la CPP
de poisson vers l’extérieur.
(en kg) de caribou ® 9 c André produit 3 pains à l’heure ; Raymond
produit 1 pain à l’heure.
Famille 10 b André détient un avantage comparatif dans
Netsilik la production de pain (coût d’opportunité
inférieur), Raymond détient un avantage
Famille comparatif dans la production de beurre.
Oonark Ile c, par définition, est un avantage comparatif;
d parce que le coût d’opportunité de Y
c) Quelle famille détient un avantage comparatif est l’inverse du coût d’opportunité de X.
pour la pêche ? Pour la chasse au caribou ? 12b Le progrès technique déplace la CPP vers
d) La spécialisation et l’échange peuvent-ils faire l’extérieur au coût de la consommation
augmenter la production totale de poisson courante.
et de caribou des deux familles ? Expliquez. 13 b Définition.
Ô 14 d Coût d’opportunité de la production
de pétrole en boisseaux de pommes — Canada
2/3, Mexique 1/2. Coût d’opportunité de

E RÉPONSES la production de pommes en barils de pétrole


— Canada 3/2, Mexique 2.
15 c Coût d’opportunité de la production de vin
Vrai/Faux/Incertain en balles de laine - Portugal 1/3, Angleterre 3.
(Justifiez votre réponse.) Coût d’opportunité de la production de laine
1 F Possible, mais n’est pas un niveau de en bouteilles de vin - Portugal 3, Angleterre, 1/3.
production efficace. 16 d Rareté —> coût d’opportunité —» relation
2 V En se déplaçant de b à d, la production du négative. Pour obtenir plus de X, on doit
bien 1 diminue de 4 unités. renoncer à Y.

3 V À l’extérieur de la CPP. 17 e Coût d’opportunité constant —> CPP,

® 4 V Voir Rappel 2. pente constante.


5 F Le coût de la croissance est le renoncement 18a La discrimination entraîne la sous-utilisation
à la consommation courante. des ressources. Les femmes ne peuvent
6 F Les points correspondant aux biens de produire à plein rendement.
production produisent une croissance plus 19 e Renoncer à 6 feuilles d’érable pour obtenir
2 bâtons de hockey : 6/2 = 3 feuilles d’érable
rapide.
7 F Les droits de propriété sont une condition par bâton de hockey.
préalable à la spécialisation et à 1 échangé. <3> 20 a Coût d’opportunité constant —A ressources
8 F On peut consommer à un point qui se situe également utiles pour la production de tous
les biens.
à l’extérieur de la CPP.
9 F Un échange mutuellement avantageux dépend 21 c Seuls des changements dans les ressources

de l’avantage comparatif, non de 1 avantage ou les techniques déplacent la CPP.


22 a La production d’une plus grande quantité de
absolu.
10 F L’échange monétaire ne requiert pas la double biens de production aujourd’hui fait déplacer
la CPP vers l’extérieur à l’avenir.
coïncidence des besoins.
32 CHAPITRE 3

23 d Les points situés à l’intérieur de la CPP


TABLEAU 3.5
représentent les ressources non utilisées,
qu’il s’agisse du travail, du capital ou
Coût d’opportunité
de la terre.
de I heure
24 d La monnaie n’a aucun rapport avec l’avantage
supplémentaire
comparatif (a) et les gains de l’échange (c) ;
(en dollars)
l’échange monétaire ne va pas sans un
intermédiaire des échanges. Services juridiques Dactylographie
25 b Définition.
Avocate 200 200
Secrétaire >200 15
Problèmes à court développement
Considérons d’abord les coûts d’opportunité
de l’avocate. La meilleure possibilité perdue
1 La pente négative de la CPP reflète le coût d’offrir 1 heure de services juridiques est le 200 $
d’opportunité. Pour avoir une plus grande quelle aurait pu gagner en offrant une autre
quantité d’un bien, il faut renoncer à un heure de services juridiques. Si elle offre
autre bien. une heure de services de dactylographie, elle
Elle est arquée vers l’extérieur, car renonce aussi à 200 $ (1 h) de services juridiques.
l’existence de ressources non homogènes À quoi devra renoncer la secrétaire pour offrir
entraîne l’augmentation du coût d’opportunité
une heure de services juridiques? Elle devra
avec l’augmentation de la production de l’un
passer 3 ans à la faculté de droit, payer des frais
ou l’autre bien. de scolarité et renoncer à avoir des revenus
pendant ces 3 ans. Son coût d’opportunité est
2 a) Dans une économie qui ne peut fabriquer très élevé, s’élevant sûrement à plus de 200 $.
d’outils, un point situé à l’intérieur de la Si elle fournit une heure de services de
CPP représente les ressources non employées dactylographie, la meilleure possibilité à laquelle
ou sous-utilisées. En se déplaçant jusqu’à elle devra renoncer est la somme de 15 $ quelle
un point sur la courbe, il est possible aurait pu gagner en occupant un autre emploi
d’augmenter la production avec les mêmes de secrétaire.
ressources en les utilisant plus efficacement. Le tableau 3.5 nous montre que l’avocate
Étant donné qu’il n’est pas nécessaire de a un coût d’opportunité plus bas (avantage
retirer des ressources pour produire tout autre comparatif), en offrant des services juridiques
bien, le coût d’opportunité du déplacement et sa secrétaire, un coût d’opportunité plus bas
vers un point situé sur la courbe est nul. (avantage comparatif), en offrant des services
C’est ce qui se rapproche le plus de la de dactylographie. C’est sur la base de l’avantage
gratuité dans le domaine de la science comparatif (et non de l’avantage absolu) qu’il
économique. y aurait un échange dans lequel les deux parties
b) Dans une économie de fabrication d’outils, pourraient gagner.
nous pouvons renoncer à la consommation
aujourd’hui, pour produire des biens de L’efficacité d’un système d’échange monétaire
production, qui, par la suite, font augmenter par rapport au troc s’explique surtout par le fait
la production et la consommation. En qu’il ne nécessite pas de double coïncidence
consommant tout ce qui est produit des besoins. Supposons, par exemple, que vous
actuellement, nous renonçons à la fabrication vous spécialisiez dans la production de pommes,
d’outils et, en définitive, à une consommation mais que vous aimiez manger des bananes.
future accrue. Dans une économie de troc, vous ne pourrez
probablement pas faire d’échange avec le premier
3 L’avocate détient un avantage absolu dans venu qui a des bananes à échanger. Il faut aussi
la prestation de services juridiques et de services que cette personne veuille les échanger contre
de dactylographie par rapport à la secrétaire. des pommes et non contre des carottes ou
Néanmoins, elle détient un avantage comparatif contre une autre marchandise. Avec un système
en services juridiques, et la secrétaire détient monétaire, il vous sera toujours possible de faire
un avantage comparatif en services de un échange avec le premier venu qui possède
dactylographie. Pour démontrer ces avantages des bananes puisque cette personne acceptera
comparatifs, nous pouvons bâtir le tableau 3.5 de la monnaie en échange. Dans le même ordre
des coûts d’opportunité. d’idées, dans un système d’échange monétaire,
LA PRODUCTION, LA CROISSANCE ET L'ÉCHANGE 33

il vous sera possible de vendre vos pommes au Le coût d’opportunité par unité de beurre
premier venu qui veut des pommes (même si serait :
cette personne n’a pas de bananes à vendre).
220/200 = 1,1 unité de canons
= 11/10 unités de canons
5 a) Le graphique de la CPP est présenté
à la figure 3.6 Le fait que les coûts d’opportunité sont
constants implique que les ressources sont
FIGURE 3.6 homogènes ; autrement dit, elles sont
également utiles pour la production de
beurre et de canons.
g) Cette combinaison se situe à l’extérieur de la
CPP et n’est donc pas réalisable. Etant donné
que, dans cette économie, cette combinaison
n’est pas réalisable, la question de l’efficacité
ne se pose pas.
h) Cette combinaison se situe à l’intérieur de
la CPP et elle est réalisable. Elle est inefficace
parce que, dans cette économie, on pourrait
produire une plus grande quantité de l’un
ou de l’autre bien ou des deux à la fois.
Par conséquent, certaines ressources ne sont
pas pleinement utilisées, ou le sont de
manière moins efficace.

a) Si la fabrication des deux biens nécessite


de l’énergie, la CPP toute entière se déplace
vers l’extérieur, en direction nord-est, comme
b) En nous déplaçant de c à d, pour gagner dans la figure 3.7(a).
60 unités de canons, nous devons renoncer b) Si des travailleurs qualifiés sont employés
à 160 - 100 = 60 unités de beurre. Le coût dans la production des deux biens, la CPP
d’opportunité par unité de canons est toute entière se déplace vers le nord-est.

-60 unités de beurre _ 1 unité de beurre c) La CPP ne se déplace pas. Une augmentation
de la production de beurre implique un
+60 unités de canons Par un*té de canons
mouvement le long de la CPP vers la gauche,
c) En nous déplaçant de d à e, pour gagner non un déplacement de la CPP même.
40 unités de canons, nous devons renoncer d) Cette nouvelle invention implique que pour
à 100 - 40 = 60 unités de beurre. Le coût chaque niveau de production de canons,
d’opportunité par unité de canons est : l’entreprise peut maintenant produire plus
de beurre. La CPP se déplace vers la droite,
60 unités de beurre _ 1,5 unité de beurre
mais reste ancrée au point f comme dans
40 unités de canons Par un‘té de canons
la figure 3.7(b).
d) Le coût d’opportunité de la production e) La CPP toute entière se déplace vers l’origine.
d’un plus grand nombre de canons augmente
avec la production de canons. FIGURE 3.7
e) Dans la même veine, le coût d’opportunité
de la production d’une plus grande quantité
de beurre augmente avec la production de
beurre. Le fait que les coûts d opportunité
augmentent implique que les ressources ne
sont pas homogènes ; autrement dit, elles ne
sont pas également utiles pour la production
de canons et de beurre.
f) Les coûts d’opportunité seraient toujours
constants, quelle que soit la production
de canons ou de beurre. Le coût d’opportunité
par unité de canons serait:

200/220 = 10/11 unités de beurre


34 CHAPITRE 3

que a. Cela déplace la CPP vers l’extérieur,


à la période suivante, mais seulement jusqu’à
la courbe 2 (où, on le suppose, Quark choisira
de produire au point b).

FIGURE 3.8

7 a) Voici le tableau 3.3 au complet.

TABLEAU 3.3 SOLUTION

Bouteilles
produites Coût d’opportunité b) En partant de la même CPP, si Quark décide
par une unité d’une bouteille maintenant d’augmenter de 20 % les ressources
de travail supplémentaire consacrées à la production de nouveaux biens
Vin Bière Vin Bière de production, elle choisira de produire à
un point tel que c. Dans ce cas, à la période
France 2 4 2,0 bières 0,50 vin
suivante, la CPP se déplacera davantage -
Allemagne 5 4 0,8 bière 1,25 vin
jusqu’à la courbe 3, et vers un point tel que d,
par exemple.
<§> b) L’Allemagne, qui peut produire plus de Par conséquent, en comparant les points
vin (5 bouteilles) par unité de facteur a et c, nous trouvons les coûts et avantages
de production, détient un avantage absolu suivants : le point a présente l’avantage
dans la production de vin. d’une plus grande consommation aujourd’hui,
® c) Aucun des deux pays ne détient d’avantage mais aux dépens d’une consommation future
absolu dans la production de bière, puisque réduite ; le point c présente l’avantage
la production de bière (4 bouteilles) par unité d’une consommation future plus grande,
de facteur de production est la même dans mais aux dépens d’une consommation réduite
les deux pays. aujourd’hui.
d) L’Allemagne, qui a le coût d’opportunité
le plus bas (0,8 bouteille de bière), détient & 9 a) Les figures 3.9 (a) et (b), respectivement,
un avantage comparatif dans la production présentent la CPP de Victoria et celle
de vin. de Robert.
e) La France, qui a le coût d’opportunité le plus
bas (0,5 bouteille de vin), détient un avantage
FIGURE 3.9
comparatif dans la production de bière.
f) L’incitation à l’échange ne dépend que de (a) Victoria
la différence entre les avantages comparatifs.
L’Allemagne se spécialisera dans la production
de vin et la France, dans la production
de bière.

8 a) La situation de Quark est présentée à la


figure 3.8. Supposons que Quark débute
sur la CPP 1. Si elle continue à consacrer
- seulement 10 % de ses ressources à la
production de nouveaux biens de production,
elle choisit alors de produire à un point tel
LA PRODUCTION, LA CROISSANCE ET L'ÉCHANGE 35

(b) Robert Victoria utilise moins de facteurs de


production que Robert dans le cas des gadgets
(1 heure par rapport à 3) et des bidules
(2 heures par rapport à 3). Etant donné que
la productivité de Victoria est supérieure à celle
de Robert dans le cas des deux biens (gadgets
et bidules), nous pouvons dire que, dans
l’ensemble, elle détient un avantage absolu.
e) Victoria détient un avantage comparatif dans
la production de gadgets, étant donné quelle
peut les produire à un coût d’opportunité
inférieur à celui de Robert (1/2 bidule par
rapport à 1 bidule). Par ailleurs, Robert
b) La forme linéaire de la CPP nous indique que détient un avantage comparatif dans la
les coûts d’opportunité sont constants le long production de bidules étant donné qu’il peut
de chaque courbe et que les ressources sont les produire à un coût d’opportunité inférieur
homogènes. à celui de Victoria (1 gadget par rapport à
Ces CPP linéaires, qui supposent des coûts 2 gadgets).
d’opportunité constants, font abstraction f) Victoria produira des gadgets et Robert des
des complexités qui caractérisent le monde bidules, ce qui donne au total, à eux deux,
réel. En effet, dans le monde réel, les coûts 30 gadgets et 10 bidules.
d’opportunité sont généralement croissants, g) Après l’échange, Victoria aura 23 gadgets et
mais il n’est pas essentiel d’en tenir compte 5 bidules (point T). Robert aura 7 gadgets
pour comprendre les gains de l’échange, et 5 bidules (point R). Ces nouveaux points
ce qui est l’objectif de ce problème. Si des possibilités respectives de consommation
on compliquait le problème, en ajoutant après l’échange se situent à l’extérieur des
des coûts d’opportunité croissants, on ne possibilités de consommation (et de
changerait pas les résultats, mais on aurait production) d’avant l’échange de Victoria et
plus de mal à les voir. de Robert. De ce fait, l’échange permet de
c) Les individus sont autosuffisants s’ils ne faire des gains, dans la mesure où les deux
consomment que ce qu’ils produisent. parties peuvent améliorer leurs possibilités de
Cela signifie qu’il n’y a pas d’échanges. Sans consommation au-delà de celles qui seraient
échanges, les possibilités de consommation possibles dans une situation d’autosuffisance.
de Victoria (maximum) sont exactement
les mêmes que ses possibilités de production :
10 a) Les CPP des familles Netsilik et Oonark sont
les points le long de sa CPP. Les possibilités
indiquées à la figure 3.10.
de consommation (maximum) de Robert
sont, dans le même ordre d’idées, les points
FIGURE 3.10
situés le long de sa CPP.
d) Victoria détient un avantage absolu dans la
production de gadgets et de bidules. On peut
définir son avantage absolu en termes d une
plus grande production par unité de facteurs
de production ou d’une moins grande quantité
de facteurs de production par unité de
production. La comparaison des CPP de
la figure 3.9 indique que, pour des facteurs
de production de 30 heures, la production
de gadgets de Victoria est supérieure a celle
de Robert (30 par rapport à 10) tout comme
sa production de bidules (15 par rapport à 10).
Ce problème nous apprend que, dans le
même ordre d’idées, par unité de production,
36 CHAPITRE 3

b) Voici le tableau 3.4 au complet. Pour illustrer l’absence de gains, supposons


que, au début, chaque famille consacre

3.4 SOLUTION
la moitié de son temps à chaque activité.
TABLEAU
Supposons ensuite que la famille Netsilik

Coût d’opportunité se spécialise entièrement dans la pêche et

d’une unité la famille Oonark, dans la chasse au caribou.

supplémentaire Le tableau 3.6 montre la production avant


et après la spécialisation.
de poisson
(en kg) de caribou TABLEAU 3.6

Famille
Avant la Après la
Netsilik 1 caribou 1 kg de poisson
spécialisation spécialisation

Famille Poisson Poisson


Oonark 1 caribou 1 kg de poisson (en kg) Caribou (en kg) Caribou

Famille
c) Ni l’une ni l’autre famille ne détient d’avantage
Netsilik 3 3 6 0
comparatif dans la pêche étant donné que le
coût d’opportunité du poisson est le même
Famille
(1 caribou) pour chaque famille. Dans le
Oonark 2 2 0 4
même ordre d’idées, aucune des familles ne
détient d’avantage comparatif dans la chasse Production
au caribou étant donné que le coût totale 5 5 6 4
d’opportunité d’un caribou est le même
(1 kg de poisson) pour chaque famille. Comparez la production totale des deux biens
d) Les gains provenant de la spécialisation et de avant et après la spécialisation. La spécialisation
l’échange sont dus à l’existence de l’avantage a permis d’augmenter la production totale de
comparatif. Dans ce cas, aucune des familles poisson de un kilogramme, mais elle a également
ne détient d’avantage comparatif dans entraîné une réduction de la production totale
la pêche ou la chasse ; par conséquent, de caribou de une unité. La spécialisation
la spécialisation et l’échange n’entraînent et l’échange n’entraînent pas d’augmentation
aucun gain. visible de la consommation.
C h a

4
L’offre et la demande

♦ Un déplacement de la courbe de demande


CONCEPTS CLÉS correspond à une modification de la demande.
La courbe de demande se déplace en raison
de modifications
Le coût d’opportunité et le prix
• du prix des autres biens.
Le prix relatif d’un bien est le rapport entre son prix • du revenu.
nominal (monétaire) et celui d’un autre bien. Le prix • des prix futurs anticipés.
relatif mesure le coût d’opportunité de l’achat d’un • de la population.
bien - les autres biens auxquels on doit renoncer. • des préférences.
La loi de l’offre et de la demande explique la notion
♦ Augmentation de la demande — déplacement de
de prix relatif et permet de faire des prédictions sur
la courbe de demande vers la droite. Diminution
la hausse ou la baisse du prix d’un bien ou d’un service
de la demande - déplacement de la courbe
par rapport au prix moyen d’autres biens et services.
de demande vers la gauche.

♦ Pour une augmentation


La demande
• du prix d’un substitut — la courbe de demande
La quantité demandée d’un bien est la quantité se déplace vers la droite.
de ce bien que les consommateurs prévoient acheter • du prix d’un bien complémentaire - la courbe
à un prix particulier durant un certain laps de temps. de demande se déplace vers la gauche.
Selon la loi de la demande, « toutes autres choses • du revenu (bien normal) - la courbe de
étant égales, plus le prix d’un bien est élevé, demande se déplace vers la droite.
moins la quantité demandée est importante». • du revenu (bien inférieur) — la courbe de
T prix —> si quantité demandée pour deux raisons : demande se déplace vers la gauche.
• des prix futurs anticipés - la courbe de
♦ effet de substitution - lorsque le prix relatif d un
demande se déplace vers la droite.
bien T, les consommateurs achètent une moins
• de la population - la courbe de demande se
grande quantité de ce bien et une plus grande
déplace vers la droite.
quantité de substituts.
• des préférences — la courbe de demande se
♦ effet de revenu — lorsque le prix d un bien T et que déplace vers la droite.
les revenus restent constants, les consommateurs
ont moins d’argent à dépenser pour tous les biens, L’offre
y compris celui dont le prix T.
La quantité offerte d’un bien est la quantité que les
La courbe de demande représente la relation inverse
producteurs prévoient vendre à un prix déterminé au
entre la quantité demandée et le prix, ceteris paribus cours d’une période donnée. Selon la loi de F offre,
(toutes autres choses étant égales). «toutes autres choses étant égales, plus le prix d’un
♦ Une variation de prix entraîne un déplacement, bien est élevé, plus la quantité offerte de ce bien est
le long de la courbe de demande. C est ce que 1 on élevée». Du fait que le coût d’opportunité d’un bien
appelle une variation de la quantité demandée. s’accroît à mesure que la quantité produite augmente,
Plus le prix d’un bien est élevé, moins la quantité les producteurs demandent un prix plus élevé pour
accroître la quantité offerte.
demandée est importante.
38 CHAPITRE 4

La courbe d’offre représente la relation positive entre ♦ augmentation de la demande -4 T P et T Q.


la quantité offerte et le prix, ceteris paribus.
♦ diminution de la demande —> -1 P et -l Q.
♦ Une variation de prix entraîne un mouvement
♦ augmentation de l’offre —> 1 P et T Q.
le long de la courbe d’offre. C’est ce que l’on
appelle une variation de la quantité offerte. ♦ diminution de l’offre —> T P et I Q.
Plus le prix d’un bien est élevé, plus la quantité
Lorsqu’il y a un changement simultané dans l’offre
offerte sera élevée.
et dans la demande, nous pouvons en déterminer les
♦ On appelle modification de l’offre, tout effets sur le prix ou sur la quantité. Toutefois, sans
déplacement de la courbe d’offre. La courbe information sur la taille relative des déplacements des
d’offre se déplace en raison de modifications courbes de demande et d’offre, l’effet sur l’une ou l’autre
des deux variables est ambigu. Ceteris paribus, une
• du prix des facteurs de production.
• du prix des autres biens produits. ♦ augmentation de la demande et de l’offre -4 T, 4-
• des prix futurs anticipés. ou aucun changement de T et Q T.
• du nombre de producteurs.
♦ diminution de la demande et de l’offre —» T, >1
• de la technologie.
ou aucun changement de T et Q •!.
♦ Augmentation de l’offre - la courbe d’offre
♦ augmentation de la demande et diminution de
se déplace vers la droite. Diminution de l’offre -
l’offre —4 P T et T, ou aucun changement de Q.
la courbe d’offre se déplace vers la gauche.
♦ diminution de la demande et augmentation de
♦ Pour une augmentation
l’offre —> P -l et T, 4- ou aucun changement de Q.
• du prix des facteurs de production - la courbe
d’offre se déplace vers la gauche.
• du prix d’un substitut de production - la courbe
d’offre se déplace vers la gauche. RAPPELS
• du prix d’un produit complémentaire
1 Lorsque vous commencez à vous familiariser
de production - la courbe d’offre se déplace
avec la notion d’offre et de demande, il est
vers la droite.
avantageux d’utiliser des exemples concrets pour
• des prix futurs anticipés - la courbe d’offre
arriver à la comprendre intuitivement. Gardez à
se déplace vers la gauche.
l’esprit des exemples qui vous sont familiers. Par
• du nombre de producteurs - la courbe d’offre
exemple, en analysant les biens complémentaires,
se déplace vers la droite.
pensez aux hamburgers et aux frites ; en analysant
• de la technologie - la courbe d’offre se déplace
les substituts, pensez aux hamburgers et aux
vers la droite.
hot dogs. Cela rendra la théorie économique
La détermination des prix moins abstraite.

Le prix d’équilibre se situe au point d’intersection 2 L’énoncé «le prix est déterminé par la demande
de la courbe de demande et de la courbe d’offre, lorsque et par l’offre» sous-entend que le prix est
la quantité demandée est égale à la quantité offerte. déterminé par tous les facteurs qui ont un effet
sur la demande (prix d’autres biens, revenu,
♦ Au-dessus du prix d’équilibre, il y a un surplus prix futurs anticipés, population, préférences)
(quantité offerte > quantité demandée), et le prix et par tous les facteurs ayant un effet sur l’offre
baisse. (prix d’autres biens produits, prix des facteurs
♦ Au-dessous du prix d’équilibre, il y a une pénurie de production, prix futurs anticipés, nombre
(quantité demandée > quantité offerte), et le prix de producteurs, technologie). L’utilisation
monte. des courbes de demande et d’offre a l’avantage
de nous permettre de séparer systématiquement
♦ C’est seulement en situation d’équilibre qu’on
les effets de chacun de ces facteurs sur les prix.
n’exerce aucune pression pour faire changer le prix.
Les modifications des facteurs qui ont un
La quantité d’équilibre est la quantité achetée et
effet sur la demande, hormis le prix du bien
vendue au prix d’équilibre.
considéré, font déplacer la courbe de demande
et nous permettent de monter ou de descendre
Les prédictions des variations
le long d’une courbe d’offre donnée. Les
dans les prix et les quantités échangées
changements dans les facteurs ayant un effet
Lorsqu’il se produit un changement unique soit dans la sur l’offre font déplacer la courbe d’offre, hormis
demande, soit dans l’offre, ceteris paribus, une le prix du bien considéré, et nous permettent de
L’OFFRE ET LA DEMANDE 39

monter ou de descendre le long d’une courbe de En revanche, si l’un des facteurs affectant
demande donnée. la quantité demandée change, la courbe de
Tout problème d’offre et de demande nous demande se déplacera et la quantité demandée
force à déterminer attentivement ces influences. pour chaque prix changera. Ce déplacement de
Ce faisant, il faut toujours tracer un graphique la courbe de demande porte le nom de variation
même si c’est seulement un petit graphique de la demande. Il est essentiel de se souvenir
dans la marge d’une question à choix multiple qu’une variation du prix d’un bien ne déplacera
ou d’un problème du type « Vrai/Faux/Incertain». pas la courbe de demande, elle entraînera
La représentation graphique constitue un moyen seulement un mouvement le long de cette courbe.
très efficace de « voir » ce qui se passe. Plus vous Dans le même ordre d’idées, il est tout
vous familiariserez avec les graphiques, plus vous aussi important de faire la différence entre
verrez à quel point ces outils peuvent vous aider déplacements de la courbe d’offre et mouvements
à organiser vos pensées. le long de la courbe d’offre.
Ne faites pas l’erreur courante de croire Pour être sûr que vous avez bien compris,
qu’un problème est tellement simple que vous considérez l’effet (tracez un graphique !)
pouvez le résoudre dans votre tête, sans tracer de l’augmentation du revenu d’un ménage
de graphique. Cette erreur risque de vous coûter sur le marché des disques compacts (DC).
très cher le jour de l’examen. De plus, lorsque Remarquez tout d’abord qu’une augmentation
vous tracez un graphique, veillez à bien nommer du revenu a une incidence sur la demande
les axes. Plus le cours avancera, plus vous verrez et non sur l’offre de DC. Ensuite, il faut
de graphiques ayant différentes variables sur déterminer si l’augmentation du revenu entraîne
les axes. Il est très facile de se tromper si l’on un déplacement de la courbe de demande ou
ne prend pas l’habitude de nommer les axes. un mouvement le long de cette courbe. Cette
augmentation du revenu fera-t-elle augmenter la
Une autre erreur très courante chez les étudiants quantité de DC demandée même si le prix des
consiste à ne pas faire correctement la distinction DC ne varie pas ? Etant donné que la réponse à
entre le déplacement d’une courbe et cette question est « oui », vous savez que la
le mouvement le long d’une courbe. Cette courbe de demande va se déplacer vers la droite.
distinction s’applique autant à la courbe d’offre Remarquez également que l’augmentation de la
qu’à la courbe de demande. De nombreuses demande de DC fera augmenter le prix
questions d’autoévaluation visent à vérifier d’équilibre. Cette hausse du prix sera indiquée
si vous avez bien compris cette distinction, par un mouvement le long de la courbe d’offre
et vous pouvez être sûr que votre professeur (augmentation de la quantité offerte) et ne
vous interrogera souvent à ce sujet. Il est essentiel déplacera pas la courbe d’offre.
de faire la distinction entre «déplacement» d’une Souvenez-vous : Ce sont les déplacements
courbe et «mouvement le long» d’une courbe des courbes d’offre et de demande qui font varier
pour comprendre systématiquement comment le prix du marché, et non les variations du prix
les facteurs influent sur l’offre et la demande qui entraînent le déplacement des courbes
et, également, comment on détermine le prix d’offre et de demande.
et la quantité d’équilibre.
Prenez l’exemple de la courbe de demande. 4 Lorsqu’on analyse les déplacements des courbes
La quantité demandée d’un bien dépend de d’offre et de demande dans des marchés
son prix, du prix d’autres biens, des revenus, apparentés (substituts comme la bière et le vin),
des prix futurs anticipés, de la population et on a souvent l’impression que les effets
des préférences. Le terme « demande » s applique de rétroaction d’un marché à l’autre peuvent
à la relation entre le prix d un bien et la quantité se poursuivre indéfiniment. Pour éviter
demandée, en maintenant constants tous la confusion, tenez-vous-en à la règle qui dit
les autres facteurs dont dépend la quantité que chaque courbe (de demande et d’offre)
demandée. Cette relation est représentée pour un marché donné ne peut se déplacer
graphiquement par la courbe de demande. De ce opxunefois au maximum. (Voir les problèmes
fait, l’effet d’une variation de prix sur la quantité à court développement 4 et 6 de la page 45
demandée s’exerce déjà sur la pente de la courbe pour obtenir plus de précisions et des exemples).

de demande; l’effet d’une variation de prix


du bien s’exerce par un mouvement le long de ® 5 Les relations entre le prix et la quantité demandée

la courbe de demande. C’est ce que 1 on appelle et offerte peuvent être représentées sous trois
formes équivalentes : barèmes, courbes et
variation de la quantité demandée.
40 CHAPITRE 4

équations d’offre et de demande. Au chapitre 4


FIGURE 4.1
du manuel, on illustre les barèmes et les courbes,
mais les équations de la demande et de l’offre Barèmes d’offre et de demande

constituent également de puissants outils Pénurie (-)/


d’analyse économique. La note mathématique Prix Surplus (+)
du chapitre 4 présente la forme générale de (en dollars) Qd Qo (Qo - Qd)
ces équations. Ce rappel et le suivant cherchent
0 5 -1 -6
à mieux expliquer les équations et la manière
1 4 0 -4
de les utiliser pour déterminer les valeurs
2 3 1 -2
d’équilibre du prix et de la quantité.
La figure 4.1 présente un exemple simple 3 2 2 0
4 I 3 +2
d’offre et de demande sous trois formes
5 0 4 +4
équivalentes : (a) barèmes, (b) courbes et
(c) équations. Les barèmes d’offre et de demande
(b) Courbes d’offre et de demande
de a) sont présentés sous la même forme qu’à
la figure 4.8 du manuel. Les combinaisons prix-
quantité provenant des barèmes sont indiquées
sur le graphique (b), produisant des courbes
d’offre et de demande linéaires. Cet exemple
a ceci de nouveau que les courbes sont
des représentations des équations de (c).
En vous souvenant de la formule (chapitre 2)
de l’équation d’une ligne droite (y = a + bx),
vous verrez que l’équation de la demande est
l’équation d’une ligne droite. Ce n’est plusjy,
mais P qui est la variable dépendante sur l’axe
vertical, et ce n’est plus x, mais Qd qui est
la variable indépendante sur l’axe horizontal.
L’ordonnée à l’origine sur l’axe vertical a est +5,
et la pente b est -1. L’équation de l’offre est (c) Équations de l’offre et de la demande
également linéaire et représentée graphiquement Demande: P=5~\Qp
de la même manière, mais avec Qa comme Offre: P= 1+lQo
variable indépendante. L’ordonnée à l’origine
de la courbe d’offre sur l’axe vertical est +1 et
Les équations de l’offre et de la demande
la pente est +1. La pente négative de la courbe
sont très utiles pour calculer les valeurs d’équilibre
de demande reflète la loi de la demande, et la
du prix et de la quantité. Comme le montrent
pente positive de la courbe d’offre reflète la loi
les barèmes et les courbes, deux choses restent
de l’offre.
vraies en situation d’équilibre: 1) le prix est
Vous pouvez démontrer l’équivalence
le même pour les consommateurs (le prix le plus
du barème, de la courbe et de l’équation de
élevé qu’ils sont disposés à payer pour obtenir
la demande en utilisant diverses valeurs de Qp
la dernière unité du bien ou service) et pour les
indiquées dans le barème dans l’équation de
producteurs (le prix le plus bas auquel ils sont
la demande et en calculant les prix associés. Ces
prêts à consentir pour vendre la dernière unité
combinaisons de quantité demandée et de prix
du bien ou service) et 2) la quantité demandée
correspondants ont les coordonnées (Qp, P) des
est égale à la quantité offerte, de sorte qu’il n’y a
points sur la courbe de demande. Vous pouvez
ni pénurie ni surplus. En termes d’équations de
démontrer de la même façon l’équivalence du
la demande et de l’offre, cela signifie qu’en
barème, de la courbe et de l’équation de l’offre.
situation A'équilibre : 1) le prix dans les deux
équations est le même. Nous appellerons le prix
d’équilibre P*; 2) Qp = Q0 = la quantité
d’équilibre achetée et vendue = la quantité
échangée. Nous appellerons la quantité
d’équilibre Q*. Cela signifie que, en situation
d’équilibre, les équations deviendront :

Demande: P* = 5 - 1Q*
Offre: P* = \ + IQ*
L'OFFRE ET LA DEMANDE 41

Ces équations d’équilibre constituent un Vous pouvez résoudre ces deux équations
simple ensemble d’équations simultanées. Étant vous-même pour voir qu elles donnent
donné qu’il y a deux équations (demande exactement les mêmes valeurs pour P* et Q*.
et offre) et deux inconnues (P* et Q*), nous {Un conseil: calculez d’abord P*, ensuite Q*).
pouvons trouver les inconnues. Quelle que soit la forme d’équation utilisée
Pour trouver la solution, commençons par par votre instructeur, la technique de résolution
établir l’égalité de la demande et de l’offre: des équations sera semblable et les résultats,
identiques.
5- 1Q*=1 + 1Q*

En rassemblant les termes semblables, nous


trouvons

4 = 2Q* AUTOÉVALUATION
2=Q*

Lorsque nous avons obtenu Q* (quantité Vrai/Faux/Incertain


d’équilibre), nous pouvons obtenir le prix (Justifiez votre réponse.)
d’équilibre en utilisant soit l’équation de la
demande, soit l’équation de l’offre. Considérons I La loi de la demande nous dit que, à mesure
d’abord la demande : que le prix augmente, la demande diminue.

P* = 5- 1Q*
P* = 5 - 1(2)
P* = 5-2
2 Une baisse du revenu fera déplacer la courbe
P* = 3 de demande vers la gauche.
Dans le même ordre d’idées, en utilisant
Q* dans l’équation d’offre, nous obtenons
les mêmes résultats :
3 Une courbe d’offre indique le prix maximum
P*= 1+ 1 Q*
auquel la dernière unité produite sera offerte.
P*= 1 + 1(2)
P* = 1+2
P* = 3

Une fois que vous avez résolu l’équation 4 Si A et B sont des substituts, une hausse du prix
pour Q*, le fait d’obtenir le bon P* en utilisant de A fera déplacer la courbe d’offre de B vers
Q* dans l’équation de la demande comme la gauche.
dans l’équation de l’offre constitue un moyen
de vérification fort utile. Si vous vous trompez
dans vos calculs, lorsque vous utilisez Q*
dans les équations de l’offre et de la demande, 5 Lorsqu’on abat un bœuf pour sa viande, on peut
vous obtiendrez deux prix différents. Si cela se utiliser sa peau pour faire du cuir. De ce fait,
produit, vous devrez vérifier vos calculs. Si vous la viande et le cuir sont des substituts de
obtenez le même prix lorsque vous utilisez Q* production.
dans les équations de l’offre et de la demande,
vous saurez alors que vos calculs sont justes.

® 6 Les économistes suivent la convention voulant 6 Si le prix de la viande augmente, il y aura à


que l’on porte sur un graphique la quantité en la fois une augmentation de l’offre de cuir et de
tant que variable indépendante et le prix en tant la quantité de viande offerte.
que variable dépendante, ce que reflètent les
équations ci-dessus. Malgré cette convention,
les économistes considèrent, en fait, que les prix
du monde réel doivent être des variables 7 Si le prix futur anticipé d’un bien augmente,
indépendantes et les quantités, des variables il y aura une hausse du prix d’équilibre et
dépendantes. Dans ce cas, les équations une diminution de la quantité d’équilibre.
prendraient la forme suivante :

Demande : QD —5-1P
Offre: Qo = —^ + ^
42 CHAPITRE 4

8 Supposons que de nouvelles entreprises entrent 4 Un déplacement de la courbe d’offre


sur le marché de l’acier. Le prix d’équilibre des rutabagas sera provoqué par
de l’acier va baisser et la quantité, augmenter. a) un changement dans la préférence
des consommateurs.
b) une variation du prix d’un substitut
de consommation des rutabagas.
9 Supposons que la demande d’ordinateurs c) une modification des revenus.
personnels augmente alors que leur coût d) une variation du prix des rutabagas.
de production diminue. La quantité d’équilibre e) Aucune de ces réponses.
des ordinateurs personnels s’élèvera et le prix
baissera. 5 Lorsqu’il est possible d’utiliser une ressource
pour produire soit le bien A, soit le bien B,
A et B sont alors
a) des substituts de production.
I0 Lorsque le prix courant est supérieur au prix b) des compléments de production.
d’équilibre, il se produit une pénurie. c) des substituts de consommation.
d) des compléments de consommation.
e) des biens normaux.

6 Si le marché des biscuits est en équilibre, alors


Questions à choix multiple
a) les biscuits doivent être des biens normaux.
b) les producteurs aimeraient en vendre
1 Si une hausse du prix du bien A fait déplacer
davantage au prix courant.
la courbe de demande du bien B vers la gauche,
c) les consommateurs aimeraient en acheter
alors
davantage au prix courant.
a) A et B sont des substituts de consommation.
d) il y aura un surplus.
b) A et B sont des compléments de
e) la quantité d’équilibre est égale à la quantité
consommation.
demandée.
c) A et B sont des compléments de production.
d) B est un bien inférieur.
7 Un groupe de directeurs des ventes parlent
e) B est un bien normal.
de leur métier. Quelles observations, parmi
les suivantes, font allusion à un mouvement
2 Quel facteur, parmi les suivants, ne peut
le long de la courbe de demande ?
entraîner l’augmentation de la demande
a) « Depuis que nos concurrents ont augmenté
d’un bien?
leurs prix, nos ventes ont doublé. »
a) Une augmentation du revenu.
b) «L’hiver a été exceptionnellement doux;
b) Une diminution du revenu.
nos ventes d’écharpes en laine ont diminué
c) Une diminution du prix d’un substitut.
par rapport à l’an dernier. »
d) Une diminution du prix d’un complément.
c) «Nous avons décidé de baisser nos prix,
e) Une augmentation des préférences pour
et nos ventes ont augmenté de façon
ce bien.
considérable. »
d) « Grâce au mouvement écologiste, nos ventes
3 Le fait que la baisse du prix d’un bien oblige
de produits biodégradables ont augmenté. »
les producteurs à réduire la quantité offerte
e) Aucune de ces réponses.
de ce bien illustre
a) la loi de l’offre.
8 Quelle situation, parmi les suivantes, entraînera
b) la loi de la demande.
sans équivoque une hausse du prix d’équilibre ?
c) une modification de l’offre.
a) Une augmentation à la fois de l’offre et
d) la nature d’un bien inférieur.
de la demande.
e) un progrès technique.
b) Une diminution à la fois de l’offre et
de la demande.
c) Une augmentation de la demande combinée
à une diminution de l’offre.
d) Une augmentation de la demande combinée
à une augmentation de l’offre.
e) Aucune de ces réponses.
L'OFFRE ET LA DEMANDE 43

9 Le prix d’un bien aura tendance à diminuer I3 Depuis 1960, le nombre de mères au travail
a) s’il existe un surplus au prix courant. a augmenté de manière spectaculaire. En nous
b) si le prix courant est supérieur au prix fondant sur cette seule information, nous
d’équilibre. pouvons prédire que le marché des services
c) si la quantité offerte est supérieure à de garderie a connu
la quantité demandée au prix courant. a) une augmentation de la demande.
d) Toutes ces réponses. b) une diminution de la demande.
e) Aucune de ces réponses. c) une augmentation de la quantité demandée.
d) une diminution de la quantité offerte.
Le marché du café est initialement en équilibre avec
e) une augmentation de l’offre.
des courbes d’offre et demande aux formes habituelles.
Le Pepsi est un substitut du café ; la crème est
14 Si le Hamburger Helper est un bien inférieur,
un complément du café. Les questions 10 à 12
alors, ceteris paribus, une diminution du revenu
se rapportent au marché du café.
a) fera déplacer la courbe de demande
Supposez que toutes les hypothèses ceteris paribus
de Hamburger Helper vers la gauche.
s’appliquent sauf dans le(s) cas suivants. Répondez à
b) fera déplacer la courbe de demande
chaque question sans vous occuper des autres.
de Hamburger Helper vers la droite.
c) entraînera un mouvement ascendant le long
10 Le café est un bien normal. Une diminution
de la courbe de demande de Hamburger
des revenus
Helper.
a) fera augmenter le prix du café et augmenter
d) entraînera un mouvement descendant le long
la quantité de café demandée.
de la courbe de demande de Hamburger
b) fera augmenter le prix du café et augmenter
Helper.
la quantité de café offerte.
e) Aucune de ces réponses.
c) fera diminuer le prix du café et diminuer
la quantité de café demandée.
I5 On peut éliminer un surplus
d) fera diminuer le prix du café et diminuer
a) en augmentant l’offre.
la quantité de café offerte.
b) lorsque le gouvernement augmente le prix.
e) c et d.
c) en diminuant la quantité demandée.
d) en laissant chuter le prix.
I I Une hausse du prix du Pepsi
e) en diminuant la quantité achetée et vendue.
a) fera augmenter le prix du café et la quantité
de café demandée.
16 Une diminution de la quantité demandée est
b) fera augmenter le prix du café et la quantité
représentée par
de café offerte. a) un déplacement vers la droite de la courbe
c) fera diminuer le prix du café et la quantité
d’offre.
de café demandée. b) un déplacement vers la droite de la courbe
d) fera diminuer le prix du café et la quantité
de demande.
de café offerte. c) un déplacement vers la gauche de la courbe
e) a et b. de demande.
d) un mouvement ascendant et vers la gauche
I2 Un progrès technique permet de réduire le coût
le long de la courbe de demande.
de production du café. En même temps, les e) un mouvement descendant et vers la droite
préférences pour le café diminuent. La quantité le long de la courbe de demande.
d’équilibre du café
a) augmentera. 17 Si A et B sont des biens (de consommation)
b) diminuera. complémentaires et que le coût d’une ressource
c) restera la même. utilisée dans la production de A diminue, alors
d) augmentera ou diminuera en fonction de le prix
l’augmentation ou de la diminution du prix a) de A et de B va augmenter.
du café. b) de A et de B va diminuer.
e) augmentera ou diminuera en fonction des c) de A va diminuer et le prix de B
déplacements relatifs des courbes d offre va augmenter.
et de demande. d) de A va augmenter et le prix de B
va diminuer.
e) de A va diminuer et le prix de B restera
le même.
44 CHAPITRE 4

I8 Le long d’une courbe de demande, tous les 23 « Le prix des voitures canadiennes ayant
éléments suivants restent les mêmes, sauf augmenté, les consommateurs trouvent
a) les revenus. les voitures étrangères plus abordables.
b) le prix des biens apparentés. Par conséquent, la vente des voitures canadiennes
c) le prix du bien lui-même. a baissé et celle des voitures étrangères a
d) les préférences. augmenté. » En vous fondant sur cette seule
e) Toutes ces réponses. information, vous direz qu’il y a eu
a) un déplacement des courbes de demande
I9 Quel facteur, parmi les suivants, fera déplacer des voitures canadiennes et des voitures
la courbe d’offre du bien Avers la gauche? étrangères.
a) Une baisse du salaire des travailleurs qui b) un déplacement des courbes d’offre
fabriquent le bien X. des voitures canadiennes et étrangères.
b) Une augmentation du coût des machines c) un mouvement le long des courbes
utilisées pour produire le bien X. de demande des voitures canadiennes
c) Un progrès technique réalisé dans et étrangères.
la production du bien X. d) un mouvement le long de la courbe
d) Une circonstance où la quantité demandée de demande des voitures canadiennes et
est supérieure à la quantité offerte. un déplacement de la courbe de demande
e) Toutes ces réponses. des voitures étrangères.
e) un déplacement de la courbe de demande
20 Quand parle-t-on de pénurie ?
des voitures canadiennes et un mouvement
a) Lorsque la quantité demandée est supérieure
le long de la courbe de demande des voitures
à la quantité offerte.
étrangères.
b) Lorsque la quantité offerte est supérieure
à la quantité demandée.
© 24 La courbe de demande des étagères est
c) Lorsque la quantité demandée augmente
P= 75 — 6Cf , et la courbe d’offre des étagères
si le prix augmente.
est P= 35 + 2Qq. Quel est le prix d’équilibre
d) Lorsque la quantité demandée est supérieure
d’une étagère ?
à la quantité d’équilibre.
a) 5$
e) Lorsque la quantité offerte est supérieure
b) 10$
à la quantité d’équilibre.
c) 40$
21 Quelques producteurs discutent autour d’une d) 45$
bière. Quelles observations se rapportent à e) Aucune de ces réponses.
un mouvement le long de la courbe d’offre ?
a) « Les augmentations de salaire nous ont forcés © 25 La courbe de demande des trampolines est
à augmenter nos prix. » P = 300 - 6Qo- La courbe d’offre des trampolines
b) « Notre nouvel équipement de pointe nous est P = 20 + 8 Q0. Si le prix d’un trampoline
permettra de semer nos concurrents. » est fixé à 120 $, le marché des trampolines
c) « Le prix des matières premières a monté a) sera en équilibre.
en flèche. Cela se répercutera sur notre prix b) connaîtra une demande excédentaire
de vente. » entraînant une hausse du prix.
d) « Nous prévoyons une forte augmentation c) connaîtra une demande excédentaire
de la demande. Le prix de notre produit entraînant une baisse du prix.
devrait augmenter. Nous prévoyons donc d) connaîtra une offre excédentaire entraînant
une augmentation de la production. » une hausse du prix.
e) « Les nouveaux concurrents de l’industrie e) connaîtra une offre excédentaire entraînant
nous obligent à baisser nos prix. » une baisse du prix.

22 Si une hausse du prix du bien A fait déplacer


la courbe d’offre du bien B vers la droite, alors Problèmes à court développement
a) A et B sont des substituts de consommation.
b) A et B sont des compléments de 1 Expliquez la différence entre les besoins et
consommation. les demandes.
c) A et B sont des substituts de production.
d) A et B sont des compléments de production. 2 Le prix des ordinateurs personnels a continué
e) A est un facteur de production qui entre de baisser malgré l’augmentation de la demande.
dans la production de B. Expliquez pourquoi.
L'OFFRE ET LA DEMANDE 45

Une taxe sur le pétrole brut fera augmenter 6 Le marché du vin au Canada est initialement
le coût de la matière première utilisée dans en équilibre et les courbes de demande et d’offre
la production de l’essence. Un adepte d’une telle ont leur forme habituelle. La bière est un
taxe a déclaré quelle ne ferait pas augmenter substitut immédiat du vin ; le fromage et le vin
le prix de l’essence en avançant les arguments sont des compléments. Utilisez les graphiques
suivants : « Bien que le prix de l’essence puisse de l’offre et de la demande pour analyser l’effet
augmenter au début, cette augmentation de chacun des événements suivants (distincts)
entraînera une diminution de la demande sur le prix d’équilibre et la quantité sur le marché
d’essence, ce qui mènera à une baisse du prix de canadien du vin. Supposez que tous les autres
ce produit. En quoi cet argument est-il erroné? facteurs restent les mêmes sauf ceux qui sont
indiqués ci-dessous. Vous indiquerez dans chaque
Les choux de Bruxelles et les carottes sont cas, pour le prix d’équilibre et pour la quantité,
des substituts de consommation et, également, pourquoi la variable augmente, diminue, reste
puisqu’on peut les cultiver sur le même type la même ou se déplace de manière ambiguë
de sol, des substituts de production. Supposons (peut augmenter ou diminuer).
qu’il y ait une augmentation de la demande a) Les revenus des consommateurs diminuent
de choux de Bruxelles. Vérifiez, étape par étape, (le vin est un bien normal).
les effets d’une telle augmentation sur le prix et b) Un gel précoce a détruit une grande partie
la quantité échangée sur les marchés des choux des récoltes mondiales de raisin.
de Bruxelles et des carottes, (Gardez à l’esprit c) Un nouveau procédé de barattage réduit
le rappel 4). le coût de la production de fromage.
d) Une nouvelle technique de fermentation
Les données fournies au tableau 4.1 se rapportent
réduit le coût de la production de vin.
au comportement des acheteurs et des vendeurs
e) Une étude gouvernementale, qui vient d’être
de poisson dans une poissonnerie, un certain
publiée, établit un lien néfaste entre la
samedi.
consommation de vin et les maladies du cœur.
f) Les coûts de production de bière et de vin
TABLEAU 4.1 BARÈMES D’OFFRE ET
augmentent considérablement.
DE DEMANDE DE POISSON
7 Un journal indique que «en dépit d’une récolte
Prix Quantité Quantité
abondante de cerises cette année, la baisse du prix
(par poisson) demandée offerte
des cerises ne sera pas aussi forte que prévue
0,50$ 280 40 en raison d’une mauvaise récolte de prunes

1,00$ 260 135 et de pêches».

1,50$ 225 225 a) Utilisez le graphique de l’offre et de

2,00$ 170 265 la demande du marché des cerises pour

2,50$ 105 290 expliquer l’effet de la récolte abondante.

3,00$ 60 310 b) Sur le même graphique, expliquez l’effet sur

3,50$ 35 320 le marché des cerises de la mauvaise récolte


de prunes et de pêches.

a) Sur du papier graphique, tracez la courbe


de demande et la courbe d offre. Veillez à bien
8 Le tableau 4.2 indique les barèmes d’offre et
de demande pour les caisses de gelée de raisin.
nommer les axes. Quel est le prix d équilibre ?
b) Nous aurons recours aux habituelles
TABLEAU 4.2 BARÈMES D'OFFRE ET
suppositions ceteris paribus sur la courbe
DE DEMANDE DE LA GELÉE
de demande afin qu elle ne se déplace pas.
DE RAISIN PAR SEMAINE
Indiquez cinq facteurs qui, selon nos
hypothèses, ne changent pas.
Quantité Quantité
c) Nous supposerons également que la courbe
Prix demandée offerte
d’offre reste constante et que cinq facteurs
(par caisse) (en caisses) (en caisses)
ne changent pas. Énumérez-les.
d) Expliquez brièvement ce qui se produirait si 70$ 20 140
le prix était initialement fixe a 3,00$. 60$ 60 120
e) Expliquez brièvement ce qui se produiiait 50$ 100 100
si le prix était initialement fixe à 1,00 $. 40$ 140 80
f) Expliquez brièvement ce qui se produiiait 30$ 180 60
si le prix était initialement fixe a 1,50 $.
a) Tracez, sur le graphique de la figure 4.2, <§> 9 L’équation de la demande de grille-pain est
les courbes d’offre et de demande de gelée
de raisin. Veillez à bien nommer les axes.
P=8-1Qd.
Nommez les courbes de demande et d’offre L’équation de l’offre de grille-pain est
D0 et 00, respectivement.
T=2+ lQo,
FIGURE 4.2 lorsque P est le prix en dollars d’un grille-pain,
Qd est la quantité de grille-pain demandée, et
Qo, la quantité de grille-pain offerte. Le marché
des grille-pain est initialement en équilibre et
le revenu est de 300 $.
a) Quelle est la quantité d’équilibre (Q*)
des grille-pain ?
b) Quel est le prix d’équilibre (P*) d’un grille-
pain ?
c) A la suite d’une augmentation du revenu,
qui est maintenant de 500 $, la courbe
de demande de grille-pain se déplace (la courbe
d’offre reste la même). La nouvelle équation
de la demande est

P=4-l Qo.
b) Quels sont le prix d’équilibre et la quantité
Utilisez cette information pour calculer la
d’équilibre sur le marché de la gelée de raisin ?
nouvelle quantité d’équilibre des grille-pain ;
Sur votre graphique, nommez le point
calculez le nouveau prix d’équilibre d’un
d’équilibre a.
grille-pain.
c) Y a-t-il un surplus ou une pénurie au prix
d) Sur le graphique de la figure 4.3, tracez
de 40$ ? De quel ordre ?
et nommez: 1) la courbe d’offre, 2) la courbe
d) Les barèmes de l’offre et de la demande
de demande initiale, 3) la nouvelle courbe
peuvent également être représentés par
de demande.
les équations de l’offre et de la demande
e) Les grille-pain sont-ils un bien normal
suivantes :
ou inférieur ? Comment le savez-vous ?
Demande: T= 75 - 0,25 Qo
Offre: P=0,5Qo FIGURE 4.3
Utilisez ces équations pour obtenir la
quantité d’équilibre (Q*) ; le prix d’équilibre
(P*). (Indice: Vos réponses devraient être
les mêmes que pour 8b.)
e) Supposez que la population augmente
suffisamment pour que la demande de gelée
de raisin augmente de 60 caisses par semaine
pour tous les prix.
i) Faites le tableau (prix, quantité demandée)
du nouveau barème de demande.
ii) Tracez la nouvelle courbe de demande
sur votre graphique d’origine et nommez-
la Dx.
iii) Nommez le nouveau point d’équilibre b.
Quantité (en grille-pain)
Quels sont les nouveaux prix et quantité
d’équilibre ?
iv) Quelle est la nouvelle équation de la
demande? (Indices: Quelle est la valeur de
la nouvelle pente ? Quelle est la nouvelle
ordonnée à l’origine sur l’axe des prix?)
L'OFFRE ET LA DEMANDE 47

® 10 L’équation de la demande de pastilles à la d’offre. Le prix T, mais la variation de la


menthe est quantité dépend de l’importance relative des
déplacements.
P=80-2Qd. 8 V T nombre d’entreprises —> déplacement vers
L’équation de l’offre de pastilles à la menthe est la droite de la courbe d’offre —> X prix et
T quantité.
P=50 + lQo, 9 I La quantité va T, mais la variation du prix
où P est le prix en dollars d’une boîte de pastilles dépend de l’importance relative des
à la menthe, QD est la quantité de pastilles à la déplacements des courbes de demande et d’offre.
menthe demandée, et Qq, la quantité de 10 F Lorsque P > P d’équilibre, il y a un surplus

pastilles à la menthe offerte. Supposons que (quantité offerte > quantité demandée).
l’hypothèse ceteris paribus soit toujours valable.
a) Si le prix des pastilles à la menthe était fixé Questions à choix multiple
à 56 $, calculez le surplus ou la pénurie exacts
de pastilles à la menthe. I b Par exemple, T du prix des frites

b) Expliquez le processus d’ajustement qui —» X demande de hamburgers.

permettra d’atteindre le prix d’équilibre. & 2 c Les deux réponses sur les revenus pourraient

c) Quelle est la quantité d’équilibre (Q*) être correctes s’il s’agissait d’un bien normal

des pastilles à la menthe ? (a) ou inférieur (b).

d) Quel est le prix d’équilibre (P*) d’une boîte 3 a La question porte sur le mouvement
descendant le long d’une courbe d’offre.
de pastilles à la menthe ?
e) Supposons maintenant que, à cause de
4 e Les réponses a, b et c déplacent la courbe

progrès techniques, la courbe d’offre des


de demande, alors que d entraîne un
mouvement le long de la courbe d’offre.
pastilles à la menthe se déplace (la courbe de
demande reste la même). La nouvelle
5 a Définition d’un substitut de production.

équation d’offre est


6 e Au prix d’équilibre, les intentions de
production correspondent aux intentions
P= 20+ IQq. des consommateurs ; la quantité demandée
= la quantité offerte.
Utilisez cette information pour calculer la
nouvelle quantité d’équilibre des pastilles à la
7 c Les autres réponses décrivent les déplacements
de la courbe de demande.
menthe ; calculez le nouveau prix d’équilibre
d’une boîte de pastilles à la menthe.
8 c Les réponses a et b ont un effet indéterminé
sur le prix, alors que d —> X prix.
9 d Toutes les réponses portent sur un prix

RÉPONSES supérieur au prix d’équilibre.


10 e La courbe de demande se déplace vers
la gauche.
Vrai/Faux/Incertain I I e La courbe de demande se déplace vers
(Justifiez votre réponse.) la droite.
I F À mesure que le prix augmente, la quantité 12 e La courbe d’offre se déplace vers la droite et la
demandée diminue, mais pas la demande. courbe de demande se déplace vers la gauche.
Ô 2 I Déplacement vers la gauche pour un bien 13 a T mères au travail —> T des préférences pour
normal, vers la droite pour un bien inférieur. les garderies —> T de la demande de services
3 F La courbe d’offre indique un prix minimum de garderies.
auquel la dernière unité est offerte. 14 b Les modifications du revenu font déplacer
Ô 4 I Vrai si A et B sont des substituts de la courbe de demande sans entraîner de
production, mais faux s ils sont des substituts mouvement le long de la courbe de demande.
de consommation. 15 d Les autres réponses font augmenter le surplus
&5 F La viande et le cuir sont des compléments (offre excédentaire).
de production, car ils sont nécessairement 16 d X quantité demandée correspond à un

produits ensemble. mouvement ascendant le long de la courbe


6 V Pour des compléments de production, de demande. Cette diminution pourrait
T du prix d’un bien —> T quantité offerte également être causée par un déplacement

et t offre de l’autre bien. vers la gauche de la courbe d’offre.


7 1 T prix futurs anticipés —> déplacement vers 0 17 c La courbe d’offre de A se déplace
la droite de la courbe de demande et vers la droite —> X prix de A, ce qui
déplacement vers la gauche de la courbe —» T demande de B —> T prix de B.
48 CHAPITRE 4

18 c Les autres éléments déplacent la courbe gauche la courbe d’offre de l’essence. Cela fera
de demande. Seul le prix peut changer le long augmenter le prix d’équilibre de l’essence et,
d’une courbe de demande fixe. de ce fait, diminuer la quantité d’essence
19 b T prix du facteur de production fait déplacer demandée. La demande en elle-même ne va pas
la courbe d’offre vers la gauche. diminuer, c’est-à-dire que la courbe de demande
20 a La pénurie correspond à la distance horizontale ne se déplacera pas. La diminution de l’offre
entre les courbes d’offre et de demande à un entraîne un mouvement le long d’une courbe
prix inférieur au prix d’équilibre. de demande inchangée.
21 d Les autres réponses décrivent les déplacements
de la courbe d’offre. 4 Pour répondre à cette question, il faut vérifier
22 d Définition des compléments de production. étape par étape les effets exercés sur les deux
Les variations de prix liées aux biens substituts graphiques de la figure 4.4 - a) pour le marché
de consommation font déplacer la courbe des choux de Bruxelles et b) pour le marché des
de demande. carottes. La séquence des effets se produit dans
23 d T prix des voitures canadiennes en raison l’ordre des chiffres indiqués sur les graphiques.
du déplacement vers la gauche de la courbe Étudiez d’abord le marché des choux
d’offre. Les substituts des voitures canadiennes de Bruxelles. L’augmentation de la demande fait
et étrangères —> le déplacement vers la droite déplacer la courbe de demande vers la droite
de la demande de voitures étrangères. de D0 à Dl (1), et le prix des choux de Bruxelles
©<3>24 d Supposez que la demande est égale à l’offre augmente. Cette hausse du prix a deux effets
et trouvez la solution pour Q* = 5. Utilisez (2) sur le marché des carottes. Étant donné
Q* = 5 soit dans l’équation de la demande, que les choux de Bruxelles et les carottes sont
soit dans l’équation de l’offre pour obtenir P*. des substituts de consommation, la courbe
©0 25 b Avec P = 120 $, Qd = 30 et Q0 = 12,5. de demande des carottes se déplace vers la droite
La demande est excédentaire de sorte que de D0 à Dp De plus, étant donné que les choux
le prix va T. de Bruxelles et les carottes sont des substituts
de production, la courbe d’offre des carottes
se déplace vers la gauche de O0 à Ol. Ces deux
Problèmes à court développement
déplacements sur le marché des carottes font
monter le prix des carottes, ce qui entraîne
1 Les besoins reflètent nos désirs illimités de biens
des effets de rétroaction sur le marché des choux
et de services indépendamment de notre capacité
de Bruxelles. Mais n’oubliez pas (Rappel 4) que
ou de notre volonté de faire les sacrifices
chaque courbe (offre et demande) d’un marché
nécessaires pour les obtenir. Lorsqu’il y a rareté,
donné ne peut se déplacer qu une fois au
un grand nombre de ces besoins ne seront
maximum. Puisque la courbe de demande
pas satisfaits. En revanche, les demandes
des choux de Bruxelles s’est déjà déplacée, nous
se rapportent aux intentions d’acheter et,
pouvons seulement déplacer la courbe d’offre
par conséquent, reflètent les décisions prises
de O0 à Ox (3) à cause des substituts dans
à propos des besoins qui seront satisfaits.
la relation de production. Chaque courbe
de chaque marché ne s’est maintenant déplacée
2 En raison du rythme accéléré des progrès
technologiques, non seulement la demande qu’une fois et l’analyse doit prendre fin. Nous
pouvons prédire que les effets nets sont des
d’ordinateurs personnels a augmenté, mais l’offre
augmentations des prix d’équilibre des choux
également. En fait, l’offre a augmenté bien plus
de Bruxelles et des carottes, et des variations
rapidement que la demande, ce qui a entraîné
indéterminées des quantités d’équilibre dans
la baisse des prix. De ce fait, une grande partie
les deux marchés.
de l’augmentation (pas toute) des ventes
d’ordinateurs personnels reflète un mouvement
descendant le long d’une courbe de demande
plutôt qu’un déplacement de la courbe de
demande.

3 Cet argument confond un mouvement le long


d’une courbe de demande inchangée avec un
déplacement de la courbe de demande. La
bonne analyse est la suivante : la hausse du prix
du pétrole (principale ressource utilisée dans la
production de l’essence) fera déplacer vers la
L’OFFRE ET LA DEMANDE 49

FIGURE 4.4 b) Prix des biens apparentés ; revenu ; prix futurs


anticipés ; population ; préférences.
(a) Marché des choux de Bruxelles
c) Prix des facteurs de production ; prix des autres
biens produits ; prix futurs anticipés ; nombre
de producteurs ; technologie.
d) Au prix de 3 $, la quantité offerte (310)
est supérieure à la quantité demandée (60).
Les vendeurs de poisson se retrouvent avec
un surplus de poisson. Plutôt que d’avoir
du poisson invendu (qui ne donne pas
de revenus), certains vendeurs baissent le prix
afin d’augmenter la quantité de poisson
demandée.
Le jeu de la concurrence force d’autres
vendeurs à agir de façon similaire, et le prix
diminue jusqu’à son prix d’équilibre de 1,50 $,
alors que la quantité demandée augmente
jusqu’à la quantité d’équilibre de 225 unités.
e) Au prix de 1 $, la quantité demandée (260)
est supérieure à la quantité offerte (135) - il
y a pénurie. Les acheteurs de poisson font
monter le prix afin d’obtenir le poisson
«rare». Comme les prix continuent de monter
aussi longtemps que la demande reste
excédentaire, la quantité offerte augmente
pour répondre aux prix plus élevés. Le prix
et la quantité offerte s’élèvent tous deux
jusqu’au prix d’équilibre (1,50 $) et à
la quantité d’équilibre (225 unités).
f) Au prix de 1,50 $, la quantité offerte est
exactement égale à la quantité demandée
(225). Il n’y a pas de demande excédentaire
(pénurie) ni d’offre excédentaire (surplus) et,
par conséquent, aucune tendance de variation
a) Les courbes d’offre et de demande sont du prix ni de la quantité demandée.
indiquées à la figure 4.5. Le prix d’équilibre
est de 1,50 $ par poisson. Les diagrammes de la demande et de l’offre pour
les parties a à e sont présentés à la figure 4.6.
FIGURE 4.5
FIGURE 4.6
50 CHAPITRE 4

f) De telles questions nous forcent à examiner


deux marchés distincts mais apparentés — les
marchés de la bière et du vin. Étant donné
que ce type de question désoriente souvent les
étudiants, la figure 4.7 offre une explication
plus détaillée de la réponse.
Étudions d’abord le marché de la bière.
É’augmentation du coût de la production de
bière déplace la courbe d’offre de la bière vers
la gauche de O0 à 01. L’augmentation du prix
de la bière, qui en découle, a une incidence
sur le marché du vin puisque la bière et le vin
sont des substituts (de consommation).
Prenons maintenant le marché du vin où
nous avons deux déplacements à examiner. La
hausse du prix de la bière entraîne le
déplacement vers la droite, de D0 à Dv de la
demande de vin. L’augmentation du coût de
la production de vin entraîne le déplacement
vers la gauche, de O0 à Ol, de la courbe
d’offre du vin. L’analyse prend alors fin
puisque la question ne porte que sur le
marché du vin. Le résultat final est une
hausse du prix d’équilibre du vin et un
changement ambigu sur le plan de la quantité
de vin. Bien que le graphique indique que Qj
= Qû, Qi peut être > ou < Qq.

FIGURE 4.7
(a) Marché de la bière

o Quantité
L’OFFRE ET LA DEMANDE 51

(b) Marché du vin de l’an dernier. La récolte abondante de l’année


en cours a fait augmenter l’offre à Oj. Toutes
autres choses étant égales, le prix des cerises
descendrait de P0 à P{.
b) Mais les autres choses ne sont pas égales.
La mauvaise récolte de prunes et de pêches
(leurs courbes d’offre se sont déplacées vers la
gauche) en a fait augmenter le prix. La hausse
du prix des prunes et des pêches, qui sont
des substituts de consommation des cerises,
fait monter la demande de cerises à Dx.
Le résultat net est que le prix des cerises
ne baisse que jusqu’à P2 au lieu d’aller
jusqu’à Px.

8 a) Les courbes de demande et d'offre de


gelée de raisin sont présentées à la figure
De nombreux étudiants demandent avec
4.2 Solution.
justesse: «Mais n’est-ce pas la hausse du prix
du vin qui fait déplacer la courbe de demande
FIGURE 4.2 SOLUTION
de la bière vers la droite, ce qui entraîne une
hausse du prix de la bière et une augmentation
supplémentaire de la demande de vin ? » Cette
question, juste en principe, se rapporte à la
dynamique de l’ajustement, et ces graphiques
permettent seulement d’analyser les déplacements
uniques de la courbe d’offre ou de la courbe de
demande. Nous pourrions déplacer la demande
de bière vers la droite, mais l’augmentation
résultante du prix de la bière nous ferait déplacer
la demande de vin une seconde fois. Dans la
pratique, adoptez la règle voulant que chaque
courbe (de demande et d’offre) d’un marché
donné ne peut se déplacer c\uune seule fois Quantité (en caisses de gelée de raisin par semaine)
au maximum.

a) Les courbes d’offre et de demande du marche b) L’équilibre est atteint à l’intersection des
des cerises sont présentées à la figure 4.8. courbes d’offre et de demande (point a).
Le prix d’équilibre est de 50 $ par caisse
et la quantité d’équilibre est de 100 caisses
par semaine.
c) Au prix de 40 $, il y a une pénurie de
60 caisses par semaine.
® d) En situation d’équilibre, les équations
deviennent:

Demande: P* = 75- 0,25 Q*


ou Qr> = 300 -4 P
Offre: P* = 0,5 Q*
ou Qo = 2P

Pour résoudre l’équation pour Q*,


supposons que la demande est égale
à l’offre :

75 - 0,25 Q* = 0,5 Q*
75 = 0,75 Q*
100 = Q*
52 CHAPITRE 4

Pour résoudre l’équation pour P*, nous demande et d’offre pour calculer les
pouvons utiliser Q* dans les équations d’offre valeurs d’équilibre du prix et de la
ou de demande. Prenons d’abord la demande : quantité, vous pouvez utiliser la nouvelle
équation de la courbe de demande avec
P* = 75 - 0,25 Q* l’équation de la courbe d’offre pour
P* = 75 - 0,25 (100) trouver les réponses obtenues à e iii.
P* = 75-25
P* = 50 ® 9 En situation d’équilibre, les équations
deviennent:
En utilisant Q* dans l’équation de l’offre
nous obtenons le même résultat : Demande : P* = 8 — 1 Q*
Offre: P* = 2+\Q*
P* = 0,5 Q*
P* = 0,5 (100) a) Pour résoudre l’équation pour Q*, supposons
P* = 50 que la demande est égale à l’offre :
e) i) Le tableau 4.3 indique aussi la quantité
8- 1Q* = 2+ 1Q*
offerte (inchangée) à titre de référence.
6 = 2Q*
3= Q*
TABLEAU 4,3 BARÈMES D'OFFRE ET
DE DEMANDE DE LA GELÉE b) Pour résoudre l’équation pour P*, nous
DE RAISIN PAR SEMAINE pouvons utiliser Q* dans les équations
de la demande ou de l’offre. Voyons d’abord
Quantité Quantité la demande :
Prix demandée offerte
(par caisse) (en caisses) (en caisses) P* = 8-\Q*
P* = 8- 1(3)
70$ 80 140
P* = 8-3
60$ 120 120
P* = 5
50$ 160 100
40$ 200 80 En utilisant Q* dans l’équation de l’offre,
30$ 240 60 nous obtenons le même résultat :

ii) Le graphique de la nouvelle courbe P* = 2+ 1Q*


de demande, Dx, est présenté dans P* = 2+ 1(3)
la figure 4.2 Solution. P* = 2+3
iii) Le nouveau prix d’équilibre est de 60 $ P* = 5
par caisse et la quantité est de 120 caisses
c) En situation d’équilibre, les équations sont
de gelée de raisin par semaine.
les suivantes :
® iv) La nouvelle équation de demande est
P =90- 0,25 Qn, (ou Qn = 360 - 4P). Demande : P* = 4 — 1 Q*
Remarquez que la pente de la nouvelle Offre: P* = 2+\Q*
équation de la courbe de demande est
la même que celle de l’équation de Pour résoudre l’équation pour Q*,
la courbe de demande d’origine. Une supposons que la demande est égale à l’offre :
augmentation de la demande de 60 caisses
4- 1Q* = 2 + 1Q*
pour tous les prix entraîne un déplacement
2 = 2 Q*
parallèle vers la droite de la courbe
1 = Q*
de demande. Les deux courbes étant
parallèles, elles ont la même pente. Pour résoudre l’équation pour P*,
Le chiffre de 90 est l’ordonnée à l’origine nous pouvons utiliser Q* dans les équations
sur l’axe des prix de la nouvelle courbe de de demande ou d’offre. Voyons d’abord
demande, que vous pouvez voir sur votre la demande :
graphique. N'oubliez pas que l’équation
de la demande est l’équation d’une ligne P* = 4- 1Q*
droite (y = a + bx) ; dans ce cas, a = 90. P* = 4- 1(1)
Si vous souhaitez vous exercer encore P* = 4-1
un peu à utiliser les équations de P* = 3
L'OFFRE ET LA DEMANDE 53

En utilisant Q* dans l’équation de La quantité demandée excède la quantité


demande, nous obtenons le même résultat: offerte de 6 unités (12 — 6), de sorte qu’il y a
pénurie de 6 pastilles à la menthe.
P* = 2+ 1Q*
b) Une pénurie signifie que le prix a été fixé
P* = 2 + 1(1)
au-dessous du prix d’équilibre. La concurrence
P* = 2+1
que se livrent les consommateurs pour obtenir
P* = 3
le nombre limité de pastilles à la menthe fera
d) La courbe d’offre, la courbe de demande monter le prix et augmenter la quantité
initiale et la nouvelle courbe de demande offerte jusqu’à ce que l’on atteigne le prix
de grille-pain sont présentées dans la figure et la quantité d’équilibre.
4.3 Solution. c) En situation d’équilibre, les équations sont les
suivantes :

Demande: P* = 80 - 2Q*
Offre: P* = 50+1Q*

Pour résoudre l’équation pour Q*,


supposons que la demande est égale à l’offre :

80 - 2Q* = 50 + 1Q*
30 = 3 Q*
10 = Q*
d) Pour résoudre l’équation pour P*, utilisez Q*
dans l’équation de la demande :

P* = 80 - 2Q*
P* = 80-2(10)
P* = 80 - 20
P* = 60

Vous pouvez vérifier cette réponse vous-


même, en utilisant Q* dans l’équation de
e) Les grille-pain sont un bien inférieur. Une l’offre.
augmentation du revenu (de 300 $ à 500 $) e) Les nouvelles équations d’équilibre sont les
entraîne une diminution de la demande - suivantes :
la courbe de demande des grille-pain s’est
Demande: P* = 80 — 2Q*
déplacée vers la gauche.
Offre: P* = 20+1Q*

® 10 a) On utilise le prix de 56 $ dans les équations Pour résoudre l’équation pour Q*,
d’offre et de demande pour calculer les supposons que la demande est égale à l’offre :
quantités demandées et offertes a ce prix.
80 - 2Q* = 20 + 1Q*
Il s’agit de l’équivalent mathématique de
60 = 3 Q*
ce que l’on fait sur un graphique pour
20 = Q*
déterminer un prix sur l’axe vertical ; suivez
des yeux la courbe de demande (ou d offre), Pour résoudre l’équation pour P*, utilisez
puis regardez vers le bas pour lire la quantité Q* dans l’équation de l’offre:
sur l’axe horizontal.
P* = 20+ 1Q*
On utilise le prix dans l’équation de
P* = 20+ 1(20)
la demande pour trouver :
P* = 20 + 20
P=80-2Qn P* = 40

,
56 = 80-20^
Vous pouvez vérifier cette réponse vous-
2Qd = 2A
même, en utilisant Q* dans l’équation de la
0 3=12
demande.
En utilisant le prix dans l’équation
de l’offre :

P= 50+ \Qo
56 = 50 + 1 Q0
6 = Qjo
L’élasticité

♦ Le temps écoulé depuis la variation de prix —


CONCEPTS CLÉS plus le temps écoulé à partir d’une variation
de prix est long —>T élasticité.

L’élasticité de la demande • La demande à court terme décrit la réaction des


acheteurs à une variation du prix d’un produit
L’élasticité-prix de la demande (t|) mesure la sensibilité avant qu’ils aient eu le temps de découvrir tous
de la quantité demandée à une variation de prix. ses substituts.
♦ T) est la mesure de la sensibilité indépendante • La demande à long terme décrit la réaction des

des unités de mesure choisies. acheteurs à une variation du prix d’un produit
après qu’ils ont découvert des produits substituts
% A quantité demandée (A Q/Qmoy) acceptables.
♦ Tl • La demande à court terme est habituellement
% A du prix (AP/Pmoy)
moins élastique que la demande à long terme.

♦ L’élasticité n’est pas synonyme de pente. En se L’élasticité et la recette totale (P X Q) — calcul


déplaçant vers le bas le long d’une courbe de des recettes totales
demande linéaire, la pente (AP/AQ) est constante,
mais l’élasticité X à mesure que P -i et Qjnoy X. ♦ Lorsque la demande est T prix entraîne
inélastique (T| < 1) T de la recette totale
à élasticité unitaire (î| = 1) aucun changement
♦ Lorsque la demande est de la recette totale
T) = 00 parfaitement élastique (horizontale) élastique (T| > 1) X de la recette totale
1 < T) < 00 élastique
Tl = 1 à élasticité unitaire
0 < T| < 1 inélastique Autres types d’élasticité de la demande
T| = 0 parfaitement inélastique (verticale)
L’élasticité-prix croisée de la demande (T|x) mesure
L’élasticité de la demande varie en fonction des facteurs la sensibilité de la quantité demandée du bien A
qui suivent. à une variation du prix du bien B.
♦ La substituabilité des produits - plus il est facile
de remplacer un bien par un autre, plus la demande % A quantité demandée du bien A

est élastique. % A prix du bien B


• Les biens de première nécessité sont des biens
♦ T|x > 0 les biens sont des substituts
difficiles à remplacer et leur demande est
TL < 0 les biens sont des compléments
inélastique.

• Les produits de luxe ont généralement L’élasticité-revenu de la demande (t| ) mesure

de nombreux substituts et leur demande la sensibilité de la quantité demandée à une variation


est élastique. du revenu.

♦ La part du revenu consacrée au produit —


% A quantité demandée
plus la part du revenu consacrée au produit
Tl, =-*-
est élevée —> T élasticité. % A revenu
L'ÉLASTICITÉ 55

♦ Lorsaue La demande est La formule complète (voir la figure 5.2 du


11, > 1 élastique par rapport au revenu manuel à la page 100) du calcul de l’élasticité
(bien normal de luxe) prix de la demande entre deux points sur
0 < Tl, < 1 inélastique par rapport au revenu la courbe de demande est :
(bien normal essentiel)
_ % A quantité demandée
R,<0 d’élasticité-revenu négative
(bien inférieur) % A du prix

' AQ ^ v AP ^
L’élasticité de l’offre
KQmoy J v m°y j

L’élasticité-prix de l’ofifre (T|0) mesure la sensibilité de Selon la loi de la demande, le prix et la


la quantité offerte à une variation du prix. quantité demandée évoluent toujours en sens
opposé le long de n’importe quelle courbe
_ % A quantité offerte de demande. Par conséquent, sans signe de valeur

% A prix absolue, la formule de l’élasticité-prix de la


demande donnerait une valeur négative. Comme
L’élasticité de l’offre varie en fonction des facteurs nous cherchons surtout à connaître l’importance
qui suivent. de l’effet d’une variation de prix sur la quantité
demandée, pour simplifier, nous utilisons la
♦ La substituabilité des facteurs de production - plus
valeur absolue pour être sûrs d’obtenir un chiffre
les facteurs de production utilisés pour produire
positif. Chaque fois que vous voyez le terme
les biens et les services sont adaptables —> T T|0.
élasticité de la demande, souvent écrit en abrégé,
♦ La durée du temps de réaction des producteurs n’oubliez pas qu’il se rapporte à la valeur absolue
aux variations de prix — T|0 T avec T du temps de l’élasticité-pnx de la demande.
écoulé depuis l’offre instantanée jusqu’à l’offre
à court terme et jusqu’à l’offre à long terme. 3 L’élasticité n’est pas synonyme de pente (bien

• L’offre instantanée indique l’effet immédiat quelles soient reliées). Le long d’une courbe
d’une variation de prix sur la quantité offerte. de demande linéaire, la pente est constante,

• L’offre à court terme décrit la réponse d’une mais l’élasticité varie de l’infini à zéro à mesure
variation de prix à la quantité offerte, une fois que nous descendons le long de la courbe

que les producteurs ont fait certains ajustements de demande. Pour le comprendre, prenez
la formule ci-dessous, qui n’est qu’une simple
techniques possibles.
• L’offre à long terme décrit la réponse d’une adaptation de la formule de l’élasticité

variation de prix à la quantité offerte, une fois du Rappel 2.

que les producteurs ont fait tous les ajustements AQ moy


techniques possibles. R= x
VAP J y Q moy J
Le terme entre les premières parenthèses
est simplement l’inverse de la pente. Étant donné
R A P P E L S que la pente est constante n’importe où le long
d’une courbe de demande linéaire, l’inverse de
I On trouve dans ce chapitre de nombreuses la pente l’est également. Toutefois, voyez ce qui
formules d’élasticité. Toutes sont fondées sur se passe lorsque le terme entre les secondes
le même principe intuitif simple — la sensibilité. parenthèses évolue à mesure que nous descendons
Toutes les formules d’élasticité de 1 offre et de le long de la courbe de demande. En haut de la
la demande mesurent la sensibilité de la quantité courbe de demande, le Pmoy est très élevé, la
(demandée ou offerte) à la variation d un est très faible, et la valeur du terme entre les
autre facteur. Par conséquent, le pourcentage secondes parenthèses est donc élevée. Comme
de changement de la quantité est toujours nous descendons le long de la courbe de
le numérateur de la formule applicable. Lorsque demande, le Pmoy devient plus bas et la Q„ , plus
vous commencerez à voir les effets des variations grande. La valeur du terme entre les secondes
des prix, du revenu et du prix des biens parenthèses diminue. Le résultat net est que
apparentés sur la quantité, vous pourrez trouver la valeur absolue de l’élasticité-prix de la demande
chaque formule d’élasticité, même si vous 1 avez diminue à mesure que nous descendons le long
de la courbe de demande.
oubliée.
56 CHAPITRE 5

4 Le concept d’élasticité de la demande est mesurent la sensibilité, l’importance et la


important et pratique en ce sens qu’il nous direction de la réaction sont essentielles. Dans le
permet de prédire l’effet d’une variation du prix cas de l’élasticité-revenu de la demande, l’effet
sur la recette totale. Une baisse du prix fera d’une augmentation du revenu sur la quantité
augmenter la recette totale si la demande est demandée sera positif pour un bien normal
élastique, ne fera pas changer le revenu si et négatif pour un bien inférieur. Dans le cas
la demande est à élasticité unitaire, et fera baisser de l’élasticité-prix croisée de la demande, l’effet
la recette totale si la demande est inélastique. d’une variation du prix d’un bien B sur la
Du fait que le prix et la quantité demandée quantité demandée du bien A sera positif si
évoluent toujours en sens opposé le long d’une les biens sont des substituts et négatif si les biens
courbe de demande, une baisse de prix entraînera sont des compléments.
une augmentation de la quantité demandée. Pour calculer une élasticité-revenu ou une
Etant donné que la recette totale est égale élasticité-prix croisée, il ne faut pas oublier que
au prix multiplié par la quantité, la baisse de l’on suppose que le prix du bien, au numérateur
prix aura tendance à faire baisser la recette dans La formule, est constant. L’élasticité du revenu
totale, alors que l’augmentation de la quantité mesure l’effet d’une variation du revenu sur
demandée fera augmenter la recette totale. la quantité, ceteris paribus. L’élasticité croisée
L’effet net dépend de l’effet individuel qui est mesure l’effet d’une variation du prix du bien B
le plus fort. sur la quantité du bien A, ceteris paribus.
Le concept d’élasticité de la demande résume Autrement dit, pour calculer ces élasticités,
de manière pratique l’effet net. Par exemple, nous ne pouvons simplement lire les valeurs
si la demande est élastique, le pourcentage numériques à partir des positions d’équilibre
de variation de la quantité demandée est plus d’un ensemble de courbes d’offre et de demande.
grand que le pourcentage de variation du prix. Ainsi, lorsque le revenu augmente, la demande
De ce fait, avec une baisse de prix, l’effet-quantité se déplace vers la droite (pour un bien normal),
dominera et la recette totale augmentera. et la nouvelle quantité d’équilibre aura également
Toutefois, si la demande est inélastique, un prix d’équilibre légèrement plus élevé. Pour
le pourcentage de variation de la quantité calculer l’élasticité-revenu correctement, il faut
demandée est inférieur au pourcentage comparer le revenu et la quantité demandée
de variation du prix. De ce fait, avec une baisse de l’équilibre initial avec les nouvelles valeurs
de prix, l’efifet-prix dominera et la recette totale du revenu et de la quantité demandée. Pour
baissera. obtenir la nouvelle quantité demandée correcte,
nous devons utiliser la nouvelle courbe
5 Deux autres concepts d’élasticité importants sont
de demande, mais calculer la quantité demandée
l’élasticité-revenu de la demande et l’élasticité-
qui aurait prévalu au prix initial, non changé.
prix croisée de la demande.
(Utilisez comme exemples les problèmes à court
L’élasticité-revenu de la demande: développement 9 et 10).

_ % À quantité demandée

% A revenu

f AQ A 'AKÏ A U T O F. V A L U A T I O N
J Y
Qwoy
V m°y y
Vrai/Faux/Incertain
L’élasticité-prix croisée de la demande: (Justifiez votre réponse.)
_ % A quantité demandée du bien A
I L’élasticité-prix de la demande mesure la
% A du prix du bien B
sensibilité des prix aux fluctuations de la
demande.
AQ/ APB

moy
Q7^ J/ y-t VôTmoy y

Remarquez que ces deux formules


d’élasticité n’ont pas de signes de valeur absolue 2 Après une augmentation du prix, la demande
et elles peuvent prendre des valeurs soit d’essence deviendra probablement plus
négatives, soit positives. Bien que ces formules inélastique avec le temps.
L’ÉLASTICITÉ 57

3 L’élasticité-prix de la demande est constante Au tableau 5.1, il y a deux points sur la courbe
le long d’une courbe de demande linéaire. de demande de ballons de volley-ball.

TABLEAU 5.1

4 Si vous aimez autant boire du Pepsi Cola que Prix Quantité


du Coca-Cola, votre demande de Pepsi sera par ballon de volley-ball demandée
probablement élastique.
19$ 55
21 $ 45

Quelle est l’élasticité de la demande entre ces


5 Si vos dépenses en dentifrice sont faibles deux points ?
par rapport à votre revenu total, votre demande a) 2,5
de dentifrice sera probablement inélastique. b) 2,0
c) 0,5
d) 0,4
e) Aucune de ces réponses.
6 Plus la définition d’un bien est étroite, plus
la demande de ce bien est élastique. 3 Le fait que la margarine soit un substitut
immédiat de la consommation de beurre
a) rend l’offre de beurre plus élastique.
b) rend l’offre de beurre moins élastique.
c) rend la demande de beurre plus élastique.
7 Un bien inférieur aura une élasticité-prix croisée
d) rend la demande de beurre moins élastique.
de la demande négative.
e) n’a pas d’incidence sur l’élasticité de l’offre
ni sur celle de la demande de beurre.

4 Une soudaine vague de chaleur à la fin de


8 Lorsqu’une augmentation de 10% du prix des l’été fait monter la demande de climatiseurs
gadgets entraîne une augmentation de 6% de la et surprend les producteurs qui n’en ont
quantité de bidules demandée, cela signifie que plus en stock. La courbe d’offre instantanée
les gadgets et les bidules sont des compléments. des climatiseurs est
a) parfaitement élastique.
b) parfaitement inélastique.
c) élastique.
9 Lorsqu’une diminution de l’offre entraîne d) à pente positive.
l’augmentation de la recette, la demande e) horizontale.
est alors inélastique.
5 Une innovation technologique fait baisser
le coût des photocopieuses. Si la demande
de photocopieuses par rapport au prix
est inélastique, on peut prévoir que la vente
10 Lorsqu’une augmentation de 9 % du prix
des photocopieuses va
entraîne un diminution de 5 % de la quantité
a) baisser et la recette totale, augmenter.
demandée, la recette totale a diminue.
b) baisser et la recette totale, diminuer.
c) augmenter et la recette totale, augmenter.
d) augmenter et la recette totale, diminuer.
e) augmenter, mais les fluctuations de la recette
Questions à choix multiple totale dépendront de l’élasticité de l’offre.

I Si l’élasticité-prix de la demande est nulle, alors, 6 L’élasticité-prix d’une courbe de demande

à mesure que le prix baisse parfaitement verticale est

a) la recette totale ne change pas. a) égale à 0.

b) la quantité demandée ne change pas. b) supérieure à 0, mais inférieure à 1.

c) la quantité demandée tombe a zéro. c) égale à 1.

d) la recette totale augmente à partir de zéro. d) supérieure à 1.


e) infinie.
e) Aucune de ces réponses.
58 CHAPITRE 5

7 Lorsqu’une augmentation de 10% du revenu I2 Lorsque le prix passe de 1,50 $ à 2,50 $, la


entraîne une augmentation de 5 % de la quantité quantité offerte passe de 9 000 à 11 000 unités.
demandée (le prix étant constant), quelle est Quelle est l’élasticité-prix de l’offre ?
l’élasticité-revenu de la demande ? a) 0,4
a) 0,5 b) 0,8
b) -0,5 c) 2,5
c) 2,0 d) 4,0
d) -2,0 e) Aucune de ces réponses.
e) Aucune de ces réponses.
13 Si la demande de jus d’orange congelé est
élastique en fonction du prix, alors un fort gel,
8 Si l’élasticité-prix de la demande est égale à 2,
qui détruit une grande quantité d’oranges, fera
alors une baisse du prix de 2 %
probablement
a) fera doubler la quantité demandée.
a) diminuer le prix d’équilibre du jus d’orange,
b) réduira de moitié la quantité demandée.
mais augmenter les dépenses totales
c) fera augmenter de 4% la quantité demandée.
des consommateurs relatives à ce bien.
d) réduira de 2 % la quantité demandée.
b) baisser la quantité d’équilibre du jus d’orange
e) fera augmenter de 0,5 % la quantité
ainsi que les dépenses totales des
demandée.
consommateurs relatives à ce bien.
c) réduire la quantité d’équilibre et le prix
9 Un pourcentage donné d’augmentation
d’équilibre du jus d’orange.
du prix d’un bien entraînera probablement
d) augmenter le prix d’équilibre du jus d’orange
un pourcentage de diminution de la quantité
ainsi que les dépenses totales des
demandée plus élevé
consommateurs relatives à ce bien.
a) lorsque moins de temps se sera écoulé.
e) augmenter le prix d’équilibre du jus d’orange,
b) lorsque le pourcentage du revenu dépensé
mais maintiendra constantes les dépenses
sur ce bien est élevé.
totales des consommateurs relatives à ce bien.
c) lorsqu’il est plus difficile d’obtenir
des substituts de ce bien.
14 Si une augmentation de 4 % du prix du beurre
d) Toutes ces réponses.
d’arachide entraîne une baisse de 8 % de
e) Aucune de ces réponses.
la recette totale, alors la demande de beurre
d’arachide
10 Une valeur négative de
a) est élastique.
a) l’élasticité-prix de l’offre implique une pente
b) est inélastique.
de courbe d’offre positive.
c) est à élasticité unitaire.
b) l’élasticité-prix croisée de la demande
d) a une élasticité égale à 0,5.
implique des biens complémentaires.
e) a une élasticité égale à 2.
c) l’élasticité-prix de la demande implique
un bien inférieur. I5 Monique et Bernard travaillent pour la même
d) l’élasticité-revenu de la demande implique maison de disques. Monique affirme qu’ils
un bien normal. auraient intérêt à augmenter le prix de leurs
e) l’élasticité-revenu de la demande implique disques compacts, mais Bernard est convaincu
une erreur de calcul de votre part. qu’ils auraient, au contraire, intérêt à le baisser.
Nous pouvons en conclure que
I I Une baisse des frais de scolarité fera baisser a) Monique pense que l’élasticité-prix de
la recette totale de l’université si l’élasticité-prix la demande de DC est égale à 0 et Bernard
de la demande des études universitaires est pense quelle est égale à 1.
a) négative. b) Monique pense que l’élasticité-prix de
b) supérieure à 0, mais inférieure à 1. la demande de DC est égale à 1, et Bernard
c) égale à 1. pense quelle est égale à zéro.
d) supérieure à 1. c) Monique pense que la demande de DC est
e) inférieure à l’élasticité-prix de l’offre. élastique en fonction du prix et Bernard pense
quelle est inélastique en fonction du prix.
d) Monique pense que la demande de DC
est inélastique en fonction du prix et Bernard
pense quelle est élastique.
e) Monique et Bernard feraient mieux d’écouter
des disques, sans s’occuper d’économie.
L'ÉLASTICITÉ 59

16 L’élasticité-prix croisée de la demande de balles 21 Si une diminution de 4 % du revenu (le prix


de tennis blanches par rapport au prix des balles étant constant) entraîne une diminution
de tennis jaunes est probablement de 2 % de la consommation des grille-pain,
a) négative et importante. alors
b) négative et faible. a) l’élasticité-revenu de la demande de grille-
c) positive et importante. pain est négative.
d) positive et faible. b) les grille-pain sont des biens nécessaires
e) nulle. et normaux.
c) les grille-pain sont des biens de luxe
17 Les préférences pour les choux de Bruxelles
et normaux.
augmentent. Le prix des choux de Bruxelles
d) les grille-pain sont des biens inférieurs.
ne changera pas si l’élasticité de
e) a et d sont justes.
a) la demande est égale à 0.
b) la demande est égale à 1.
22 La courbe d’offre à long terme sera probablement
c) l’offre est égale à 0.
a) plus élastique que l’offre instantanée,
d) l’offre est égale à 1.
mais moins élastique que l’offre à court
e) l’offre est infinie.
terme.
18 Un dirigeant syndical qui affirme que b) moins élastique que l’offre instantanée,
des « salaires plus élevés améliorent le niveau mais plus élastique que l’offre à court
de vie sans entraîner de chômage » pense que terme.
la demande de travail est c) moins élastique que la courbe d’offre
a) élastique en fonction du revenu. instantanée et que la courbe d’offre
b) inélastique en fonction du revenu. à court terme.
c) parfaitement élastique. d) plus élastique que la courbe d’offre instantanée
d) parfaitement inélastique. et que la courbe d’offre à court terme.
e) à élasticité unitaire. e) verticale.

I9 Les produits de luxe ont tendance à avoir


23 Des gens d’affaires parlent de l’élasticité-prix
une élasticité-revenu de la demande
de la demande sans réellement la nommer.
a) supérieure à 1.
Lequel des énoncés suivants reflète la demande
b) supérieure à 0, mais inférieure à 1.
élastique d’un produit ?
c) positive.
a) « Une baisse de prix ne m’aidera pas. Elle
d) négative.
ne fera pas augmenter les ventes et je gagnerai
e) d’abord positive, ensuite négative, à mesure
moins d’argent par unité qu’avant. »
que s’accroît le revenu.
b) «Je ne crois pas qu’une baisse de prix modifiera
20 En se fondant sur la relation entre la recette mes résultats. Ce que je pourrais gagner
totale provenant de la vente d’un bien et la en vendant plus, je le perdrais à cause
quantité de ce bien vendue (voir la figure 5.1), de cette baisse. »
on peut affirmer c) «Mes clients sont toujours à la recherche
a) qu’il s’agit d’un bien inférieur. d’aubaines. Depuis que mon prix est inférieur
b) qu’il s’agit d’un bien normal. de quelques cents à celui de mes concurrents,
c) que l’élasticité de la demande est égale à 0. mon magasin ne désemplit pas et les ventes
d) que l’élasticité de la demande est infinie. montent en flèche. »
e) que l’élasticité de la demande est égalé a 1. d) « Grâce à la récente reprise économique, les
consommateurs ont plus d’argent et les ventes
FIGURE 5.1 montent en flèche. »
JD
e) Aucune de ces réponses.
O
O

<D
24 Si les Jets baissent le prix des billets et découvrent
que cette baisse n’influe pas sur la recette
totale, alors l’élasticité-prix de la demande de
billets est
a) égale à 0.
b) supérieure à 0, mais inférieure à 1.
c) égale à 1.
0 Quantité vendue d) supérieure à 1.
e) inférieure.
60 CHAPITRE 5

25 L’élasticité à long terme de l’offre d’un bien est


FIGURE 5.2
supérieure à l’élasticité à court terme de l’offre
parce que
a) il y a un plus grand nombre de substituts
de consommation pour ce produit.
b) il y a un plus grand nombre de compléments
de consommation pour ce produit.
c) il est possible d’exploiter un plus grand
nombre de moyens techniques qui permettent
d’ajuster l’offre.
d) le revenu augmente plus le temps s’écoule.
e) la courbe d’offre à long terme est plus abrupte.

Problèmes à court développement

1 Pourquoi est-il préférable d’utiliser l’élasticité


utilisée comme mesure de l’effet d’une variation
5 Considérez la courbe de demande de la figure
de prix sur la quantité demandée plutôt que
5.3(a). Le barème de demande indiqué au
la pente ?
tableau 5.2 décrit également une portion de
la courbe de demande.
2 Dans chacun des cas suivants, comparez
l’élasticité de la demande de chaque ensemble de
FIGURE 5.3
biens et expliquez pourquoi la demande de l’un
des biens est plus élastique que celle de l’autre.
a) Les ordinateurs personnels IBM avant la mise
au point d’autres ordinateurs personnels
compatibles par rapport aux ordinateurs
personnels IBM après la fabrication d’autres
ordinateurs compatibles.
b) Les téléviseurs par rapport aux allumettes.
c) L’électricité juste après une augmentation
de son prix par rapport à l’électricité deux ans
après cette augmentation.
d) L’acétaminophène par rapport à une marque
déposée d’acétaminophène, comme Tylénol.

3 Pourquoi l’offre a-t-elle tendance à être plus Quantité

élastique à long terme ?

4 Dans la figure 5.2, laquelle de ces deux courbes


de demande (DÂ ou DB) est plus élastique dans
la fourchette de prix Px à P2 ? Justifiez votre
réponse. (Conseil: Utilisez la formule de
l’élasticité-prix de la demande.)

Quantité
L'ÉLASTICITÉ 6f

TABLEAU 5.2 b) Supposons que le revenu augmente et qu’il


passe de 10 000 $ à 14 000 $, entraînant

Prix une augmentation de la demande : la quantité


Quantité Recette
(en dollars) demandée augmente de 2 unités pour chaque
demandée totale
prix. Tracez la nouvelle courbe de demande
22 0 sur la figure 5.3(a) et nommez-la D'. Utilisez
cette nouvelle courbe de demande pour
20 1 remplir les deux dernières colonnes du
tableau 5.3 pour T)' (la nouvelle élasticité-
18 2
prix de la demande) et ART'(en dollars)
(la nouvelle variation de la recette totale).
16 3
c) En utilisant une fourchette de prix entre
14 4 16 $ et 14 $, expliquez pourquoi D' est
plus inélastique que D.
12 5 d) Calculez Félasticité-revenu de la demande,
en supposant que le prix reste constant à 12 $.
10 6 S’agit-il d’un bien inférieur ou d’un bien
normal ? Expliquez pourquoi vous pourriez
8 7 répondre à cette question sans même calculer
l’élasticité-revenu de la demande.
6 8

e® 7 L’équation de la demande de bidules est


Inscrivez les chiffres qui correspondent
P = 60 — 2 Qa- L’équation de l’offre de bidules
à la recette totale dans la dernière colonne
est P = 32 + 5Q0> P étant le prix d’un bidule
du tableau 5.2 et tracez le graphique
en dollars, Qd la quantité de bidules demandée
de vos résultats à la figure 5.3 (b). Décrivez et Q0, la quantité de bidules offerte.
la forme de la courbe de recette totale à mesure a) Calculez f élasticité-prix de la demande
que la quantité s’accroît (le prix diminue). de bidules entre Qo = 4 et Qo = 6.
b) Tout d’abord, le marché des bidules est en
6 En vous servant de la même information que équilibre avec P* = 52 et Q* = 4. Ensuite,
pour la courbe de demande dont il est question la courbe d’offre se déplace (la courbe
au problème à court développement 5 : de demande reste constante) et produit
a) Complétez les deuxième et troisième colonnes un nouveau prix d’équilibre de P* = 48.
du tableau 5.3: T| (l’élasticité-prix de la En utilisant toutes les informations
demande) et ART (variation de la recette pertinentes données précédemment, présentez
totale), à mesure que le prix diminue, allant deux méthodes distinctes permettant de
du prix le plus élevé au prix le moins élevé. déterminer si la recette totale augmente,
Décrivez la relation entre l’élasticité et diminue ou reste constante en se déplaçant de
la variation de la recette totale à mesure que l’équilibre initial vers un nouvel équilibre.
le prix diminue (en descendant sur la courbe
de demande). Supposons que Jeanne perde son emploi et que
son revenu mensuel passe de 10 000 $ à 6 000 $,
TABLEAU 5.3 alors que ses achats mensuels de gruau passent
de 200 sacs à 400 sacs.
AP ri ART X]' ART' a) Calculez l’élasticité-revenu de sa demande
(en dollars) (en dollars) (en dollars) de gruau.
b) Le gruau est-il un bien inférieur ou un bien
16-14
normal ? Expliquez pourquoi.

14- 12

12- 10

10-8

8-6
62 CHAPITRE 5

© 9 L’équation de la demande des trampolines est


RÉPONSES
P = 50 - IQû- L’équation de l’offre de
trampolines est P — 20 + 0,5 Qo> P étant le prix
d’un trampoline en dollars, Qd, la quantité Vrai/Faux/Incertain
de trampolines demandée, et Qa, la quantité (Justifiez votre réponse.)
de trampolines offerte. Le marché des trampolines
est initialement en équilibre, et le revenu est 1 F Réponse de la Qd à la AP.
de 180 $. 2 F L’élasticité T avec le temps écoulé.
a) Quelle est la quantité d’équilibre (Q*) 3 F Pente constante ; T| X à mesure que l’on
des trampolines ? descend le long de la courbe de demande.
b) Quel est le prix d’équilibre (P*) d’un 4 V Pour vous les colas sont des substituts.
trampoline ? 5 V X proportion du revenu consacrée au bien,
c) À la suite de la baisse du revenu qui est demande plus inélastique.
maintenant de 120 $, la courbe de demande 6 V Définition plus étroite —> davantage
de trampolines s’est déplacée (la courbe de substituts.
d’offre est restée la même). La nouvelle <•) 7 I Un bien inférieur a une élasticité-revenu
équation de la demande est : négative ; le signe de l’élasticité croisée varie
selon qu’il s’agit d’un complément ou
P= 110- lQo
d’un substitut.
Utilisez cette information pour calculer la <J) 8 F TL positive, les biens sont donc des substituts.
nouvelle quantité d’équilibre des trampolines ; 9 V J- O —> î P —> î recette totale, la demande
calculez le nouveau prix d’équilibre d’un est donc inélastique.
trampoline. 10 F (% T P) > (% -1 Q), la recette totale
d) Calculez l’élasticité-revenu de la demande (P X Q) T. La demande est inélastique.
de trampolines. Les trampolines sont-ils
un bien normal ou un bien inférieur ?
(Conseil: 1) veillez à utiliser l’information sur
Questions à choix multiple
les conditions d’équilibre initiales (prix,
1 b La courbe de demande est verticale donc
quantité et revenu) ; 2) lorsque vous calculez
si P —> pas de À quantité demandée.
l’élasticité en fonction du revenu, n’oubliez
2 b | (—10/50)/(2/20) | = 2.
pas de supposer que le prix est constant:
seuls le revenu et la quantité changent.)
3 c Substituts immédiats —> T élasticité
de la demande.
4 b Définition.
10 Le tableau 5.4 présente les barèmes de demande
5 d Déplacement de l’offre vers la droite
du bien A lorsque le prix du bien B (PB) est
de 8 $ et de 12 $. Remplissez la dernière colonne
—> T quantité vendue et X P, avec
du tableau en calculant l’élasticité-prix de la
demande inélastique -A X recette totale.
6 a Définition.
demande entre les biens A et B pour chacun
7 a r| = (% À Qd) / (% A revenu) = 5/10 = 0,5.
des trois prix de A. Les biens A et B sont-ils
8 c T| = |(% A Qd)/(% A P)| = 2 = |2/-l|.
des compléments ou des substituts ?
Qd et P sont toujours inversement reliés sur
la courbe de demande.
TABLEAU 5.4 BARÈMES DE DEMANDE
9 b rit avec T proportion du revenu consacrée
DU BIEN A
au bien. Tl T lorsque plus de temps s’est
écoulé et qu’il est plus facile d’obtenir des
PB = 8$ PB= 12$
substituts.
Pa Qa Q) 1x 10 b T|0 et r) ne sont jamais négatives. T)y est
négative dans le cas d’un bien inférieur.
±p>
CO

2 000 4 000
11 b X P —> t recette totale lorsque T) < 1.

7$ 4 000 6 000 r) n’est jamais négative.


12 a r)0 = ( % A Qo) / ( % A P)
6$ 6 000 8 000 = (2 000/10 000)/(l/2) = 0,4.
13 b i O -A i Qo et T P. Étant donné que T) > 1,
T P —> X dépenses.
& 14 a Si t R -A X recette totale, alors r) doit
être > 1. Doit connaître (% À Qp) pour
calculer T) avec précision.
L'ÉLASTICITÉ 63

15 d « Ils auraient intérêt » signifie T recette totale. c) La demande d’électricité, après une période
Par définition, relation entre T P, ri et recette de deux ans, sera plus élastique car les
totale. consommateurs auront eu plus de temps
16 c Substituts immédiats de sorte que T|x est pour trouver des substituts, par exemple,
positive et très élastique (élevée). la cuisinière à gaz.
17 e T D —> pas de A P si la courbe d’offre est d) La demande de Tylénol est plus élastique.
horizontale (r)0 = infini). Il y a beaucoup plus de substituts de Tylénol
18 d La courbe de demande de travail serait (autres marques d’acétaminophène) que
verticale. de l’acétaminophène générique.
19 a Voir la réponse à la question 21 ci-dessous,
c est correct, mais a est une meilleure réponse. 3 L’offre est plus élastique à long terme parce
& 20 e Remarquez la recette totale (RT) sur l’axe des qu’avec le passage du temps, les producteurs
y. Etant donné que RT est constante à mesure pourront trouver de meilleurs moyens (plus
que Q T (et on suppose que P-l), T) = 1. a, b efficaces) de production que ceux qui sont
dépendent de r)r disponibles à court terme. La réponse augmentera
& 21 b T| > 0, le bien est donc normal. Les produits puisque les entreprises auront le temps de
essentiels ont tendance à avoir T(y < 1, alors découvrir et de mettre en œuvre de nouvelles
que les produits de luxe ont tendance à avoir techniques ou d’augmenter leur production.
T\y > 1 •
22 d Définition. La courbe d’offre instantanée 4 Da est plus élastique que DB. Pour voir
est verticale. pourquoi, considérez la formule de l’élasticité-
23 c Petite si prix —> grande T quantité demandée, prix de la demande :
a inélastique, b à élasticité unitaire,
d élasticité-revenu. % A quantité demandée
24 c Calcul de la recette totale. % A du prix
25 c Définition, a, b, d, effet sur la demande et
pas sur l’offre. La courbe d’offre à long terme Le pourcentage de variation du prix est le même
est plus élastique que la courbe d’offre à pour les deux courbes de demande. Toutefois, le
court terme. pourcentage de variation de la quantité est plus
grand pour DA. Avec Px, la quantité initiale
demandée est la même pour les deux courbes de
Problèmes à court développement
demande (Qj). Avec la baisse de prix à P2,
l’augmentation de la quantité demandée est plus
1 La pente d’une courbe de demande indique la
grande pour DA (vers QjA) que pour DB (vers
conséquence de l’effet d’une variation de prix
Qis). Par conséquent, DA est plus élastique
sur la quantité demandée. Toutefois, la valeur
que Db.
numérique de la pente dépend des unités qu on
a utilisées pour mesurer le prix et la quantité,
FIGURE 5.2SOLUTION
et elle changera si l’on change l’unité de mesure,
même si la demande ne change pas. Si on
change l’unité de mesure de la quantité,
par exemple, et qu’on prend des kilogrammes au
lieu de tonnes, la nouvelle valeur de la pente de
la (même) courbe de demande sera 1 000 fois
supérieure à celle de l’ancienne pente.
Par ailleurs, l’élasticité donne une mesure,
indépendante de l’unité choisie, de la sensibilité
d’une variation du prix à la quantité demandée.

2 a) La demande d’ordinateurs personnels IBM


sera plus élastique après la production
d’ordinateurs personnels compatibles étant
donné qu’il y aura plus de substituts
immédiats disponibles,
b) La demande de téléviseurs sera plus
élastique étant donné qu ils représenteront 5 Au tableau 5.2 Solution, on présente les chiffres
de la recette totale. La figure 5.3(b) Solution
une plus grande proportion du revenu
illustre la courbe de recette totale.
du consommateur.
64 CHAPITRE 5

Celle-ci commence par augmenter (à un taux


TABLEAU 5.3 SOLUTION
décroissant - sa pente décroît à mesure que
la courbe monte), et atteint son maximum
AP 9 ART 9' ART'
entre Q = 5 et Q = 6 pour diminuer ensuite.
(en dollars) (en dollars) (en dollars)
Nous retrouverons ce type de variation de la
recette totale au chapitre 12 lorsque nous 16 - 14 2,14 +8 1,36 +4
étudierons le monopole. 14- 12 1,44 +4 1,00 0
12- 10 1,00 0 0,73 -4
10-8 0,69 -4 0,53 -8
TABLEAU 5.2 SOLUTION 8-6 0,47 -8 0,37 -12

Prix Quantité Recette b) La nouvelle courbe de demande est nommée


(en dollars) demandée totale D’a la figure 5.3(a) Solution. Les deux
dernières colonnes du tableau ont été remplies,
22 0 0
compte tenu de la nouvelle courbe de
20 1 20
demande.
18 2 36
16 3 48
FIGURE 5.3(a) SOLUTION
14 4 56
12 5 60
10 6 60
8 7 56
6 8 48

FIGURE 5.3(b) SOLUTION

Quantité

c) Étant parallèles, D' et D ont exactement


la même pente. De ce fait, nous savons que,
pour une variation de prix donnée, la variation
de la quantité demandée sera identique pour
les deux courbes. Toutefois, l’élasticité est
déterminée par les pourcentages de variation,
et le pourcentage de variation de la quantité
demandée est différent pour les deux courbes,
même si le pourcentage de variation du prix
est le même. Pour un pourcentage de variation
Quantité du prix donné, le pourcentage de variation
de la quantité demandée sera toujours inférieur
pour D'. Par exemple, lorsque le prix tombe
6 a) Nous voyons ici les colonnes du tableau 5.3 de 16 $ à 14 $ (une variation de 13%), la
dûment remplies. Les deuxième et troisième quantité demandée grimpe de 5 à 6 unités
colonnes du tableau indiquent que la recette le long de D', mais de 3 à 4 unités le long
totale augmente en fonction de la baisse de D. Le pourcentage de variation de la
du prix lorsque la demande est élastique ; quantité demandée n’est que de 18% le long
la recette totale reste constante lorsque de D' et de 29 % le long de D. Comme
la demande est à élasticité unitaire ; la recette le pourcentage de variation du prix est le
totale diminue lorsque la demande est même pour les deux courbes, D' est plus
inélastique. inélastique que D.
L'ÉLASTICITÉ 65

d) Le revenu augmente, passant de 10 000 $ à Nouvel équilibre:


14 000 $. Au prix constant de 12 $, la hausse En utilisant P* — 48, nous pouvons calculer
de revenu, qui déplace la courbe de demande Q* dans l’équation de la demande :
vers D', fait grimper de 5 à 7 unités la
48 = 60 - 2 Q*
quantité que les consommateurs demanderont.
2Q* = 12
En substituant ces chiffres dans la formule
Q*= 6
de l’élasticité de la demande par rapport
au revenu, on obtient : Donc, la recette totale = (P X Q) — (48 X 6)
= 288

/
<i
cy

<1
En se déplaçant de l’équilibre initial vers un
v^ o / \ Ym°y y nouvel équilibre, la recette totale a augmenté.
Une seconde méthode consiste à utiliser
(2') / f 4 000 ^ les informations de la partie a sur l’élasticité-
U J/ Q2 000 J prix de la demande. Nous savons que la
demande est élastique entre Qd = 4 et Qn — 6
L’élasticité-revenu de la demande est un sur la courbe de demande. Ces deux points
chiffre positif, étant donné que les ÀQ et AY correspondent à l’équilibre initial et, après
sont toutes deux positives. Par conséquent, le déplacement de la courbe d’offre, au nouvel
il s’agit d’un bien normal. Nous le savions équilibre. Comme la demande est ici élastique,
déjà, grâce aux informations données à la nous savons que la baisse du prix, qui passe
partie b qui indique que la courbe de demande de P— 52 à P= 48, fera augmenter la recette
se déplace vers la droite lorsque le revenu totale.
augmente. S’il s’agissait d’un bien inférieur,
la hausse du revenu aurait déplacé la courbe a) En utilisant la formule de l’élasticité-revenu
de demande vers la gauche et l’élasticité- de la demande, on obtient :
revenu de la demande aurait été négative.
AQ AY
7 a) Pour calculer l’élasticité-prix de la demande, Ç Q-moy J Ç
Ymoy y

nous devons connaître deux points (chaque


point étant une combinaison du prix 400 - 200 6 000-10 000
et de la quantité demandée) sur la courbe de X (200 + 400) j X (10 000+ 6 000)
demande. Nous ne connaissons que
la coordonnée de la quantité demandée de 200 V f-4 000^1 4
chaque point. En utilisant Qp = 4 et Qx, = 6 300 [ 8 000 J 3
dans l’équation de la demande, nous pouvons
obtenir les deux coordonnées de prix : b) Le gruau est un bien inférieur, car l’élasticité-
revenu de la demande est négative.
lorsque Qr> = 4; P= 60 - 2(4) = 52
lorsque Qb — 6; P= 60 - 2(6) = 48
@9 a) La quantité d’équilibre initiale (Q*) des
Nous pouvons alors substituer ces trampolines est 20. (Consultez le chapitre 4
coordonnées des prix et des quantités du Guide de l’étudiant, si vous avez besoin
demandées dans la formule de l’élasticité-prix d’aide en ce qui concerne les équations
de la demande : de l’offre et de la demande).
b) Le prix d’équilibre initial (P*) d’un trampoline

il = { 1
/ ( AP \ est de 30 $. (Consultez le chapitre 4
du guide, si vous avez besoin d’aide
. Q~moy j/ y
Pmoy y
en ce qui concerne les équations de l’offre
et de la demande).
v fi-1 c) La nouvelle quantité d’équilibre des
J/ J .50 trampolines est de 60. Le nouveau prix
d’équilibre d’un trampoline est de 50 $.
b) Selon une première méthode, nous pouvons
® d) Avec un revenu initial de 180 $ et un prix
tout simplement comparer la recette totale
de 30 $, la quantité de trampolines demandée
(Px Q) pour chaque équilibre.
est de 20. Pour trouver la nouvelle quantité
demandée avec un revenu de 120 $,
Équilibre initial-.
Recette totale = (PX Q) = (52 X 4) = 208 nous devons utiliser la nouvelle équation
66 CHAPITRE 5

de demande. Mais comme nous supposons 10 Les élasticités croisées de la demande entre A
que le prix est constant lorsque nous et B sont indiquées au tableau 5.4 Solution.
calculons l’élasticité de la demande par Comme les élasticités croisées sont positives,
rapport au revenu, nous devons utiliser nous savons que A et B sont des substituts.
P— 30 dans la nouvelle équation de demande
pour obtenir la nouvelle quantité. Il serait TABLEAU 5.4 SOLUTION
incorrect d’utiliser la nouvelle quantité BARÈMES DE DEMANDE
d’équilibre, car la quantité correspond DU BIEN A
à un prix différent (50 $).

00
II
PB= 12$


P= 110- 1
30 = 110 - lQn PA Qa Q'a Tlx
00 = 80
8$ 2 000 4 000 1,67
Nous avons maintenant la bonne réponse
7$ 4 000 6 000 1,00
en ce qui concerne la quantité initiale
6$ 6 000 8 000 0,71
demandée (20) et le revenu (180 $), et en ce
qui concerne la nouvelle quantité demandée
(80) et le revenu (120 $). En appliquant cette
donnée dans la formule de l’élasticité-prix de
la demande nous obtenons :

f AQ ' /
<i

yQmoy
/ Y, y moy y

f_60l /{
CN
°

l J /{
1

50 150 J

Étant donné que l’élasticité-revenu de


la demande des trampolines est négative,
les trampolines sont un bien inférieur.
Les marchés en action

♦ Le salaire minimum entraîne une offre excédentaire


CONCEPTS CLÉS de main-d’œuvre et réduit la quantité de main-
d’œuvre demandée et embauchée.
Le marché du logement ♦ Les travailleurs au chômage qui sont prêts
et le plafonnement des loyers à travailler à un salaire inférieur passent plus
de temps à chercher du travail.
Dans un marché du logement non réglementé
♦ Le salaire minimum contribue au taux élevé
♦ Pénurie —> T loyers —> taux d’occupation plus
de chômage chez les jeunes travailleurs non
élevé, t quantité offerte (à court terme) et activité
qualifiés.
de construction font déplacer la courbe d’offre
vers la droite (à long terme).
Les taxes
♦ Loyers si et quantité de logements offerte T.
Dans un marché du logement réglementé Les taxes de vente font déplacer vers le haut
la courbe d’offre d’une distance verticale égale
♦ Un plafonnement des loyers (prix plafond au montant de la taxe. L’élasticité de l’offre et
ou maximum) rend illégale î des loyers. l’élasticité de la demande déterminent qui paie
♦ La quantité de logements offerte est moins la taxe.
importante que dans un marché non réglementé
Elasticité de l’offre et partage de la taxe
à long terme et à court terme. Il n’existe aucune
incitation quant à l’augmentation du taux ♦ Offre parfaitement inélastique - les vendeurs
d’occupation ou à la construction de nouveaux paient intégralement la taxe.

logements. ♦ Offre plus inélastique - les vendeurs paient


♦ Pénurie —> demande excédentaire —> une part plus élevée de la taxe.

• activité de prospection (temps passe a chercher ♦ Offre plus élastique - les acheteurs paient une part
un fournisseur avec lequel s’entendre) et plus élevée de la taxe.
• marchés noirs (contrat d’échange illégal en
♦ Offre parfaitement élastique - les acheteurs paient
vertu duquel les prix pratiqués sont supérieurs
intégralement la taxe.
aux plafonds fixés par la loi).
Élasticité de la demande et partage de la taxe
♦ Le coût total de logement dans un marché
réglementé, en comprenant le coût de 1 activité de ♦ Demande parfaitement inélastique - les acheteurs
prospection, peut être > au coût pratique dans un paient intégralement la taxe.
marché non réglementé.
♦ Demande plus inélastique - les acheteurs paient
une part plus élevée de la taxe.
Le marché du travail
et le salaire minimum ♦ Demande plus élastique - les vendeurs paient
une part plus élevée de la taxe.
Les lois sur le salaire minimum stipulent qu il est
illégal d’engager de la main-d œuvre a un tarif inférieur ♦ Demande parfaitement élastique — les vendeurs
paient intégralement la taxe.
au tarif fixé par la loi.
68 CHAPITRE 6

Les marchés des produits illégaux prix sont contrôlés par l’État, le mécanisme
des prix ne peut plus remplir cette fonction.
Les sanctions imposées aux vendeurs d’un produit M’équilibre doit donc être rétabli d’une autre
illégal —> T prix de vente de ce produit et si l'offre. manière. S’il y a plafonnement des prix, les
Les sanctions imposées aux acheteurs de produits marchés noirs vont probablement se développer.
illégaux —> si propension à payer et 1 la demande. S’ils ne peuvent le faire, en raison d’une
application rigoureuse du plafonnement des prix,
♦ (Sanctions imposées aux vendeurs > sanctions les demandeurs seront forcés de supporter le coût
imposées aux acheteurs) —> si Q, T P. d’activités de prospection plus soutenues,
♦ (Sanctions imposées aux vendeurs < sanctions d’attendre en file, etc.
imposées aux acheteurs) —> i Q, -I- P.
2 Ce chapitre nous permet d’étudier les limites
♦ Taxer (décriminaliser) les produits peut donner
de prix imposées par l’État dans trois marchés
le même résultat que les interdire. précis : le marché du logement, le marché
du travail et le marché des produits agricoles.
La stabilisation des revenus agricoles Toutefois, les principes soulevés dans ces cas
peuvent s’appliquer à d’autres marchés.

Dans le cas de la plupart des produits agricoles, Dans tout marché assujetti à un prix plafond

la demande est inélastique —> de fortes variations inférieur au prix d’équilibre, il y a demande

des revenus agricoles. excédentaire puisqu’on ne peut augmenter


le prix pour l’éliminer. Par conséquent, la valeur
♦ Mauvaise récolte (si O) —> T P, T revenu total. de la dernière unité disponible du bien sera
♦ Récolte abondante (T O) —> 4- P, -l revenu total. supérieure au prix réglementé. Les demandeurs
sont prêts à se lancer dans des activités coûteuses
Par la spéculation, les détenteurs de stocks qui achètent correspondant à la valeur de cette dernière unité
à bas prix et vendent à prix fort permettent de stabiliser (activité de prospection, files d’attente, marché
les revenus agricoles. noir) pour pouvoir se procurer ce bien.

Les organismes de stabilisation des revenus (régies En outre, s’il est permis d’augmenter le prix
des marchés agricoles) limitent les fluctuations pour répondre à une diminution de l’offre ou à
des prix en fixant une augmentation de la demande, les fournisseurs
seront incités à produire davantage et les
♦ des prix plancher —> T du prix du marché demandeurs à acheter moins (mouvements
et des surplus. le long des courbes d’offre et de demande). En
♦ des quotas (restrictions imposées à la quantité fait, c’est la réaction à ces incitations qui restaure
produite) —> T P et -i Q produite. l’équilibre dans les marchés où les prix s’ajustent
librement. Toutefois, si le prix ne peut s’ajuster,
♦ des subventions (paiements versés aux producteurs ces effets d’incitation, induits par le prix, ne
par les gouvernements) —»? offre et ? le prix peuvent s’exercer. Dans le cas du plafonnement
du marché (mais les agriculteurs reçoivent le prix des loyers en particulier, l’impossibilité
du marché + la subvention). d’augmenter les loyers (prix) signifie que :
1) rien n’incite à utiliser plus intensivement
le stock de logements actuel à court terme ;
2) rien n’incite à construire des logements à long
RAPPELS terme. Dans le même ordre d’idées, les effets
sur tout marché où le prix minimum (prix
I Dans le monde réel, les gouvernements plancher) est supérieur au prix d’équilibre seront
interviennent fréquemment dans le marché semblables aux effets que nous avons étudiés
en réglementant les prix de différentes manières. à propos du salaire minimum, dans le cas
Il est donc important d’étudier les effets de cette du marché du travail (voir le manuel) ou des
réglementation. Cela nous permettra également prix planchers, dans le cas des marchés agricoles.
de comprendre comment fonctionnent
les marchés lorsque le gouvernement ^intervient 3 On appelle également incidence fiscale le fardeau
pas dans les activités normales des marchés. du partage de la taxe. Les élasticités respectives
Chaque fois qu’un événement perturbe de l’offre et de la demande déterminent
l’équilibre d’un marché déréglementé (libre), les qui paiera une taxe de vente imposée aux
actions des acheteurs et des vendeurs s’équilibrent producteurs. Les principes généraux de
grâce aux mouvements des prix. Toutefois, si les l’incidence fiscale sont :
LES MARCHÉS EN ACTION 69

• Plus la demande est inélastique, plus la part 7 Supposons que la courbe d’offre de maïs fluctue
que paient les consommateurs est élevée. considérablement, mais que la courbe de
• Plus la demande est élastique, plus la part demande reste stable. Le prix du maïs fluctuera
que paient les producteurs est élevée. davantage si la demande est élastique plutôt
• Plus l’offre est inélastique, plus la part qu’inélastique.
que paient les producteurs est élevée.
• Plus l’offre est élastique, plus la part que paient
les consommateurs est élevée.
8 Si le prix est supérieur au prix futur anticipé,
le détenteur des stocks les vendra.

AUTOÉVALUATION

9 Si des sanctions sont imposées aux acheteurs


Vrai/Faux/Incertain et aux vendeurs dans un marché de produits
(Justifiez votre réponse.) illégaux, le prix reste constant et la quantité
de produits achetée diminue.
I Lorsque les loyers d’un marché du logement
déréglementé augmentent en raison d’une
diminution de l’offre, les consommateurs
qui ne peuvent payer les loyers plus élevés 10 Une subvention fait déplacer la courbe
ne pourront se loger. de demande d’un bien vers la droite.

2 Plus la demande d’un produit est élastique,


plus la part de la taxe de vente payée par Questions à choix multiple
les consommateurs est élevée.
1 La courbe d’offre à court terme des logements
locatifs a une pente positive parce que
a) l’offre de logements est fixe à court terme.

3 Lorsqu’un plafond des loyers est supérieur au b) le taux d’occupation des logements augmente

montant qu’un consommateur est prêt à payer, à mesure que le loyer augmente.

les activités de prospection et de marché noir c) le coût de construction de nouveaux


logements augmente à mesure que leur
augmentent.
nombre augmente.
d) le coût de construction de nouveaux
logements est à peu près le même, quel
que soit le nombre de logements existants.
4 L’énoncé «si nous légalisons et taxons les drogues,
e) de nouveaux logements seront construits
les recettes provenant de leur vente pourraient
à mesure que le prix des loyers augmente.
être utilisées pour financer des programmes
de sensibilisation aux dangers des drogues»
2 Le plafonnement des loyers imposé par les
est un énoncé normatif.
gouvernements
a) maintient le prix des loyers au-dessous
du prix en vigueur dans le marché non
réglementé.
5 Une augmentation du salaire minimum fera
b) maintient le prix des loyers au-dessus du prix
diminuer le nombre de travailleurs employés.
en vigueur dans le marché non réglementé.
c) maintient le prix des loyers égal au prix
en vigueur dans le marché non réglementé.
d) augmente le stock de logements locatifs.
6 Les effets sur le chômage des lois sur le salaire
e) augmente le taux d’occupation du stock
minimum ont tendance à être les mêmes chez
de logements locatifs courant.
les jeunes que chez les autres travailleurs.
70 CHAPITRE 6

3 La figure 6.1 présente le marché des frisbees 7 D’après la figure 6.1, nous pouvons déduire
avant et après l’imposition d’une taxe. La taxe que, entre 4 000 et 5 000 unités, la demande
de vente de chaque frisbee est de : de frisbee
a) 0,40 $. a) est inélastique.
b) 0,60 $. b) est à élasticité unitaire.
c) 1,00$. c) est élastique.
d) 5,60 $. d) est plus élastique que l’offre (O) de frisbees.
e) 6,60 $. e) monte en flèche.

FIGURE 6.1 8 Les pays de l’Union européenne (UE) ont


accumulé des quantités immenses de beurre,
de fromage et de vins. Ces surplus sont
conformes aux
a) prix plancher fixés pour les produits agricoles
qui sont inférieurs aux prix du marché.
b) prix plancher fixés pour les produits agricoles
qui sont supérieurs aux prix du marché.
c) prix plafond fixés pour les produits agricoles
qui sont inférieurs aux prix du marché.
d) prix plafond fixés pour les produits agricoles
qui sont supérieurs aux prix du marché.
e) quotas fixés pour les produits agricoles.

9 Une courbe d’offre instantanée est


Quantité de frisbees (en milliers) a) élastique puisque le prix est fixe.
b) relativement plus élastique qu’une courbe
d’offre à court terme.
c) relativement inélastique puisque la quantité
4 Reportez-vous à la figure 6.1. Pour chaque disponible est connue, mais pas le volume
frisbee, la part de la taxe payée par le vendeur des ventes.
est de : d) inélastique puisque la production est fixe.
a) 0,40 $. e) éphémère.
b) 0,60 $.
c) 1,00$. 10 Un prix plafond inférieur au prix d’équilibre
d) 5,60 $. entraînera
e) 6,60 $. a) une offre excédentaire.
b) une demande excédentaire.
5 Reportez-vous à la figure 6.1. Pour chaque c) le prix d’équilibre.
frisbee, la part de la taxe payée par l’acheteur d) une augmentation de l’offre.
est de : e) une diminution de la demande.
a) 0,40 $.
b) 0,60 $. 11 Si le salaire minimum est fixé à 2 $ l’heure
c) 1,00$. dans la figure 6.2, quel est, exprimé en millions
d) 5,60 $. d’heures, le niveau du chômage ?
e) 6,60 $.
a) 50
b) 40
6 Reportez-vous à la figure 6.1. Dans ce cas, c) 20
la recette fiscale de l’Etat est de : d) 10
a) 4 000 $.
e) 0
b) 5 000 $.
c) 22 400 $.
d) 26 400 $.
e) 30 000 $.
LES MARCHÉS EN ACTION 71

FIGURE 6.2 15 Quel énoncé, parmi les suivants, est faux?


a) Acheter des biens pour les mettre en stock
revient à diminuer la quantité offerte.
b) Vendre des biens en stock revient à augmenter
la quantité offerte.
c) Dans un marché des stocks, l’offre
des producteurs diffère de l’offre du marché.
d) L’offre des producteurs correspond à la somme
de l’offre du marché et de l’offre de stocks.
e) Les détenteurs de stocks doivent prévoir
les prix futurs.

16 Quel résultat, parmi les suivants, rîtst pas


un résultat probable du plafonnement des loyers ?
a) Un marché noir des logements à loyer
Quantité de travail (en millions d'heures) contrôlé.
b) De longues listes d’attente de locataires qui
cherchent des logements à loyer contrôlé.

12 Si on spécule sur les stocks, une récolte c) Une pénurie à court terme de logements.

abondante d’orge fera d) Des prix du marché noir inférieurs aux prix

a) diminuer les revenus agricoles, alors qu’une plafonds des loyers.

mauvaise récolte les fera augmenter. e) Une intensification des activités de

b) augmenter les revenus agricoles, alors qu’une prospection de logements à loyer contrôlé.

mauvaise récolte les fera diminuer.


c) diminuer les revenus agricoles tout autant 17 La figure 6.3 illustre les courbes de demande et
d’offre à court terme dans le marché du son. Si
qu’une mauvaise récolte.
la Commission canadienne du blé fixe un quota
d) augmenter les revenus agricoles tout autant
de 50 millions de boisseaux, quel sera le prix
qu’une mauvaise récolte.
d’équilibre d’un boisseau de son ?
e) Aucune de ces réponses.
a) 1,00$.
b) 3,00 $.
13 Si les vendeurs d’un produit illégal risquent
c) 5,00 $.
des sanctions
d) Deux cuillerées de raisins secs.
a) le prix et la quantité achetée diminueront.
e) Aucune de ces réponses.
b) le prix et la quantité achetée augmenteront.
c) le prix augmentera et la quantité achetée
FIGURE 6.3
diminuera.
d) le prix diminuera et la quantité achetée
augmentera.
e) la variation de prix sera incertaine,
et la quantité achetée diminuera.

14 Lorsque le prix d’un bien n’est pas touché par


la taxe de vente, alors
a) l’offre est parfaitement élastique.
b) la demande est parfaitement élastique.
c) l’élasticité de l’offre est plus grande
que l’élasticité de la demande.
d) 1 'élasticité de la demande est plus grande
que l’élasticité de l’offre.
e) Aucune de ces réponses.

0 10 20 30 40 50 60 70 80 90
Quantité de son (en millions de boisseaux)
72 CHAPITRE 6

18 Reportez-vous à la figure 6.3. Avec le quota 22 Quel type de main-d’œuvre, parmi les suivants,
de 50 millions de boisseaux, les agriculteurs serait plus touché par une augmentation du
sont incités à salaire minimum légal ?
a) diminuer la production, car le prix d’équilibre a) Athlètes professionnels.
est inférieur au coût de production. b) Jeunes travailleurs non qualifiés.
b) diminuer la production, car il y a surplus au c) Travailleurs qualifiés syndiqués.
prix d’équilibre. d) Professeurs d’université.
c) augmenter la production, car le prix e) Travailleurs autonomes.
d’équilibre est supérieur au coût de
production. 23 Quelles combinaisons, parmi les suivantes,
d) augmenter la production, car il y a pénurie au pourraient entraîner les plus grandes fluctuations
prix d’équilibre. de prix ?
e) se contenter du quota. a) Des déplacements considérables de l’offre
et une demande inélastique.
I9 Reportez-vous à la figure 6.3. Supposons que b) Des déplacements considérables de l’offre
l’Office de commercialisation établisse un et une demande élastique.
nouveau type de subvention, appelée plan de c) De larges déplacements de l’offre et une
paiement direct. On dit aux agriculteurs : « Nous demande parfaitement élastique.
vous garantirons 5 $ par boisseau, mais tout ce d) De faibles déplacements de l’offre et une
que vous produirez doit être vendu à n’importe demande inélastique.
quel prix que l’acheteur est prêt à payer. Ensuite, e) De faibles déplacements de l’offre et
nous vous payerons la différence (aux frais une demande élastique.
du contribuable) entre le prix du marché
et le montant de 5 $ par boisseau. » Combien 24 Grâce aux marchés spéculatifs des stocks
de boisseaux de son seront vendus ? a) les prix se stabilisent, mais pas les revenus
a) 20 millions agricoles.
b) 50 millions b) les prix ne se stabilisent pas, mais les revenus
c) 60 millions agricoles se stabilisent.
d) 70 millions c) les récoltes abondantes font diminuer
e) Aucune de ces réponses. les revenus.
d) les mauvaises récoltes font augmenter
20 Reportez-vous à la figure 6.3. Le montant puisé les revenus.
à même les taxes des contribuables que l’Office e) c et d.
de commercialisation paie aux agriculteurs
(question 19) est 25 Une subvention agricole
a) nul. a) fait déplacer la courbe d’offre vers la droite.
b) de 70 millions de dollars. b) profite aux consommateurs en faisant baisser
c) de 250 millions de dollars. le prix des produits.
d) de 280 millions de dollars. c) profite aux agriculteurs en faisant monter
e) de 350 millions de dollars. le prix qu’ils reçoivent.
d) impose de grosses taxes aux contribuables.
21 Lequel des énoncés suivants, relatifs aux produits e) Toutes ces réponses.
illégaux, est juste?
a) L’imposition de taxes est plus efficace que
Problèmes à court développment
l’interdiction pour modifier les préférences.
b) L’interdiction est plus efficace qu’une taxe 1 Supposons que l’offre d’essence soit
équivalente pour générer des revenus. considérablement réduite. Expliquez comment
c) Les taxes et les sanctions ne peuvent être s’ajuste un marché déréglementé. Qu’est-ce
imposées de manière à entraîner des résultats qui pousse les consommateurs à réduire
équivalents. volontairement leur consommation d’essence ?
d) Les taxes génèrent des revenus, alors que
l’interdiction implique de plus grandes 2 Les gouvernements ont tendance à imposer des
dépenses pour faire appliquer la loi. taxes de vente élevées (souvent appelées « taxes
è) Aucune de ces réponses. sur les vices ») sur les alcools et le tabac. A part
les raisons de santé et les principes moraux,
pourquoi choisit-on ces biens comme source
de recettes fiscales?
LES MARCHÉS EN ACTION 73

L’offre de fruits est assujettie à des fluctuations b) Supposons que la quantité d’essence offerte
imprévisibles. Pourquoi le prix des fruits diminue brusquement de 8 millions de
frais fluctue-t-il plus que le prix des fruits litres par jour pour chaque prix. Préparez
en conserve? un nouveau tableau des prix, de la quantité
demandée et de la quantité offerte, et tracez
Supposons que le Parti des nudistes gagne le graphique de la courbe de demande et
les prochaines élections fédérales parce que tous de la courbe initiale ainsi que de la nouvelle
les citoyens habillés ont oublié d’aller voter. courbe d’offre. Supposez que le marché
Les nudistes adoptent une loi qui rend illégal de l’essence est déréglementé, et utilisez votre
le port de vêtements. Malheureusement pour tableau ou votre graphique pour trouver
les nudistes, la police ne prend pas cette loi le nouveau prix et la nouvelle quantité
au sérieux (car où les agents de police pourraient- d’équilibre de l’essence.
ils accrocher leur insigne ?) et ne déploie pas c) Quel est l’effet de la variation de prix
beaucoup d’effort pour la faire respecter. À l’aide sur le comportement des demandeurs ?
d’un graphique, expliquez pourquoi le prix du Le comportement des producteurs ?
marché noir des vêtements, qui sont maintenant
illégaux, se rapprochera du prix d’équilibre 7 Supposons que le gouvernement impose un
prévalant dans un marché non réglementé ? prix plafond de 1 $ par litre d’essence alors
que l’offre diminue comme il est indiqué
Supposons que le marché des logements locatifs au problème 6b.
soit initialement en équilibre à long terme. a) Quelle est la quantité d’essence demandée ?
En vous servant de graphiques, répondez La quantité offerte ?
aux questions suivantes : b) Quelle est la quantité d’essence véritablement
a) Comment s’ajusterait un marché de logements vendue ?
locatifs libre si, soudain, la demande c) Quelle est la quantité d’essence excédentaire
augmentait considérablement? Quels seraient demandée ?
les effets sur les loyers et la quantité d) Quel est le prix le plus élevé que les
de logements loués à court terme et à long consommateurs sont prêts à payer pour
terme ? N’oubliez pas de parler des effets le dernier litre d’essence disponible ?
d’incitation exercés sur les producteurs e) Georges est prêt à payer le litre d’essence
(autant à court terme qu’à long terme) au prix indiqué en d. Combien de temps
à mesure que varie le prix (loyer) déterminé Georges est-il prêt à faire la queue pour
par le marché. acheter 10 litres d’essence, si la meilleure
b) Expliquez maintenant comment le marché possibilité qui lui reste est de travailler
s’ajusterait à l’augmentation de la demande, au taux horaire de 8 $ ?
si les prix plafond étaient fixés au prix
d’équilibre initial. Quels seraient les effets 8 Le Conseil du Trésor a été autorisé à percevoir
d’incitation sur les producteurs, dans ce cas ? une taxe d’accise de 0,15 $ par unité sur l’un de
ces deux biens : bandes dessinées ou biscuits
La demande et l’offre d’essence sont présentées pour chien. Vous êtes étudiant occupant un
au tableau 6.1 emploi d’été au Conseil et le directeur du fisc,
a) Quels sont le prix et la quantité d’équilibre M. Mauret, vous donne la tâche suivante :
de l’essence? choisir entre ces deux biens celui qui répond à
deux objectifs: 1) produire la recette fiscale la
TABLEAU 6.11 plus élevée, et 2) imputer la majeure partie de
cette taxe au consommateur. Les chercheurs du
Quantité Quantité Conseil ont déterminé les courbes d’offre et de
Prix demandée offerte demande (sans la taxe) de chaque marché. Le
( en dollars (en millions de litres marché des bandes dessinées est indiqué à la
par litre) par jour) figure 6.4 et le marché des biscuits pour chiens,
à la figure 6.5. Les questions suivantes vous
1,40 8 24
aideront à accomplir votre tâche.
1,30 10 22
1,20 12 20
1,10 14 18
1,00 16 16
0,90 18 14
74 CHAPITRE 6

e) Calculez la recette fiscale totale perçue


FIGURE 6.4
et indiquez-la sous la forme d’une surface
à la figure 6.5.
f) Quelle part de la taxe paient
les consommateurs ? Les producteurs ?
g) Que recommandez-vous à M. Mauret ?
Justifiez votre réponse.

9 M. Mauret a été satisfait de votre


recommandation sur la taxe d’accises
de la question précédente, mais vous ne cessez
de vous demander ce que son commentaire :
« Hum, je croyais que vous vous y prendriez
autrement ...» pouvait signifier. Si vous n’avez
pas deviné à quoi il a fait allusion, les questions
suivantes vous aideront.
Bandes dessinées (en milliers d'exemplaires)
a) Pour le marché des bandes dessinées, calculez
l’élasticité de la demande entre le point

a) Pour le marché des bandes dessinées, déplacez d’équilibre initial et le nouveau point

la courbe appropriée afin de refléter la taxe et d’équilibre. La demande est-elle élastique

tracez-la sur la figure 6.4. Nommez la courbe ou inélastique ?

soit O + taxe, soit D + taxe. Déterminez b) Faites le même calcul pour le marché

les nouveaux prix et quantité d’équilibre. des biscuits pour chiens. La demande est-elle
Comparez les dépenses totales pour l’équilibre élastique ou inélastique?

initial et le nouvel équilibre. c) Que savez-vous des élasticités de la demande


b) Calculez la recette fiscale totale perçue, qui vous auraient permis d’accomplir votre
et indiquez-la sous la forme d’une surface tâche « autrement » et de prédire quel marché
à la figure 6.4. aurait rapporté la recette fiscale maximale,
c) Quelle part de la taxe paient les et dans le cas de quel marché les
consommateurs? Les producteurs? consommateurs auraient à payer la part
de la taxe la plus élevée ?
FIGURE 6.5
10 La figure 6.6 présente la courbe de demande
du marché des graines de grinola. Le grinola
est difficile à cultiver - un simple froncement
l de sourcil de l’agriculteur dans le champ suffit

I pour fendre la graine en deux, ce qui la rend


inutilisable. De ce fait, les récoltes de grinola
i
i connaissent de grosses fluctuations.

FIGURE 6.6

Biscuits pour chiens (en milliers)

d) Procédez de la même manière pour le marché


des biscuits pour chiens. Déplacez la courbe
appropriée afin de refléter la taxe et tracez-la
à la figure 6.5. Nommez la courbe soit
O + taxe, soit D + taxe. Déterminez
les nouveaux prix et quantité d’équilibre.
Comparez les dépenses totales pour l’équilibre
initial et le nouvel équilibre.
LES MARCHÉS EN ACTION 75

a) En l’absence de spéculateurs sur les stocks Questions à choix multiple


sur le marché du grinola, quel sera le prix
du marché lorsque l'offre instantanée est 1 b c, d et e se rapportent au long terme.
de 25 tonnes? de 20 tonnes? de 15 tonnes? 2 a Définition, d et e entraînent T loyer dans un
b) Supposons que les spéculateurs découvrent marché libre.
que le grinola peut s’entreposer facilement 3 c égale à la distance verticale entre les deux
et qu’ils entrent sur ce marché pour faire courbes d’offre.
des profits. Ils prévoient un prix de 60 $ 4 a A initial = 6 $, nouveau P = 6,60 $. Les
la tonne. Expliquez ce qui se passe lorsque vendeurs paient une taxe de 1 $ et obtiennent
l’offre instantanée est de 25 tonnes. Trouvez 0,60 $ de plus que les acheteurs, les vendeurs
la quantité de stock que les spéculateurs paient donc 0,40 $.
achètent ou vendent, la quantité achetée 5 b Voir la réponse précédente.
par les consommateurs et le prix du marché. 6 a 1$ par frisbee X 4 000 unités vendues.
c) Procédez de la même manière qu’à la partie b 7 c T A (6 $ à 6,60 $) —» -l recette totale
pour une offre instantanée de 20 tonnes. (30 000 $ à 26 400 $). d faux - r\ = 2,3
d) Procédez de la même manière qu’à la partie b et Éo ~ 3,2.
pour une offre instantanée de 15 tonnes. 8 b Prix plancher supérieur au prix de marché
e) Décrivez les fluctuations des prix dans —> offre excédentaire.
le marché du grinola. 9 d Définition.
f) Les spéculateurs ne spéculent pas par bonté 10 b Tracez le graphique. Pas de À hypothèses
d’âme. Ont-ils réalisé des profits avec ceteris paribus —> pas de déplacement
le grinola? Justifiez vos calculs. de l’offre ou de la demande.
Ile Le prix plancher inférieur au prix d’équilibre
n’empêche pas le marché d’atteindre
l’équilibre.
12b Le prix reste constant de sorte que la recette
R É P O N S E S
totale et la production sont corrélées
positivement.
Vrai/Faux/1 ncertain I3 c L’offre se déplace vers la gauche à mesure que
(Justifiez votre réponse.) les sanctions s’ajoutent aux autres coûts.
14b a ferait T A du montant intégral de la taxe.
1 V Au prix d’équilibre, tous ceux qui ont c et d ont un effet sur A, mais les montants
la capacité de payer pour se loger obtiennent sont incertains.
un logement, mais pas forcément ceux qui I5 d Offre du marché = offre des producteurs
ont besoin de logement. + offre de stocks.
2 F Demande plus élastique -A plus de substituts 16 d Les prix du marché noir seront supérieurs
—> plus facile de ne pas payer de taxes au prix plafond.
en n’achetant pas. 17c À l’intersection de la courbe d’offre
3 F Le prix plafond est supérieur au prix instantanée de 50 millions de boisseaux
de marché, par conséquent l’activité et de la courbe de demande.
de prospection ne T pas et il n’y aura pas I8 c Les agriculteurs reçoivent 5 $, mais ils sont
de marché noir. prêts à offrir cette quantité à 1 $ (leur coût).
Ô 4 F Énoncé positif; peut être vérifié par I9 d Quantité offerte lorsque A = 5 $.
l’observation. 20 d 70 millions de boisseaux se vendent à raison
5 V T salaire minimum —> >1 quantité de 1 $ le boisseau. Les contribuables paient
de travailleurs demandée. la différence de 4 $ par boisseau X 70 millions.
6 F Plus grande incidence sur les jeunes 21 d Inversez taxes et interdiction pour que a et b
travailleurs, car leurs salaires sont plus bas. soient justes.
7 F Fluctue davantage si la demande est 22 b Taux salarial le plus bas.
inélastique. 23 a Tracez les graphiques pour le voir.
8 V Achète à bas prix, vend à prix fort. 24 a Les quantités (et les recettes) continueront
& 9 i L’effet sur A dépend de l’importance relative de fluctuer, c et d sont justes si l’on inverse
des sanctions. diminuer et augmenter.
10 F Déplace l’offre vers la droite. 25 e Définition.
76 CHAPITRE 6

Problèmes à court développement fait, les courbes qui comprennent CIL sont très
proches des courbes d’offre et de demande
1 Si le marché de l’essence est initialement en initiales. L’intersection de D — CIL et de O + CIL
équilibre et qu’il y a une réduction importante produit un nouveau prix d’équilibre, identique
de l’offre, la quantité demandée au prix courant au prix initial (le prix peut être légèrement plus
sera excédentaire. De ce fait, le prix de l’essence élevé ou plus bas, selon l’importance relative
augmentera, ce qui entraînera des mouvements des déplacements de l’offre et de la demande)
le long de la nouvelle courbe d’offre et de et une nouvelle quantité d’équilibre (Qj> )
la courbe de demande. À mesure que le prix légèrement inférieure à la quantité originale.
s’élève, la quantité offerte augmente et
la quantité demandée diminue. Le prix FIGURE 6.7
continuera de monter jusqu’à ce que la quantité
excédentaire soit éliminée. L’augmentation
du prix incite les consommateurs à réduire
leur consommation d’essence.

2 Du fait que l’alcool et la nicotine induisent


l’accoutumance, les demandes de boissons
alcoolisées et de cigarettes sont relativement
inélastiques. Lorsque les « taxes sur les vices »
viennent s’ajouter au prix des boissons alcoolisées
et des cigarettes, le pourcentage de réduction de
la quantité demandée est inférieur au pourcentage
d’augmentation du prix. De ce fait, la recette Quantité de vêtements

totale augmente parallèlement à l’augmentation


des recettes fiscales. Pour une augmentation
donnée de la recette fiscale, il faut que la taxe
par unité soit bien plus faible sur les produits a) La figure 6.8 correspond à un marché de
à demande inélastique que sur les produits logements locatifs non réglementé. Les
à demande élastique. En outre, une demande courbes de demande, d’offre à court terme,
inélastique signifie, ceteris paribus, que les et d’offre à long terme initiales sont D0, OCT0
consommateurs paieront une part plus élevée et OIT, respectivement. Le marché est d’abord
de la taxe que les producteurs. en équilibre au point a, correspondant au
loyer L0 et à la quantité de logements locatifs
3 La principale différence entre les deux produits Q0. La demande passe alors a. Dv créant une
est que l’on ne peut pas vraiment stocker demande excédentaire de Q2 - Qq au prix de
la plupart des fruits frais alors que les fruits en loyer initial. À court terme, dans un marché
conserve se stockent facilement. Faute de stocks, libre, le loyer s’élèvera à Ll pour équilibrer
il est difficile de spéculer, et le prix des fruits les marchés, et la quantité d’équilibre des
frais fluctue fortement selon les variations de la logements loués est Qi (point b). Remarquez
courbe d’offre instantanée. Puisqu’il est possible que, à mesure que le loyer s’élève, la quantité
de stocker les fruits en conserve, les détenteurs de logements locatifs offerte augmente (un
de stocks tentent de rendre l’offre de fruits en mouvement du point a au point b le long
conserve parfaitement élastique, au prix qu’ils de la courbe d’offre OCT0) puisque le taux
prévoient. De ce fait, les prix induits par l’offre d’occupation des logements existants
de fruits en conserve ne fluctuent pas. Les prix augmente. De plus, la quantité de logements
des fruits en conserve peuvent quand même demandée diminue (un mouvement du point
fluctuer si le prix anticipé par les détenteurs de c au point b le long de la courbe de demande
stocks varie. Z)j). Ensemble, ces mouvements éliminent la
demande excédentaire. De plus, le montant
4 Le marché du vêtement est illustré à la figure 6.7. du loyer plus élevé stimule la construction
La courbe de demande est D et la courbe d’offre, domiciliaire à long terme. C’est ce qu’illustre
O. Le prix d’équilibre est Pc et la quantité, Qc. le déplacement de la courbe d’offre de OCT0
Étant donné que la police ne cherche pas à OCTl. Enfin, un nouvel équilibre à long
réellement à faire appliquer la loi, le coût terme est atteint au point c, le loyer étant
d’infraction à la loi (CIL) est très faible, autant revenu à son niveau initial (L0) et le nombre
pour les acheteurs que pour les vendeurs. De ce de logements loués égal à Qj-
LES MARCHÉS EN ACTION 77

FIGURE 6.8 TABLEAU 6.2

Quantité Quantité
demandée offerte
(en millions de (en millions de
Prix litres par jour) litres par jour)

1,40 8 16
1,30 10 14
1,20 12 12
1,10 14 10
1,00 16 8
0,90 18 6

b) Nous pouvons utiliser de nouveau la figure 6.8


pour examiner le comportement d’un marché
lorsque le prix plafond du loyer est fixé à L0.
Commençons au point a, avec le même
équilibre à long terme. Là encore, nous
observons une augmentation de la demande
de Dq à D\. Dans ce cas, cependant, il n’est
pas possible d’augmenter le loyer pour
restaurer l’équilibre. Rien n’incitera à faire
augmenter le taux d’occupation des logements
existants à court terme ni la construction
domiciliaire à long terme. La quantité
de logements locatifs offerte sera la même
à Qo- Étant donné que la dernière unité
de logement locatif est évaluée à L2 mais
que le loyer est fixé à L0, les demandeurs
de logements locatifs seront prêts à payer
Le nouveau prix d’équilibre est de 1,20 $ par
des coûts supplémentaires jusqu’à concurrence
litre puisque, à ce prix, la quantité d’essence
de L2 — Lq (sous forme d’une activité de
demandée est égale à la nouvelle quantité
prospection supplémentaire ou de paiements
offerte (12 millions de litres par jour).
illégaux) afin de se trouver un logement.
La nouvelle quantité d’équilibre est de
12 millions de litres d’essence par jour.
6 a) Le prix d’équilibre de l’essence est de 1 $ par
c) La hausse du prix a entraîné la diminution de
litre puisque, à ce prix, la quantité d’essence
la quantité d’essence demandée de 4 millions
demandée est égale à la quantité offerte
de litres par jour (de 16 à 12 millions).
(16 millions de litres par jour). La quantité
Compte tenu de la nouvelle courbe d’offre
d’équilibre de l’essence est de 16 millions
Ov la hausse du prix de 1 $ à 1,20 $ le litre
de litres par jour. fait augmenter la quantité d’essence offerte
b) Les nouvelles données sont indiquées de 4 millions de litres par jour (de 8 à
au tableau 6.2 et à la figure 6.9. 12 millions).

7 a) Au prix plafond de 1 $, la quantité demandée


est de 16 millions de litres par jour et la
quantité offerte est de 8 millions de litres
par jour.
b) La quantité d’essence réellement vendue est
de 8 millions de litres par jour. Lorsque,
à un prix donné, la quantité demandée et la
quantité offerte diffèrent, la quantité la moins
78 CHAPITRE 6

importante détermine la quantité réellement b) La recette fiscale totale est de 15 <f par bande
vendue. dessinée X 100 000 unités vendues
c) La quantité excédentaire d’essence demandée = 15 000 $, ce qu’indique la surface ombrée à
est de 8 millions de litres par jour. la figure 6.4 Solution.
d) Le prix le plus élevé que les consommateurs c) À cause de la taxe, le prix imposé aux
sont prêts à payer pour la dernière unité consommateurs a augmenté de 5 <£ (passant
d’essence offerte (le 8 millionième litre par de 25 <t à 30 <t). La part de la taxe que
jour) est de 1,40 $. Vous pouvez obtenir cette paient les consommateurs est de 5 000 $
réponse grâce à votre graphique, en imaginant (5 <f X 100 000). Les producteurs paient
une ligne verticale partant de la quantité de la taxe de 15 <f au gouvernement, mais ne
8 millions de litres jusqu’à son intersection avec reçoivent que 5 <t du consommateur. Par
la courbe de demande au point correspondant conséquent, la part de la taxe que paient les
à 1,40 $. La courbe de demande indique le producteurs est de 10 000 $ (10 é X 100 000).
prix le plus élevé que les consommateurs sont d) Voir la figure 6.5 Solution. Sur le marché des
prêts à payer pour le dernier litre offert. biscuits pour chiens, la courbe d’offre
e) Le prix réglementé de l’essence est de 1 $ le se déplace verticalement vers le haut d’une
litre, mais la valeur qu’accorde Georges au distance égale au montant de la taxe (15 C).
dernier litre d’essence est de 1,40$; par Le prix d’équilibre initial est de 25 <t et
conséquent, il serait prêt à payer un montant la quantité, de 200 000. Le nouveau prix
supérieur de 4 $ au prix réglementé de l’essence d’équilibre est de 35 <f et la quantité, de
afin d’obtenir 10 litres (0,40 $ X 10 litres). 150 000. La dépense totale a augmenté,
Si sa meilleure possibilité est de gagner 8 $ de 50 000 $ (25 <f x 200 000) à 52 500 $
l’heure, Georges sera prêt à faire la queue (35 <t x 150 000).
pendant près d’une demi-heure pour acheter
10 litres. FIGURE 6.5 SOLUTION

8 a) Voir la figure 6.4 Solution. Dans le marché


des bandes dessinées, la courbe d’offre se
déplace verticalement vers le haut d’une
distance égale au montant de la taxe (15 <£)•
Le prix d’équilibre original est de 25 <f et
la quantité, de 200 000. Le nouveau prix
d’équilibre est de 30 <t et la quantité, de
100 000. La dépense totale a diminué,
passant de50 000 $ (25 4 X 200 000)
à 30 000$ (30 <t x 100 000).

FIGURE 6.4 SOLUTION

e) La recette fiscale totale est de 15 par biscuit


pour chiens X 150 000 unités vendues
= 22 500 $, ce qu’indique la surface ombrée
de la figure 6.5 Solution.
f) À cause de la taxe, le prix que doit payer le
consommateur a augmenté de 10 è
(de 25 <1 à 35 <f). La part de la taxe que
paient les consommateurs est de 15 000 $
(10 <t X 150 000). Les producteurs paient
la taxe de 15 <t au gouvernement et reçoivent
10 <t du consommateur. Par conséquent, la
portion de la taxe payée par les producteurs
est de 7 500$ (5 <t x 150 000).
LES MARCHÉS EN ACTION 79

g) Vous recommandez confidentiellement à augmentation des recettes fiscales ; mais,


M. Mauret que le Conseil du Trésor impose si l’on dépense plus, on dépensera plus en
une taxe sur les biscuits pour chiens. Une taxes. Vous avez également oublié que plus
taxe d’accises imposée sur les biscuits pour la demande est inélastique, plus le
chiens a deux avantages: 1) elle permettra consommateur paie une grande portion
de percevoir plus de recettes fiscales qu’une de la taxe.
taxe sur les bandes dessinées (22 500 $ par Si vous avez pensé tout seul à l’approche
rapport à 15 000 $) et 2) elle frappera plus relative à l’élasticité, bravo ! M. Mauret
lourdement le consommateur. Sur le marché n’hésitera pas à vous demander des conseils.
des biscuits pour chiens, les consommateurs Si vous n’y avez pas pensé, c’est le moment de
paieront 15 000 $, soit 67% de la taxe. consulter Parkin et Bade pour vous rafraîchir
Sur le marché des bandes dessinées, les la mémoire.
consommateurs ne paieront que 5 000 $,
soit 33 % de la taxe. ® 10 Examinez la figure 6.6 Solution.

9 Vous vous en voulez d’avoir oublié qu’il existe FIGURE 6.6 SOLUTION
une relation entre l’élasticité de la demande et,
en même temps, 1) une variation de la recette
totale en raison d’une hausse du prix (engendrée
par la taxe) et 2) le fait que ce soit les
consommateurs ou les producteurs qui paient
une part plus élevée de la taxe,
a) Pour le marché des bandes dessinées, le calcul
de l’élasticité de la demande est:

f A Q ^ / ( AP "j
. J
Q-moy
/P
\ moy y

Quantité de grinola (en tonnes)


f 0,30-0,25^1
y
J/ l 0,275 J

^-0,667^
3,67 a) Lorsque l’offre instantanée est de 25 tonnes,
v 0,182 , P — 40 $. Lorsque l’offre instantanée est de
La demande est élastique, 20 tonnes, P - 60 $. Lorsque l’offre
b) Pour le marché des biscuits pour chiens, le instantanée est de 15 tonnes, P — 80 $.
calcul de l’élasticité de la demande est : b) Le prix futur anticipé est de 60 $. Lorsque
l’offre instantanée est de 25 tonnes, le prix du
AQ )/ ( AP "| marché de 40 $ est inférieur au prix futur

Q moy]/ P
\ moy y
anticipé, et les spéculateurs achètent du
grinola pour en stocker. Ils commencent à en
acheter au prix de 40 $, mais leur demande
^150 — 200 y f 0,35-0,25^1
supplémentaire fait monter le prix jusqu’à
v 175 J/l 0,30 J
ce qu’il atteigne rapidement les 60 $. Les
spéculateurs achètent 5 tonnes, en payant
"-0,286"
0,87 de 40 $ à 60 $ la tonne. Les consommateurs
^ 0,333 y achètent les 20 tonnes restantes, et le prix

La demande est inélastique, du marché final est de 60 $.


c) Vous avez oublié qu’une hausse de prix fait c) Dans ce cas, le prix futur anticipé et le prix
augmenter le montant des dépenses totales du marché sont identiques (60 $), de sorte
lorsque la demande est inélastique, et baisser que les spéculateurs ne font rien. Le prix du
ce montant lorsque la demande est élastique. marché est de 60 $.

Par conséquent, vous pourriez avoir utilise d) Le prix futur anticipé est de 60 $. Lorsque

les données d’élasticité pour prédire que les l’offre instantanée est de 15 tonnes, le prix

dépenses allaient augmenter sur le marché du marché de 80 $ est supérieur au prix

des biscuits pour chiens. Une augmentation futur anticipé, de sorte que les spéculateurs

des dépenses n’est pas la même chose qu une vendent le grinola à partir de leurs stocks. Ils
80 CHAPITRE 6

commencent à vendre au prix de 80 $, mais f) Il est probable que les spéculateurs ont
leur offre supplémentaire fait baisser réalisé un profit. Dans le cas b, ils ont
ce prix jusqu’à ce qu’il atteigne rapidement acheté 5 tonnes au prix de 40 $ à 60 $
60 $. Les spéculateurs vendent 5 tonnes, par tonne environ. Par conséquent, ils ont
recevant entre 80 $ et 60 $ par tonne. dépensé entre 200 $ et 300 $. Ils n’ont
Les consommateurs achètent 20 tonnes effectué aucune transaction dans le cas c.
(15 aux producteurs et 5 aux spéculateurs), Dans le cas d, ils ont vendu 5 tonnes à un
et le prix final du marché est de 60 $. prix variant entre 80 $ et 60 $ la tonne.
e) Pour les consommateurs, les fluctuations Par conséquent, ils ont gagné entre 400 $
du prix ont été largement éliminées étant et 300 $. Leur profit net se situe entre
donné que, quelle que soit l’offre instantanée, un maximum de 200 $ (400 $ - 200 $)
le prix du marché atteint rapidement 60 $. et un minimum de 0 (300 $ - 300 $).
Révision
re
PARTIE

Chapitres 1 à 6

PROBLÈME FIGURE RI. I MARCHÉ DES CIGARETTES

AU CANADA

Le ministre des Finances a demandé à votre


patronne, Mme Irma Losique, d’évaluer
les effets de l’imposition d’une taxe de 24 $
sur la cartouche de cigarettes. Mme Losique
prédit que la taxe fera monter sensiblement
les recettes fiscales (d’au moins 30 millions
de dollars), car la demande de cigarettes
est inélastique.
Malheureusement, Mme Losique
n’est pas au courant d’une récente étude
que vous avez lue dans le Globe and Mail
sur les augmentations de taxes passées
et qui indiquait que « Ottawa n’a pas retiré
tout le bénéfice prévu de l’augmentation
de la taxe sur les cigarettes à cause
de la contrebande accrue. »
Pour pouvoir faire ses prévisions, elle
vous a demandé d’effectuer l’analyse détaillée
suivante. Les courbes d’offre et de demande
(sans taxes) des cigarettes sont présentées
à la figure R 1.1. Le prix avant taxe Quantité de cigarettes (en millions de cartouches) par année
est de 28 $ la cartouche et la quantité
de cartouches vendues avant taxe est
de 2,25 millions de cartouches par année. & e) Jusqu’à aujourd’hui, Mme Losique
a) Tracez la nouvelle courbe O + taxe s’est toujours montrée forte en économie.

sur la figure R 1.1. Néanmoins, vous lui signalez respectueusement


b) Selon les prédictions de Mme Losique, quel que sa prédiction au sujet des recettes est
est le prix d’équilibre après taxe ? la quantité fausse, car votre patronne n’a pas pris en compte
les effets de la contrebande. Faisant preuve
d’équilibre ?
c) Calculez le montant des recettes fiscales d’une flexibilité qui ne lui ressemble pas, elle
vous répond: «Je vous écoute. Quelles sont vos
quelle a prévues.
d) Vérifiez si Mme Losique a raison de dire prédictions en matière de recettes fiscales ? »
que la demande de cigarettes est inélastique, Voilà donc l’occasion d’impressionner

en calculant T| entre l’équilibre avant taxe votre patronne. Vous savez que la courbe
d’offre de cigarettes aux États-Unis est
et l’équilibre après taxe.
la même que la courbe d’offre de cigarettes
avant taxe au Canada. Vous avez calculé
le coût d’infraction à la loi (C/L) des
82 RÉVISION I PARTIE

fournisseurs à 16 $ la cartouche, sachant que 3 Une CPP classique montre


le risque que courent les consommateurs en a) qu’il existe une limite à la production
achetant des cigarettes de contrebande est de n’importe quel bien.
tellement mince que leur CIL est, en fait, nul. b) que, pour produire une plus grande quantité
À l’aide de cette information, répondez aux d’un bien, nous devons produire une moins
questions suivantes. grande quantité d’un autre bien.
i) Tracez et nommez toutes les nouvelles c) qu’il y a des limites à la production totale
courbes appropriées sur la figure Rl.l. avec des ressources et une technologie
ii) Quel sera le prix après taxe d’une données.
cartouche de cigarettes ? la quantité d) Toutes ces réponses.
de cigarettes vendues légalement ? e) Aucune de ces réponses.
la quantité de cigarettes vendues
en contrebande? (Indice: le prix 4 Si deux personnes a et b peuvent chacune
des cigarettes vendues légalement produire les biens A et Y seulement et que
et illégalement sera le même étant a ne détient pas d’avantage comparatif dans
donné qu’il s’agit de substituts parfaits.) la production du bien X ou. Y, nous savons
iii) Que prédisez -vous en matière de recettes alors que
fiscales ? a) b a un avantage absolu dans
la production de A et Y
b) a et b ont le même coût
d’opportunité pour A et pour Y.
EXAMEN DE MI-ÉTAPE c) b a un avantage comparatif dans
la production de A et de K

Allouez 48 minutes pour cet examen (24 questions, d) il y aura des gains de l’échange.

2 minutes par question). Pour chaque question, e) b et d.

choisissez la meilleure réponse.


5 La valeur de l’élasticité de la demande

1 L’économie canadienne repose et de l’élasticité de l’offre dépend

a) exclusivement sur le mécanisme a) des possibilités de substitution des facteurs.

de coordination par le marché. b) de la part du revenu dépensée sur un bien.

b) surtout sur le mécanisme de coordination c) du temps écoulé depuis la variation du prix.

par le marché, dans une économie mixte. d) des conditions techniques de production.

c) exclusivement sur le mécanisme de e) Aucune de ces réponses.

coordination par directives.


d) largement sur le mécanisme de coordination 6 Si une augmentation de prix entraîne une

par directives, dans une économie mixte. diminution de la recette totale, alors

e) également sur les mécanismes de coordination l’élasticité-prix de la demande est

par directives et par le marché, dans une a) négative.

économie mixte. b) égale à 0.


c) supérieure à 0 mais inférieure à 1.

2 Francine gagne 25 $ l’heure comme soudeuse. d) égale à 1.

Elle doit s’absenter deux heures de son travail e) supérieure à 1.

(qui ne lui seront pas payées) afin d’aller chez


le dentiste pour un plombage. Cette séance chez 7 Les données du tableau Rl.l ne pourraient

le dentiste lui coûte 100 $. Le coût d’opportunité pas être représentées par
de la séance chez le dentiste de Francine est de a) deux graphiques de série

a) 125 $. chronologique à une variable.


b) 100$. b) un graphique de série
c) 50 $. chronologique à deux variables.
d) 150$. c) un graphique de série

e) Aucune de ces réponses. chronologique à trois variables.


d) un diagramme de dispersion.
e) Toutes ces réponses.
CHAPITRES I À 6 83

présentées au tableau RI .2. En vous servant


TABLEAU Rl.l
des données du tableau R1.2, répondez
aux questions 11 et 12.
Année X y
1990 6,2 143 TABLEAU RI.2 POSSIBILITÉS
1991 5,7 156 DE PRODUCTION

1992 5,3 162


Unités Unités
Possibilité de beurre de canons

8 Si le prix d’un parapluie est peu élevé et qu’il y a a 8 0


de nombreux jours de pluie, un plus grand b 6 1
nombre de parapluies sera vendu. Si le prix d’un c 0 3
parapluie est élevé et qu’il y a peu de jours de
pluie, un moins grand nombre de parapluies sera
vendu. Compte tenu de cette donnée, le nombre Il En se déplaçant de la combinaison
a) de parapluies vendus et le prix d’un parapluie b à la combinaison c, le coût d’opportunité
sont positivement reliés, en supposant que de la production d’une unité supplémentaire
le nombre de jours de pluie reste constant. de canons est de
b) de parapluies vendus et le prix d’un parapluie a) 2 unités de beurre.
sont négativement reliés, en supposant que b) 1/2 unité de beurre.
le nombre de jours de pluie reste constant. c) 6 unités de beurre.
c) de jours de pluie et le nombre de parapluies d) 1/6 d’unité de beurre.
vendus sont négativement reliés, en supposant e) 3 unités de beurre.
que le prix d’un parapluie reste constant.
d) de jours de pluie et le prix d’un parapluie I2 Selon cette CPP,
sont négativement reliés, en supposant que a) une combinaison de 6 unités de beurre
le nombre de parapluies vendus reste constant. et de 1 canon n’utiliserait pas toutes
e) de fois que vous vous posez cette question les ressources.
est de 44. b) une combinaison de 0 unité de beurre
et de 4 canons est réalisable.
9 Les détenteurs de stocks c) les ressources sont homogènes.
a) vendent des biens en stock lorsque le prix d) le coût d’opportunité de la production
est inférieur au prix futur qu’ils ont prévu. de canons augmente à mesure que l’on
b) achètent des biens pour les mettre en stock produit davantage de canons.
lorsque le prix est supérieur au prix futur e) le coût d’opportunité de la production
qu’ils ont prévu. de canons diminue à mesure que l’on produit
c) rendent la courbe d’offre du marché davantage de canons.
parfaitement élastique au prix futur
qu’ils ont prévu. 13 Quel énoncé, parmi les suivants, est un énoncé
d) Toutes ces réponses. normatif?
e) Aucune de ces réponses. a) La pollution est un exemple de coût externe.
b) La pollution nuit au bien-être des gens.
I0 La part de la taxe que doivent payer c) On devrait forcer les entreprises pollueuses
les consommateurs augmente plus à fermer leurs portes.
1 la demande est élastique. d) La pollution impose des coûts d’opportunité
2 la demande est inélastique. aux autres.
3 l’offre est élastique. e) Aucune de ces réponses.
4 l’offre est inélastique.
a) 2 seulement 14 Les décisions économiques concernant
b) 1 et 3 les marchés sont coordonnées par
c) 1 et 4 a) les entreprises.
d) 2 et 3 b) les ménages.
e) 2 et 4 c) les gouvernements.
d) les ajustements de prix.
Supposons qu’une entreprise ne produise e) Toutes ces réponses.
que deux biens : des canons et du beurre.
Trois combinaisons possibles sur sa CPP sont
84 RÉVISION - I re PARTIE

I5 Si l'offre et la demande augmentent toutes deux, I9 Si une augmentation du revenu de 10 % entraîne


alors le prix d’équilibre une diminution de 10 % de la consommation
a) augmentera et la quantité augmentera. de gadgets (en supposant que le prix reste
b) baissera et la quantité augmentera. constant), alors
c) pourrait augmenter ou baisser, mais a) l’élasticité-prix de la demande
la quantité augmentera. de gadgets est égale à 1.
d) augmentera et la quantité pourrait b) l’élasticité-revenu de la demande
à la fois augmenter ou baisser. de gadgets est négative.
e) baissera et la quantité pourrait c) l’élasticité-revenu de la demande
à la fois augmenter ou baisser. de gadgets est égale à 1.
d) les gadgets sont des biens normaux.
16 Si les navets sont un bien inférieur, alors, e) Aucune de ces réponses.
ceteris paribus, une augmentation du prix
du navet entraînera 20 Une diminution du prix de X, qui passe
a) une diminution de la demande de navets. de 6 $ à 4 $ fait grimper la quantité de
b) une augmentation de la demande de navets. Y demandée (au prix courant de F) de 900
c) une diminution de l’offre de navets. à 1 100 unités. Quelle est l’élasticité-prix croisée
d) une augmentation de l’offre de navets. de la demande entre X et F?
e) Aucune de ces réponses. a) 0,5
b) -0,5
I7 Dans la figure RI .2, si le salaire minimum c) 2
est fixé à 6 $ l’heure, quel est, exprimé d) -2
en millions d’heures, le niveau du chômage ? e) a ou b, selon que A et F sont des substituts
a) 50 ou des compléments.
b) 40
c) 0 21 Le graphique de la relation entre deux variables
d) 10 qui sont négativement reliées
e) 20 a) est horizontal.
b) a une pente ascendante vers la droite.
FIGURE RI.2 c) est vertical.
d) a une pente descendante vers la droite.
e) est linéaire.

22 Dans la figure RI.3, la pente de la droite est de


a) 1,00.
b) 1,25.
c) 1,50.
d) 0,75.
e) 0,50.

FIGURE RI.3

Quantité de travail (en millions d'heures)

18 Un prix plancher inférieur au prix d’équilibre


entraîne
a) une offre excédentaire.
b) une demande excédentaire.
c) le prix d’équilibre.
d) une augmentation de l’offre.
e) une diminution de la demande.
CHAPITRES I À 6 85

23 Une diminution de la quantité offerte


est représentée par b) Le prix d’équilibre après taxe est de 46 $
a) un mouvement vers le bas le long par cartouche. La quantité d’équilibre après
de la courbe d’offre. taxe est de 1,5 million de cartouches par an.
b) un mouvement vers le haut le long c) La recette fiscale est de 1,5 million
de la courbe d’offre. de cartouches X 24 $ par cartouche
c) un déplacement vers la gauche de la courbe = 36 $ millions par année.
d’offre.
d) un déplacement vers la droite de la courbe AQ
d’offre.
e) un déplacement vers la droite de la courbe
de demande.
moy

24 La terre agricole peut être utilisée pour élever


du bétail ou produire du maïs. Si la demande 1,5-2,25 -0,75
de bétail augmente, alors 1,875 1,875 -40 %
a) la demande de maïs va augmenter. 46-28 18 +48,6%
b) l’offre de maïs va augmenter.
37 37
c) la demande de maïs va diminuer.
d) l’offre de maïs va diminuer. = 0,82
e) b et c.
La courbe de demande est inélastique
(t| < 1) entre les deux points d’équilibre.
Les prédictions de Mme Losique sur
RÉPONSES les recettes fiscales (sans contrebande)
et l’élasticité étaient bonnes.
Problème & e) i) Voir la solution de la figure Rl.l.
Etant donné que le CIL est nul pour
a) Voir la figure Rl.l Solution. Ne vous les consommateurs, la courbe de demande
occupez pas de la courbe O + CIL pour ne se déplace pas. La courbe d’offre des
l’instant. producteurs de cigarettes de contrebande
est O + CIL.
FIGURE RI. I SOLUTION ii) Étant donné que les consommateurs
MARCHÉ DES CIGARETTES
peuvent acheter des cigarettes de
AU CANADA contrebande au prix de 40 $ la cartouche,
ils seront peu enclins à les payer au prix
légal plus élevé de 46 $. Le prix légal
devra être ramené à 40 $. À ce prix,
les producteurs du marché légal
fourniront 0,75 million de cartouches
alors que la demande totale du marché
est de 1,75 million de cartouches.
Les contrebandiers fournissent
la différence, soit 1 million de cartouches.
iii) La recette fiscale est de 0,75 million
de cartouches X 24 $ par cartouche
= seulement 18 millions de dollars
par an. Mme Losique est impressionnée.
Elle vous dit : «Lorsque je faisais mes
études d’économie, on nous enseignait
que les activités illégales relevaient
de la police et non des économistes.
Vous nous avez montré qu’il est possible
d’analyser presque n’importe quelle
activité dans une perspective économique.
Bien joué. Allons préparer notre réponse
au ministre. »
86 RÉVISION I re PARTIE

Examen de mi-étape 13 c a et b sont des énoncés justes (positifs) ; d est


un énoncé positif qui peut être vérifié.
14 d Les ajustements de prix permettent
1 b Définition.
de coordonner les décisions prises par les
2 d 100 $ qui pourraient avoir servi à acheter
entreprises, les ménages et les gouvernements.
d’autres biens, plus le coût du temps
I5 c Les déplacements vers la droite de la demande
(25 $ X 2 heures).
et de l’offre doivent —>î quantité d’équilibre,
3 d a et c ont trait au phénomène de rareté,
mais l’effet sur le prix est indéterminé.
b illustre le coût d’opportunité.
Ô 16 e î P —> mouvement le long de la courbe
4 b L’absence d’avantage comparatif signifie un
de demande.
coût d’opportunité égal. Aucune information
17b La quantité offerte (50) > quantité
sur l’avantage absolu,
demandée (10).
& 5 c a, d ne modifient que T|0 (offre), b
6) 18 c Tracez le graphique. Le prix plafond
ne modifie que t|D (demande).
n’empêche pas T P lorsque la demande
6 e Calcul de la recette totale.
est excédentaire.
7 c U y a deux variables plus le temps, qui
19 b T\y = (%A Qd) / (% A revenu) = —10%/10%
pourraient être représentées par a, b ou d.
= — 1 ; les gadgets sont des biens inférieurs.
& 8 b c serait juste si l’on changeait «négativement»
& 20 b r\x= (%A Qyd) / (%AP*)
par «positivement». On ne peut répondre
= (200/1 000) / (-2/5) = -0,5.
pour d sans informations supplémentaires,
21 d À mesure que xî, y -t.
9 c a et b seraient justes, si l’on inversait
22 d Par exemple, en utilisant les points (0, 4) et
«inférieur» et «supérieur».
(16, 16): Ay= 12 (16-4), Ax = 16 (16-0).
10 d Définition.
23 a Illustration de la loi de l’offre.
11 e Renoncer à 6 unités de beurre pour obtenir
24 d Le bétail et le maïs sont des substituts
2 canons: 6/2 = 3 unités de beurre par canon.
de production. T demande de bétail
12 d Le coût d’opportunité d’un canon se situe
—> T prix du bétail —> -l offre de maïs.
entre a et b = 2 unités de beurre ; entre
b et c — 3 unités de beurre, a sur la CPP,
b à l’extérieur de la CPP.
L’utilité et la demande

♦ L’utilité totale est maximisée lorsque tout le revenu


CONCEPTS CLÉS est dépensé et que l’utilité marginale par dollar
dépensé est égale pour tous les biens. Pour des
biens substituts comme les films (f) et les sodas
Les choix de consommation
{s), cela se produit lorsque:
des ménages
Umf _ Ums
Les choix de consommation des ménages
sont déterminés par les éléments suivants.

♦ Les contraintes budgétaires La capacité de l’analyse marginale de prédire les

• Le revenu et les prix (P) des biens et des services décisions économiques est due à une simple règle -

limitent les possibilités de consommation. lorsque le gain marginal d’une décision est supérieur

• La droite de budget délimite les choix qui sont à la perte marginale, il est temps d’agir.

possibles (points sur la droite ou à l’intérieur ♦ La règle qui s’applique à l’exemple de maximisation
de la droite) et ceux qui ne le sont pas (points de l’utilité est : lorsque l’utilité marginale par dollar
à l’extérieur de la droite). dépensé en films est supérieure à l’utilité marginale
par dollar dépensé en sodas, il faut acheter plus
♦ Les préférences — ce que l’on aime et ce que
de films et moins de sodas.
l’on n’aime pas ; elles sont fondées sur l’utilité.
♦ Le gain marginal que procure le fait de voir
L’utilité est l’avantage ou la satisfaction qu’une personne
un plus grand nombre de films est supérieur
retire de la consommation d’un bien ou d’un service.
à la perte marginale entraînée par une moins
♦ L’utilité totale {UT) est l’avantage total grande consommation de sodas.
ou la satisfaction totale qu’une personne retire
de la consommation d’un bien ou d’un service.
T consommation —> T utilité totale. Les prédictions de la théorie
d’utilité marginale
♦ L’utilité marginale {Uni) est la A utilité totale
que procure la dernière unité consommée.
Ceteris paribus, X Pr —> T Qf consommée (mouvement
L’Um est positive, mais en raison d’une utilité
vers le bas le long de la courbe de demande de films )
marginale décroissante, elle X à mesure que la
et —» déplacement vers la gauche de la courbe de
consommation du bien T.
demande de sodas.

Ceteris paribus., T Ps —» -l Q, consommée (mouvement


La maximisation de l’utilité vers le haut le long de la courbe de demande de sodas)
et —> déplacement vers la droite de la courbe de
Les consommateurs cherchent à maximiser 1 utilité. demande de films.

♦ Étant donné le revenu et les prix des biens et des^ Les films et les sodas sont des biens normaux. Ceteris
services, il y a équilibre du consommateur lorsqu il paribus, T revenu —» T consommation de films
répartit ses dépenses de manière à maximiser son et de sodas (déplacements vers la droite des deux
courbes de demande).
utilité totale.
88 CHAPITRE 7

La théorie de l’utilité marginale nous permet de dériver


et de prédire les résultats précédents qui, au chapitre 4, RAPPELS
n’étaient que de simples suppositions à propos de
la demande du consommateur. 1 Le concept de futilité est un concept abstrait,
mais extrêmement utile ; il nous permet de voir
La demande individuelle et la demande du marché :
plus clairement le choix des consommateurs.
♦ demande individuelle — relation entre la quantité Ne vous laissez pas dérouter par le fait que l’on
demandée et le prix pour chaque individu, mesure futilité par des unités arbitraires. Ce
ceteris pari bus ; qui compte dans le cas de la théorie de futilité
marginale, c’est que vous puissiez juger si
♦ demande du marché - somme des demandes
l’avantage supplémentaire retiré par dollar
individuelles ; relation entre la quantité demandée
dépensé sur le bien X est supérieur ou inférieur
totale et le prix; à l’avantage supplémentaire par dollar dépensé
♦ courbe de demande du marché - somme sur le bien Y. S’il est plus grand, alors vous
horizontale des courbes de demande individuelles, déciderez de consommer une unité supplémentaire
pour tous les prix possibles. de X. Pour prendre cette décision, il n’est pas
nécessaire de connaître le niveau de satisfaction
supplémentaire.
Les critiques de la théorie
de l’utilité marginale
2 On peut exprimer futilité marginale par dollar
dépensé sur le bien X sous la forme Umx /Px,
L’utilité ne peut être observée. Toutefois, il n’est
Umx étant futilité marginale de la dernière unité
pas nécessaire de l’observer pour l’utiliser. Nous
de X consommée et PX> le prix d’une unité
observons, en fait, les revenus, les quantités et les prix
du bien X. La condition d’équilibre du
des biens et des services. La théorie de l’utilité nous
consommateur (maximisation de futilité) pour
permet de comprendre les choix des consommateurs
les biens X et Y peut donc s’exprimer ainsi :
et de prédire les effets des variations des prix et des
revenus sur leurs choix. Umx _ UmY
Les consommateurs ne sont pas aussi malins que
l’implique la théorie. Toutefois, la théorie ne fait
aucune prédiction quant au processus de la pensée Cela implique que, en condition d’équilibre
des consommateurs. Elle ne fait que prédire leurs du consommateur, le ratio des utilités marginales
vraies actions. sera égal au ratio des prix des deux biens :

Umx _ Px
Les implications de la théorie UmY PY
de l’utilité marginale
Ce résultat est souvent utile.
Le surplus du consommateur :

♦ la valeur d’un bien est le montant maximum


3 Lorsque la condition d’équilibre du
consommateur n’est pas satisfaite, l’équation
qu’une personne est disposée à payer pour se
le procurer; précédente n’est pas vérifiée. Supposons,
par exemple, que le consommateur dépense
♦ le surplus du consommateur est la différence tout son revenu sur des biens lorsque
entre la valeur d’un bien et son prix;
Umx UmY
♦ le surplus du consommateur est faire située
au-dessous de la courbe de demande, mais
au-dessus du prix du marché (voir la figure 7.7, Ou, de façon équivalente,
p. 159 du manuel).
Utnx > P^
La distinction entre futilité totale et futilité marginale
permet de résoudre le paradoxe eau-diamant. UmY PY

♦ L’eau est plus utile (UT élevée), mais son prix Puisque Px et PY sont donnés, cela signifie
est plus bas (faible Um). que Umx est « trop grande » et UmY « trop
petite». On peut accroître futilité en augmentant
la consommation de X (et donc en diminuant
Umx conformément au principe de futilité
L'UTILITÉ ET LA DEMANDE 89

marginale décroissante) et en diminuant 7 Du fait que l’on ne peut observer ni mesurer


la consommation de Y (et donc, en augmentant Futilité, la théorie de Futilité marginale est tout
UmYt, conformément au principe de l’utilité à fait inutile.
marginale décroissante).

4 Le tableau 7.7 du manuel (p. 156) constitue


un bon exercice de révision. 8 La valeur d’un bien est égale au prix de ce bien.

AUTOÉVALUATION 9 Le surplus du consommateur de biens


relativement peu coûteux, comme l’eau,
sera relativement faible.
Vrai/Faux/Incertain
(Justifiez votre réponse.)

I La courbe de demande du marché est la somme


10 Lorsqu’un déplacement de l’offre fait baisser
horizontale de toutes les courbes de demande
le prix d’un bien, le surplus du consommateur
individuelles.
augmente.

2 Le principe de Futilité marginale décroissante


signifie que, à mesure que la consommation Questions à choix multiple
d’un bien augmente, Futilité totale augmente,
mais à un rythme décroissant. I Si Mme Choquette maximise son utilité par
la consommation des biens A et B, quel énoncé,
parmi les suivants, est juste?
a) UmA = UmB
3 Lorsque le prix du bien X augmente, Futilité
marginale que procure la consommation du bien b) UmA
«5
1

X diminue. Pa Pb
c) UmA UmB
Pb Pa
4 Lorsque les utilités marginales que procure
la consommation de deux biens ne sont d) UTa = utb
pas égales, la condition d’équilibre
du consommateur ne peut être satisfaite. e) UTa UTb
Pa Pb

5 Lorsque Futilité marginale par dollar dépensé 2 Lucien consomme des pommes et des
sur le bien X est plus grande que 1 utilité bananes, il est donc en condition d’équilibre
marginale par dollar dépensé sur le bien Y, du consommateur. L’utilité marginale
Futilité totale augmentera en augmentant de la dernière pomme est 10 et celle de
la consommation de X et en diminuant la dernière banane, 5. Si le prix d’une pomme
la consommation de Y. est de 0,50 $, quel serait le prix d’une banane?
a) 0,05 $
b) 0,10$
c) 0,25 $
6 Lorsque le revenu diminue, 1 utilité marginale d) 0,50 $

que procure un bien augmente. e) 1,00$


90 CHAPITRE 7

La demande du marché est b) Alfred est plus disposé que Bernard à payer
a) la somme des prix que chaque personne un montant plus élevé pour obtenir la
est disposée à payer pour chaque quantité dernière barre de chocolat.
demandée. c) le surplus du consommateur de Bernard
b) la somme des quantités demandées est supérieur à celui d’Alfred.
par chaque personne pour chaque prix. d) le surplus du consommateur d’Alfred
c) la somme du surplus du consommateur est supérieur à celui de Bernard.
par individu. e) le surplus du consommateur de Bernard
d) la différence entre le montant maximal est égal à celui d’Alfred.
que chaque individu est disposé
à payer pour obtenir un bien et le prix 8 Un consommateur se trouve en situation
du marché. d’équilibre lorsque
e) la différence entre le prix du marché et a) Futilité totale est maximisée compte
le montant maximal que chaque individu tenu de son revenu et des prix
est prêt à payer pour obtenir un bien. des biens.
b) Futilité marginale est maximisée
Si Sylvie maximise son utilité et que deux compte tenu de son revenu et des prix
biens ont la même utilité marginale, alors elle des biens.
a) achètera un bien seulement. c) Futilité marginale par dollar dépensé
b) achètera des quantités égales des deux biens. est maximisée compte tenu de son revenu
c) sera disposée à payer le même prix pour et des prix des biens.
chaque bien. d) Futilité marginale est la même
d) retirera la même utilité totale de chaque bien. pour tous les biens.
e) Aucune de ces réponses. e) Aucune de ces réponses.

Les bicyclettes et les patins à roues alignées Le kilogramme de calmars coûte 2 $ et le


sont des substituts. Selon la théorie de Futilité kilogramme de pieuvre, 1 $. Jacques n’achète
marginale, lorsque le prix des bicyclettes que de la pieuvre et obtient 10 unités
augmente, la quantité de bicyclettes demandée d’utilité du dernier kilogramme qu’il achète.
a) diminue, et la courbe de demande des patins Si Jacques a maximisé son utilité, Futilité
à roues alignées se déplace vers la droite. marginale, exprimée en unités, que lui
b) diminue, et la courbe de demande des patins procure le dernier kilogramme de calmars
à roues alignées se déplace vers la gauche. doit être
c) diminue, et la courbe de demande des patins a) supérieure à 10.
à roues alignées ne se déplace pas. b) inférieure à 10.
d) augmente, et la courbe de demande des patins c) supérieure à 20.
à roues alignées se déplace vers la droite. d) inférieure à 20.
e) augmente, et la courbe de demande des patins e) égale à zéro.
à roues alignées se déplace vers la gauche.

10 Supposons qu’Alain dépense tout son revenu


L’utilité totale est toujours
de 10 $ en livres de droit et en cravates
a) supérieure à Futilité marginale.
de soie. Les livres de droit coûtent 2 $ et
b) inférieure à Futilité marginale.
les cravates en soie, 4 $ (voir le tableau 7.1).
c) décroissante lorsque Futilité marginale
L’utilité marginale de chaque bien est
est décroissante.
indépendante de la quantité consommée de
d) décroissante lorsque Futilité marginale
l’autre bien.
est croissante.
e) croissante lorsque Futilité marginale
TABLEAU 7.1
est positive.

Utilité marginale
Bernard et Alfred consomment chacun 15 barres
de chocolat au prix courant. Si la courbe Quantité Livres de droit Cravates de soie
de demande de Bernard est plus élastique | 12 16
que la courbe de demande d’Alfred, alors
2 10 12
a) Bernard est plus disposé qu’Alfred à payer
3 8 8
un montant plus élevé pour obtenir la dernière
4 6 4
barre de chocolat.
L'UTILITÉ ET LA DEMANDE 91

Si Alain maximise son utilité, combien Si le prix courant est de 7 $, quel est le surplus
de cravates achètera-t-il ? du consommateur total ?
a) 0 a) 3$
b) l b) 4$
c) 2 c) 6 $
d) 3 d) 12$
e) 4 e) 27$

I I L’utilité totale est égale à I5 Serge maximise l’utilité de sa consommation


a) la somme des utilités marginales de chaque de bière et de gomme à mâcher. Si le prix
unité consommée. de la bière est supérieur au prix de la gomme
b) l’aire située au-dessous de la courbe à mâcher, nous savons alors avec certitude que
de demande mais au-dessus du prix a) Serge achète plus de bières que de gommes
du marché. à mâcher.
c) la pente de la courbe d’utilité marginale. b) Serge achète plus de gommes à mâcher
d) l’utilité marginale de la dernière unité, divisée que de bières.
par le prix. c) l’utilité marginale de la dernière bière achetée
e) l’utilité marginale de la dernière unité est supérieure à l’utilité marginale
consommée, multipliée par le nombre total de la dernière gomme à mâcher achetée.
d’unités consommées. d) l’utilité marginale de la dernière gomme
à mâcher achetée est supérieure à l’utilité
12 Les choix de consommation des ménages marginale de la dernière bière achetée.
sont déterminés par e) les utilités marginales de la dernière bière
a) les prix des biens et des services. et de la dernière gomme à mâcher achetées
b) le revenu. sont égales.
c) les préférences.
d) Toutes ces réponses. 16 Martin et Roland ont des préférences
e) a et b seulement. semblables, mais Martin a des revenus
plus élevés. Si chacun d’eux maximise
13 Le prix élevé des diamants par rapport son utilité, alors
à celui de l’eau reflète le fait qu’aux niveaux a) ils ont la même utilité totale.
de consommation habituels b) l’utilité totale de Martin est inférieure
a) l’utilité totale de l’eau est relativement faible. à celle de Roland.
b) l’utilité totale des diamants est relativement c) l’utilité marginale de Martin est inférieure
élevée. à celle Roland pour la plupart des biens.
c) l’utilité marginale de l’eau est relativement d) l’utilité marginale de Martin est supérieure
élevée. à celle de Roland pour la plupart des biens.
d) l’utilité marginale des diamants e) ils ont la même utilité marginale pour
est relativement faible. la plupart des biens.
e) Aucune de ces réponses.
I7 Quel énoncé, parmi les suivants, Test pas une
14 Le tableau 7.2 présente le barème de demande prédiction de la théorie de l’utilité marginale?

de billes de verre. a) Toutes choses étant égales par ailleurs, plus


le prix d’un bien est élevé, plus la quantité
TABLEAU 7,2 BARÈME DE DEMANDE demandée de ce bien est faible.
DE BILLES DE VERRE b) Toutes choses étant égales par ailleurs,
plus le prix d’un bien est élevé,
Prix Quantité plus la consommation de substituts
(en dollars par bille) demandée de ce bien est élevée.
c) Toutes choses étant égales par ailleurs,
10 1
moins le prix d’un bien est élevé, plus
9 2
la consommation de substituts de ce
8 3
bien est faible.
7 4
d) La loi de la demande.
6 5
e) L’utilité marginale est décroissante.
92 CHAPITRE 7

18 D’après le principe de l’utilité marginale 22 Selon la définition, la valeur d’un bien


décroissante, à mesure que la consommation correspond
d’un bien augmente, l’utilité totale a) au prix du marché.
a) diminue et finit ensuite par augmenter. b) au prix moyen que payent les consommateurs
b) diminue à un taux croissant. sur un marché.
c) diminue à un taux décroissant. c) au coût de production du bien.
d) augmente à un taux croissant. d) au prix le plus élevé qu’un acheteur
e) augmente à un taux décroissant. est disposé à payer pour obtenir ce bien.
e) à l’utilité totale qu’un acheteur donné retire
I9 Supposons que Madonna dépense tout son
de la consommation de toutes les unités
revenu de 6 $ en vernis à ongles violet et du bien.
en parures de cuir. Le vernis à ongles coûte
1 $ le flacon et les parures de cuir, 2 $ pièce
23 Si les croustilles étaient gratuites, combien
(voir le tableau 7.3). L’utilité marginale de en consommerait-on ?
chaque bien est indépendante de la quantité
a) Une quantité illimitée.
de l’autre bien consommée.
b) La quantité de croustilles à laquelle
leur utilité totale tombe à zéro.
TABLEAU 7.3 c) La quantité de croustilles à laquelle
leur utilité marginale tombe à zéro.
Utilité marginale
d) Aucune croustille puisque l’utilité marginale
Quantité Vernis à ongles Parures de cuir et le prix seraient égaux.
e) Aucune de ces réponses.
| 8 16
2 6 12
24 Dans un premier temps, tante Sophie maximise
3 4 10
l’utilité de sa consommation de biens X et Y
4 3 6
Le prix du bien X double, ceteris paribus. Pour
que tante Sophie puisse maximiser de nouveau
Si Madonna maximise son utilité, quelle
son utilité, la quantité du bien X quelle
est son utilité totale? consomme doit
a) 19
a) augmenter jusqu’à ce que l’utilité marginale
b) 28
du bien X ait doublé.
c) 38
b) diminuer de moitié par rapport au niveau
d) 42
précédent.
e) Aucune de ces réponses.
c) diminuer jusqu’à ce que l’utilité marginale

20 En condition d’équilibre, le consommateur


du bien X double.
d) diminuer jusqu’à ce que l’utilité marginale
égalise
a) l’utilité totale retirée de chaque bien.
du bien X diminue de moitié par rapport
à son niveau précédent.
b) l’utilité marginale retirée de chaque bien.
e) lui procurer un plaisir énorme.
c) l’utilité totale par dollar dépensé
sur chaque bien.
d) l’utilité marginale par dollar dépensé
25 Béatrice est en situation d’équilibre
du consommateur. Une augmentation
sur chaque bien.
de son revenu devra
e) le revenu total dépensé sur chaque bien
a) augmenter son utilité totale.
et l’utilité totale retirée de chaque bien.
b) diminuer son utilité totale.
21 Le prix de la bière est deux fois plus élevé que c) augmenter l’utilité marginale quelle retire
le prix du bacon. Si la consommation actuelle de tous les biens.
de Robert est telle que le ratio Umhière /Um hacon d) diminuer Futilité marginale quelle retire
est de 1 à 2, alors pour maximiser son utilité, de tous les biens.
Robert doit e) augmenter sa consommation de tous
a) consommer plus de bière et moins de bacon. les biens.
b) consommer la même quantité de bière
et de bacon.
c) consommer moins de bière et plus de bacon.
. d) faire augmenter le prix de la bière.
e) consommer deux fois plus de bière et réduire
de moitié sa consommation de bacon.
L’UTILITÉ ET LA DEMANDE 93

Problèmes à court développement 4 Un paradoxe apparent, qui gêne un grand


nombre de gens, est que, bien que le personnel
I Un consommateur maximise Futilité des garderies soit habituellement mal payé,
de sa consommation des biens A et B on dit souvent « que c’est le travail le plus
de manière à ce que important du monde». En vous fondant
sur un raisonnement économique, expliquez
UmA _ UmB
ce paradoxe.
Pb
5 Le tableau 7.4 donne les barèmes de demande
Le prix de A augmente alors en raison
de brocoli de trois personnes : Thomas, Jeanne
du déplacement de l'offre, comme il est
et Louis.
indiqué à la figure 7.1

TABLEAU 7.4 DEMANDE INDIVIDUELLE


FIGURE 7.1
DE BROCOLI

Quantité demandée
p . (en kilogrammes

<en dollars) _par semaine) _


par kilogramme) Thomas Jeanne Louis

0,50 10 4 10
0,75 9 2 7
1,00 8 0 4
1,25 7 0 1

a) Calculez le barème de demande du marché.


b) Sur un simple graphique, tracez les courbes
de demandes individuelles de Thomas,
de Jeanne et de Louis, ainsi que la courbe
D’après ce que vous savez sur les conditions
de demande du marché.
qui permettent de maximiser Futilité, expliquez
comment le consommateur arrivera à un nouvel
6 Les tableaux 7.5 et 7.6 indiquent Futilité que
équilibre. Montrez le lien entre votre explication
retire Anne de la consommation hebdomadaire
et le changement qui se produit sur le graphique.
de maïs soufflé et de barres de chocolat.

2 Supposez que les informations suivantes


TABLEAU 7.5 UTILITÉ QUE RETIRE ANNE
se rapportent à une consommatrice qui tente
DE LA CONSOMMATION
de répartir son revenu entre les biens X et Y,
DE MAÏS SOUFFLÉ
afin de maximiser son utilité. Le bien X coûte
2 $ l’unité et le bien F, 1 $ l’unité. Lorsqu’elle
Nombre de Utilité Utilité
dépense tout son revenu, Futilité marginale
sacs de maïs soufflé totale marginale
de la dernière unité du bien X est de 20 et celle
de la dernière unité du bien Y, de 16. 1 20
a) Pourquoi cette consommatrice n’est-elle pas 2 36
en situation d’équilibre? 3 50
b) Pour accroître son utilité, quel bien devrait- 4 12
elle consommer en plus grande quantité et 5 72
quel bien devrait-elle consommer en moins 6 80
grande quantité ?

3 Expliquer de quelle façon l’équilibre du


consommateur et le principe d utilité marginale
décroissante démontrent la loi de la demande.
94 CHAPITRE 7

7 Supposons que les préférences d’Anne soient


TABLEAU 7.6 UTILITÉ QUE RETIRE ANNE
les mêmes que celles indiquées au problème 6,
DE LA CONSOMMATION
mais que le prix d’une barre de chocolat double,
DE BARRES DE CHOCOLAT
passant à 1 $.
a) Construisez un nouveau tableau (semblable
Nombre de Utilité Utilité
au tableau 7.7) de Um/P pour le maïs soufflé
barres de chocolat totale marginale
et les barres de chocolat.
1 14 b) Anne continue de recevoir 4 $ par semaine.
2 26 Après le changement de prix, combien de sacs
3 10 de maïs soufflé et de barres de chocolat
4 44 consommera-t-elle par semaine ?
5 51 c) Le maïs soufflé et les barres de chocolat
6 57 constituent-ils des substituts ou des
compléments pour Anne ? Pourquoi ?
a) Ajoutez les données manquantes aux tableaux. d) En vous fondant sur les résultats obtenus,
b) Supposons que le prix d’un sac de maïs tracez la courbe de demande de barres de
soufflé soit de 1 $ et celui d’une barre chocolat d’Anne.
de chocolat, de 0,50 $. En vous servant e) Supposons que les sacs de maïs soufflé et les
des données présentées aux tableaux 7.5 barres de chocolat coûtent toujours 1 $ pièce,
et 7.6, remplissez les deux colonnes mais que maintenant les parents d’Anne lui
du tableau 7.7 lorsque Um/P correspond donnent 6 $ par semaine.
à utilité marginale divisée par le prix, i) Combien de sacs de maïs soufflé
c’est-à-dire à futilité marginale par dollar et de barres de chocolat Anne
dépensé. consommera-t-elle par semaine ?
ii) Le maïs soufflé et les barres de chocolat
TABLEAU 7.7 sont-ils des biens normaux? Pourquoi
ou pourquoi pas ?
Nombre de Nombre de
sacs de maïs barres de 8 Supposons qu’André Agassi consacre tout son
soufflé Um/P chocolat Um/P revenu de 8 $ à l’achat de bandeaux serre-tête et
de raquettes de tennis miniature (voir le tableau
1 1
7.8). Le prix d’un bandeau est de 2 $ et celui
2 2
d’une raquette de tennis, de 4 $. L’utilité
3 3
marginale de chaque bien est indépendante de la
4 4
quantité de l’autre bien qui est consommée.
5 5
6 6
TABLEAU 7.8
c) Ses parents lui donnent 4 $ par semaine.
Utilité marginale
Si Anne dépense tout ce montant en maïs
soufflé et en barres de chocolat, quelles Quantité Bandeaux Raquettes
réponses donneriez-vous aux questions
1 20 36
suivantes ?
2 18 32
i) Combien de sacs de maïs soufflé et de
3 16 20
barres de chocolat Anne consommera-
4 8 16
t-elle par semaine si elle maximise son
utilité ?
a) Si André maximise son utilité, combien
ii) Comment les conditions qui permettent
d’unités de chaque bien devra-t-il acheter?
de maximiser l’utilité seraient-elles
b) Si le revenu d’André s’élève à 24 $, combien
satisfaites ?
d’unités de chaque bien devra-t-il acheter ?
iii) Quelle est l’utilité totale?
c) D’après les données fournies, calculez
iv) Si Anne consommait trois sacs de maïs
l’élasticité-revenu de la demande de bandeaux
soufflé et 2 barres de chocolat, en
d’André.
utilisant les chiffres de futilité totale et de
Um/P, expliquez pourquoi elle ne peut
maximiser son utilité dans ces conditions.
L’UTILITÉ ET LA DEMANDE 95

Supposons que Victor maximise son utilité en 4 F Si les prix ne sont pas égaux, alors les utilités
consacrant la totalité de son revenu à l’achat marginales sont inégales lorsque la condition
de chauves-souris et de lézards (voir le tableau d’équilibre du consommateur est satisfaite.
7.9). L’utilité marginale de chaque bien O 5 V T consommation X —> -l Umx.
est indépendante de la quantité de l’autre -1 consommation Y —>î UmY■ Déplace
bien qui est consommée. le ratio Um/P vers l’égalité.
& 6 I Vrai pour un bien normal ; peut être
faux pour un bien inférieur.
TABLEAU 7.9
7 F La théorie fait des prédictions qui peuvent
être vérifiées à l’aide de données observables.
Utilité marginale
@ 8 I Vrai si c’est la dernière unité consommée,
Quantité Chauves-souris Lézards faux autrement (valeur > prix).
9 F Puisque de nombreuses unités sont
1 20 45
consommées, la disposition à payer
2 18 40
sera supérieure au prix.
3 16 25
10 V Déplacement de l’offre vers la droite (—» -l P)
4 8 20
—> T quantité consommée, car la disposition
à payer un plus grand nombre d’unités est
Victor a un revenu de 16 $. Le prix d’une
supérieure au prix.
chauve-souris est de 2 $, et il en achète 3.
Si l’utilité marginale que procure l’achat
du dernier lézard est de 40, calculez le
Questions à choix multiple
prix d’un lézard en utilisant deux méthodes
distinctes.
1 Définition.
2 c 10/0,5 = 5IPh pour Ph.
10 Le tableau 7.10 présente le barème de demande
3 b Voir le manuel.
hebdomadaire de pizzas de Dédé.
4 En raison de la condition maximale d’égalité
Um/P. Il n’est pas nécessaire qu’il y ait
TABLEAU 7.10 BARÈME DE DEMANDE de relation entre Um et la quantité ou UT.
DE PIZZAS Voir le manuel.
UT T lorsq ue Um positive, que Um soit 4-
Prix Quantité ou T. UT peut être § Um.
(par pizza) demandée 7 d La pente plus abrupte de la courbe
de demande d’Alfred —> T disposition
15$ 1
à payer pour les unités précédentes. Même
12$ 2
disposition à payer pour la dernière unité.
10$ 3
8 a Les consommateurs maximisent F UT. c et d
9$ 4
sont de mauvaises réponses parce que l’égalité
8$ 5
de Um/P correspond à une UT maximisée.
<3> 9 d Pour la pieuvre, UmJPp =10. Pour le calmar,
Si une pizza coûte 9 $, quel est le surplus
il faudrait que UmJ2 =10 afin que Umc soit
du consommateur de Dédé pour les pizzas
< 20.
suivantes achetées à ce prix ?
10 b Il achète 1 cravate (Um/P = 16/4) et 3 livres
a) première pizza
(Um/P= 8/2).
b) deuxième pizza
I I b est le surplus du consommateur. Pour c,
c) nombre total de pizzas
Um = pente de la courbe UT. Les réponses
d et e n’ont pas de sens.
12 Préférences et contraintes.
13 Pour les diamants: UT relativement faible,
RÉPONSES Um relativement élevée. Pour l’eau : UT
relativement élevée, Um relativement faible.

Vrai/Faux/1 ncertain 14 Pour 4 billes de verre achetées, le surplus du


consommateur (en dollars) = (10 — 7) +
(Justifiez votre réponse.)
( 9 - 7) + (8 - 7) + (7 - 7).

1 V Définition.
& 15 À partir d’une condition maximale d’égalité

2 V Parce que l’Um est positive mais décroissante. des Um/P. Il n’est pas nécessaire qu’il y ait

3 F î /> -> I Q -> î t/w. une relation entre Um et la quantité.


96 CHAPITRE 7

16 c Martin consomme une plus grande quantité b) Pour rendre égales les utilités marginales par
de chaque bien, son Um est donc plus faible. dollar dépensé (et, donc, augmenter Futilité),
17 e i Um est l’hypothèse de la théorie. elle devra augmenter sa consommation
G) 18 e Du fait que Futilité marginale est positive du bien Y et diminuer celle du bien X.
mais décroissante lorsque la consommation T Le principe de Futilité marginale décroissante
19 d Elle achète 2 vernis à ongle et 2 parures. implique qu’ainsi on baissera Futilité
UT=8 + 6+ 16+ 12. marginale du bien Y et on augmentera
20 d Définition. Um/P est la clé celle du bien X.
de la maximisation de l’utilité.
3 Supposons qu’un individu, en situation
21 c Pour égaliser Um/P, il faut T Umhière
d’équilibre du consommateur, consomme
et -l Umhacon. d est faux car Robert
X0 unités du bien X et Y0 unités du bien Y,
n’a aucune emprise sur les prix.
les prix des biens A et de Y étant Px et PY,
22 d Définition.
respectivement. Cela signifie qu’aux niveaux de
G) 23 c On maximise UT. Avec moins de croustilles,
consommation X0 et Y0, Futilité marginale par
I ’Um est toujours positive, Y UT pourrait
dollar dépensé pour le bien X est égale à Futilité
donc T. Avec plus de croustilles, 1 ’Um devient
marginale par dollar dépensé pour le bien Y.
négative, F UT devrait donc -t.
Voyons ce qui se produit lorsque le prix du bien
24 c De la condition maximale d’égalité Um/P.
X augmente pour passer à Px. L’utilité marginale
II n’est pas nécessaire qu’il y ait de relation
par dollar dépensé pour le bien X diminue
entre YUm et la quantité consommée.
et elle est donc maintenant inférieure à Futilité
G) 25 a Pour des biens inférieurs, la consommation
marginale par dollar dépensé pour le bien Y.
peut si et Um î.
Pour rétablir l’équilibre, le consommateur doit
augmenter Futilité marginale du bien X et
Problèmes à court développement diminuer l’utilité marginale du bien Y. Selon
le principe de l’utilité marginale décroissante,
1 Lorsque le prix de A augmente, ceteris paribus : nous savons que le seul moyen de le faire
est de diminuer la consommation du bien X
UmA UmB
et d’augmenter celle du bien Y. La loi de
la demande est ainsi démontrée, car on voit
qu’une augmentation du prix du bien X devait
Le consommateur n’est plus en situation
être accompagnée d’une diminution de la
d’équilibre. Pour rétablir l’égalité, il doit modifier
consommation du bien X pour que l’équilibre
sa consommation de sorte que UmA augmente
du consommateur soit rétabli.
et UmB diminue. (Le consommateur ne peut
modifier les prix des biens A et de B). Étant 4 Alors que la majorité des parents sont conscients
donné que Futilité marginale diminue avec de la valeur des services de garde d’enfants,
l’augmentation de la quantité consommée, le de nombreux travailleurs de garderie sont prêts
consommateur doit réduire sa consommation à offrir ces services. Bien que Futilité totale des
du bien A et augmenter sa consommation du services de garde soit élevée, Futilité marginale
bien B. Une consommation plus faible du bien de ces services (offre abondante) est faible.
A entraîne un mouvement vers la gauche le long
5 a) On obtient le barème de demande du marché
de la courbe de demande du consommateur,
en ajoutant à chaque prix les quantités
à partir du point d’intersection initial
demandées par Thomas, Jeanne et Louis
de D et de O0 et jusqu’au nouveau point
(voir le tableau 7.11).
d’intersection de D et Ot. Au nouvel équilibre
du consommateur, les deux termes de l’équation
TABLEAU 7.11 BARÈME DE DEMANDE
sont devenus égaux.
DU MARCHÉ DU BROCOLI

2 a) Cette consommatrice n’est pas en équilibre,


Prix Quantité demandée
car Futilité marginale par dollar dépensé n’est
(en dollars (en kilogrammes
pas la même pour les biens X et Y. L’utilité
par kilogramme) par semaine)
marginale par dollar dépensé pour le bien X
est Umx! Px= 20/2 = 10, ce qui est inférieur 0,50 24
à Futilité marginale par dollar dépensé pour 0,75 18
- le bien Y: UmY/ PY — 16. 1,00 12
1,25 8
L'UTILITÉ ET LA DEMANDE 97

b) La figure 7.2 illustre les courbes de demande b) Voir le tableau 7.7 Solution,
individuelles de Thomas, Jeanne et Louis _________________________
ainsi que la courbe de demande du marché. TABLEAU 7.7 SOLUTION

FIGURE 7.2 Nombre de Nombre de


sacs de maïs barres de
soufflé UmlP chocolat UmlP

1 20 1 28
2 16 2 24
3 14 3 20
4 12 4 16
5 10 5 14
6 8 6 12

c) i) 2 sacs de maïs soufflé et 4 barres


de chocolat.
ii) Les conditions qui permettent de
6 a) Voir les tableaux 7.5 et 7.6 Solution. maximiser Futilité sont satisfaites puisque
Anne dépense tout son revenu (4 $),
TABLEAU 7.5 SOLUTION et Futilité marginale par dollar dépensé
UTILITÉ QUE RETIRE ANNE est la même pour les deux biens (16).
DE LA CONSOMMATION iii) L’utilité totale est Futilité retirée de
DE MAÏS SOUFFLÉ la consommation de 2 sacs de maïs (36)
plus Futilité retirée de la consommation

Nombre de Utilité Utilité de 4 barres de chocolat (44) = 80.

sacs de maïs soufflé totale marginale iv) Si Anne consomme 3 sacs de maïs soufflé
et 2 barres de chocolat, Futilité totale
1 20 20 serait de 76, donc inférieure à 80, soit
2 36 16 Futilité totale quelle retire de la
3 50 14 consommation de 2 sacs de maïs soufflé
4 62 12 et de 4 barres de chocolat. En ce qui
5 72 10 concerne la combinaison de 3 sacs de
6 80 8 maïs soufflé et de 2 barres de chocolat,
XUm/P pour le maïs est de 14, alors
queXUmIP pour les barres de chocolat
TABLEAU 7.6 SOLUTION est de 24. Comme le ratio Um/P n’est
UTILITÉ QUE RETIRE ANNE pas le même pour ces deux biens, cette
DE LA CONSOMMATION combinaison ne lui permet pas
DE BARRES DE CHOCOLAT de maximiser son utilité.

Nombre de Utilité Utilité 7 a) Voir le tableau 7.12.


barres de chocolat totale marginale

14 14 TABLEAU 7.12
1
2 26 12
Nombre de Nombre de
3 36 10
sacs de maïs barres de
4 44 8
soufflé UmlP chocolat UmlP
5 51 7
6 57 6 1 20 1 14
2 16 2 12
3 14 3 10
4 12 4 8
5 10 5 7
6 8 6 6
98 CHAPITRE 7

b) 3 sacs de maïs soufflé et 1 barre de chocolat. 8 a) Le tableau 7.3 donne la combinaison de biens
Anne dépense tout son revenu (4 $) et son qui permet de maximiser Futilité.
utilité marginale par dollar dépensé est la
même pour le maïs soufflé et pour les barres TABLEAU 7.13
de chocolat (14).
c) Le maïs soufflé et les barres de chocolat UmlP
sont des substituts pour Anne, puisqu’une
Quantité Bandeaux Raquettes
augmentation du prix des barres de chocolat
fait augmenter la demande de maïs soufflé. 1 10 9
d) La courbe de demande de barres de chocolat 2 9 8
de Anne est illustrée à la figure 7.3. Nous 3 8 5
connaissons deux points situés sur cette 4 4 4
courbe de demande : lorsque le prix d’une
barre de chocolat est de 1 $, Anne en achète André devra acheter 2 bandeaux et 1 raquette.
une, et lorsque le prix est de 0,50 $, elle André dépense tout son revenu (8 $) et l’utilité
en achète quatre. La courbe de demande marginale par dollar dépensé est identique pour
est la droite qui relie ces deux points. les bandeaux et les raquettes (9).
b) André devra acheter 4 bandeaux et 4
FIGURE 7.3 raquettes. Il dépense tout son revenu (24 $),
et l’utilité marginale par dollar dépensé est
identique pour les bandeaux et les raquettes (4).
c) L’élasticité-revenu de la demande de
bandeaux est

AQ 4—2 2
= Qny _ i (4 + 2) _ 3 _ 2
AY_ 24-8 16 3

7(24 + 8) 16

9 II y a deux façons de calculer le prix d’un lézard.


L’une se fonde sur le fait que Victor dépense
tout son revenu en chauves-souris et en lézards.
Ainsi

4"- PcsQcs + PlQl

e) i) Dans ces conditions, Anne consommera où Y est le revenu, Pcs — le prix d’une chauve-
4 sacs de maïs soufflé et 2 barres de souris, Qcs = la quantité de chauves-souris
chocolat (au lieu de 3 sacs de maïs soufflé achetée, PL = le prix d’un lézard, et Qz = la
et 1 barre de chocolat). Anne dépense quantité de lézards achetée. Nous savons que Y
tout son revenu (6 $) et son utilité = 16, Pcs = 2, Qcs = 3, et si l’utilité marginale
marginale par dollar dépensé est la même du dernier lézard est de 40, alors Qi = 2. En
pour le maïs soufflé et les barres de utilisant ces valeurs, nous pouvons obtenir PL.
chocolat (12).
16 = 2(3) + PL( 2)
ii) Le maïs soufflé et les barres de chocolat
10 = PL( 2)
sont tous deux des biens normaux pour
Anne puisque la demande de ces deux
5 =PL
biens augmente avec son revenu (l’argent La seconde méthode de calcul se fonde sur
que ses parents lui donnent par semaine). cette condition de la maximisation de Futilité :

Umcs _ UmL
L'UTILITÉ ET LA DEMANDE 99

Nous savons que UmL = 40, Pcs = 2, et si 15 $, mais le prix d’une pizza n’est que
Victor achète 3 chauves-souris, Umcs— 16. de 9 $. Le surplus du consommateur est
En utilisant ces valeurs, nous pouvons calculer PL. 6$ (15$-9$).
b) Le surplus du consommateur de Dédé pour
la deuxième pizza est la différence entre
16 _ 40
le prix le plus élevé qu’il serait prêt à payer
2 " PL (12 $) et le prix actuel (9 $). Son surplus
du consommateur est de 3 $.
80 = 1 6Pl
c) Au prix de 9 $, Dédé achètera 4 pizzas.
5 = Pl Son surplus du consommateur sur les trois
premières pizzas s’élève donc à 6 $, 3 $ et
10 a) Le prix le plus élevé que Dédé est disposé 1 $, respectivement. De ce fait, son surplus
à payer pour la première pizza est de du consommateur total est égal à 10 $.
Chapitre

Contraintes budgétaires,
préférences et choix
de consommation

Les préférences et
CONCEPTS CLÉS les courbes d’indifférence

La courbe d’indifférence illustre les préférences des


Les possibilités de consommation
ménages, en reliant les combinaisons de biens qui leur
procurent une satisfaction égale.
La droite de budget indique les limites de la
consommation des ménages, compte tenu du ♦ En général, les courbes d’indifférence ont
revenu et des prix des biens et des services. Prenons une pente négative et sont arquées par rapport
l’exemple des films (abscisse) et des sodas (ordonnée). à l’origine (convexes).

♦ Contrainte du budget: Y— PjQy + PsQ ♦ Les courbes d’indifférence plus éloignées de


l’origine représentent des niveaux de satisfaction
Isolez Q — —-—Qf plus élevés.
P P
♦ Les courbes d’indifférence ne se croisent jamais.

♦ L’équation de budget est ♦ Le taux marginal de substitution ( TmS)


correspond à la valeur de la pente de la courbe
d’indifférence — le taux auquel les ménages
renoncent au bien y (soda) pour obtenir
des unités supplémentaires du bien x (films)
♦ La valeur de la pente de l’équation de budget qui leur procurent une égale satisfaction.
{Pf/P) est égale au prix relatif d’un film exprimé ♦ Le taux marginal de substitution décroissant est
en sodas. la tendance du TmS à X en se déplaçant vers le bas
♦ Les valeurs sur les axes mesurent le revenu réel du le long d’une courbe d’indifférence. On lui doit la
ménage exprimé en films (abscisse) et en sodas forme arquée (convexe) par rapport à l’origine des
(ordonnée). courbes d’indifférence.

♦ T Pf-> droite de budget plus abrupte, ordonnée ♦ T substituabilité entre les biens —> des courbes
à l’origine (sodas) fixe. d’indifférence plus droites.
substituabilité entre les biens —> des courbes
♦ T Ps —» droite de budget plus plate, abscisse d’indifférence plus arquées.
à l’origine (films) fixe.

♦ T y —> déplacement parallèle vers la droite de la Les choix de consommation des ménages
droite de budget.

Un ménage répartit ses dépenses en fonction


de son revenu et des prix des biens, de manière
à en retirer le plus de satisfaction possible. Le ménage
CONTRAINTES BUDGÉTAIRES, PRÉFÉRENCES ET CHOIX DE CONSOMMATION 101

choisit le meilleur point accessible situé sur sa droite que les questions étudiées ici se placent dans
de budget et sur la courbe d’indifférence la plus élevée. un contexte plus large. D’un point de vue
À ce meilleur point accessible : économique très général, le consommateur doit
prendre les meilleures décisions, compte
♦ le ménage dépense tout son revenu et retire
tenu de ses contraintes. Celles-ci, qui limitent
le plus de satisfaction ;
son éventail de choix, varient en fonction
♦ la droite de budget et la courbe d’indifférence du revenu et du prix des biens. Elles sont
sont tangentes et elles ont la même pente - le TmS représentées graphiquement par la droite
de la courbe d’indifférence est égal au prix relatif de budget. Pour le consommateur, prendre les
(de la droite de budget). meilleures décisions possibles, c’est obtenir les
meilleurs résultats compte tenu des contraintes
qui limitent ses choix. Dans ce chapitre, les
Comment prédire le comportement
préférences sont représentées graphiquement
des consommateurs
par les courbes d’indifférence.
Le problème du consommateur, illustré
L’effet de prix est la À de la consommation découlant
sur le graphique, est de trouver la courbe
de la À de prix d’un bien. L’effet de prix = effet de
d’indifférence accessible la plus éloignée,
substitution + effet de revenu.
compte tenu de sa droite de budget. Pour
L’effet de substitution est la A de la consommation faciliter l’analyse graphique, nous limiterons
résultant d’une A de prix accompagnée d’une A notre choix à deux biens seulement, mais
(hypothétique) du revenu de sorte que le ménage les mêmes principes s’appliquent aux situations
peut opter indifféremment pour la combinaison initiale réelles, où l’éventail de choix est bien plus large.
ou pour la nouvelle combinaison. Pour les biens 2 L’équation du budget pour les sodas et les films
normaux et les biens inférieurs, l’effet de substitution (page 169 du manuel) est
de la nI du prix —» T de la consommation.
P,
♦ L’effet de revenu est la A de la consommation Q/
P,
résultant du rétablissement (hypothétique)
du revenu initial, tout en maintenant les prix Il s’agit du type d’équation de droite
constants au nouveau niveau. Pour les biens de budget (y — a + bx) dont nous avons parlé
normaux, l’effet de revenu de T (hypothétique) au chapitre 2. Les différences sont que Q est
du revenu —> T consommation. Pour les biens la variable dépendante (au lieu de y) et Qy,
inférieurs, la T (hypothétique) du revenu la variable indépendante (au lieu de x). Nous

—> 'l de la consommation. pouvons utiliser l’équation de budget pour tracer


la droite de budget en trouvant l’ordonnée à
♦ La courbe de demande à pente négative est la l’origine Qs (point où la droite croise l’ordonnée
conséquence du choix de la meilleure combinaison Qs), en trouvant l’abscisse à l’origine Qy(point
de biens accessible au consommateur. On peut où la droite croise l’abscisse Qy) et en reliant
tracer la courbe de demande en se fondant ces deux points par une ligne droite.
sur l’effet de prix — en déterminant la meilleure Pour trouver l’ordonnée à l’origine Q,,
quantité d’un bien accessible au fur et à mesure supposons que Qy = 0
que le prix de ce bien change.

♦ Pour les biens normaux, l’effet de substitution Q, = —-^(


P P 0)
et l’effet de revenu s’exercent dans le même sens,
aussi une >1 du prix —> T de la consommation.
Q
♦ Pour les biens inférieurs, l’effet de substitution P
et l’effet de revenu s’exercent en sens oppose, mais Pour trouver l’abscisse à l’origine Qy,
les effets nets d’une si de prix —> habituellement supposons que Q^ = 0
une T (plus faible) de la consommation.

p
ü = --—Qf
p

rappels y_
/ -
p.
I Bien que l’analyse du modèle propose dans
ce chapitre puisse sembler étroite et restrictive Q
à première vue, il importe de se rappeler
102 CHAPITRE 8

L’ordonnée à l’origine Qs, Qs — y/Ps> marginale décroissante qui sous-tend l’équation


correspond au revenu réel du consommateur suivante :
exprimé en sodas. Il nous indique le nombre
de canettes que le consommateur peut s’acheter
Taux marginal _ U^x
en dépensant tout son revenu. L’abscisse à l’origine
de substitution UmY
Qf, Qf= y/Pf, correspond au revenu réel du En haut de la courbe d’indifférence, la
consommateur exprimé en films. Il nous indique consommatrice consomme une petite quantité
le nombre de films que le consommateur peut du bien X et une grande quantité du bien Y,
aller voir, s’il consacre tout son revenu à l’achat de sorte que l’unité marginale du bien X (Umx)
de billets de cinéma. L’ordonnée et l’abscisse est élevée et celle du bien Y (Umr), faible.
à l’origine nous offrent un moyen facile de tracer En descendant le long de la courbe, à mesure
une droite de budget. Chacun des deux points que la quantité consommée du bien X augmente,
extrêmes représentent le revenu divisé par le prix YUmx diminue; et à mesure que la quantité
du bien sur l’axe considéré. En reliant ces deux consommée du bien T diminue, \’UmY augmente.
points par une ligne droite, nous obtenons la Par conséquent, le principe de l’utilité marginale
droite de budget. décroissante nous permet de comprendre
La pente de la droite de budget nous intuitivement pourquoi le TmS diminue lorsqu’on
informe aussi sur le choix du consommateur. descend le long d’une courbe d’indifférence.
La valeur (absolue) de la pente est égale au prix
relatif (ou coût d’opportunité) des films exprimé ® 4 Au meilleur point accessible, la droite de budget
en sodas. Autrement dit, la valeur de la pente est du consommateur est tangente à la courbe
égale au nombre de canettes de sodas nécessaires d’indifférence la plus élevée, de sorte que
pour acheter un film. D’une manière plus la valeur de la pente de la droite de budget
générale, la valeur de la pente de la droite est égale à la valeur de la pente de la courbe
de budget (Px/P) est égale au prix relatif d’indifférence. Sachant que la valeur de la pente
(ou coût d’opportunité) du bien mesuré sur de la droite de budget est égale à Px/Py (voir
l’axe horizontal, l’axe des x, exprimé en unités le Rappel 2) et que la valeur de la pente de
de biens mesuré sur l’axe vertical, l’axe des y, la courbe d’indifférence est égale à Umy/Umy
ou encore, elle correspond au nombre d’unités (voir le Rappel 3)on peut conclure que:
du bien mesuré en ordonnée auquel il faut
renoncer pour pouvoir se procurer une unité Px_ _ Umx
du bien mesuré en abscisse. PY UmY

Ou encore :
® 3 Le taux marginal de substitution (TmS)
correspond à la quantité d’un produit Y que Umx _ UmY
le consommateur est prêt à sacrifier pour
se procurer une unité supplémentaire du
bien X, en retirant la même satisfaction de la Cette dernière équation permet de
nouvelle combinaison. Le TmS est égal à la maximiser l’utilité (chapitre 7). On peut
valeur de la pente de la courbe d’indifférence, donc constater que l’analyse de l’équation
AQy/AQx- du budget/courbe d’indifférence des choix
Les courbes d’indifférence étant convexes de la consommatrice présentée ici complète
par rapport à l’origine, la valeur absolue de l’analyse de l’utilité marginale que nous avons
la pente et, par voie de conséquence, le TmS, étudiée au chapitre 7.
baissent à mesure que l’on descend le long
de la courbe d’indifférence. La décroissance 5 Les étudiants ont parfois du mal à faire la
du TmS signifie que cette consommatrice est distinction entre l’effet de revenu et l’effet de
prête à renoncer à une moins grande quantité substitution d’une variation du prix d’un bien.
du bien Y pour se procurer chaque unité Considérons une baisse du prix du bien A.
supplémentaire du bien X. À mesure que Celle-ci a deux effets qui s’exerceront sur la
la consommatrice descend le long de la courbe consommation de ce bien. D’abord, la baisse
d’indifférence, elle accorde de plus en plus du prix du bien A fera baisser le prix relatif de
de valeur au bien Y et de moins en moins ce bien et, ensuite, fera augmenter le revenu réel.
de valeur au bien X. Ce comportement peut L’effet de substitution nous permet de répondre
facilement s’expliquer par le principe de futilité à la question suivante : De combien
la consommation du bien A varie-t-elle quand
CONTRAINTES BUDGÉTAIRES, PRÉFÉRENCES ET CHOIX DE CONSOMMATION 103

le prix relatif de ce bien diminue si, par 7 Au meilleur point de consommation de


hypothèse, le revenu baisse suffisamment films et de sodas accessible, le taux marginal
pour que le consommateur puisse opter de substitution est égal au ratio du prix
indifféremment pour la combinaison initiale des films sur le prix des sodas.
ou pour la nouvelle combinaison ? Quant à
l’effet de revenu, il nous permet de répondre
à la question suivante : De combien la
consommation du bien A variera-t-elle si, par 8 Lorsque le prix relatif d’un bien diminue,
hypothèse, le revenu réel est rétabli à son niveau l’effet de revenu fait augmenter la consommation
initial, mais que les prix relatifs restent constants de ce bien.
à leur nouveau niveau ?

9 On peut déduire la loi de la demande


d’un modèle fondé sur la courbe d’indifférence
AUTOÉVALUATION à partir de l’effet qu’exerce une augmentation
de prix sur la quantité demandée.
Vrai/Faux/Incertain
(Justifiez votre réponse.)
10 Selon la théorie des choix du consommateur,
i La droite de budget est convexe par rapport les acheteurs calculent effectivement les taux
à l’origine. marginaux de substitution et arrêtent leurs
décisions quant aux dépenses de façon
à ce que les TmS soit égaux aux prix relatifs.

2 Ceteris paribus, la hausse du prix des biens


se traduit pour une diminution du revenu réel.
Questions à choix multiple

1 Quel énoncé, parmi les suivants, décrit le mieux

3 On suppose que, pour tous les biens, la droite de budget d’un consommateur ?

les consommateurs préfèrent en consommer a) La quantité de chaque bien qu’il peut acheter.
b) Les limites des possibilités de choix
plus que moins.
de consommation accessibles.
c) Son niveau désiré de consommation.
d) Ses choix de consommation.
e) L’ensemble de tous les choix
4 Les courbes d’indifférence plus élevées
de consommation accessibles.
correspondent à des niveaux de revenu
plus élevés.
2 Le revenu réel est exprimé en
a) unités monétaires.
b) unités de prix.
c) unités de satisfaction.
5 Le principe du taux marginal de substitution
d) unités d’indifférence.
décroissant permet d’expliquer pourquoi
e) unités de biens.
les courbes d’indifférence sont convexes
par rapport à l’origine.
3 Pour une augmentation de prix, l’effet
de substitution
a) fait toujours accroître la consommation.
b) fait accroître la consommation des biens
6 Les courbes d’indifférence des substituts parfaits
normaux seulement.
sont en forme de L.
c) fait baisser la consommation des biens
normaux seulement.
d) fait baisser la consommation des biens
inférieurs seulement.
e) Aucune de ces réponses.
104 CHAPITRE 8

Lorsque deux biens sont des substituts parfaits, c) Les courbes d’indifférence sont concaves
alors leurs par rapport à l’origine.
a) courbes d’indifférence sont des lignes droites d) Le taux marginal de substitution correspond
à pente positive. à la valeur de la pente d’une courbe
b) courbes d’indifférence sont des lignes droites d’indifférence.
à pente négative. e) Le taux marginal de substitution augmente
c) courbes d’indifférence sont en forme de L. alors que la courbe d’indifférence se déplace
d) taux marginaux de substitution sont égaux vers le haut.
à zéro.
e) taux marginaux de substitution sont infinis. 8 La droite de budget initiale {RS à la figure 8.2)
se déplace vers TR à la suite
Examinez la droite de budget et la courbe a) d’une augmentation du prix du bien X.
d’indifférence de la figure 8.1. Lorsque le prix b) d’une diminution du prix du bien X.
du bien X est de 2 $, quel est le prix du bien Y? c) d’une diminution des préférences pour le bien X.
a) 0,37 $ d) d’une augmentation du prix du bien Y
b) 0,67 $ e) d’une augmentation du revenu réel.
c) 1,50$
d) 2,67 $ FIGURE 8.2
e) Impossible à calculer avec les données dont
on dispose.

FIGURE 8.1

9 Lorsque la droite de budget initiale


(RS à la figure 8.2) se déplace vers TR,
l’effet de substitution est illustré par
le mouvement du point
a) a au point b.
L’équation initiale du budget pour les sodas et b) a au point c.
les films est Qs = 20 - 4Qf, et le prix du soda c) a au point d.
(Ps) est de 5 $. Lorsque le prix du soda diminue, d) b au point d.
passant à 4 $, quelle est l’équation du nouveau e) d au point c.
budget ?
a) Qs = 25 - 2 Qf I0 Lorsque la droite de budget initiale (RS à la
b) CL = 25 - 4 Qy figure 8.2) se déplace vers TR, l’effet de revenu
c) QJ = 25-5Q/ est illustré par le mouvement du point
d) Q_s = 20 - 5 Qf a) a au point b.
e) Aucune de ces réponses. b) a au point c.
c) a au point d.
Quel énoncé, parmi les suivants, est faux? d) b au point c.
a) Les courbes d’indifférence ont une pente e) b au point d.
négative.
b) Une carte des préférences consiste en
une série de courbes d’indifférences qui
ne se croisent pas.
CONTRAINTES BUDGÉTAIRES. PRÉFÉRENCES ET CHOIX DE CONSOMMATION 105

I I Le revenu de Carine lui permet d’acheter


FIGURE 8.3
3 tomates si elle n’achète pas de brosse à dents
ou 2 brosses à dents et si elle n’achète pas de
tomates. Le prix relatif des brosses à dents (prix
d’une brosse à dents/ prix d’une tomate) est
a) 2/3.
b) 3/2.
c) 6/1.
d) 1/6.
e) Impossible à calculer avec les données
dont on dispose.

I2 En général, à mesure que le consommateur


descend le long d’une courbe d’indifférence,
en augmentant sa consommation du bien X
(mesuré sur l’axe horizontal)
a) il doit renoncer à consommer davantage
de biens Y pour chaque unité supplémentaire I5 Lorsque le revenu augmente, la droite de budget
du bien X. a) devient plus abrupte.
b) il doit renoncer à une quantité constante b) devient moins abrupte.
du bien Y pour chaque unité supplémentaire c) se déplace vers la gauche, mais parallèlement
du bien X. à la droite de budget initiale.
c) il doit renoncer à une moins grande quantité d) se déplace vers la droite, mais parallèlement
du bien Y pour chaque unité supplémentaire à la droite de budget initiale.
du bien X. e) se déplace parallèlement, mais vers la gauche
d) le prix relatif du bien Y augmente. ou vers la droite, selon qu’il s’agit d’un bien
e) le prix relatif du bien Y diminue. normal ou d’un bien inférieur.

13 Si le prix du bien mesuré sur l’axe vertical I6 Lorsque le prix du bien X (mesuré en abscisse)
augmente, la droite de budget baisse, l’effet de substitution se traduit par
a) devient plus abrupte. un déplacement
b) devient moins abrupte. a) sur une courbe d’indifférence plus élevée.
c) se déplace vers la gauche, mais parallèlement b) sur une courbe d’indifférence plus basse.
à la droite de budget initiale. c) vers une partie plus abrupte de la même
d) se déplace vers la droite, mais parallèlement courbe d’indifférence.
à la droite de budget initiale. d) vers une partie moins abrupte de la même
e) se déplace vers la gauche et devient courbe d’indifférence.
plus abrupte. e) vers une partie moins abrupte d’une courbe
d’indifférence plus élevée.
14 Parmi les énoncés suivants se rapportant à la
figure 8.3, trouvez celui ou ceux qui sont justes. I7 Une variation du prix du bien mesuré en
a) Le consommateur préfère le point s au point abscisse (x) fait varier un ou plusieurs éléments
q, mais le point r n’est pas accessible. de l’équation de budget. De quels éléments
b) Les points q et r procurent la même utilité, s’agit-il ?
mais le point q est plus accessible. a) Pente et ordonnée à l’origine
c) Le consommateur préfère le point t au point b) Pente et abscisse à l’origine
q, mais le point t n’est pas accessible. c) Abscisse et ordonnée à l’origine, mais non
d) Le coût des combinaisons de consommation la pente
aux points q et s est le même, mais le d) Pente seulement
consommateur préfère le point q au point s. e) Aucune de ces réponses.

e) Toutes ces réponses.


18 Bernard consomme des pommes et des bananes.
Supposons que le revenu de Bernard double
et que le prix des pommes et des bananes double
également. La droite de budget de Bernard
a) se déplacera vers la droite, mais la pente
ne changera pas.
106 CHAPITRE 8

b) restera identique. c) sera relativement constant.


c) se déplacera vers la droite, mais la pente ne d) diminuera plus vite que le TmS des substituts.
changera pas e) diminuera moins vite que le TmS
d) se déplacera vers la droite et la pente des substituts.
deviendra plus abrupte.
e) se déplacera vers la droite et deviendra moins 23 L’effet de revenu
abrupte. a) est habituellement plus grand que l’effet
de substitution.
19 Lorsque le prix d’un bien inférieur diminue, b) est habituellement plus grand que l’effet
1 l’effet de revenu et l’effet de substitution font de prix.
tous deux déplacer la quantité demandée dans c) ajouté à l’effet de prix est égal à l’effet
le même sens. de substitution.
2 l’effet de revenu et l’effet de substitution font d) moins l’effet de prix est égal à l’effet
déplacer la quantité demandée en sens opposé. de substitution.
3 l’effet de revenu est habituellement plus grand e) ajouté à l’effet de substitution est égal
que l’effet de substitution. à l’effet de prix.
4 l’effet de substitution est habituellement
plus grand que l’effet de revenu. 24 La droite de budget dépend
a) 1 et 2 a) du revenu.
b) 1 et 4 b) des prix.
c) 2 et 3 c) du revenu et des prix.
d) 2 et 4 d) des préférences.
e) Aucune de ces réponses. e) des préférences et des prix.

20 Quel énoncé, parmi les suivants, concernant 25 La forme d’une courbe d’indifférence dépend
la droite de budget est faux ? La droite de budget a) du prix des biens.
a) sépare les points de consommation accessibles b) du revenu du ménage.
et les points de consommation inaccessibles. c) du degré de substituabilité des biens
b) est fondée sur les prix fixes. du ménage.
c) est fondée sur le revenu fixe. d) du niveau de satisfaction du ménage.
d) est fondée sur des quantités fixes. e) Toutes ces réponses.
e) limite les choix du consommateur.

21 Lorsque Clark Gable a enlevé sa chemise Problèmes à court développment


dans le film C’est arrivé une nuit, il ne portait
pas de maillot de corps. Ce qui a provoqué 1 Expliquez pourquoi une courbe d’indifférence
une forte baisse des ventes de maillots de corps. a une pente négative ?
Ceteris paribus, nous pouvons conclure qu’en
matière de maillots de corps, 2 Expliquez pourquoi il est logiquement impossible
a) les préférences des hommes ont changé que les courbes d’indifférence se croisent, si l’on
en fonction des variations de prix. compare les points a, b et c de la figure 8.4
b) les préférences des hommes ont changé
en fonction des variations de revenu. FIGURE 8.4
c) les choix des hommes ont changé lorsque
les préférences ont changé.
d) les choix ont changé lorsque les prix
ont changé.
e) les choix ont changé lorsque les revenus
ont changé.

22 En descendant le long d’une courbe


d’indifférence, le taux marginal de substitution
(TmS) des compléments
a) augmentera plus vite que le TmS
' des substituts.
b) augmentera moins vite que le TmS
des substituts.
CONTRAINTES BUDGÉTAIRES, PRÉFÉRENCES ET CHOIX DE CONSOMMATION 107

© 3 Au Rappel 3, il est établi que le


FIGURE 8.5
Taux marginal _ utilité marginale des films
de substitution utilité marginale des sodas

En descendant le long d’une courbe


d’indifférence, utilisez le principe de l’utilité
marginale décroissante pour expliquer pourquoi
le taux marginal de substitution diminue.

4 Pour les biens normaux,


• une augmentation du revenu fait augmenter
la demande (la courbe de demande se déplace
vers la droite).
• l’effet de revenu (dû à une baisse de prix)
fait augmenter la quantité demandée.
Ces deux énoncés sont-ils contradictoires ?
Expliquez pourquoi, en définissant bien
clairement toute notion importante.

5 Jeanne et Daniel aiment tous deux le pain


et le beurre d’arachide et ils ont le même revenu.
Comme les prix de ces biens sont les mêmes a) i) Initialement, le prix d’une unité de
pour tous deux, leurs droites de budget sont vêtement Pv est de 1 $ et celui d’une
identiques. Actuellement, Jeanne et Daniel unité de nourriture Pn est de 1,50 $.
consomment exactement la même quantité Si le revenu y est de 9 $, tracez la droite
de pain et de beurre d’arachide ; leur meilleur de budget du consommateur de la figure
point de consommation accessible est le même. 8.5 et nommez le meilleur point
Toutefois, Jeanne considère le pain et le beurre accessible a. Quelle est la quantité
d’arachide comme étant des substituts immédiats de nourriture consommée ?
(bien qu’imparfaits), alors que Daniel les ii) Quelle est l’équation de la droite de
considère comme des biens complémentaires budget exprimée en Pv, Pn et j/? Quelle
(mais pas parfaitement). est l’équation de la droite de budget
a) Sur le même graphique, tracez une droite exprimée en chiffres ? Exprimez la valeur
de budget ainsi que les courbes d’indifférence (absolue) de la pente de la droite de
représentatives de Jeanne et de Daniel. budget sous forme de ratio de Pv sur Pn.
(La quantité de pain est mesurée en abscisse.) iii) Au point a, quelle est la valeur du taux
b) Supposons maintenant que le prix du pain marginal de substitution (ou, de façon
diminue. Représentez graphiquement les équivalente, de Umn/Umv) ?
effets de substitution pour Jeanne et pour iv) À l’aide des résultats obtenus aux
Daniel. Pour lequel des deux l’effet questions ii et iii, trouvez la formule
de substitution est-il le plus important? de la maximisation de l’utilité. Expliquez
votre raisonnement.
& 6 La figure 8.5 illustre la carte d’indifférence b) Si Pn passe à 3 $ l’unité alors que le revenu
d’un consommateur en matière de nourriture et Pv ne changent pas, tracez la nouvelle droite
et de vêtements. de budget et appelez b le meilleur point
accessible. Quelle est la nouvelle quantité
de nourriture consommée ?
c) Si Pn augmente encore, passant à 4,50 $
l’unité, alors que le revenu et Pv ne changent
pas, tracez la droite de budget correspondante
et appelez c le meilleur point accessible. Quelle
est la quantité de nourriture consommée ?
d) Sur un autre graphique, tracez la courbe
de la demande de nourriture qui correspond
aux points a, b et c. N’oubliez pas de
nommer les axes.
108 CHAPITRE 8

e) Supposons que les prix ne varient pas ii) Sur le nombre total d’heures de squash
(Pv = 1 $, Pn = 1,50 $), mais que le revenu supplémentaires combien correspondent
baisse, passant à 3 $. Tracez la nouvelle droite à l’effet de substitution entraîné par la
de budget et nommez z le meilleur point baisse de prix ?
accessible. Quelle est la quantité de nourriture iii) Sur le nombre total d’heures de squash
consommée ? La nourriture est-elle un bien supplémentaires combien correspondent
normal ou un bien inférieur? Expliquez votre à l’effet de revenu entraîné par la baisse
raisonnement. Dans votre graphique de la de prix?
courbe de demande de la question d, trouvez c) Sur un autre graphique, représentez deux
le point ^ et tracez approximativement points sur la courbe de demande d’heures
une courbe de demande correspondant de squash de Sylvie et tracez sa droite
au nouveau niveau de revenu. de demande.
d) Le prix de location du court de squash
7 Sylvie, une fanatique de la condition physique, est toujours de 1 $, mais le budget que Sylvie
joue au squash et suit des cours de danse consacre à son conditionnement physique
aérobique à son centre de culture physique. est maintenant de 14 $. Combien d’heures de
La location du court de squash coûte de 2 $ squash et de danse aérobique Sylvie peut-elle
l’heure et les cours de danse aérobique coûtent se payer maintenant?
1 $ l’heure. Sylvie a décidé de dépenser 12 $
par semaine pour ses activités de conditionnement <5) 8 Mlle Mouffette consomme du lait et du
physique. La figure 8.6 présente plusieurs fromage. Le prix initial du fromage est de 1 $
courbes d’indifférence relatives au squash le morceau, et celui du lait, 1,50 $ le berlingot.
et à la danse aérobique de la carte des préférences Le revenu initial de Mlle Mouffette est de 12 $.
de Sylvie. a) Quel est le prix relatif du fromage ?
b) Quelle est l’équation de budget de Mlle
FIGURE 8.6 Mouffette ? Tracez sa droite de budget sur
un graphique. (Mesurez le fromage sur l’axe
horizontal).
c) Sur votre graphique, tracez une courbe
d’indifférence de manière à ce que le meilleur
point accessible corresponde à 6 morceaux
de fromage et à 4 berlingots de lait.
d) Quel est le taux marginal de substitution
du fromage au lait à ce point ?
e) Montrez que tous les autres points
de la droite de budget représentent une
combinaison moins intéressante.

9 En reprenant les mêmes données, supposons


maintenant que le revenu de Mlle Mouffette
augmente.
a) Illustrez graphiquement la variation de la
consommation de fromage et de lait lorsque
Squash (en heures par semaine)
ces deux biens sont des biens normaux.
(Il n’est pas nécessaire d’indiquer les chiffres.
a) Combien d’heures de squash et de danse Indiquez simplement si la consommation
aérobique Sylvie pourra-t-elle se payer augmente ou diminue.)
à son meilleur point accessible? b) Tracez un nouveau graphique montrant l’effet
b) Supposons que la location du court de squash d’une augmentation du revenu de Mlle
baisse, passant à 1 $ l’heure. Mouffette lorsque le lait est un bien inférieur.
i) Combien d’heures de squash et de
danse aérobique Sylvie peut-elle se payer 10 Revenons aux conditions initiales décrites au
maintenant ? Combien d’heures de squash problème 8. Supposons maintenant que le prix
supplémentaires peut-elle se payer ? du fromage double, passant à 2 $ le morceau
CONTRAINTES BUDGÉTAIRES, PRÉFÉRENCES ET CHOIX DE CONSOMMATION 109

alors que celui du lait reste le même à 1,50 $ le


©6c En prenant Q = (y/PJ - (P/PJ Q/-
berlingot et que le revenu est toujours de 12 $.
Si Ps = 5 $, alors y— 100 $ et Pf~ 20 $.
a) Tracez la nouvelle droite de budget.
Recalculez ensuite l’équation pour Ps = 4 $.
b) Pourquoi le meilleur point accessible initial 7 c Les courbes d’indifférences sont convexes
(nommez-le r) n’est-il plus le meilleur point
par rapport à l’origine.
accessible ?
8 a Avec le même revenu, il est possible d’acheter
c) En vous servant de votre graphique, indiquez ilc a trait aux courbes d’indifférence.
le nouveau meilleur point accessible et 9 a Avec les nouveaux prix, la droite de budget
nommez-le t. Comment évolue la est tangente à la courbe d’indifférence initiale.
consommation de fromage ? 10 d Rétablit hypothétiquement le revenu initial
d) Décomposez l’effet sur la consommation (renversez T revenu réel) mais gardez
de X en effet de substitution et en effet de les prix au même niveau (de bac).
revenu. Sur votre graphique, indiquez l’effet ©lî b (3 X prix d’une tomate) = (2 X prix d’une
de substitution sous forme de déplacement brosse à dents). Divisez les deux parties
du point r au point s (que vous devez trouver) de l’équation par le prix d’une tomate et par 2.
ainsi que l’effet de revenu sous forme 12 c En raison du TmS décroissant, d et e sont
de déplacement du point s au point t. faux puisque le prix relatif se rapporte
à la droite de budget et non à la courbe
d’indifférence.
RÉPONSES 13 b L’ordonnée à l’origine y se déplace vers
le bas, l’abscisse à l’origine x ne bouge pas.
Vrai/Faux/Incertain 14 c Le point rsur la courbe d’indifférence
(Justifiez votre réponse.) plus élevée, mais à l’extérieur de la droite
de budget. Les points s et q donnent la même
I F Les lignes de budget sont droites. Les courbes utilité et donc, les réponses a et d sont
d’indifférence sont convexes par rapport fausses. Le point q est préféré au point r,
à l’origine. la réponse b est donc fausse.
2 V Revenu réel = revenu/prix des biens. 15 d T revenu ne change pas la pente, mais
3 V Voir le manuel. abscisse et ordonnée à l’origine T.
4 F Elles représentent des niveaux de satisfaction 16 d La nouvelle droite de budget est plus plate
plus élevés. et étirée et elle est tangente à la même courbe
5 V Voir le manuel. d’indifférence.
6 F Vrai pour les compléments parfaits. ©17 b Rapportez-vous au Rappel 2 et analysez A Pr
© 7 V La droite de budget est tangente à la courbe (films en abscisse).
d’indifférence. 18 b Les numérateurs et dénominateurs des
© 8 I Vrai pour les biens normaux, faux pour deux axes doublent, de sorte que l’ordonnée
les biens inférieurs. et l’abscisse à l’origine ne changent pas.
9 V Voir le texte d’introduction du manuel. 19 d Voir le manuel.
10 F Selon cette théorie, seuls les choix faits 20 d Les quantités varient le long de la droite
par des acheteurs réels ressemblent de budget.
aux choix prédits par le modèle. 21 c L’influence de Clark Gable —> X des
préférences pour les maillots de corps.
Combinée aux prix et revenus restés
Questions à choix multiple
identiques —» X consommation.
1 b a devrait être les combinaisons de biens ; ©22 d Le TmS se déplace toujours X le long
c porte sur les courbes d’indifférence; d porte de la courbe d'indifférence. Les courbes
sur le meilleur point accessible ; e comprend d’indifférence des compléments sont
l’aire à l’intérieur de la droite de budget. plus abruptes.

2 e Voir le manuel.
23 e Voir le manuel. En inversant a, on constate

3 e Pour T prix, l’effet de substitution toujours que la réponse b est juste.


—» si consommation pour les biens normaux 24 c Voir le manuel.

et les biens inférieurs.


25 c La droite de budget dépend de a et de b.
La courbe d’indifférence peut prendre
4 b Avec une pente constante.
5 c Revenu = 12 $ (2 $ X 6 unités du bien X), n’importe quelle forme à n’importe quel
niveau de satisfaction.
donc le prix Y— 12 $/8 unités du bien Y
110 CHAPITRE 8

Problèmes à court développement conséquent, la valeur du dénominateur


du terme droit de l’équation augmente. L’effet
combiné d’une diminution du numérateur
1 Une courbe d’indifférence nous indique de
combien d’unités la consommation d’un bien et d’une augmentation du dénominateur est

doit augmenter lorsque la quantité consommée une baisse du ratio Um^lmJUmsodas à mesure
que nous descendons le long d’une courbe
d’un autre bien diminue de manière à ce que
le consommateur en retire le même niveau d’indifférence, ce qui correspond à un taux
marginal de substitution décroissant au niveau
de satisfaction. Elle a une pente négative puisque
du terme gauche de l’équation.
les biens sont tous deux désirables. Lorsque
nous diminuons la consommation d’un bien,
pour retirer le même niveau de satisfaction, Ces deux énoncés sont apparemment
contradictoires, car une hausse du revenu
nous devons augmenter notre consommation
entraîne, dans le premier énoncé, un
de l’autre bien. C’est pourquoi la pente des
courbes d’indifférence est négative. déplacement de la courbe de demande,
mais, dans le second, il entraîne un mouvement

2 Nous répondons à cette question en avançant le long de la courbe de demande. Ces énoncés

des arguments contradictoires. Puisque les points


ne sont pas contradictoires si on fait la
a ex b sont situés sur la même courbe distinction essentielle entre le revenu nominal

d’indifférence (/j), le niveau de satisfaction et le revenu réel.


Le premier énoncé (une augmentation
du consommateur est le même entre ces deux
du revenu entraîne une augmentation de
points. Les points a et c étant situés sur la même
la demande (la courbe de demande se déplace
courbe d’indifférence (/2), le niveau de
satisfaction du consommateur est également le vers la droite) nous dit comment une

même entre ces deux points. Lorsque le niveau augmentation du revenu nominal déplace

de satisfaction du consommateur est le même la courbe de demande. Le revenu nominal


se mesure en dollars.
entre les points a et b et entre les points a et c,
Le second énoncé (l’effet de revenu —
il devrait en être de même pour les points b et c.
causé par une baisse du prix — entraîne une
Mais cela est logiquement impossible, car nous
avons supposé que le consommateur préfère augmentation de la quantité demandée) nous
dit comment une augmentation du revenu réel
consommer plus que moins de chaque bien.
entraîne un mouvement le long de la courbe
Comme le point c comprend plus de biens
de demande qui reste fixe.
X et Y que le point b, le consommateur
ne peut retirer le même niveau de satisfaction Le revenu réel se mesure par le pouvoir
d’achat ou par la quantité de biens que
pour les points b et c. Ce qui est contraire
le revenu nominal permet d’acheter. Lorsque
à la supposition qu’il préfère consommer plus
le prix d’un bien diminue, votre revenu réel
que moins de biens. De ce fait, les courbes
d’indifférence ne peuvent pas se croiser. augmente, car vous pouvez maintenant acheter
une plus grande quantité de ce bien, même si
votre revenu nominal ne change pas. Du fait que
® 3 À mesure que nous descendons le long d’une
le revenu nominal ne change pas dans ce second
courbe d’indifférence relative aux sodas et aux
énoncé, la courbe de demande ne se déplace pas.
films, (voir la figure 8.5 du manuel, p. 173),
Cependant, le revenu réel ayant augmenté,
nous augmentons la quantité de films
la baisse de prix entraîne une augmentation
consommée et diminuons la quantité de sodas
de la quantité demandée.
consommée. À mesure que la consommation
de films augmente, chaque film supplémentaire
procure une utilité marginale plus faible, a) Initialement, Jeanne et Daniel se trouvent au
point c de la droite de budget AB de la figure
conformément au principe de l’utilité marginale
décroissante. De ce fait, la valeur du numérateur 8.7. La courbe d’indifférence de Jeanne
est illustrée par Ij. Notons que sa courbe
du terme droit de l’équation suivante diminue.
d’indifférence s’approche d’une ligne droite
puisque, pour elle, le pain et le beurre
Taux marginal _ utilité marginale des films
d’arachide sont des substituts proches. En
de substitution utilité marginale des sodas
revanche, du fait que pour Daniel le pain
À mesure que la consommation de sodas et le beurre d’arachide sont des biens
diminue, chaque soda consommé au préalable complémentaires, sa courbe d’indifférence,
procure une utilité marginale plus élevée. Par ID, est plus arquée.
CONTRAINTES BUDGÉTAIRES, PRÉFÉRENCES ET CHOIX DE CONSOMMATION

FIGURE 8.7 a) i) 4 unités de nourriture


ii) la droite de budget du consommateur
nous est donnée par

pvQ* +pnQn = y
Pour obtenir l’équation de budget, procédez
de la même manière qu’à la page 169 du
manuel. Si on divise par Pv, on obtient

K
Qv +
pv
On soustrait ensuite (PJPV) Q„ des deux
membres de l’équation pour obtenir

= y_ P
Q,
P P.,
Pour obtenir l’équation de budget en
b) Lorsque le prix du pain diminue, la droite de termes numériques, on substitue dans
budget est moins abrupte, comme le montre l’équation les valeurs Pv = 1 $, Pn = 1,50 $
la droite AD de la figure 8.7. Pour mesurer et y = 9 $
l’effet de substitution, trouvez sur la courbe 1,50
d’indifférence initiale le point qui a la même Qn
1
pente que la nouvelle droite de budget.
Q,=9-3/2Q„
Comme la courbe d’indifférence de Daniel
est plus arquée, elle s’aplatit très rapidement La valeur (absolue) de la pente de la
à mesure qu’on s’éloigne du point c. C’est droite de budget est 3/2, ce qui correspond
pourquoi l’effet de substitution est très faible au ratio de PJPV. Remarquez que l’on
lorsqu’on se déplace du point c au point e. ne peut lire directement ce ratio des prix
Comme la courbe d’indifférence de Jeanne est sur le graphique puisque les axes mesurent
presque droite, l’effet de substitution doit être les quantités et non les prix.
plus important du point c au point f. iii) Le taux marginal de substitution au point
a correspond à la valeur de la pente de la
Ô 6 La droite de budget du consommateur est courbe d’indifférence au point a. Étant
illustrée à la figure 8.5 Solution. donné que la pente de la courbe
d’indifférence au point a est égale à la
FIGURE 8.5 SOLUTION pente de la tangente au point a, et que la
droite de budget est tangente au point a,
la valeur de la pente de la courbe
d’indifférence est de 3/2. Cette valeur est
équivalente à UmJUmv.
iv) La valeur de la pente de la droite de
budget est PJPV et la valeur de la pente
de la courbe d’indifférence au point a est
UmJUmv. Étant donné que ces deux
valeurs sont égales, Pn/Pv = Um„/Umv ou
Umn/Pn = UmvIPv.
b) Voir la figure 8.5 Solution: 2 unités de
nourriture.
c) Voir la figure 8.5 Solution: 1 unité de
nourriture.
d) La courbe de demande de nourriture est
illustrée à la figure 8.8.
112 CHAPITRE 8

i) Maintenant, le meilleur point accessible


FIGURE 8.8
de Sylvie est le point e sur la courbe
d’indifférence I3. Cela correspond à la
consommation de 6 heures de squash et
6 heures de danse aérobique par semaine.
Sa consommation de squash a augmenté
de 3 heures.
ii) Pour calculer l’effet de substitution,
déplacez parallèlement la nouvelle droite
de budget vers la gauche jusqu’à ce qu’elle
soit tangente à la courbe d’indifférence I2
avant la baisse du prix de l’heure de
squash. En effet, la droite de budget A'B'
supprime l’augmentation du revenu réel
due à la baisse du prix de la location du
court de squash, ce qui permet d’isoler
l’effet de substitution de la baisse de prix

e) Voir la figure 8.5 Solution. 1 unité de sur la consommation d’heures de squash

nourriture. La nourriture est un bien normal de Sylvie. Le point/"correspond au

parce qu’une diminution du revenu (les prix nombre d’heures qu’aurait pu se payer

étant constants) entraîne une diminution de Sylvie aux nouveaux prix si son revenu

la consommation de nourriture (de 4 unités à avait baissé suffisamment pour la ramener

1 unité). La nouvelle courbe de demande à sa courbe d’indifférence initiale. L’effet

(revenu = 3 $) est illustrée à la figure 8.8. de substitution de la baisse de prix de


l’heure de squash correspond au
mouvement du point initial c au point f,

7 a) Pour trouver le meilleur point accessible de ce qui représente une augmentation de

Sylvie, tracez la droite de budget à la figure 2 heures des heures de squash consommées

8.6 Solution. La droite de budget initiale (de 3 à 5 heures).


est AB et le meilleur point accessible est le iii) Pour calculer l’effet de revenu, déplacez

point c sur la courbe d’indifférence /2. Par parallèlement la droite de budget A'B'

conséquent, Sylvie peut se payer 3 heures vers la droite jusqu’à ce quelle soit

de squash et 6 heures de danse aérobique tangente à la courbe d’indifférence I3

par semaine. de Sylvie avant que nous ayons baissé


(hypothétiquement) son revenu. En effet,

FIGURE 8.6 SOLUTION ce déplacement permet d’isoler l’effet de


revenu en rétablissant l’augmentation du
revenu réel due à la baisse de prix tout
en maintenant les prix constants à leurs
nouvelles valeurs. L’effet de revenu de
la baisse de prix de l’heure de squash
correspond au mouvement du point/
au point e, ce qui représente une
augmentation de 1 heure des heures de
squash consommées (de 5 à 6 heures).

Le tableau 8.1 résume ces résultats.

TABLEAU 8.1 EFFET DE PRIX -


FIGURE 8.6 SOLUTION

Mouvement T heures
Effet du point de squash
Squash (en heures par semaine)
Effet de substitution c à f 2
+ effet de revenu fà e 1
b) Lorsque le prix de location du court de squash = effet de prix c à e 3
diminue, la droite de budget devient AD.
CONTRAINTES BUDGÉTAIRES, PRÉFÉRENCES ET CHOIX DE CONSOMMATION I 13

c) Dans les questions a et b, le revenu dépensé La ligne AB de la figure 8.10 constitue la


pour le conditionnement physique était droite de budget, qui est la représentation
constant à 12 $ et le prix du cours de danse graphique de cette équation.
aérobique était constant à 1 $ l’heure.
Lorsque le prix d’une partie de squash était FIGURE 8.10
de 2 $ l’heure, Sylvie souhaitait consommer
3 heures de squash et lorsque ce prix était
de 1 $, Sylvie désirait en consommer 6 heures.
Cela nous donne deux points sur la courbe
de demande de squash de Sylvie, nommés
a et b à la figure 8.9. En traçant une ligne
passant par ces points, nous obtenons une
courbe de demande linéaire, appelée D.

FIGURE 8.9

c) Si le meilleur point accessible correspond à


6 morceaux de fromage et à 4 berlingots de
lait, alors la courbe d’indifférence pertinente
sera tangente à la droite de budget AB au
point c, qui est la courbe d’indifférence II.
d) Le taux marginal de substitution est donné
par la valeur de la pente de la courbe
Squash (en heures par semaine)
d’indifférence au point c. Nous ne
connaissons pas la valeur de la pente de la
d) Lorsque le budget que consacre Sylvie au courbe d’indifférence, mais nous pouvons
conditionnement physique augmente, passant facilement calculer la pente de la droite de
à 14 $, la droite de budget appelée GH à la budget. Étant donné qu’au point c, la courbe
figure 8.6 Solution, se déplace vers la droite. d’indifférence et la droite de budget ont une
Le meilleur point accessible de Sylvie est pente identique, nous pouvons obtenir le taux
maintenant le point j, qui correspond à marginal de substitution du lait au fromage.
7 heures de squash et à 7 heures de danse Étant donné que la valeur de la pente de
aérobique par semaine. la droite de budget est de -2/3, le taux
marginal de substitution est égal à 2/3.
Ainsi, Mlle Mouffette est prête à renoncer
& 8 a) Le prix relatif du fromage est le prix du à 2 berlingots de lait pour obtenir 3 morceaux
fromage divisé par le prix du lait : supplémentaires de fromage, tout en restant
satisfaite.
Pf 1$ _ 2
e) Comme les courbes d’indifférence ne peuvent
^ ~ 1,50$ ” 3 se croiser et que la courbe d’indifférence Ix
b) Pf étant égal au prix du fromage et Pi étant relie tous les points au-dessus de la droite de
égal au prix du lait, Qf = morceaux de budget (sauf le point c), nous savons que tous
fromage, Q/ = berlingots de lait, et les autres points sur la droite de budget se
y — revenu. L’équation du budget est trouvent sur une courbe d’indifférence moins
la suivante : élevée. Par exemple, le point d se trouve sur la
courbe d’indifférence 70. Par conséquent, tout
autre point situé sur la droite de budget est
inférieur au point c.
Puisque Pf — 1 $, P/= 1,50 $ et y — 12 $,
l’équation de budget de Mlle Mouffette 9 a) Une augmentation du revenu entraînera un
devient : déplacement parallèle de la droite de budget
Q/ = 8 - 2/3Qf vers la droite, par exemple, jusqu’à EF de la
114 CHAPITRE 8

figure 8.10. Si le fromage et le lait sont tous le point r de la figure 8.12 est identique au
deux des biens normaux, Mlle Mouffette point c de la figure 8.10. La nouvelle droite
se déplacera vers un point g, par exemple, de budget, à la suite d’une augmentation du
qui représente une augmentation de sa prix du fromage, qui est maintenant de 2 $,
consommation des deux biens, est AH (le revenu reste à 12 $).
b) Si le lait est un bien inférieur, alors la
consommation de lait diminuera à mesure FIGURE 8.12
que le revenu augmentera (figure 8.11). Là
encore, la droite de budget se déplace de AB
vers EF, mais les préférences de Mlle
Mouffette sont telles que son nouveau point
de consommation correspond au point g ',
point où la consommation de lait a, en
réalité, diminué.

FIGURE 8.1 I

Quantité de fromage (en morceaux)

b) Après l’augmentation du prix, le point r ne


correspond plus au meilleur point accessible,
étant donné qu’il n’est même plus accessible.
c) Le nouveau meilleur point accessible (t à la
figure 8.12) indique une diminution de la
consommation de fromage.
d) L’effet de substitution de l’augmentation
du prix du fromage est indiqué par le
Quantité de fromage (en morceaux)
déplacement de r à r (figure 8.12), ce qui
illustre l’effet de la variation des prix relatifs,
Mlle Mouffette se maintenant sur la même
10 a) La droite de budget initiale de Mlle courbe d’indifférence. L’effet de revenu est
Mouffette est AB et le meilleur point indiqué par le déplacement du point s au
accessible est r (figure 8.12). Remarquez que point t.
9 j
L’organisation
de la production

Le financement des entreprises


CONCEPTS CLÉS
Les entreprises obtiennent les fonds dont elles
ont besoin de deux façons.
L’entreprise et ses problèmes
♦ Par émission d’actions : pour se procurer des
économiques
capitaux, les sociétés vendent des parts de leur
capital. Les actionnaires sont les propriétaires
Une entreprise est une organisation qui mobilise d’une société par actions. Les capitaux propres
et gère des facteurs de production afin de produire sont les sommes que le propriétaire investit
et de vendre des biens et des services. Les entreprises dans son entreprise.
visent à réaliser le maximum de profit de la manière
la plus efficace et la plus rentable possible. Les ♦ Par vente d’obligations : une obligation est

entreprises organisent la production en recourant l’engagement de payer un certain montant


à des dates déterminées.
♦ aux systèmes de surveillance, fondés
• Un gain en capital est le profit que l’on retire
sur la hiérarchie ;
lorsqu’on vend une action ou une obligation
♦ aux systèmes d’incitation qui permettent à un prix supérieur au prix d’achat initial.
de résoudre les problèmes liés à l’incertitude
La valeur actuelle d’une somme à payer ou à percevoir
et à l’information incomplète.
dans l’avenir correspond au montant que l’on doit
Le problème principal-agent se pose lorsque investir aujourd’hui pour obtenir la valeur de cette
les agents (ceux qui sont employés par d’autres) somme future, en tenant compte de l’intérêt perçu.
n’agissent pas selon l’intérêt des principaux (les
L’actualisation est la conversion de la valeur future
employeurs des agents).
d’une somme d’argent en sa valeur actuelle.
♦ Les principaux moyens de résoudre le problème
somme d’argent à percevoir
principal-agent et d’inciter les agents à agir
... .. dans n années (valeur future)
selon l’intérêt des principaux sont la propriété, ♦ Valeur actuelle =-
la rémunération à la performance et les contrats (1 +r)n

à long terme.
♦ Pour décider si elle doit ou non emprunter,
♦ Pour résoudre le problème lié à l’incertitude l’entreprise doit calculer l’avantage net de
et à l’information incomplète, les entreprises l’emprunt = (avantage marginal — coût marginal).
ont recours à différentes formes d’organisation : La valeur actuelle d’un avantage net correspond
à la valeur actuelle nette. Lorsque la valeur nette
• L'entreprise individuelle, qui appartient à un seul
actuelle est positive, l’entreprise î profits en
propriétaire, dont la responsabilité est illimitée.
empruntant.
• La société de personnes, qui appartient a au
moins deux propriétaires, dont la responsabilité
est illimitée. Coût d’opportunité et profit économique
• La société par actions, qui appartient à un
ou plusieurs actionnaires, dont la responsabilité Les coûts d’opportunité de la production d’une
entreprise englobent les coûts explicites (exprimés
est limitée.
116 CHAPITRE 9

directement en numéraire) et les coûts implicites Les entreprises et les marchés


(possibilités auxquelles on a renoncé mais qui ne sont
pas exprimées en numéraire). Les principaux coûts Les entreprises coordonnent l’activité économique
implicites sont les suivants. lorsqu’elles peuvent accomplir une tâche de façon plus
efficiente que les marchés, pour les raisons suivantes :
♦ Les coûts d’utilisation du capital : l’amortissement
économique, qui est la variation du prix sur • coûts de transaction moins élevés — les coûts
le marché d’un bien de production (les provenant de la recherche de partenaires
comptables appliquent au prix d’achat un taux commerciaux ;
d’amortissement conventionnel pour calculer • économies d’échelle — -l coût unitaire
la valeur de l’amortissement), et le revenu des de la production d’un bien à mesure que
intérêts auquel on a renoncé. T l’échelle de production;
• économies de production en équipe —
• Le loyer implicite (= amortissement économique
processus de production où est engagé
+ intérêts auxquels on a renoncé).
un groupe de personnes spécialisées dans
• Le coût irrécupérable (= amortissement
des tâches complémentaires.
économique passé) «est ^as un coût
d’opportunité.

♦ Le coût d’utilisation des stocks - coût


de remplacement des stocks utilisés. RAPPELS
♦ Le coût des ressources du propriétaire —
le propriétaire fournit l’esprit d’entreprise. 1 La notion de valeur actuelle est fondamentale,

Le profit normal est le profit engendré par car elle désigne la valeur d’aujourd’hui d’un

l’esprit d’entreprise. Il fait partie du coût investissement ou d’autres sommes à percevoir

d’opportunité de l’entreprise. à l’avenir. Elle nous offre un moyen de comparer


des investissements dont les paiements ont lieu
Coûts et profits pour l’économiste : à des dates différentes. La valeur actuelle repose
sur la notion intuitive qu’un dollar d’aujourd’hui
♦ coûts d’opportunité = coûts explicites + coûts
vaut plus qu’un dollar de demain, car le dollar
implicites ;
d’aujourd’hui peut être investi pour produire
♦ profits économiques = revenu total — (coûts de l’intérêt. Pour calculer la valeur à’aujourd’hui
explicites + coûts implicites) ; d’une somme monétaire qui sera payée demain,
on doit actualiser ce montant futur en y incluant
♦ le profit normal faisant partie des coûts
le revenu d’intérêt qui serait gagné. La valeur
implicites, le profit économique (lorsqu’il
actuelle d’une somme d’argent à payer ou à
est positif) est supérieur au profit normal.
percevoir dans l’avenir équivaut au montant qui,

Coûts et profits pour le comptable : s’il est investi aujourd’hui, sera égal à la somme
future plus élevée, si l’on prend en compte
♦ coûts comptables = coûts explicites + l’intérêt obtenu.
amortissement conventionnel ;

♦ profit comptable = revenu total - (coûts 2 Dans ce chapitre, nous retrouvons la fameuse

explicites + amortissement conventionnel) ; notion de coût d’opportunité. Ici, nous analysons,


en effet, les coûts de l’entreprise, mais en insistant
♦ le profit, pour le comptable, fait abstraction sur les différences entre les méthodes de calcul
du profit normal ou des autres coûts du coût d’opportunité, utilisées par les économistes,
implicites. et les méthodes comptables. Le coût d’opportunité,
qui est le coût sur lequel les entreprises se fondent
pour prendre des décisions économiques,
L’efficience économique
comprend les coûts implicites et les coûts
explicites. Entre autres coûts implicites, citons
Efficience technique — impossible T production
l'intérêt auquel a renoncé le propriétaire ou
sans T facteurs de production.
l’investisseur, le loyer implicite, l’amortissement
Efficience économique - coût de production le plus économique et les profits normaux. Les coûts
bas possible. comptables ne comprennent que les coûts
L’ORGANISATION DE LA PRODUCTION I 17

explicites remboursables et l’amortissement augmenter la quantité de facteurs de production.


conventionnel. Ces différences entre les Les coûts des facteurs de production ne sont
méthodes de calcul des coûts par les comptables pas pris en compte. L’efficience économique est
et par les économistes entraînent également atteinte lorsque le coût de production
des différences dans les mesures des profits. d’une quantité donnée de produit est aussi
bas que possible. Les méthodes de production
Économistes
techniquement efficientes ne sont pas toutes
Coûts d’opportunité = coûts explicites
efficientes économiquement. Mais toutes les
+ coûts implicites
méthodes efficientes économiquement sont
Profits économiques = revenus — (coûts
également efficientes techniquement. Le jeu
explicites + coûts
de la concurrence favorise les entreprises
implicites)
qui choisissent des méthodes de production
Comptables
économiquement efficientes et pénalise celles
Coûts comptables = coûts explicites
qui ne les choisissent pas.
+ amortissement
conventionnel
Profits comptables = revenus — (coûts
explicites + amortissement
conventionnel)
AUTOÉVALUATION

Les coûts implicites, que calculent les


économistes, mais pas les comptables, sont la
Vrai/Faux/Incertain
clé de la différence entre les méthodes de calcul
(Justifiez votre réponse.)
des coûts et des profits par les économistes
et par les comptables. Les profits comptables I On résout le problème principal-agent,
en donnant aux gestionnaires des parts
ne soustraient pas les profits normaux ou
dans l’entreprise.
autres coûts implicites, de sorte que les profits
comptables sont généralement plus élevés
que les profits économiques.
Du fait que les profits normaux font partie
des coûts implicites, les profits économiques 2 Le profit normal est le rendement prévu,

sont des profits supérieurs aux profits normaux. attribuable à l’esprit d’entreprise.

Si nous considérons les profits normaux comme


des profits moyens, les profits économiques sont
des profits supérieurs à la moyenne. Les profits
économiques signalent aux propriétaires des 3 Si le taux d’intérêt est de 10% par an, la valeur

entreprises qu’ils enregistrent un rendement de actuelle de 100 $ reçus au bout d’un an est

leur investissement plus élevé que le rendement de 110$.

moyen qu’ils auraient enregistré en se consacrant


à d’autres activités.
Les profits économiques peuvent également
être négatifs, si les revenus sont inférieurs aux 4 Dans une relation principal-agent entre
coûts d’opportunité. Comme nous le verrons au les actionnaires et les directeurs de la Banque
chapitre 11, ces pertes économiques signalent aux de Montréal, les actionnaires sont les agents
propriétaires des entreprises qu’ils enregistrent et les directeurs, les principaux.

un rendement de leur investissement moins


élevé que le rendement moyen qu’ils auraient
enregistré en se consacrant à d autres activités.
5 Le coût d’opportunité est supérieur au coût
3 II importe de faire la différence entre efficience comptable.
technique et efficience économique. Cette différence
est critique, car les décisions économiques ne
sont prises qu’en fonction de l’efficience
économique. L’efficience technique est du ressort 6 Lorsqu’une entreprise utilise sa propre machine,
de l’ingénieur ; elle est atteinte lorsqu il n est son coût d’opportunité est plus bas que si elle
pas possible d’augmenter la production sans l’avait louée.
I 18 CHAPITRE 9

7 Le coût d’opportunité de l’utilisation de stocks 4 Benoît achète une entreprise de nœuds papillon
est le prix courant du marché. pour la somme de 100 000 $. Pour la payer,
il emprunte 60 000 $ à la banque à un taux
d’intérêt de 12% et vend des obligations d’une
valeur de 40 000 $. Les obligations rapportent
8 Un processus de production économiquement 10% d’intérêt par année. Le coût d’opportunité
efficient devient économiquement inefficient si de l’exploitation de l’entreprise, durant sa

le prix relatif des facteurs de production change. première année, comprend


a) seulement les 7 200 $ d’intérêt à payer
à la banque.
b) seulement les 4 000 $ d’intérêt à payer
aux détenteurs d’obligations.
9 Dans le cas d’une immobilisation,
c) les 7 200 $ d’intérêt bancaire et les 4 000 $
l’amortissement économique est inférieur
d’intérêt dus aux détenteurs d’obligation.
à l’amortissement conventionnel.
d) ni les 7 200 $ d’intérêt bancaire ni les 4 000 $
d’intérêt dus aux détenteurs d’obligation.
e) Aucune de ces réponses.

10 Les marchés coordonnent l’activité économique 5 En général

chaque fois qu’on fait des économies d’échelle. 1 le coût d’opportunité est plus élevé
que le coût comptable.
2 le coût d’opportunité est inférieur
au coût comptable.
3 le profit économique est supérieur au profit
Questions à choix multiple comptable.
4 le profit économique est inférieur au profit
comptable.
1 Parmi les énoncés suivants, lequel est faux:?
a) 1 seulement
a) Les entreprises et les marchés sont des
b) 1 et 3
institutions qui coordonnent l’activité
c) 1 et 4
économique.
d) 2 et 3
b) Les entreprises gèrent les facteurs de
e) 2 et 4
production de façon à produire des biens
et des services. 6 Si le taux d’intérêt est de 10% par année, et si
c) Les entreprises vendent des biens vous investissez 50 000 $ de votre argent dans
et des services. une entreprise et réalisez un profit comptable de
d) Les entreprises techniquement efficientes 20 000 $ au bout d’un an, ceteris paribus, quel
peuvent résoudre les problèmes de rareté. est votre profit économique ?
e) Les entreprises se servent de systèmes a) 20 000 $
de surveillance pour organiser la production. b) 15 000 $
c) 5 000 $
2 Comment appelle-t-on une entreprise dont d) 2 000 $
les propriétaires (deux au moins) ont une e) -15 000 $
responsabilité illimitée ?
7 On appelle la participation du propriétaire dans
a) Une entreprise individuelle
son entreprise
b) Une société de personnes
a) valeur actuelle.
c) Un conglomérat
b) valeur de rachat.
d) Une société par actions
c) coût irrécupérable.
e) Aucune de ces réponses.
d) capitaux propres.
e) stocks.
3 Quel facteur, parmi les suivants, peut changer
entre la date de vente d’une obligation et son 8 Quel coût, parmi les suivants, «est pas
échéance ? un coût implicite?
a) La valeur actuelle a) L’amortissement économique
b) La valeur de rachat b) Le revenu d’intérêt auquel on a renoncé
c) La date du remboursement c) Le loyer implicite
d) La valeur annuelle du coupon d) Le coût irrécupérable
e) Aucune de ces réponses. e) Le profit normal
L'ORGANISATION DE LA PRODUCTION I 19

9 Parmi les méthodes de fabrication d’un 13 Les entreprises coordonnent de manière plus
torpilleur à photons (tableau 9.1), déterminez efficiente l’activité économique que le marché
laquelle ou lesquelles sont techniquement a) quand leurs coûts de transactions sont plus
efficientes ? faibles.
a) 1 b) quand leurs coûts de surveillance sont plus
b) 2 faibles.
c) 3 c) quand elles font des économiques d’échelle.
d) Toutes ces réponses. d) quand elles bénéficient d’économies
e) 1 et 3 seulement. de production en équipe.
e) Toutes ces réponses.

TABLEAU 9.1 TROIS MÉTHODES 14 Quelle est la valeur actuelle d’une somme de
DE FABRICATION D’UN 100 $ à recevoir dans un an, si le taux d’intérêt
TORPILLEUR À PHOTONS annuel est de 10% ?
a) 90,00 $
Quantités de facteurs b) 90,91 $
de production c) 95,45 $

Méthode Travail Capital


d) 100,00 $
e) 110,00$
1 5 10
2 Î0 7 15 Si un employé donne un mauvais rendement,

3 15 5 il s’agit d’un
a) problème de responsabilité limitée.
b) problème principal-agent.
10 Reprenez l’exemple du tableau 9.1. Si le coût c) problème de coût de transaction.
du travail est de 10 $ l’unité et le coût du capital d) problème d’efficience technique.
de 20 $ l’unité, déterminez laquelle ou lesquelles e) problème de partenariat.
des méthodes sont économiquement efficientes ?
a) 1 16 Quelles sont les entreprises qui émettent
b) 2 des actions ?
c) 3 a) Les entreprises individuelles
d) Toutes ces réponses. b) Les sociétés de personnes
e) 1 et 3 seulement. c) Les sociétés par actions
d) Toutes ces réponses.
I I Énoncez l’un des inconvénients de la société e) b et c seulement
par actions par rapport à l’entreprise individuelle
ou à la société de personnes. I7 Quelles personnes, parmi les suivantes, sont des
a) Les propriétaires ont une responsabilité ayants droit résiduels d’une société par actions ?
illimitée. a) Les actionnaires
b) Les profits des actionnaires sont taxés b) Les détenteurs d’obligation
en tant que profits d’entreprise et revenus c) Les banques et les autres créanciers

de dividendes. d) Les autorités fiscales


c) Il est difficile de mobiliser des capitaux. e) Les directeurs

d) La perpétuité.
I8 Le coût de construction d’un bâtiment est
e) Aucune de ces réponses.
de 100 000 $. La provision pour amortissement
I2 Quel énoncé, parmi les suivants est vrai ? est de 5 % par année. À la fin de la première
a) Toutes les méthodes techniquement année, la valeur marchande du bâtiment est
efficientes le sont également de 80 000 $. Pour la première année, le coût
économiquement. d’amortissement est de
b) Toutes les méthodes économiquement a) 20 000 $ pour un comptable ou pour un
efficientes le sont également techniquement. économiste.
c) L’efficience technique change en fonction b) 5 000 $ pour un comptable ou pour un
des changements des prix relatifs des facteurs économiste.

de production. c) 5 000 $ pour un comptable, mais de 20 000 $


d) Les entreprises techniquement efficientes pour un économiste.

ont plus de chances de survie que les d) 20 000 $ pour un comptable, mais de 5 000 $
entreprises économiquement efficientes. pour un économiste.
e) Aucune de ces réponses.
e) Aucune de ces réponses.
120 CHAPITRE 9

19 Hector exploite sa propre entreprise et se paie 24 Si la valeur actuelle de 500 $ reçus dans un an
un salaire de 20 000 $ par année. Il a refusé un est de 463 $, quel est le taux d’intérêt annuel?
emploi pour lequel on l’aurait payé 30 000 $ par a) 5%
année. Quel est le coût d’opportunité du temps b) 8%
qu’Hector consacre à son entreprise? c) 10%
a) 10 000$ d) 20,8%
b) 20 000 $ e) 37%
c) 30 000 $
d) 50 000 $ 25 Si le taux d’intérêt annuel est 10%, quelle sera

e) nul la valeur actuelle de 100 $ dans deux ans ?


a) 80,00$

20 Le profit économique est égal au revenu moins b) 82,64$

a) les coûts explicites. c) 90,91$


b) les coûts implicites. d) 120,00 $
c) les coûts d’opportunité. e) 121,00$
d) les coûts comptables.
e) (les coûts explicites + l’amortissement
Problèmes à court développement
conventionnel).
1 Expliquez la notion de valeur actuelle
21 On parle d’économies d’échelle lorsque en indiquant (en mots) pourquoi la valeur
a) les coûts de transaction sont élevés. actuelle de 110 $ à recevoir dans un an, vaut
b) les coûts de transaction sont faibles. actuellement 100 $ si le taux d’intérêt est
c) l’embauche de facteurs de production de 10 % par année.
supplémentaires ne fait pas augmenter le prix
des facteurs de production. 2 Dans un restaurant, le pourboire habituel est de
d) le coût de la production d’une unité diminue 15%. Les restaurants pourraient augmenter leurs
à mesure que la taille de l’usine augmente. prix de 15%, éliminer les pourboires et verser
e) l’entreprise est trop grosse et trop diversifiée. aux serveurs ces 15 % supplémentaires. Utilisez
l’analyse principal-agent pour expliquer pourquoi
22 Supposons que la marque déposée «Choix la plupart des restaurants préfèrent le principe
du vice-président» ait une valeur de 10 000 $ du pourboire.
sur le marché. Au bout d’un an, cette valeur
augmente, passant à 15 000 $. Le coût 3 Comparez les approches relatives au coût
d’opportunité de l’utilisation de la marque comptable et au coût d’opportunité dans chaque
durant l’année est cas suivant :
a) nul a) L’entreprise calcule le coût d’amortissement.

b) -5 000 $ b) L’entreprise emprunte des capitaux pour

c) 5 000 $ financer ses opérations.


d) 10 000$ c) L’entreprise utilise ses propres capitaux plutôt
e) 15 000$ que d’emprunter.
d) L’entreprise calcule la valeur de ses ressources
23 La valeur actuelle d’une somme future propres.
est d’autant plus élevée
4 Expliquez la différence entre efficience technique
a) que le taux d’intérêt est plus élevé ou que
et efficience économique.
le versement de cette somme est plus éloigné
dans le temps. 5 Les marchés et les entreprises ont des moyens
b) que le taux d’intérêt est plus faible ou que différents permettant de coordonner l’activité
le versement de cette somme est plus éloigné économique que la rareté des facteurs de
dans le temps. production rend indispensable. Pourquoi existe-
c) que le taux d’intérêt est plus élevé ou t-il des entreprises et des marchés ?
que le versement de cette somme est plus
rapproché dans le temps. 6 Complétez le tableau 9.2 en calculant la valeur
d) que le taux d’intérêt est plus faible ou actuelle de trois versements futurs, en supposant
que le versement de cette somme est plus d’abord que le taux d’intérêt annuel est de 10%
rapproché dans le temps. (r= 0,1), ensuite de 5% (r= 0,05). Les trois
e) que le taux d’intérêt est plus faible, sans égard versements futurs sont les suivants :
à la date de versement de cette somme. A: 100 $ payés au bout d’un an.
L'ORGANISATION DE LA PRODUCTION 121

B: 100 $ payés au bout de deux ans. 8) La valeur de l’esprit d’entreprise de François


C: Obligation de 1 000 $ arrivant à échéance est de 14 000 $.
dans deux ans, avec coupon de 100 $ par 9) Les revenus de ia première année sont
année. Ainsi le détenteur reçoit un coupon de 100 000$.
de 100 $ à la fin de la première année et a) Dressez le tableau qui montre comment
un autre coupon de 100 $ à la fin de l’année Angèle et Édith calculeraient les coûts
suivante. Au bout de deux ans, il reçoit de l’entreprise de François. Ne tenez pas
également 1 000 $ (la valeur nominale compte de l’amortissement. Quel est le
de l’obligation). montant des coûts de l’entreprise de François
selon Angèle ? selon Édith ?
TABLEAU 9.2 b) Quel est le profit (ou la perte) de François
selon Angèle ? selon Édith ?
Valeur actuelle
(en dollars) 9 Votre colocataire affirme qu’il est plus
économique de récolter du blé à la machine
Versements futurs r= 0,1 r = 0,05
qu’à la main. Supposons qu’il existe
A deux méthodes techniquement efficientes
8 pour récolter une tonne de blé.
C
Méthode 1 : il faut 20 heures de travail
mécanique et 20 heures de travail humain.

& 7 Soit deux obligations, A et B, qui présentent Méthode 2: il faut 100 heures de travail
les caractéristiques suivantes : humain.
A rapporte 220 $ au bout de deux ans.
Le pays A est un pays nettement plus
B rapporte 110 $ au bout d’un an et 110 $
industrialisé que le pays B. Dans le pays A,
au bout de deux ans.
le coût de la main d’œuvre (c’est-à-dire le salaire
Pour laquelle de ces deux obligations seriez-
horaire) est de 8$ ; il n’est que de 4 $ dans le
vous disposé à payer plus? Pourquoi? (Indice: Il
pays B. Le coût d’une heure-machine est de
n’est pas nécessaire de connaître le taux d’intérêt
20 $ dans les deux pays. Quelle méthode est
pour répondre à cette question.)
économiquement efficiente dans le pays A ?
dans le pays 5? Expliquez vos réponses.
8 François, qui est maçon, a fondé, il y a un an, sa
propre entreprise de fabrication de meubles pour
10 Reprenons l’exemple des pays A et B du
poupées. François a deux sœurs : Angèle, qui est
problème précédent.
comptable, et Édith, qui est économiste. (Les
a) À combien faudrait-il que le salaire horaire
deux sœurs sont bonnes en calcul, mais Édith
s’élève dans le pays B pour que les deux
n’a pas la personnalité qu’il faut pour être
méthodes soient économiquement aussi
comptable). Chacune d’elles calcule les coûts et
efficientes l’une que l’autre dans le pays B?
les profits de la première année d’exploitation de
b) À combien faudrait-il que le coût d’une
François à partir des données suivantes :
heure-machine s’élève dans le pays A pour
1) François n’a retiré aucun revenu personnel de
que les deux méthodes soient
son entreprise. Il peut reprendre son ancien
économiquement aussi efficientes l’une
métier de maçon, n’importe quand, au salaire
que l’autre dans le pays A ?
annuel de 30 000 $.
2) François loue ses machines 9 000 $ par an.
3) Le garage dans lequel son entreprise est
installée lui appartient; s’il ne s en servait pas,
il pourrait le louer 3 000 $ par année.
4) Pour lancer son entreprise, François a investi RÉPONSES
10 000 $ de ses propres économies
et a emprunté 30 000 $ au taux d intérêt
Vrai/Faux/Incertain
du marché, qui est de 10% par annee. (Justifiez votre réponse.)
5) François emploie une personne
qu’il paie 20 000 $ par année.
6) Le coût du matériel pour sa première année 1 V On incite de la même façon les gestionnaires
d’exploitation est de 40 000 $. (agents) que les actionnaires (principaux).
7) Les stocks de fin d’année sont nuis. 2 V Définition.
122 CHAPITRE 9

3 F La VA de 110 $, reçus au bout d’un an, 21 d Voir le texte du manuel.


est de 100 $. 22 b T valeur = coût d’opportunité négatif.
4 F Les profits des actionnaires (principaux) 23 d Puisque (1 + r)n du dénominateur de la
dépendent du travail des gestionnaires formule VA, des r et n moins élevés donnent
(agents). une VA plus élevée.
5 I Habituellement vrai, mais cela dépend @24 b 463 $ = 500 $ / (1 + r). Trouvez
de l’existence de coûts implicites et des la réponse pour r.
différences entre l’amortissement économique @25 b VA = 100 $ / (1 + 0,1)2.
et l’amortissement conventionnel.
6 F Le coût d’opportunité est le même, que
l’entreprise loue ou possède sa propre Problèmes à court développement
machine.
7 V Voir le texte du manuel. I La valeur actuelle des 110 $ reçus dans un an est
8 I Cela dépend de combien le prix change. le montant qui, s’il est investi aujourd’hui au
9 I Cela dépend si la -l de la valeur sur taux d’intérêt du marché, permettra de recevoir
le marché est fi au montant de 110 $ dans un an. Comme le taux d’intérêt est
l'amortissement conventionnel. de 10%, 100 $ investis aujourd’hui à ce taux,
I0 F Les entreprises sont plus efficientes chaque rapporteront 110 $ dans un an. Par conséquent,
fois qu’on fait des économies d’échelle. 100 $ est la valeur actuelle de 110 $ dans un an.

2 Les restaurants connaissent le problème de


Questions à choix multiple
l’agent-principal parce que les serveurs peuvent
1 d La rareté ne peut jamais être éliminée. donner un mauvais service aux clients, lesquels
2 b Définition risquent de ne plus revenir. Plutôt que de devoir
3 a La VA change lorsque r change. Tous les surveiller de près les serveurs, il vaut mieux
autres paramètres restent fixes. déléguer cette tâche aux clients. Les clients
4 c Les deux coûts explicites, une partie du donnent des pourboires en fonction de la qualité
coût d’opportunité. du service, incitant ainsi le serveur - l’agent -
5 c Voir les formules du Rappel 2. à rendre un bon service. C’est exactement ce que
@ 6 b Profits économiques = profits comptables souhaite le propriétaire du restaurant, qui est
— coûts implicites = 20 000 $ — (0,10 le principal.
X 50 000 $).
7 d Définition. 3 a) Selon la méthode des coûts comptables,
8 d Le coût irrécupérable, qui correspond le coût d’amortissement est calculé en
à l’amortissement économique passé ; la appliquant un taux préalablement spécifié
possibilité à laquelle on a renoncé ne peut au prix d’achat initial du bien de production,
être retrouvée. indépendamment de la valeur du marché.
9 d Toutes les méthodes sont efficientes sur La méthode du coût d’opportunité mesure
le plan technique et il est impossible de le coût d’amortissement économique en tant
les départager puisque les quantités des que variation de la valeur du marché du bien
deux facteurs changent simultanément. au cours de la période concernée.
10 b 2: 240 $ alors que 1 et 3: 250 $. b) Lorsqu’une entreprise emprunte des capitaux,
11 b d, avantage, a et c, inconvénients les méthodes de calcul des coûts
pour l’entreprise individuelle et pour d’opportunité et des coûts comptables seront
la société de personnes, les mêmes ; les deux prendront en compte les
12 b c vrai pour l’efficience économique. paiements d’intérêt explicites.
L’inverse de d est vrai. c) Lorsqu’une entreprise utilise ses propres
13 e Voir le texte du manuel. capitaux plutôt que d’emprunter, les
14 b VA - 100$ /(I +0,1). méthodes de calcul des coûts d’opportunité
15 b Voir le texte du manuel. et des coûts comptables sont différentes.
16 c Voir le texte du manuel. Le coût comptable sera de nul puisqu’il n’y
17 a Les autres ont des droits fixes. a pas de paiements d’intérêt explicites. La
18 c Amortissement comptable = (5 %) méthode de calcul des coûts d’opportunité
X 100 000 $. Amortissement économique tient compte du fait que ces capitaux auraient
= A valeur sur le marché. pu être prêtés et que le revenu d’intérêt
19 c Revenu auquel on a renoncé. (implicite) qui aurait été réalisé représente
20 c Définition. le coût d’opportunité.
L'ORGANISATION DE LA PRODUCTION 123

d) Si le propriétaire s’était consacré à d’autres valeur actuelle de 2) est 1 100 $ / ( 1,05)2


activités, le salaire qu’il aurait gagné serait = 997,73 $. Par conséquent, la valeur actuelle
un coût d’opportunité, mais pas un coût de l’obligation de deux ans est la somme
comptable. Si le propriétaire se paie un de 95,24 $ + 997,73 $ = 1 092,97 $.
salaire, celui-ci comptera à la fois comme
coût d’opportunité et comme coût comptable. TABLEAU 9.2 SOLUTION
En plus de fournir le travail, le propriétaire
fournir l’esprit d’entreprise qu’il aurait pu Valeur actuelle
également fournir ailleurs. Le coût de l’esprit (en dollars)
d’entreprise est le profit normal, qui est un
Versements futurs r = 0,1 r = 0,05
coût d’opportunité, mais pas un coût
comptable. 4 90,91 95,24
B 82,64 90,70
Une méthode est techniquement efficiente s’il C 1 000,00 1 092,97
n’est pas possible d’augmenter la production sans
augmenter la quantité de facteurs de production.
Une méthode est économiquement efficiente & 7 Le détenteur d’obligation serait prêt à

lorsque le coût de production d’une quantité payer davantage pour l’obligation B que

donnée est le plus bas possible. L’efficience pour l’obligation A, car la valeur actuelle de

technique est indépendante des prix alors que l’obligation B est plus élevée. Bien que les deux
obligations rapportent le même montant total
l’efficience économique dépend du prix des
en dollars pour deux ans, l’obligation B rapporte
facteurs de production (prix relatifs). Une
la moitié de ce montant au bout d’un an
méthode de production économiquement
seulement. Le détenteur de l’obligation A,
efficiente est toujours techniquement efficiente
toutefois, doit attendre deux ans pour recevoir
alors qu’une méthode techniquement efficiente
le plein montant.
n’est pas forcément économiquement efficiente.
Si nous considérons le paiement de 220 $
de l’obligation A équivalant à deux paiements
Comme nous l’avons vu à l’exemple de la de 110 $, chacun versé après deux ans, chaque
page 201 du manuel, la réparation de voitures paiement est actualisé par (divisé par) (1 + r)2.
peut être coordonnée par le marché ou par Un paiement de 110 $ de l’obligation A et un
une entreprise. Quoi qu’il en soit, l’institution paiement de 110 $ de l’obligation B ont donc
(marché ou entreprise) qui coordonne des valeurs actuelles égales. Toutefois, le premier
réellement est l’institution qui est la plus efficiente. paiement de 110 $ de l’obligation B est versé
Dans les cas où les coûts de transaction, les après un an et il n’est donc actualisé que par
économies d’échelle ou les économies de (1 + r), actualisation inférieure à celle de (1 + r)2,
production en équipe sont élevés, les entreprises qui s’applique à l’autre paiement de 110 $ de
sont probablement plus efficientes et ce sont l’obligation A. Comme ce facteur d’actualisation
elles qui coordonneront l’activité économique. est le dénominateur de la formule de la valeur
Cependant, l’efficience des entreprises est actuelle, le plus petit dénominateur du calcul de
limitée et, dans de nombreux cas, c’est le marché l’obligation B nous permet de voir que la valeur
qui coordonne l’activité économique parce actuelle de l’obligation A est plus élevée.
qu’il est plus efficient. L’équation suivante, qui compare les valeurs
actuelles des obligations A (VAa) et B (VAB)
Le tableau complet est présenté ici au tableau présente ce résultat.
9.2 Solution. Bien que chaque réponse
corresponde à une application directe de la
110 110 ,.
formule de la valeur actuelle, voici comment
B 1 +r (1 + r)2 *
calculer la valeur actuelle des versements futurs
C lorsque r — 0,05. Il faut considérer les ... 110 110 220
versements futurs C comme une promesse VA. —-1-=-
A (1 + r)2 (1 + r)2 (1 +r)2
à deux volets: 1) 100 $ au bout d un an et
2) 1 100 $ au bout de deux ans (la somme de la
valeur de l’obligation et le second paiement de 8 a) Au tableau 9.3, on présente les calculs
coupon). La valeur actuelle est alors la somme d’Angèle et d’Édith. Les références à gauche
des valeurs actuelles des deux volets. La valeur correspondent aux numéros des données

actuelle de 1) est 100 $ / 1,05 = 95,24 $ et la du problème.


124 CHAPITRE 9

la main-d’œuvre, il sera de 400 $. La méthode


TABLEAU 9.3
2 est donc économiquement efficiente pour
le pays B.
Calculs d’Angèle Calculs d’Édith
La raison de cette différence est que
(coûts (coûts
l’efficience économique suppose le coût de
Références comptables) d’opportunité)
production le plus bas possible. Lorsque les prix
1 0 $ 30 000 $ relatifs des facteurs de production sont différents
2 9 000 9 000 dans les deux pays, les coûts relatifs de production
3 0 3 000 seront différents aussi, selon les deux méthodes.
4 3 000 4 000 Votre colocataire a donc tort.
5 20 000 20 000
6 40 000 40 000 <S> 10 a) Si le salaire horaire dans le pays B passe à
7 0 0 5 $, la production d’une tonne de blé coûte
8 0 14 000 500 $, quelle que soit la méthode utilisée.
Coût total 72 000 $ 120 000 $ Pourquoi ? Déterminez le coût avec la
méthode 1 (Q) et avec la méthode 2 (C2)
b) Le revenu est de 100 000 $. Angèle calcule de la manière suivante :
le profit comptable en utilisant la formule Cl=20Pm+20Ph
suivante :
C2=l00Ph
Profits
comptables = revenus — coûts explicites
= 100 000 $-72 000$ où Pm est le coût d’une heure-machine et
= 28 000 $ Ph, le salaire horaire. Nous savons que Pm
Edith calcule le profit économique = 20 $ et nous devons trouver la valeur de Ph
en utilisant la formule suivante : pour que les deux méthodes soient également
Profits efficientes ; c’est-à-dire Ph tel que Cl — C2.
économiques = revenus - coûts d’opportunité Donc:
= 100 000$- 120 000$
= -20 000 $ 20 Pm + 20 Ph = 100 Ph
(une perte économique) 20(20$)+ 20 Ph =100 Ph

400 $ = 80Ph
Ces deux méthodes de production sont
techniquement efficientes. La méthode
n=pl
de production économiquement efficiente
b) Si le coût d’une heure-machine est de 32 $,
a le coût de production d’une tonne de blé
le coût de production d’une tonne de blé est
le plus bas possible. Dans le pays A, le salaire
de 800 $ dans le pays A quelle que soit la
horaire est de 8 $ et le coût d’une heure-machine
méthode utilisée. La question est celle-ci :
est de 20 $. Le coût de production d’une tonne
Étant donné un salaire horaire de 8 $ (Ph)
de blé est de 560 $ avec la méthode 1 et de
dans le pays A, à combien faut-il que le Pm
800 $ avec la méthode 2. Par conséquent,
s’élève pour que C\ = C2 ? Nous devons donc
la méthode 1 est économiquement efficiente
résoudre l’équation suivante:
pour le pays A.
Le salaire horaire est de 4 de $ dans
20Pm+20Ph = 100 Ph
le pays B et, par conséquent, les coûts de
20Pm +20(8$) = 100(8$)
production d’une tonne de blé seront différents.
Avec la méthode 1, le coût sera de 480 $, mais 20 Pm =640$
avec la méthode 2, qui ne fait appel qu’à de Pm =32$
La production et les coûts

♦ à mesure que la quantité du facteur de production


CONCEPTS CLÉS variable T, le Pm T (rendement marginal
croissant) atteint le maximum, et ensuite
si (rendement marginal décroissant). Lorsque
Les objectifs et les contraintes
Pm > PM, PM T. Lorsque Pm < PM, PM i.
de l’entreprise
Lorsque Pm — PM, le PM est au maximum ;

Les entreprises visent à maximiser leurs profits. ♦ la loi des rendements décroissants - la quantité
Les profits que peut réaliser une entreprise sont limités de facteurs fixes étant constante, l’entreprise utilise
par des une plus grande quantité de facteurs de production
variables et son Pm finit par diminuer.
♦ contraintes de marché :

• demande limitée de produits ou de services


et quantité limitée de facteurs de production ;
Le coût à court terme
• pour les petites entreprises, les contraintes
de marché sont les prix donnés pour les produits
La forme des courbes de coût à court terme dépend
et les services et les facteurs de production.
des techniques utilisées et des courbes de produits.
♦ contraintes techniques.
♦ Coût total (CT) = CFT+ CVT
L’entreprise prend ses décisions selon deux laps de temps.
• Coût fixe total (CFT) — coût des facteurs
♦ Le court terme — un facteur de production au de production fixes.
moins est fixe. Les autres facteurs de production • Coût variable total (CVT) - coût des facteurs
sont variables. de production variables.
• Coût marginal (Cm) — A CT résultant
♦ Le long terme — tous les facteurs de production
de la production d’une unité T.
sont variables.
♦ Coût total moyen {CTM) — CFM + CVM

• Coût fixe moyen (CFM) - coût fixe total


Les contraintes techniques à court terme par unité de production.
• Coût variable moyen (CVM) - coût variable
La production à court terme est décrite par total par unité de production.
♦ la courbe de produit total (PT) — la production • Courbe CFM X constamment à mesure
maximale possible avec une quantité fixe de que la production T.
capital, lorsqu’on fait varier la quantité de main- • La forme des courbes CVM, CTM
et Cm est en forme de U.
d’œuvre ;
• À mesure que la production T, le Cm 4-, atteint
♦ la courbe de produit marginal {Pm) — APL que un minimum et ensuite T.
l’on obtient de T d’une unité de la quantité du • Lorsque Cm < CTM, CTM X. Lorsque
facteur de production variable ; Cm > CTM, CTM î. Lorsque Cm = CTM,

♦ la courbe de produit moyen {PM) — PI par unité le CTM est au minimum. La relation est
la même avec le Cm et le CVM.
de facteur de production variable;
126 CHAPITRE 10"

La taille des installations et le coût facteurs de production fixes, le produit marginal


du facteur de production variable finit par
Coût à long terme - coût de production lorsque
diminuer. Cette loi explique pourquoi les courbes
tous les facteurs de production, y compris la taille
de produit marginal et de produit moyen finissent
des installations, sont économiquement efficients.
par décroître et pourquoi la courbe du produit
Fonction de production - relation entre la production
total s’aplanit. Lorsque la productivité diminue,
maximale possible et les quantités de facteurs de
les coûts augmentent et la pente de la courbe de
production, lorsque tous les facteurs de production coût marginal finit par devenir positive. Lorsque
sont variables.
le produit marginal diminue, le coût marginal
♦ Rendements d’échelle - î production résultant augmente.
du même pourcentage T de tous les facteurs La courbe de coût marginal, quant à elle,
de production. Trois possibilités : explique la forme en U des courbes de coût

• rendements d’échelle constants - pourcentage variable moyen et de coût total moyen. Lorsque

T de la production de l’entreprise la courbe de coût marginal se situe au-dessous

= pourcentage T de la quantité des facteurs de la courbe de coût variable (ou total) moyen,

de production ; la courbe de coût variable (ou total) moyen est

• rendements d’échelle croissants (économies descendante. Lorsque la courbe de coût marginal

d’échelle) - pourcentage î de la production se situe au-dessus de la courbe de coût variable

de l’entreprise > pourcentage T de la quantité (ou total) moyen, la courbe de coût variable

des facteurs de production ; (ou total) moyen est ascendante. Le point

• rendements d’échelle décroissants d’intersection de la courbe de coût marginal

(déséconomies d’échelle) - pourcentage et de la courbe de coût variable (ou total) moyen

T de la production de l’entreprise < pourcentage représente le coût variable (ou total) moyen le

T de la quantité des facteurs de production. plus bas.


La loi des rendements décroissants vous
♦ La courbe de coût moyen à long terme (CMLT) -
aidera à comprendre les relations qui existent
indique la relation en forme de U qui se noue
entre les nombreux graphiques et notions se
entre le CTM le moins élevé accessible et la
rapportant au court terme qui sont présentés
production, lorsque tous les facteurs de production
dans ce chapitre. Attardez-vous plus
sont variables. Lorsque la CMLT I-, les rendements
particulièrement aux notions et graphiques
d’échelle sont croissants. Lorsque la CMLT est
relatifs au coût unitaire, notamment le coût
horizontale, les rendements d’échelle sont
marginal, le coût variable moyen et le coût total
constants. Lorsque la CMLT T, les rendements
moyen, car ce sont eux qui reviendront le plus
d’échelle sont décroissants.
souvent dans les chapitres ultérieurs quand nous
analyserons le comportement des entreprises.

RAPPELS 2 Assurez-vous que vous comprenez bien la


figure 10.1 (figure 10.5b, p. 217 du manuel).
I Ce chapitre nous présente de nombreuses
C’est le graphique le plus important du chapitre
notions nouvelles et des graphiques que nous
et l’un des plus importants de la microéconomie.
n’avons pas vus jusqu’à maintenant, ce qui peut
inquiéter de prime abord. Mais ne laissons pas
FIGURE 10.1
l’arbre nous cacher la forêt. Il existe une relation
simple et fondamentale entre les fonctions de
production et les fonctions de coût.
Nous étudions tout d’abord la fonction
de production à court terme et les notions
de produit total, de produit marginal, et
de produit moyen. Nous abordons ensuite
la fonction de coût à court terme ainsi que les
notions de coût total, de coût marginal, de coût
variable moyen et de coût total moyen.
Cependant, tous ces notions, en apparence
disparates, sont reliées à la loi des rendements
décroissants. Cette loi nous indique que,
.lorsqu’une entreprise utilise des unités
supplémentaires d’un facteur de production
variable tout en maintenant la quantité des
LA PRODUCTION ET LES COÛTS 127

On obtient les courbes de coût total moyen


FIGURE 10.2
0CTM), de coût fixe moyen (CFM) et de coût
variable moyen (CVM) en divisant les valeurs
du CT, du CFT et du CVT par la quantité
produite. Comme il s’agit de valeurs moyennes
pour une production fixe, on les place
directement au-dessus des unités de production
correspondantes. Par contre, le coût marginal
(Cm) correspond au changement du coût total
(ou du coût variable total) obtenu à partir d’une
unité de production supplémentaire. On le situe
à mi-chemin entre les unités de production
correspondantes. Les courbes CTM, CVM et
Cm sont extrêmement importantes. Les courbes
CTM et CVM ont toutes deux la forme d’un U.
La courbe Cm est, elle aussi, en forme de U
et elle croise les courbes CTM et CVM à leurs
points minimum. La courbe Cm est au-dessous
si certains facteurs de production sont variables,
des courbes CTM et CVM lorsque le CTM et le
il y en a au moins un qui reste fixe. Le long
CVM diminuent, et au-dessus des courbes CTM
terme se rapporte à un horizon de planification
et CVM lorsqu’ils augmentent. La courbe CFM,
éloigné où tous les facteurs de production
moins importante, décroît continuellement
utilisés sont appelés à varier.
à mesure que la production augmente.
5 Les dernières rubriques de ce chapitre portent
Vous tracerez probablement un graphique sur la fonction de production à long terme
comme celui de la figure 10.1 une centaine et la fonction de coût, lorsque la taille des
de fois au moins durant ce cours. Voici quelques installations varie. Alors que les rendements
conseils pour le tracer rapidement et facilement. décroissants permettent de comprendre les
N’oubliez pas de nommer les axes ; coûts à court terme, les rendements d’échelle
la quantité de production (Q) en abscisse permettent de comprendre les coûts à long
et le coût unitaire en ordonnée. terme. On appelle rendements d’échelle
Tracez une courbe de coût marginal à pente l’augmentation de la production par rapport
positive comme celle de la figure 10.2. La courbe à l’augmentation des facteurs de production,
de coût marginal peut avoir une petite portion lorsque tous les facteurs de production augmentent
à pente négative au début, mais cela ne change proportionnellement. Les rendements d’échelle
rien pour l’analyse ultérieure. Tracez ensuite une peuvent être croissants, constants ou décroissants,
courbe en U (courbe CVM), qui est à pente et ils correspondent aux portions à pente négative,
négative jusqu’à ce qu elle touche la courbe de horizontale et croissante de la courbe de coût
coût marginal, puis qui remonte. Choisissez, moyen à long terme.
enfin, un point situé plus à droite sur la courbe
de coût marginal et tracez une autre courbe en
U (courbe CTM), dont le point le plus bas sera
AUTOÉVALUATION
le point que vous venez de choisir. Pour finir,
nommez les courbes.
Chaque fois qu’on vous pose une question Vrai/Faux/Incertain
sur ces courbes (y compris dans 1 autoevaluation), (Justifiez votre réponse.)
tracez un graphique avant de répondre.

Assurez-vous que vous comprenez bien comment I Les contraintes qui limitent les possibilités
les économistes utilisent les expressions court qu’a l’entreprise de transformer les facteurs
terme et long terme. Ces expressions ne font pas de production en biens et en services sont
référence à des échéances précises ni à des dates des contraintes du marché qui restreignent
spécifiques du calendrier. Elles constituent plutôt sa rentabilité.
des horizons de planification. Le court terme
est un horizon de planification bref où, même
128 CHAPITRE 10

Avec une quantité fixe de capital, si 2 travailleurs Questions à choix multiple


supplémentaires produisent 15 unités
supplémentaires, le produit marginal du travail
1 En économie, le court terme désigne une période
est de 15 unités de production.
a) de un an ou moins.
b) durant laquelle tous les facteurs
de production sont variables.
c) durant laquelle tous les facteurs
La courbe de produit moyen croise la courbe de
de production sont fixes.
produit marginal au maximum de cette dernière.
d) durant laquelle au moins un facteur
de production est fixe et au moins
un facteur de production est variable.
e) durant laquelle tous les facteurs de
4 La loi des rendements décroissants implique production sont variables, tandis que
que la courbe de produit marginal finit par être la technologie est fixe.
négative à mesure que la quantité de facteurs
de production variables augmente. 2 La courbe de coût marginal {Cm) croise
a) les courbes CTM, CVM et CFM
à leurs points minimum.
b) les courbes CTM et CFM à leurs points
La loi des rendements décroissants implique minimum.
que les rendements marginaux ne peuvent être c) les courbes CVM et CFM à leurs points
croissants. minimum.
d) les courbes CTM et CVM à leurs points
minimum.
e) les courbes CT et CVT à leurs points
6 Le coût variable moyen atteint son minimum minimum.
au niveau de production pour lequel le produit
moyen est à son maximum. 3 Le coût marginal est le montant correspondant à
a) l’augmentation du coût total, lorsqu’un
travailleur supplémentaire est engagé.
b) l’augmentation du coût fixe, lorsqu’un
7 Si le coût variable moyen est décroissant, le coût travailleur supplémentaire est engagé.
marginal décroît aussi. c) l’augmentation du coût variable, lorsqu’un
travailleur supplémentaire est engagé.
d) l’augmentation du coût total, lorsqu’une unité
supplémentaire est produite.
e) l’augmentation du coût fixe, lorsqu’une unité
Aucune portion d’une courbe de coût total
supplémentaire est produite.
moyen à court terme ne peut se situer au-dessous
de la courbe de coût moyen à long terme.
4 Une hausse de prix d’un facteur de production
fixe entraînera
a) le déplacement vers le haut de la courbe
de coût variable moyen d’une entreprise.
9 Des rendements d’échelle croissants signifient
b) le déplacement vers le haut de la courbe
que la pente de la courbe de coût moyen à long
de coût total moyen d’une entreprise.
terme est négative.
c) le déplacement vers le bas de la courbe
de coût total moyen d’une entreprise.
d) le déplacement vers le haut de la courbe
de coût marginal d’une entreprise.
V 10 À long terme, les courbes de coût total e) le déplacement vers le bas de la courbe
et de coût variable total se confondent. de coût marginal d’une entreprise.
'uXnJl
5 Prenons l’exemple d’un champ de maïs qu’on
s’apprête à moissonner. Le travail est le seul
facteur de production variable qui intervient
dans le résultat. Le produit total (en boisseaux)
LA PRODUCTION ET LES COUTS 129

en fonction du nombre de travailleurs engagés c) est égale au CFM.


est donné au tableau 10.1 d) est égale au CFT.
e) est égale au Cm.
TABLEAU 10.1
10 Prenons un coût total de 20 $ avec
Nombre de travailleurs Produit total une production de 4 unités et de 36 $
avec une production de 6 unités. Entre
0 0
4 et 6 unités de production, le coût marginal
1 3
a) est inférieur au coût total moyen.
2 7
b) est égal au coût total moyen.
3 10
c) est égal au coût variable moyen.
4 12
d) est supérieur au coût total moyen.
e) ne peut être comparé à aucun coût moyen
À partir de quel travailleur supplémentaire la
sans données supplémentaires.
productivité marginale commence-t-elle à
décroître ? I I La courbe de coût moyen à long terme
a) À partir du premier a) se déplace vers le haut lorsque les coûts
b) À partir du deuxième fixes augmentent.
c) À partir du troisième b) se déplace vers le bas lorsque les coûts
d) À partir du quatrième fixes augmentent.
e) La productivité marginale ne décroît pas, c) correspond à la courbe de coût total moyen
puisque le produit total augmente toujours. à court terme ayant le coût le plus bas.
d) passe par les points minimum de toutes
6 Lorsque le produit marginal du travail les courbes de coût total moyen à court terme
est inférieur au produit moyen du travail pour chaque taille d’installation.
a) le produit moyen du travail augmente. e) donne le coût total moyen à court terme le
b) le produit marginal du travail augmente. plus bas pour chaque niveau de production.
c) la pente de la courbe de produit total
est négative. 12 La courbe de coût variable moyen
d) la productivité marginale de l’entreprise se déplacera vers le haut si
est décroissante. a) les coûts fixes augmentent.
e) Aucune de ces réponses. b) il y a un progrès technique.
c) le prix des facteurs de production
7 Que peut-on conclure si la quantité de tous les variables baisse.
facteurs de production augmente de 10% et que d) le prix de la production augmente.
la production augmente de moins de 10%? e) Aucune de ces réponses.
a) Le coût total moyen diminue.
13 Les coûts fixes d’une entreprise sont de 100 $.
b) Le coût total moyen augmente.
Si les coûts totaux sont de 200 $ pour une unité
c) La pente de la courbe CMLT est négative.
de production et de 310 $ pour deux unités,
d) Les rendements d’échelle sont croissants.
quel est le coût marginal de la seconde unité ?
(ej) Les rendements d’échelle sont décroissants.
a) 100$

8 Un progrès technique fera déplacer


(0)110$
c) 200 $
1 les courbes PT, PM et Pm vers le haut.
d) 210 $
2 les courbes PT, PM et Pm vers le bas.
e) 310$
3 les courbes CT, CTM et Cm vers le haut.
4 les courbes CT, CTM et Cm vers le bas.
14 Une entreprise veut augmenter la taille de son
a) 1 et 3 installation lorsque
b) 1 et 4
a) son niveau de production correspond
c) 2 et 3 constamment à la portion de sa courbe
d) 2 et 4
de coût total moyen à court terme qui a une
e) Aucune de ces réponses. pente positive.
9 La distance verticale entre les courbes b) son niveau de production correspond
constamment à la portion de sa courbe
CT et CVT
a) diminue à mesure que la production de coût total moyen à court terme qui a une
pente négative.
augmente.
b) augmente à mesure que la production augmente.
130 CHAPITRE 10

c) son niveau de production est inférieur 20 À court terme, une entreprise qui maximise
à sa capacité de production. ses profits est soumise à certaines contraintes.
d) le coût marginal est inférieur au coût Lesquelles ?
total moyen. a) Aux limites de la demande de son produit.
e) le coût marginal est inférieur au coût variable b) Aux limites de l’offre de facteurs
moyen. de production.
c) Aux limites des possibilités de production
15 Si le CTM diminue, alors le Cm doit être techniquement efficientes.
a) croissant. d) Aux limites de l’horizon de planification.
b) décroissant. e) Toutes ces réponses.
c) égal auCTM.
d) supérieur auCTM. 21 Quel facteur, parmi les suivants, «entraîne
(çPinférieur au CTM. pas de baisse du CTM?
a) Un coût marginal décroissant
16 À long terme, b) Un coût variable moyen décroissant
a) seule la taille de l’installation est fixe. c) Un coût fixe moyen décroissant
(b)) tous les facteurs de production sont variables, d) Un produit marginal croissant
cj tous les facteurs de production sont fixes. ^ <g)D es rendements d’échelle croissants
d) une entreprise doit connaître des rendements
22 Reportez-vous à la figure 10.3, qui illustre la
d’échelle décroissants.
courbe de produit total à court terme de Maille
e) Aucune de ces réponses.
Maille. Quel énoncé, parmi les suivants est vrai ?
a) Les points situés au-dessus de la courbe
17 Des rendements d’échelle constants signifient
sont accessibles et inefficaces.
qu’à mesure que tous les facteurs de production
b) Les points situés au-dessous de la courbe
augmentent,
sont accessibles et inefficaces.
a) la production totale reste constante.
c) Les points situés au-dessous de la courbe
b) le coût total moyen reste constant.
sont inaccessibles et inefficaces.
c) le coût total moyen augmente au même
d) Les points situés sur la courbe
rythme que la quantité de facteurs
sont inaccessibles et efficaces.
de production.
e) Les points situés sur la courbe ont tous
d) le coût moyen à long terme reste constant.
des produits marginaux égaux.
e) le coût moyen à long terme augmente
au même rythme que la quantité des facteurs
FIGURE 10.3
de production.

18 D’après la loi des rendements décroissants


1 la productivité marginale finit par
augmenter.
2 la productivité marginale finit par diminuer.
3 le coût marginal finit par augmenter.
4 le coût marginal finit par diminuer.
a) 1 et 3
b) 1 et 4
c) 2 et 3
d) 2 et 4
e) 4 seulement.

19 Le produit moyen du travail correspond


a) à la pente de la courbe de produit total.
b) à la pente de la courbe de produit
marginal.
c) à l’augmentation du produit total, divisée 23 Reportez-vous à la figure 10.3, qui illustre la
par l’augmentation de la quantité de travail courbe de produit total à court terme de Maille
utilisé. Maille. À partir de quel travailleur le produit
d) au produit total, divisé par la quantité marginal devient-il maximal ?
de travail utilisé. a) À partir du premier
e) Aucune de ces réponses. b) À partir du deuxième
LA PRODUCTION ET LES COÛTS 131

c) À partir du troisième secondaire Les Histrions. Est-il possible que


d) À partir du quatrième la moyenne des notes des élèves des deux écoles
e) À partir du cinquième augmente ? Expliquez votre raisonnement.

24 Le coût variable moyen est au minimum


5 Quelle est la différence, le cas échéant, entre
les rendements décroissants et les rendements
au même niveau de production lorsque
d’échelle décroissants ?
a) le produit moyen est au maximum.
b) le produit moyen est au minimum. 6 Comment trace-t-on la courbe de coût moyen
c) le produit marginal est au maximum. à long terme et en quoi diffère-t-elle de la courbe
d) le produit marginal est au minimum. erronée que traçait Jacob Viner ?
e) le coût marginal est au minimum.
7 Le tableau 10.2 donne la production mensuelle
totale possible de voiturettes de golf pour une
25 Un propriétaire de restaurant vous parle de
taille donnée d’installations et pour différentes
son établissement. Quelle affirmation, parmi
quantités de travail.
les suivantes, se rapporte à la loi des rendements
décroissants ?
a) « Plus la qualité de nos ingrédients est grande, TABLEAU 10.2 PRODUCTION MENSUELLE
plus le coût de production de chaque repas DE VOITURETTES DE GOLF
est élevé. »
b) « Si nous doublons la taille de nos locaux, Production

ainsi que le nombre de cuisiniers et de Travailleurs (en unités Produit Produit


serveurs, tout comme notre matériel, nous (par mois) par mois) marginal moyen
pouvons augmenter le nombre de repas que 0 0
nous servons, sans pouvoir doubler le niveau
de production courant. » |1 11
c) « Nous pouvons augmenter le nombre
de repas que nous servons simplement en 2 3
augmentant le nombre de cuisiniers, mais
le nombre de repas produits par chaque 3 6
cuisinier supplémentaire sera inférieur
à celui du cuisinier précédent, car il 4 12
y aurait trop de monde dans la cuisine. »
d) «Nous pouvons servir le même nombre 5 17
de repas avec un moins grand nombre
de cuisiniers, mais nous devrons acheter 6 20
plus de matériel de cuisine pour pouvoir
économiser sur la main-d’œuvre. » 7 22
e) « Nous pouvons servir le même nombre de
repas avec moins de matériel de cuisine, mais 8 23
nous devrons engager plus de cuisiniers. »
a) Remplissez les colonnes «produit marginal»
Problèmes à court développement
et « produit moyen (de travail) ». (Remarquez
1 Pourquoi la courbe de produit marginal qu’il faut inscrire le produit marginal entre
doit-elle croiser la courbe de produit moyen les lignes pour souligner qu’il s’agit du résultat
au point maximum de celle-ci ? de la variation de la quantité de facteurs de
production, c’est-à-dire au passage d’une ligne
2 Pourquoi la courbe de coût marginal doit-elle
à la suivante. Le produit moyen correspond
croiser la courbe de coût total moyen au point
à une quantité fixe de travail et on doit
minimum de celle-ci ?
l’inscrire sur la ligne même.)
3 Expliquer le lien existant, le cas échéant, b) Identifiez les axes d’un graphique et tracez sur
entre la forme en U de la courbe de coût ce graphique la courbe du produit total (PT).
total moyen et a) les coûts fixes, b) la loi c) Sur une autre feuille, identifiez les axes
des rendements qui finissent par diminuer. d’un graphique et tracez sur ce graphique

& 4 Utilisez les concepts de coût marginal et de coût


les courbes de produit marginal (Pm)
moyen pour répondre à la question suivante. et de produit moyen (PM). Il faut situer

Supposons que le plus mauvais élève de 1 école le produit marginal à mi-chemin des points
représentatifs des quantités de travail
secondaire Les Érables soit muté a 1 ecole
132 CHAPITRE 10

correspondantes, comme à la figure 10.2(b), d) Supposons maintenant que le prix de la


de la page 212 du manuel, alors qu’il faut main-d’œuvre augmente, passant à 2 500 $
placer le produit moyen directement au-dessus par mois. Construisez un tableau qui rend
des points représentatifs des quantités de compte des nouvelles courbes Cm et CTM
travail correspondantes, comme à la figure (production, Cm et CTM). Identifiez les
10.3 de la page 213 du manuel). axes d’un graphique et tracez sur ce graphique
les nouvelles courbes Cm et CTM. Quel
8 Examinons maintenant les coûts à court
est l’effet de l’augmentation du prix du facteur
terme de la production de voiturettes
de production variable sur ces courbes ?
de golf. Les deux premières colonnes du
tableau 10.2 sont reproduites au tableau 10.3.
Le coût de 1 travailleur (le seul facteur de Revenons au prix initial de la main-d’œuvre,

production variable) est de 2 000 $ par mois. soit 2 000 $ par mois. Supposons maintenant
Le coût fixe total s’élève à 2 000 $ par mois. que l’on double la quantité des facteurs de
production fixes, de manière à ce que les coûts
TABLEAU 10.3 COÛTS À COURT TERME fixes totaux doublent également, pour s’élever

(MENSUELS) à 4 000 $ par mois. De ce fait, la production


mensuelle de voiturettes de golf augmente pour
Dollars/mois Dollars/voiturette tous les niveaux de main-d’œuvre possibles,
T CFT CVT CT Cm CFM CVM CTM comme l’indique le tableau 10.4.
Q
0 0 2 000 TABLEAU 10.4 NOUVELLE PRODUCTION
MENSUELLE DE
VOITURETTES DE GOLF

2 3 Travailleurs Production
(par mois) (en unités par mois)
3 6
0 0
4 12 11 11
2 4
5 17 3 10
4 19
6 20 5 26
6 31
7 22 7 34
8 36
8 23
a) Construisez un tableau pour les nouvelles

a) À l’aide de ces données, remplissez le tableau courbes Cm et CTM (production, Cm,


10.3 en calculant le coût fixe total (CFT), CTM). Identifiez les axes d’un graphique

le coût variable total (CVT), le coût total et tracez sur ce graphique les nouvelles

{CT), le coût marginal {Cm), le coût fixe courbes Cm et CTM.


moyen {CFM), le coût variable moyen b) Quel est l’effet sur ces courbes (par

{CVM) et le coût total moyen {CTM). Une rapport aux courbes initiales Cm et CTM
fois terminé, votre tableau devrait ressembler du problème 8c) d’une augmentation

au tableau 10.2 de la page 215 du manuel, de la « taille des installations » ?

le coût marginal étant inscrit entre les lignes. Observe-t-on des économies d’échelle ?

b) Identifiez les axes d’un graphique et tracez c) Tracez la courbe de coût moyen à long

les courbes CT, CVT et CFT sur ce graphique. terme {CMLT) dans l’hypothèse que ces

c) Identifiez les axes d’un graphique et tracez deux tailles d’installations sont les seules

les courbes Cm, CTM, CVM et CFM possibles.

sur ce graphique. N’oubliez pas de placer


les points représentatifs de Cm à mi-chemin I La figure 10.4 donne une série de courbes CTM
des points représentatifs des quantités à court terme, numérotées de 1 à 7, et qui
- produites correspondantes. correspondent à 7 tailles d’installations possibles.
LA PRODUCTION ET LES COÛTS 133

6 d Lorsque Pm < PM, Pm 4- (rendements


FIGURE 10.4
décroissants), PM -l, et PT à. pente positive.
7 e Définition. Puisque tous les facteurs
de production sont variables,
a et b ne sont pas pertinents.
8 b La productivité T, les coûts si.
9 d CT — CPT + CVT. Distance constante.
@10 d Cm — 16/2 = 8. Le CTM est de 5 $ avec
4 unités et de 6 $ avec 6 unités.
I I e Définition, d est l’erreur de Jacob Viner.
a et b court terme puisque les coûts sont fixes.
12 e a et d n’ont pas d’effet sur le CVM.
b et c font déplacer le CVM vers le bas.
13 b Coûts fixes non pertinents. ÀC77AQ

a) Tracez la courbe de coût moyen à long terme = (310 $-200 $)/(2 - 1).

sur la figure 10.4. 14 a b et c, les installations sont trop grosses.

b) Si l’entreprise souhaitait produire 100 unités d et e ont trait au court terme.

par jour, quelle serait la taille idéale des Ô 15 e Le Cm pourrait être T ou -l au-dessous

installations ? (Indiquez le numéro de la de la courbe CTM lorsque le CTM si.


courbe CTM à court terme correspondante.) 16 b Définition. Tous les rendements d’échelle

Quelle serait la taille idéale si l’entreprise possibles à long terme.

souhaitait produire 200 unités par jour?


17 d La courbe CMLT est horizontale.
18c 4- Pm —> T Cm.
19 d Définition.
20 e Les contraintes de marché et les contraintes
techniques.
RÉPONSES 21e T rendements d’échelle portent sur le long
CTM porte sur le court terme.
terme ; le
22 b PT accessible mais pas maximal.
Vrai/Faux/1 ncertain 23 b Pm — pente de la courbe PT = 6 pour
(Justifiez votre réponse.) le deuxième travailleur.
24 a CVM I aussi longtemps que PM T.
1 F Exemple de contrainte technique. Voir la figure 10.6 du manuel.
2 F A PT/A T= 15/2 = 7,5. 25 c Pm si car le restaurant utilise une plus grande
@ 3 F Vrai si l’on inverse les expressions quantité du facteur de production variable
«produit moyen» et «produit marginal». (travail).
4 V Pm doit finir par 4- avec T T.
5 F Rendements décroissants. Le Pm Problèmes à court développement
peut initialement T.
@ 6 V CVM = CVT/Q = sT/Q = s/(Q/T) I Étant donné que la courbe de produit moyen
= s/PM. (s : salaire)
monte pour finir par descendre, lorsque le
<0 7 I Cm doit être inférieur au CVM, mais produit moyen augmente, le produit marginal
Cm doit être T ou 4-. doit être supérieur au produit moyen, et lorsque

8 V Définition CMLT. le produit moyen diminue, le produit marginal

9 V Voir le texte du manuel. doit être inférieur au produit moyen. Par


10 V Tous les coûts sont variables à long terme. conséquent, la courbe de produit marginal
doit croiser la courbe de produit moyen à son
point maximum. Pour que le produit moyen
Questions à choix multiple puisse augmenter, il doit être le résultat d’une
augmentation plus grande de la quantité produite
par la dernière unité du facteur de production.
1 d Définition.
2 d CFM toujours i, CT et CVT toujours T. De ce fait, le produit marginal est supérieur
au produit moyen. Dans le même ordre d’idées,
3 d Définition.
4 b Elle n’aura pas d’effet sur le CVM ni sur lorsque le produit moyen diminue, c’est parce
qu’il est le résultat d’un produit marginal moins
leCm.
5 c Pm à partir du premier = 3, Pm à partir du élevé. Lorsque le produit moyen est au maximum,
deuxième = 4, Pm à partir du troisième — 3. il ne peut ni augmenter ni diminuer, de sorte
134 CHAPITRE 10

que le produit marginal ne peut être ni supérieur (g) 4 Oui, il est possible que la moyenne des notes
ni inférieur au produit moyen. Par conséquent, des élèves de chaque école augmente. L’élève muté
le produit marginal doit être égal au produit peut être considéré comme un élève marginal.
moyen. Si la moyenne de ses notes, même s’il est le plus
mauvais élève de l’école Les Erables, est supérieure
La réponse à ce problème fait écho à la réponse à la moyenne des notes des élèves de l’école
du problème 1. La courbe de coût total moyen Les Histrions, alors les résultats sont les suivants :
est en forme de U, d’abord décroissante puis la moyenne des notes des élèves de l’école Les
croissante, à mesure que la production augmente. Érables augmente avec l’élimination des notes les
Lorsque le coût total moyen diminue, le coût plus faibles; et la moyenne des notes des élèves
marginal doit être inférieur au coût total moyen, de l’école Les Histrions augmente parce que
et lorsque le coût total moyen augmente, le coût l’élève muté (marginal) fait monter la moyenne
marginal doit être supérieur au coût total moyen. des notes des élèves.
Par conséquent, la courbe de coût marginal
doit croiser la courbe de coût total moyen 5 Selon la loi des rendements décroissants,
à son point minimum. Pour que le coût total à mesure qu’une entreprise utilise des unités
moyen diminue, il doit être le résultat d une supplémentaires d’un facteur de production
augmentation plus petite du coût de la dernière variable, tout en maintenant constante la quantité
unité produite. Par conséquent, le coût marginal des facteurs de production fixes, le produit marginal
est inférieur au coût total moyen. Dans le même du facteur de production variable finira par
ordre d’idées, lorsque le coût total moyen diminuer. On observe des rendements d’échelle
augmente, c’est parce qu’il est le résultat d’un décroissants lorsque l’entreprise augmente au
coût marginal plus élevé. Lorsque le coût total même rythme tous ses facteurs de production, ce
moyen est au minimum, il ne peut ni augmenter qui entraîne un plus faible rythme d’augmentation
ni diminuer, de sorte que le coût marginal ne de la production. Les rendements décroissants
peut être ni supérieur ni inférieur au coût total (marginaux) portent sur le court terme puisqu’il
moyen. Par conséquent, le coût marginal doit doit y avoir des facteurs de production fixes.
être égal au coût total moyen. Les rendements d’échelle décroissants portent
sur le long terme puisque tous les facteurs
La courbe de coût total moyen (CTM) de production doivent être variables.
doit sa forme en U à deux forces opposées :
a) l’étalement des coûts frxes sur une plus 6 On peut atteindre n’importe quel niveau de
grande production et b) la loi des rendements production en changeant la taille des installations,
décroissants. mais pour chaque niveau de production, une
À mesure que la production augmente, taille d’installations entraîne le coût total moyen
les coûts fixes sont étalés sur une plus grande à court terme le plus bas possible. La taille des
production de sorte que le coût fixe moyen installations peut être changée à long terme. La
(CFM) diminue. Au début, à mesure que la courbe de coût moyen à long terme passe alors
production augmente, la productivité marginale par le coût total moyen à court terme le plus bas
du facteur variable augmente, ce qui fait baisser possible de chaque niveau de production. Jacob
le coût variable moyen (CVM). La baisse du Viner s’est trompé au sujet de la courbe de coût
CVM et CFM entraîne la baisse du coût
du moyen à long terme, car a), selon lui, elle doit
total moyen (CTM), ce qui explique en partie passer par le point minimum de chaque courbe
pourquoi une portion de la courbe CTM a de coût moyen à court terme (ce qui est faux)
une pente négative. À mesure que la production et b) elle ne doit pas se situer au-dessus de
augmente, les rendements finissent par décroître, n’importe quel courbe de coût à court terme
la productivité marginale diminue et le CVM à n’importe quel point (ce qui est juste). L’erreur
augmente. En fin de compte, l’augmentation est que ces deux conditions ne peuvent être
du CVM est plus rapide que la baisse remplies simultanément. En fait, la courbe de
du CFM, ce qui explique en partie pourquoi coût moyen à long terme ne passe pas par le
une portion de la courbe CTM a une pente point minimum de chacune des courbes de coût
positive. moyen à court terme.
LA PRODUCTION ET LES COÛTS 35

7 a) Voici le tableau 10.2 Solution. c) La figure 10.6 présente les graphiques


du produit marginal et du produit moyen.
TABLEAU 10.2 SOLUTION
PRODUCTION MENSUELLE FIGURE 10.6
DE VOITURETTES DE GOLF
Production
Travailleurs (en unités Produit Produit
(par mois) par mois) marginal moyen

0 0 0
.1
1 1 1,00
.2
2 3 1,50
.3
3 6 2,00
.6
4 12 3,00
.5
5 17 3,40
.3
6 20 3,33
8 a) Voici le tableau 10.3 Solution.
.2
7 22 3,14
1 TABLEAU 10.3 SOLUTION
COÛTS MENSUELS
8 23 2,88
À COURT TERME

b) La figure 10.5 présente le graphique


Dollars/mois Dollars/voiturette
de la courbe de produit total.
T Q CFT CVT CT Cm CFM CV7V1 CTM
FIGURE 10.5
0 0 2 000 0 2 000 - - -

2 000
1 1 2 000 2 000 4 000 2 000 2 000 4 000
1 000
2 3 2 000 4 000 6 000 667 1 333 2 000
667
3 6 2 000 6 000 8 000 333 1 000 1 333
333
4 12 2 000 8 000 10 000 167 667 833
400
5 17 2 000 10 000 12 000 1 18 588 706
667
6 20 2 000 12 000 14 000 100 600 700
1 000
7 22 2 000 14 000 16 000 91 636 727
2 000
8 23 2 000 16 000 18 000 87 696 783
136 CHAPITRE 10

b) Les courbes CT, CVT et CFT


FIGURE 10.9
sont représentées à la figure 10.7.

FIGURE 10.7

c) Les courbes Cm, CTM, CVM et CFM sont Cm CTM


représentées à la figure 10.8. Production (en dollars) (en dollars)

0 0
FIGURE 10.8
.2 500
I 4 500
.I 250
3 2 333
. 833
6 I 583
. 417
12 I 000
. 500
17 853
. 833
20 850
.I 250
22 886
.2 500
23 957
d) Les nouvelles courbes Cm et CTM
(et le tableau correspondant) sont présentées
à la figure 10.9. Les courbes initiales, 9 a) Les nouvelles courbes Cm et CTM (et
Cmx et CTMX sont indiquées à titre le tableau correspondant) sont présentées
de renseignement. Les nouvelles courbes à la figure 10.10. Les nouvelles courbes sont
sont nommées Cm2 et CTM2. Ces deux nommées Cm3 et CTMy Les courbes initiales,
courbes se sont déplacées à la suite d’une Cmx et CTMX sont indiquées à titre de
augmentation du coût de la main-d’œuvre. renseignement.
LA PRODUCTION ET LES COÛTS 137

des économies d’échelle (rendements


FIGURE 10.10
d’échelle croissants) jusqu’au niveau
Coût par unité (en dollars)

de production où Cm3 croise CTM3


(environ 32 unités).
c) La courbe de coût moyen à long terme
est indiquée à la figure 10.10 par la ligne
en gras qui passe par le coût total moyen
à court terme le plus bas possible à chaque
niveau de production. Dans cet exemple,
il se trouve quelle se confond avec la courbe
CTMy

10 a) La courbe de coût moyen à long terme


est indiquée à la figure 10.4 Solution. Cette
courbe parapluie indique le coût moyen
à court terme le plus bas pour chaqde niveau
de production.
b) Pour une production de 100 unités,
Cm CTM la taille idéale d’installations est celle qui
Production (en dollars) (en dollars) correspond à la courbe du coût total moyen

0 0 à court terme 2. Pour une production

.2 000 de 200 unités, la taille idéale d’installations

I 6 000 correspond à la courbe du coût total à

. . .667 court terme 5.

4 2 000
. . .333 FIGURE 10.4 SOLUTION

10 I 000
. . .222
19 632
. . .286
26 538
. . .400
31 516
. . .667
34 529
.1 000
36 556

b) Les courbes se déplacent (généralement)


vers le bas et vers la droite lorsque la taille
des installations augmente. On observe
La concurrence

• P (— RM — Rm) > CTM —> profits


CONCEPTS CLÉS économiques.
• P (= RM = Rm) = CTM —> profits
économiques nuis (seuil de rentabilité
La concurrence parfaite
avec CTM minimum ; l’entreprise ne réalise
que des profits normaux).
La concurrence parfaite est un modèle de marché
• P (= RM = Rm) < CTM —> pertes économiques
qui répond aux conditions suivantes : nombreuses
(les profits de l’entreprise sont inférieurs aux
entreprises ; produits identiques ; nombreux acheteurs ;
profits normaux).
accès libre ; information parfaite.
♦ Pour l’entreprise qui subit des pertes économiques :
♦ Chaque entreprise est un preneur de prix.
• Si P > CVM, elle continuera de produire.
♦ Chaque entreprise fait face à une courbe
• Si P < CVM, elle cessera pour un certain temps
de demande parfaitement élastique au prix
ses activités.
du marché.
• Le seuil de fermeture correspond au CVM
L’entreprise maximise ses profits (profit normal minimum.
+ profit économique).
La courbe d’offre d’une entreprise en situation
♦ Profit normal = rendement prévu de l’esprit de concurrence parfaite est sa courbe Cm située
d’entreprise. au-dessus du CVM minimum.
♦ Profit économique = recette totale - coût total
(le coût total inclut le profit normal). La production, le prix
et le profit à court terme
♦ Recette totale (RT) - prix (P) X quantité (Q).
• Recette moyenne {RM) - RT/Q. La courbe d’offre de l’industrie à court terme est
• Recette marginale (Rm) — ART/AQ. la somme horizontale des courbes d’offre des entreprises
• En situation de concurrence parfaite, individuelles.
RM = Rm = R
La quantité et le prix du marché d’équilibre sont
À court terme, l’entreprise décide des quantités
déterminés par les courbes d’offre et de demande
à produire ou de la nécessité de cesser ses activités.
de l’industrie.
♦ Le profit est maximisé avec la quantité pour
♦ À court terme, les entreprises en situation de
laquelle Rm = Cm :
concurrence parfaite peuvent réaliser un profit
• Si Rm > Cm, l’entreprise T Q pour T les profits. économique, un profit normal (profit économique
• Si Rm < Cm, l’entreprise Q pour î les profits. nul), ou subir une perte économique.
♦ La maximisation des profits à court terme ♦ À court terme, le nombre d’entreprises et la taille
peut donner trois résultats : de leurs installations sont fixes.
LA CONCURRENCE 139

La production, le prix et le profit La forme de la courbe d’offre à long terme de l’industrie


à long terme dépend de l’existence d’économies externes ou de
déséconomies externes. La courbe d’offre à long terme
À long terme, le nombre d’entreprises de l’industrie de l’industrie illustre comment change la quantité
et la taille de leurs installations peuvent s’ajuster. offerte à mesure que change le prix du marché,
Les profits ou les pertes économiques indiquent lorsque tous les ajustements possibles ont été faits,
aux entreprises s’il convient d’entrer dans l’industrie y compris les changements apportés à la taille
ou d’en sortir, et entraînent la redistribution des des installations et au nombre d’entreprises.
ressources. La courbe peut être
♦ horizontale pour l’industrie à coûts constants ;
♦ Profits économiques —> nouvelle entrée
♦ à pente positive pour l’industrie à coûts croissants,
—» déplacement vers la droite de la courbe
en raison des déséconomies externes ;
d’offre de l’industrie —> -!• P -A élimination
♦ à pente négative pour l’industrie à coûts
des profits.
décroissants, en raison des économies externes.
♦ Pertes économiques —» sortie des entreprises
Nouvelle technique —» i coûts, T offre de l’industrie
du marché —> déplacement vers la gauche
-A si P. Les entreprises qui utilisent de nouvelles
de la courbe d’offre de l’industrie —» TP techniques réalisent des profits économiques
—> élimination des pertes.
et entrent sur le marché. Les entreprises qui
En situation d’équilibre concurrentiel à long terme s’en tiennent aux anciennes techniques subissent
des pertes économiques et sortent du marché
♦ Rm = P — Cm. Les entreprises maximisent
ou adoptent de nouvelles techniques. À long terme,
les profits à court terme.
toutes les entreprises utilisent les nouvelles
♦ P = CTM minimum. Les profits économiques techniques et réalisent des profits normaux.
sont nuis. Les entreprises ne sont incitées ni
à entrer sur le marché ni à en sortir. La concurrence et l’effîcacité
♦ P = CMLT minimum. La taille de l’installation
est optimale. Les entreprises ne sont pas incitées Allocation optimale des ressources — on ne peut
améliorer la situation d’un individu sans porter
à changer la taille de leurs installations.
atteinte à la situation d’un autre ; il n’y a aucun
gaspillage de ressources. Trois conditions doivent
L’évolution des préférences être remplies.
et l’innovation technique
♦ Leffîcacité dans la consommation — utilité
Avec un déplacement permanent de la demande maximale. Tous les points situés sur la courbe de
demande du marché permettent l’efficacité dans la
♦ -l demande —» -i P, perte économique et sortie consommation en l’absence d’avantages externes -
—> si offre de l’industrie —» T P. À long terme,
les avantages que retirent les personnes autres que
un nombre suffisamment grand d entreprises le consommateur du bien.
sortent de l’industrie, si bien que les entreprises
restantes réalisent des profits normaux. ♦ L’effîcacité dans la production — efficacité
technique et efficience économique. Tous les
♦ T demande -» î P, profit économique et entrée points situés sur la courbe d’offre de l’industrie
—A T offre de l’industrie —» -i P. À long terme, permettent l’efficacité dans la production en
un nombre suffisamment élevé d entreprises l’absence de coûts externes - les coûts subis
entrent dans l’industrie, si bien que les profits par les personnes autres que le producteur
économiques sont éliminés et que les entreprises du bien.
réalisent des profits normaux.
♦ L’efficacité dans les échanges - tous les gains
Le changement du prix d’équilibre à long terme, possibles découlant des échanges ont été obtenus.
dû à un déplacement permanent de la demande, Les gains totaux découlant des échanges sont la
dépend des somme du

♦ conomies externes — les facteurs sur lesquels • surplus du consommateur — la valeur


entreprise n’a aucune prise et qui font baisser les qu’attribue le consommateur au bien — le prix
oûts à mesure que la production de 1 industrie . payé = aire située sous la courbe de demande
et au-dessus du prix du marché ;
♦ déséconomies externes — les facteurs sur lesquels
l’entreprise n’a aucune prise et qui font monter les
coûts à mesure que la production de 1 industrie .
140 CHAPITRE II

• surplus du producteur - recette totale de fonctionner au seuil de rentabilité, où les


- coût d’opportunité = aire située sous le prix profits économiques sont nuis. La définition
du marché et au-dessus de la courbe d’offre. des types de coûts inclus dans la courbe
de coût total moyen permet de le comprendre.
La concurrence parfaite permet une allocation
Souvenez-vous, au chapitre 9, nous avons
optimale des ressources au prix du marché
dit que, pour les économistes, les coûts totaux
et à la quantité d’équilibre en l’absence de coûts
d’une entreprise sont des coûts d’opportunité
et d’avantages externes.
qui comprennent à la fois les coûts explicites
et les coûts implicites.
Les coûts implicites comprennent le revenu
d’intérêt et la rente qu’aurait pu gagner le
RAPPELS propriétaire des ressources en poursuivant une
autre activité ainsi que le coût de ces ressources.
1 Bien que les marchés parfaitement concurrentiels Les propriétaires consacrent à leur entreprise
soient rares dans le monde réel, il y a trois du temps qu’ils auraient pu utiliser pour gagner
raisons importantes pour lesquelles il faut bien un revenu ailleurs. Les propriétaires fournissent
comprendre leur comportement. également l’esprit d’entreprise. Le profit normal
D’abord, de nombreux marchés ressemblent est le rendement qu’ils espèrent retirer de l’esprit
d’assez près à des marchés en situation d’entreprise, et il fait partie des coûts implicites
de concurrence parfaite. Dans l’analyse de de l’entreprise.
ce chapitre, nous trouvons des renseignements Au seuil de rentabilité, où la recette totale
directs et utiles concernant le comportement est égale au coût total (ou, dans le même ordre
de ces marchés. d’idée, où la recette moyenne est égale au coût
Deuxièmement, la théorie de la concurrence total moyen), les propriétaires obtiennent
parfaite nous permet d’isoler les effets des forces toujours un rendement du capital investi dans
concurrentielles en jeu dans tous les marchés, l’entreprise, du temps qu’ils ont consacré à cette
même ceux ne correspondant pas aux hypothèses entreprise et de leur esprit d’entreprise, qui est
du modèle de la concurrence parfaite. égal au meilleur rendement qu’ils auraient pu
Troisièmement, le modèle de concurrence gagner ailleurs. Il s’agit de la définition du coût
parfaite constitue un repère utile pour évaluer d’opportunité - la meilleure possibilité à laquelle
l’allocation optimale relative. ils ont dû renoncer. Comme l’implique
l’expression «profits normaux», il s’agit de
2 À court terme, une entreprise en concurrence profits qui auraient pu être normalement réalisés
parfaite ne peut changer la taille de ses en tant que rendements moyens de l’esprit
installations - certains de ses facteurs de d’entreprise dans n’importe quelle autre
production sont fixes. L’entreprise est également industrie. Au seuil de rentabilité, l’entreprise
un preneur de prix ; elle vend toujours au prix gagne des profits normaux même lorsque ses
du marché, quelle ne peut influencer. La seule profits économiques (parfois appelés profits
variable sur laquelle l’entreprise peut influer est «extra-normaux» ou profits «supérieurs à la
le niveau de production. La condition à court moyenne») sont nuis. En réalisant des profits
terme de la maximisation des profits est de normaux, l’entreprise atteint le même niveau
choisir le niveau de production auquel la recette de profit quelle aurait obtenu en poursuivant
marginale est égale au coût marginal. Il s’agit d’autres activités et, par conséquent, elle est
d’une condition générale qui, comme nous entièrement satisfaite de continuer de produire
le verrons aux chapitres suivants, s’applique dans cette industrie.
à d’autres structures de marché, comme le
monopole et la concurrence monopolistique. Lorsque le prix de production diminue et
Puisque, pour l’entreprise en concurrence devient inférieur au seuil de rentabilité, tout en
parfaite, la recette marginale est égale au prix, étant supérieur au seuil de fermeture, l’entreprise
cette condition de maximisation des profits continue de produire même si elle subit des
prend une forme particulière ; il faut choisir le pertes économiques. Dans ce cas, elle ne réalise
niveau de production où le prix est égal au coût plus de profit normal et, théoriquement, elle
marginal {P = Cm). pourrait en réaliser davantage en changeant
d’industrie. Néanmoins, elle continue de
3 De nombreux étudiants ont du mal à fonctionner à court terme parce que, si elle
comprendre pourquoi une entreprise continue change d’industrie, elle devra subir des coûts.
LA CONCURRENCE 141

Pour changer d’industrie, l’entreprise doit cesser


ses activités ce qui implique, continuer de payer AUTOÉVALUATION
ses coûts fixes totaux (CFT).
Aussi longtemps que le prix est supérieur
Vrai/Faux/Incertain
au seuil de fermeture (coût variable moyen
(Justifiez votre réponse.)
minimum), l’entreprise poursuivra ses activités
puisqu’elle couvrira son coût variable total
I La courbe de demande d’une industrie
et une partie de son coût fixe total. Sa perte
parfaitement concurrentielle est horizontale.
sera moins grande si elle continue de produire
au niveau où P = Cm que si elle cesse ses
activités.
U
Si le prix baisse au-dessous du seuil de
L’objectif des entreprises d’une industrie
fermeture, l’entreprise qui continuera de produire,
concurrentielle est la maximisation des recettes.
perdra non seulement ses coûts fixes totaux,
mais également des montants supplémentaires F
sur chaque unité de production puisque la
recette moyenne est inférieure au coût variable
Les entreprises peuvent subir des pertes à long
moyen. C’est pourquoi, lorsque le prix est
terme, mais pas à court terme. __
inférieur au coût variable moyen, l’entreprise
décide de réduire ses pertes au minimum
en cessant ses activités.
4 Une industrie parfaitement concurrentielle saura
À long terme, les coûts fixes disparaissent et allouer ses ressources de façon optimale, s’il n’y a
l’entreprise peut changer d’industrie et modifier pas de coûts ni d’avantages externes.
la raille de ses installations sans subir de coûts.
Les profits économiques signalent à l’entreprise
qu’il est temps de changer ou de répartir
différemment ses ressources jusqu’à ce quelle 5 Lorsque les prix sont inférieurs au coût moyen
atteigne l’équilibre à long terme. Les entreprises total minimum, l’entreprise cessera ses activités.
sortiront des industries où leurs profits
économiques sont négatifs, et elles entreront
dans celles qui leur permettent de réaliser des
profits économiques positifs. C’est seulement 6 La courbe d’offre d’une entreprise en situation
lorsque les profits économiques seront nuis que de concurrence parfaite indique les quantités de
les entreprises ne chercheront ni à entrer dans produits mis sur le marché à des prix différents,
une industrie ni à en sortir. aussi longtemps que cette entreprise réalise des
Puisque l’entrée dans l’industrie n est profits économiques.
pas limitée, les profits économiques peuvent
être nuis, et les entreprises produisent au
point minimum de leurs courbes de coûts
totaux moyens à long terme lorsqu elles 7 En situation d’équilibre à long terme, chaque
se trouvent en situation d’équilibre a long entreprise d’une industrie parfaitement
terme. concurrentielle choisira la taille d’installations
qui lui permet de produire à un coût total
En situation d’équilibre à long terme, trois moyen à long terme minimum.
conditions doivent être remplies par chaque
entreprise d’une industrie:
i) Rm - p = Cm. Cela indique que les profits
sont maximisés pour chaque entreprise. 8 Dans une industrie concurrentielle, en présence
ii) P = CTM. Cela indique que les profits d’économies externes, la pente de la courbe
économiques sont nuis et que chacune des d’offre à long terme de l’industrie est positive.
entreprises ne peut réaliser que des profits
normaux.
iii) P = CMLT minimum. Cela indique
que la production prend place au point
correspondant au coût moyen à long
terme minimum.
142 CHAPITRE II

9 Supposons qu’une industrie concurrentielle 4 Examinez le cas 11.1 qui suit. Si, le mois
soit en équilibre à long terme lorsque ses coûts dernier, le prix des crosses de fougère était de
fixes totaux augmentent de manière substantielle. 15 $ le sac, François
Toutes les entreprises subiront alors des pertes a) aurait dû cesser ses activités parce que sa
économiques et certaines cesseront leurs recette totale ne lui permettait pas de couvrir
activités. , / son coût variable total.
b) a enregistré une perte économique de 135 $.
c) a réalisé un profit économique nul.
d) a réalisé un profit économique de 50 $.
I0 On peut dire qu’il y a efficacité dans la e) a réalisé un profit économique de 100 $.
consommation lorsque la quantité d’un bien
achetée par un consommateur à un prix donné CAS I l.l
correspond à un point situé sur la courbe de
demande. François est cultivateur de crosses de fougère.
Il a vendu 10 sacs de crosses de fougère, le mois
dernier à un coût fixe total de 100 $ et à un
Questions à choix multiple coût variable total de 50 $.

1 Quelle caractéristique, parmi les suivantes,


«est pas une caractéristique d’une industrie
5 Si le prix des crosses de fougère tombe à 10 $
parfaitement concurrentielle ?
le sac, alors que la production et les coûts restent
a) La pente de la courbe de demande
les mêmes, François
de l’industrie est négative.
a) doit cesser immédiatement ses activités.
b) La courbe de demande est parfaitement
b) atteint le seuil de rentabilité puisque
élastique pour chacune des entreprises.
sa recette totale lui permet tout juste
c) Chaque entreprise décide de la quantité
de couvrir ses coûts fixes.
quelle produira.
c) peut tout aussi bien cesser ses activités
faj)Les produits sont légèrement différenciés,
que continuer de produire, puisque sa perte
e) Il existe de nombreuses entreprises, chacune
de 50 $ lui permet juste de couvrir ses coûts
produisant une petite fraction des produits
variables totaux.
offerts par l’industrie.
d) peut continuer à produire malgré une perte
2 Lorsqu’une entreprise doit faire face à une économique de 50 $.

demande parfaitement élastique de son produit, e) continue de produire en dépit de sa perte


alors de 100$.
a) elle n’est pas un preneur de prix.
b) elle cherchera à baisser son prix 6 Supposez maintenant que le prix des crosses de
pour augmenter ses ventes. fougère reste à 10 $ le sac et que la production de
c) elle cherchera à augmenter son prix François et ses coûts restent les mêmes. Son coût
pour augmenter sa recette totale, fixe total correspond au loyer de la terre et son
sa courbe de recette marginale sera égale bail de cinq ans expire à la fin du mois. François

au prix du produit. devra-t-il renouveler son bail ?


e) elle réalisera toujours des profits a) Oui, parce que sa recette totale lui permettra
économiques nuis. encore de couvrir son coût fixe total.
b) Oui, parce que sa recette totale lui permettra
3 Prenons l’exemple d’une industrie parfaitement encore de couvrir son coût variable total
concurrentielle où le prix du marché est de 10 $. et une portion de son coût fixe total.
Une entreprise produit au niveau où Cm = c) Non, parce que la recette totale doit couvrir
CTM= 15$. À ce niveau de production, le tous les coûts des facteurs pour qu’il puisse
CVM est de 10 $. Que devra faire l’entreprise continuer de cultiver des crosses de fougère
pour maximiser ses profits à court terme ? à long terme.
a) Cesser ses activités. d) Non, parce que, à long terme, les profits
b) Augmenter sa production. économiques nuis indiquent qu’il
O Diminuer sa production. lui faut sortir de la culture des crosses
d) Ne pas modifier sa production. de fougère.
e) On ne possède pas assez de données pour e) Nous ne disposons pas d’assez de données
répondre à cette question. pour répondre à cette question.
LA CONCURRENCE 143

7 Dans quel cas, parmi les suivants, une entreprise 12 Lorsqu’une entreprise en situation de concurrence
en situation de concurrence parfaite réalisera-t-elle parfaite cherche à maximiser ses profits et réalise
des profits économiques? des profits économiques, son niveau de
a) Rm > CVM production est tel que
b) Rm > CTM a) le prix est supérieur au coût marginal.
c) CTM > Cm b) le prix est supérieur à la recette marginale.
d) CTM > RM c) le coût marginal est supérieur à la recette
e) RM > CVM -marginale.
(dHe coût marginal est supérieur au coût total
8 Dans la fourchette des prix inférieurs au coût
moyen.
variable moyen minimum, la courbe d’offre
e) le coût total moyen est supérieur au coût
d’une entreprise parfaitement concurrentielle est
marginal.
a) horizontale au prix du marché.
b) verticale lorsque la production est nulle.
13 Lorsqu’une industrie connaît des économies
c) identique à sa courbe de coût marginal.
externes à mesure quelle s’agrandit à long terme,
d) identique à sa courbe de coût variable moyen.
la courbe d’offre de l’industrie à long terme
e) Aucune de ces réponses.
a) est parfaitement inélastique.
9 À court terme, la courbe d’offre de l’industrie est b) est parfaitement élastique.
a) la somme horizontale des courbes d’offre c) a une pente positive.
des entreprises individuelles. d) a une pente négative.
b) la somme verticale des courbes d’offre e) illustre une allocation inefficace
des entreprises individuelles. des ressources.
c) verticale au point correspondant au niveau
de production totale de toutes les entreprises. 14 Dans quelle condition, parmi les suivantes,
d) horizontale au prix courant du marché. une industrie parfaitement concurrentielle et
e) Aucune de ces réponses. en équilibre à long terme sera-t-elle incapable
de réaliser une allocation optimale ?
10 Prenons l’exemple d’une entreprise qui produit
a) Lorsque les entreprises sont des preneurs
des grandes tasses à partir du travail seulement.
de prix.
Le tableau 11.1 indique deux combinaisons
b) Lorsque de nouvelles techniques sont mises
de production/facteur de production.
au point.
c) Lorsqu’il y a des économies ou des
TABLEAU ll.l déséconomies externes.
d) Lorsqu’il y a des coûts ou des avantages
Grandes tasses Heures de travail
externes.
25 4 e) Lorsque l’entrée dans cette industrie est
27 5 accessible.

Si une tasse coûte 100 $ et que le salaire horaire I5 La figure 11.1 présente les courbes de coût

est de 50 $, quel est le coût marginal de d’une entreprise parfaitement concurrentielle.

production à ces niveaux de production ? Le prix courant du marché est de 11 $ et la


taille des installations est illustrée par la
a) 25$
b) 40$
courbe CMCTX. La production d’équilibre
à court terme de l’entreprise est de
c) 50$
a) 7 unités.
d) 100$
b) 9 unités.
e) Aucune de ces réponses.
c) 10 unités.
I I Une entreprise d’une industrie parfaitement d) 17 unités.
concurrentielle maximise ses profits à court e) 18 unités.
terme en produisant 500 unités. Avec
une production de 500 unités, quel élément,
parmi les suivants ne peut être juste?
a) Cm < CVM
b) Cm < CTM
c) Cm > CTM
d) RM < CTM
e) RM > CVM
144 CHAPITRE II

20 Lorsqu’une entreprise parfaitement


concurrentielle à court terme peut payer ses
coûts variables et une partie de ses coûts fixes,
elle fonctionne dans la portion de sa courbe
de coût marginal qui est située
a) au-dessus du seuil de rentabilité.
b) au-dessous du seuil de rentabilité.
c) au-dessus du seuil de fermeture.
d) au-dessous du seuil de fermeture.
e) entre les seuils de rentabilité et de fermeture.

2I Une entreprise parfaitement concurrentielle


maximise ses profits lorsque
^aple coût marginal est égal au prix et le prix est
supérieur au coût variable moyen minimum.
16 Reportez-vous à la figure 11.1. Le prix courant b) le coût marginal est égal au prix et le prix
du marché est de 11 $ et la taille des installations est supérieur au coût fixe moyen minimum.
est illustrée par la courbe CMCTV À long terme, c) la recette totale est au maximum.
l’entreprise d) le coût variable moyen est au minimum.
a) sortira de l’industrie. e) le coût total moyen est au minimum.
b) conservera la taille actuelle de ses installations,
et d’autres entreprises entreront dans l’industrie. 22 La perte maximale qu’une entreprise subira
c) conservera la taille actuelle de ses installations, à long terme est
et d’autres entreprises sortiront de l’industrie. a) nulle.
d) augmentera la taille de ses installations, et b) égale à son coût total.
d’autres entreprises entreront dans l’industrie. c) égale à son coût variable total.
e) augmentera la taille de ses installations, d) égale à son coût total moyen.
et d’autres entreprises sortiront de l’industrie. e) Aucune de ces réponses.

17 Toujours d’après la figure 11.1, la combinaison


prix et quantité d’équilibre à long terme est 23 Pour une entreprise parfaitement concurrentielle
a) de 6 $ et de 7 unités. et en équilibre à long terme, quel facteur, parmi
b) de 6 $ et de 17 unités. les suivants «est pas égal au prix ?
c) de 8 $ et de 9 unités. a) Le coût total moyen à court terme
d) de 8 $ et de 18 unités. b) Le coût variable moyen à court terme
e) de 9 $ et de 7 unités. c) Le coût marginal à court terme
d) Le coût total moyen à long terme
18 Une industrie parfaitement concurrentielle est
e) La recette moyenne
en équilibre à court terme lorsque le prix est
inférieur au coût total moyen. Quelle prédiction,
parmi les suivantes «est pas une prédiction des 24 Lorsque les profits économiques sont nuis
conséquences à long terme d’une telle situation? a) il n’y a pas de production à court terme.
a) Le prix augmentera. b) il n’y a pas de production à long terme.
b) La production de l’industrie augmentera. c) les entreprises sortent de l’industrie.
c) Les entreprises quitteront l’industrie. d) les recettes ne couvrent pas les coûts
d) La production de chaque entreprise restante implicites.
augmentera. e) Aucune de ces réponses.
e) Les profits économiques seront nuis.
©19 La courbe d’offre d’une entreprise individuelle 25 Quel énoncé, parmi les suivants, est faux si
d’une industrie parfaitement concurrentielle un nouvel équilibre à long terme est entraîné
est P = 1 + 2Qo- Si l’industrie comprend par une nouvelle technique adoptée dans
100 entreprises identiques, quelle est alors une industrie parfaitement concurrentielle ?
l’offre de l’industrie lorsque P= 7? a) Le prix sera inférieur.
a) 300 b) La production de l’industrie sera plus élevée.
b) 400 c) Les profits des entreprises seront plus élevés.
c) 600 d) Toutes les entreprises de l’industrie utiliseront
d) 800 la nouvelle technique.
e) Aucune de ces réponses. e) Le coût total moyen sera plus faible.
LA CONCURRENCE 145

Problèmes à court développement


TABLEAU 1 1.2

I Pourquoi une entreprise d’une industrie En dollars/jour En doilars/quantité


parfaitement concurrentielle choisira-t-elle
Quantité Coût Coût Coût
de ne pas imposer un prix supérieur ou
(en unités Coût variable total variable Coût
inférieur au prix du marché ?
par jour) total total moyen moyen marginal

Pourquoi les profits économiques tendent-ils 0 12 0 — —


vers zéro dans une industrie parfaitement
12
concurrentielle en équilibre à long terme ? 1 24 /Z- /z

3 Supposons qu’une industrie parfaitement 2 32 w fb


concurrentielle en équilibre à long terme
— lo
connaisse une baisse permanente de la demande 3 42 3o H
10, — IZ
de son produit. Il n’y a ni économies ni {OP
déséconomies externes. Comment l’industrie 4 54 4Z --/*/■
s’ajustera-t-elle à un nouvel équilibre à long Ib^-
13? i
terme? Comment évolueront le prix, la quantité,
les profits et le nombre d’entreprises durant le
5 68 ït> -J6
processus d’ajustement? 6 84 7Z
IL. iz
4 Supposons que dans une industrie parfaitement b) Remplissez le tableau 11.3 en calculant le
concurrentielle, sans avantages ni coûts externes, profit (par jour) de l’entreprise pour chaque
la production soit limitée à une quantité niveau de production lorsque le prix de
inférieure à la quantité d’équilibre. Expliquez production est de 9 $, de 11 $ et de 15 $.
pourquoi ce niveau de production ne permet pas
une allocation optimale des ressources. TABLEAU I 1.3

5 Le tableau 11.2 indique la structure du Quantité


q coût total d’une des nombreuses entreprises (unités Profit Profit Profit
identiques d’une industrie parfaitement par jour) P= 9$ P= 1 1 $ P = 15 $

~
concurrentielle.
Remplissez le tableau en calculant le coût
0
(2P -J Z -/z
| /'•/ ' variable total, le coût total moyen, le coût | ~15 "Il -°i
variable moyen et le coût marginal pour —Z
-ia
chaque niveau de production. (N’oubliez pas, 2 -74
3
comme vous l’avez fait pour les problèmes
à court développement du chapitre 10,
d’inscrire le coût marginal entre les lignes.)
3 -t*? Q) /
—lù
4

5
-Z?
-il m
6 -/?> ê
c) Considérez la décision de production que
prend cette entreprise pour maximiser ses
profits à divers prix. Quelle sera la quantité
produite si le prix de production est de 9 $ ?
de 11 $ ? de 15 $ ? Expliquez chacune de
vos réponses.
146 CHAPITRE II

6 Une entreprise maximisera ses profits si elle (N’oubliezpas d’indiquer les valeurs
produit chaque unité au niveau de production de coût marginal de votre tableau entre
auquel la recette marginale est supérieure au les unités correspondantes de production.)
coût marginal. C’est ce que l’on appelle c) Considérez (séparément) les prix de marché
l’approche marginale à la maximisation des possibles suivants de l’entreprise: P — 3,20 $,
profits. En utilisant cette approche marginale, P - 2 $, P= 1,65 $, P = 1,40 $. En supposant
déterminez le niveau de production qui permet que l’entreprise souhaite maximiser ses profits,
de maximiser les profits dans le cas de l’entreprise ■ pour chacun des prix indiqués précédemment,
dont il est question au problème 5, lorsque le répondez aux questions suivantes : Combien
prix de production est de 15 $. Comparez votre de tasses environ l’entreprise devra-t-elle
réponse à celle que vous avez donnée à la produire par jour? Comment le savez-vous?
question 5c). L’entreprise réalise-t-elle un profit à ce prix?
Si la réponse est oui, à combien ce profit
7 Cette question se rapporte à une fabrique fictive, s’élève-t-il approximativement?
qui produit des tasses de céramique quelle
vend dans un marché purement concurrentiel. 8 Supposez que l’industrie des tasses de céramique
À raison d’une taille d’installations donnée, comprenne 60 entreprises, chacune étant
l’entreprise peut produire les quantités de tasses identique à la fabrique décrite au problème 7.
en céramique indiquées au tableau 11.4, Le tableau 11.5 représente certains points
en variant la quantité utilisée d’un seul facteur du barème de demande de tasses de céramique
de production variable, le travail. de cette industrie.

TABLEAU 1 1.4 TABLEAU 1 1.5

Nombre de tasses Heures de travail Prix Quantité


(par jour) (par jour) (en dollars) demandée

20 6,50 1,00 15 900


40 1 1,00 1,60 14 400
60 14,50 2,20 12 900
80 17,50 2,80 1 1 400
100 20,50 3,40 9 900
120 23,75 4,00 8 400
140 27,50 4,60 6 900
160 32,00 5,20 5 400
180 37,50 5,80 3 900
200 44,50 6,40 2 400
220 53,50 7,00 900
240 65,00
260 79,50 a) Sur un nouveau graphique, tracez la courbe
280 97,50 d’offre à court terme de l’industrie. Tracez
la courbe de demande de l’industrie sur le
Supposons que l’entreprise puisse engager même graphique.
tous les travailleurs quelle désire au salaire b) Quel est le prix d’équilibre à court terme
horaire courant de 8 $. Les coûts fixes des tasses de céramique?
totaux de l’entreprise s’élèvent à 64 $ c) L’industrie des tasses de céramiques est-elle
par jour. en équilibre à long terme ? Expliquez votre
a) Construisez un tableau indiquant la réponse.
production, le coût variable total (CVT),
le coût total {CT), le coût variable moyen 9 Une industrie parfaitement concurrentielle
(CVM), le coût total moyen (CTM) comprend 100 entreprises identiques à court
et le coût marginal (Cm). (N’oubliez pas terme, chacune d’elles présentant les courbes
d’inscrire le coût marginal entre les lignes de coût à court terme indiquées au tableau 11.6
de production.)
b) Sur un graphique, les tasses (par jour) étant
portées en abscisse, tracez les trois courbes
de coût «par unité», CVM, CTM et Cm.
LA CONCURRENCE 147

a) Quelle est la quantité de production


TABLEAU 1 1.6
correspondant au seuil de rentabilité
de l’entreprise ? au seuil de fermeture ?
Coût Coût
Expliquez vos réponses.
total variable Coût
b) Quel est le prix d’équilibre à court
Production moyen moyen marginal
terme de ce marché ? Expliquez comment
(en unités) (en dollars) (en dollars) (en dollars)
vous obtenez vos réponses.
1 1 20,5 13,1 c) Quel est le montant du profit ou de la perte
.12 de chaque entreprise au prix d’équilibre
12 19,8 13,0 à court terme ? À sa taille actuelle, cette
.14 industrie est-elle en équilibre à long terme ?
13 19,3 13,1 Pourquoi l’est-elle ? Pourquoi ne l’est-elle pas ?
.16 d) Combien d’entreprises cette industrie
14 19,1 13,3 comprendra-t-elle à long terme exactement ?
.18 Expliquez votre réponse. Quel profit
15 19,0 13,6 économique réalisera chaque entreprise
.20 à long terme ?
16 19,1 14,0
.22 I0 Prenons l’exemple d’une industrie parfaitement
17 19,2 14,5 concurrentielle en équilibre à long terme. Toutes
.24 les entreprises de cette industrie sont identiques.
18 19,5 15,0 a) Tracez un graphique en deux parties illustrant
.26 l’équilibre à long terme pour cette industrie
19 19,8 15,6 - partie a) à gauche - et pour l’entreprise
.28 type — partie b) à droite. Le graphique
20 20,3 16,2 de l’entreprise doit comprendre les courbes
.30 Cm, CTM, Rm et CMLT. Supposez que
21 20,7 16,9 CMLT soit en forme de U, comme
la courbe
à la figure 11.9 de la page 244 du manuel.
Cette courbe de coût total moyen à court Nommez le prix d’équilibre P0, la quantité
terme est tangente à la courbe de coût moyen d’équilibre de l’industrie Q, et la production
à long terme au point minimum de la courbe de l’entreprise q0.
de coût moyen à long terme, appelé point m b) Supposez maintenant que l’industrie
à la figure 11.9 de la page 244 du manuel. connaisse une baisse de la demande.
Le barème de demande de l’industrie est le En utilisant vos graphiques de la partie a,
même à court terme et à long terme. Le tableau montrez comment évoluent le prix
11.7 représente certains points du barème du marché, la production des entreprises,
de demande. leurs profits et la quantité d’équilibre
de l’industrie à court terme (supposez

TABLEAU 1 1.7 que l’on n’ait pas atteint le seuil de fermeture).


Montrez ensuite comment évoluent le
Prix Quantité prix du marché, la production des entreprises,
(en dollars) demandée leurs profits et la quantité d’équilibre de
l’industrie à long terme (supposez qu’il
1 1 3 200 n’y ait ni économies ni déséconomies
13 3 000 externes). Comment a évolué le nombre
15 2 800 d’entreprises ?
17 2 600
19 2 400
21 2 200
23 2 000
25 1 800
27 1 600
29 1 400
31 1 200
148 CHAPITRE II

<Ô I I a Implique la fermeture de l’entreprise.


RÉPONSES D’autres réponses possibles avec pertes
(b, d) ou profits (c, e).
Vrai/Faux/Incertain 12 d Cm = P = RM > CTM. a, b, c ne sont pas
(Justifiez votre réponse.) compatibles avec la maximisation des profits,
e implique des pertes.
I F La courbe de demande de l’entreprise 13 d Parce que les coûts >1 à mesure que la Q
individuelle est horizontale. La courbe de de l’industrie T.
demande de l’industrie a une pente négative. 14 d a, e conditions de concurrence parfaite, b, c,
2 F Du profit total. incidence sur la forme de la courbe d’offre à
3 F Pertes à court terme mais pas à long terme. long terme mais pas sur l’allocation optimale.
4 V «Main invisible» —> efficience économique 15 c Il faut choisir Q, lorsque P = Cm.
et efficacité dans la consommation. 16 d Profits —» nouvelle entrée —> si P et incitation
& 5 I Vrai si P < CVM minimal. Faux si P > CVM à construire des installations plus grandes
minimal mais < CTM minimal. à moindre coût.
6 F Aussi longtemps que P > CVM minimal. 17 b Équilibre à long terme à l’intersection
7 V Autrement, l’entreprise serait éliminée par des courbes CMCT, CmCT et CMLT.
les entreprises ayant des coûts moins élevés. 18 b À mesure que les entreprises sortent de
8 F Pente négative. l’industrie, l’offre se déplace vers la gauche
9 V CTM se déplace T sans ACm. Les pertes —» -l Q industrie et T P —> T Q des
économiques -+ la sortie. entreprises restantes.
10 V Définition. @ 19 a Pour une entreprise: 7 = 1 + 2Q, ;
2Qj, = 6; Q, = 3. Pour 100 entreprises,
3 x 100 = 300.
Questions à choix multiple 20 e Si elle est incapable de payer les coûts
variables —> au-dessous du seuil de fermeture.
1 d Produits identiques. Si elle paie tous les coûts variables et fixes
2 d Les entreprises peuvent T Q sans AP, -+ seuil de rentabilité.
Rm provenant
de sorte que la de la Q 21 a Le CFM n’est pas pertinent. Maximisation
supplémentaire = P. des profits + maximisation des recettes.
® 3 c Tracez le graphique. L’entreprise devrait Les possibilités d et e pourraient être vraies
choisir une plus petite Q lorsque P = Cm. selon P.
Si le CVM = 10 $ avec la quantité courante, 22 a Définition équilibre à long terme —» profits
le CVM doit être < 10 $, de sorte que la économiques nuis.
nouvelle Q > CVM minimum. 23 b Équilibre à long terme à l’intersection
4 c Profits = RT (15 $ x 10) — CT des courbes CTM, Cm et CMLT.
(100 $ + 50 $) = 0. &24 e Production à court et à long termes, les
@ 5 d RT (10 $ x 10) - CT (100 $ + 50 $) recettes couvrent exactement tous les coûts
= - 50 $. Cesser les activités entraînerait (y compris les coûts implicites).
une perte plus grande, soit de 100 $ 25 c En situation d’équilibre à long terme,
(coûts fixes). les profits économiques sont toujours nuis.
® 6 c II prend une décision à long terme, a et b se
rapportent au court terme, d n’est pas juste, Problèmes à court développement
car les pertes indiquent qu’il faut déplacer
les ressources. I Si une entreprise d’une industrie parfaitement
7 b Puisque Rm = RM, RM > CTM. Multipliez concurrentielle impose un prix supérieur au prix
par Q —> RT > CT, donc des profits du marché, même si ce prix n’est que légèrement
économiques. supérieur, elle perdra toutes ses ventes. Par
8 b L’entreprise cesse toujours ses activités conséquent, elle n’imposera pas un prix
(Q = 0) jusqu’à ce que P atteigne le CVM supérieur au prix du marché. Puisqu’elle peut
minimum. vendre autant quelle le souhaite au prix du
9 a Définition, c est la courbe d’offre instantanée. marché, elle ne pourrait augmenter ses ventes
d est la courbe de demande de l’entreprise en baissant son prix. C’est pourquoi elle ne
individuelle. demandera pas un prix inférieur au prix
10 a Cm = ACT/AQ = 50 $/2. Le prix des tasses du marché, car elle risquerait de diminuer
n’est pas pertinent. sa recette et ses profits totaux.
LA CONCURRENCE 149

Dans une industrie parfaitement concurrentielle, une situation avantageuse parce qu’ils reçoivent
des profits économiques positifs susciteront un prix supérieur au minimum nécessaire pour
l’accès au marché de nouvelles entreprises, ce qui stimuler la production. Les consommateurs et
fera déplacer la courbe d’offre de l’industrie vers les producteurs seraient donc placés dans une
la droite, entraînant la chute du prix du marché situation avantageuse sans que personne n’en
et la diminution du profit des entreprises. On souffre. Par conséquent, avec la quantité Q0, ils
observera cette tendance aussi longtemps qu’il y retirent des gains des échanges (on peut avoir
aura des profits économiques positifs. De même, de plus grands surplus du consommateur et
des pertes économiques inciteront les entreprises du producteur) et, à ce niveau de production,
à quitter l’industrie, ce qui fera déplacer la courbe l’allocation des ressources n’est pas optimale.
d’offre de l’industrie vers la gauche, entraînant
la hausse du prix du marché et du profit des 5 a) Voici le tableau 11.2 Solution.
entreprises (les pertes diminuent). On observera
cette tendance aussi longtemps qu’il y aura des TABLEAU 1 1.2 SOLUTION
pertes. C’est pourquoi, le seul point d’équilibre
à long terme est atteint lorsque les profits En dollars/jour En doliars/quantité
économiques sont nuis.
Quantité Coût Coût Coût
(en unités Coût variable total variable Coût
La diminution de la demande du marché
par jour) total total moyen moyen marginal
provoque la chute du prix du marché. Puisque,
dans la situation d’équilibre à long terme initiale, 0 12 0 - -
chaque entreprise ne réalisait pas de profit . . . 12
économique, la baisse de prix signifie que les 1 24 12 24 12
profits vont baisser et que les entreprises . . . 8
subiront maintenant des pertes. Étant donné 2 32 20 16 10
que la diminution de la demande est permanente, . . . 10
ces pertes inciteront certaines entreprises à sortir 3 42 30 14 10
de l’industrie, ce qui entraînera le déplacement . . . 12
de la courbe d’offre de l’industrie vers la gauche, 4 54 42 13,5 10,5
provoquant une augmentation du prix du . . . 14
marché. Le prix plus élevé entraînera 5 68 56 13,6 1 1,2
l’augmentation des profits des entreprises . . . 16
restantes, et donc la diminution de leurs pertes. 6 84 72 14 12
Les entreprises continuent de sortir de 1 industrie
jusqu’à ce que les pertes soient éliminées. Les b) Voici le tableau 11.3 Solution. Les valeurs du
coûts n’ont pas été touchés par la baisse de profit correspondent à la différence entre la
la demande, et s’il n’y a pas d’économies recette totale et le coût total, la recette totale
ou de déséconomies externes, le prix continuera étant le prix multiplié par la quantité ; le coût
d’augmenter jusqu’à ce qu’il atteigne son niveau total nous est indiqué au tableau 11.2.
initial dans le nouvel équilibre à long terme. La
production de chaque entreprise sera aussi égalé TABLEAU 11.3 SOLUTION
à son niveau initial, mais la quantité d équilibre
au niveau de l’industrie sera moins grande Quantité
du fait qu’il y a moins d’entreprises. (unités Profit Profit Profit
11 P=
TJ

O
-w

par jour) P = $ 15 $
II

(Voir la figure 11.12 de la page 250 du manuel).


0 -12 -12 -12
Si la production est limitée à la quantité Q0,
1 -15 -13 -9
pour les consommateurs, la valeur d une unité
2 -14 -10 -2
de production supplémentaire est V0, alors que
3 -15 -9 3
les producteurs souhaiteraient offrir les biens
4 -18 -10 6
supplémentaires au prix C0 ou à n importe quel
5 -23 -13 7
prix supérieur à C0. Si l’on produit des biens
6 -30 -18 6
supplémentaires et qu’on les vend à n importe
quel prix entre V0 et C0, les consommateurs
seront placés dans une situation plus avantageuse
parce qu’ils valorisent davantage les biens que
le prix payé et les producteurs seront placés dans
150 CHAPITRE I I

c) Si le prix est de 9 $, le profit est maximisé


TABLEAU I 1.8
(en fait, la perte est réduite au minimum)
lorsque l’entreprise cesse ses activités et ne
En dollars En dollars
produit aucune unité. Si l’entreprise choisit
par jour par unité
de produire, sa perte sera au moins de 14 $, Production
montant qui est supérieur à celui de la perte (par jour) CVT CT CV/V1 CTM Cm
de coût fixe de 12 $. Si le prix est de 11 $,
0 0 64 - -
l’entreprise est toujours incapable de réaliser
2,60
un profit économique positif. La perte est
20 52 1 16 2,60 5,80
réduite au minimum (9 $) si l’entreprise
1,80
produit 3 unités. À ce prix, tous les coûts
40 88 152 2,20 3,80
variables et une partie des coûts fixes peuvent
1,40
être récupérés. Au prix de 15 $, l’entreprise
60 1 16 180 1,93 3
maximisera son profit (7 $) avec une
1,20
production de 5 unités par jour.
80 140 204 1,75 2,55
1,20
Selon l’approche marginale de la maximisation
100 164 228 1,64 2,28
des profits, l’entreprise devrait produire toutes
1,30
les unités qui donneront une recette marginale
120 190 254 1,58 2,12
supérieure au coût marginal. Pour une entreprise
1,50
parfaitement concurrentielle, la recette marginale
140 220 284 1,57 2.03
est égale au prix, donc l’entreprise devrait
1,80
produire toutes les unités qui lui permettent
160 256 320 1,60 2
d’obtenir un prix supérieur au coût marginal.
2,20
Si le prix de la production est de 15$, nous
180 300 364 1,67 2,02
pouvons voir au tableau 11.2 Solution que
2,80
l’entreprise devrait produire 5 unités. Comme
200 356 420 1,78 2,10
le coût marginal du déplacement de la 4e à la
3,60
5e unité (14 $) est inférieur au prix (15 $), c’est
220 428 492 1,95 2,24
la 5e unité qui devrait être produite. Toutefois,
4,60
le coût marginal du déplacement vers la 6e unité
240 520 584 2,17 2,43
(16 $) est supérieur au prix. Elle ne doit pas être
5,80
produite. Ici, la réponse est identique à celle de
260 636 700 2,45 2,69
la question 5 c).
7,20
280 780 844 2,79 3,01
a) Avec seulement un facteur de production
variable (travail), le CVT (pour n’importe
quel niveau de production) = (heures
de travail X taux salarial). Le tableau 11.8
donne la réponse juste.
LA CONCURRENCE 151

b) La figure 11.2 présente le graphique. Elle Remarques :


illustre également les réponses à la question c. 1. Le coût marginal est de 3,20 $ pour environ
200 unités.
FIGURE 11.2 2. Le profit par unité est P — CTM.
3. 160 unités représentent le seuil de rentabilité
de la production, lorsque Cm — CTM minimum
et que l’entreprise couvre tout juste ses coûts
d’opportunité.
4. Avec une production de 140 unités, l’entreprise
continue de produire à court terme, car elle ne
se limite pas à couvrir ses coûts variables. Si elle
ne produit aucune unité, sa perte sera plus grande,
soit de 64 $, ce qui correspond au montant de
ses coûts fixes.
5. Toute production positive fait augmenter
les pertes lorsque le prix est inférieur au prix
de fermeture, qui s’élève ici à 1,57 $. Lorsque
le prix est inférieur au CVM minimum, l’entreprise
perdra non seulement ses coûts fixes, mais
également tout montant supplémentaire sur
chaque unité quelle produit.

8 II existe 60 preneurs de prix identiques.


Pour chaque prix possible (au-dessus du CVM
minimum de 1,57 $ environ), chaque entreprise
fournira la quantité à laquelle P — Cm. Nous
pouvons calculer (au tableau 11.10) le barème
d’offre de l’industrie à partir de la courbe
c) Dans chaque cas, au niveau de production
Cm d’une entreprise individuelle.
qui permet de maximiser les profits, la recette
marginale (qui, pour un preneur de prix dans
TABLEAU 11.10
un marché parfaitement concurrentiel, est
la même que le prix du marché) est égale
Quantité Quantité
au coût marginal, si le prix est supérieur
Prix offerte par offerte par
au coût variable moyen.
(en dollars) 1 entreprise 60 entreprises
Le profit « par unité de production » est
la différence entre la recette moyenne (prix) et 1,57 134 8 040
le coût total moyen au niveau de production. 1,80 150 9 000
Le profit total est « le profit par unité » 2,20 170 10 200
X le nombre d’unités de production. Le profit 2,80 190 1 1 400
économique peut être nul ou négatif (perte) 3,60 210 12 600
et il peut correspondre au meilleur profit que 4,60 230 13 800
peut réaliser l’entreprise à court terme. Les 5,80 250 15 000
calculs du profit total pour chaque prix sont 7,20 270 16 200
présentés au tableau 11.9
a) Le graphique de la courbe d’offre de
l’industrie et de la courbe de demande de
TABLEAU 1 1.9
l’industrie est représenté à la figure 11.3.

Profit par Profit


Prix Quantité unité total
(en dollars) choisie (en dollars) (en dollars)

3,20 200 1,10 220


2 160 0 0
1,65 140 -0,38 -53,20
1,40 0 0 -64
152 CHAPITRE I I

b) Le prix d’équilibre à court terme est de 25 $.


FIGURE 11.3
Il s’agit du prix auquel la quantité offerte
de l’industrie est égale à la quantité demandée,
comme l’indique le tableau 11.11.
c) Au prix de 25 $, chaque entreprise produit
18 unités. Avec 18 unités, le CTM est de
19,50 $, de sorte que l’entreprise réalise un
profit économique. Le montant du profit
est de (25 $ — 19,50 $ par unité = ) 5,50 $
l’unité. Le profit économique total par
entreprise est de 5,50 $ l’unité X 18 unités
= 99 $. Cela signifie que de nouvelles
entreprises seront incitées à entrer dans
l’industrie. Nous en concluons que l’industrie
«estpas en équilibre à long terme.
d) Les entreprises continuent d’entrer
dans l’industrie jusqu’à ce que les profits
économiques soient éliminés par la
b) Le prix d’équilibre à court terme est de 2,80 $ concurrence et que toutes les entreprises
(lorsqu’au total 11 400 tasses par jour fonctionnent au point minimum de la
sont offertes et vendues ; ou lorsque courbe de CMLT (qui, dans ce problème,
190 tasses par jour sont offertes par chaque est également le point minimum de la courbe
entreprise au Cm de 2,80 $). de coût total moyen à court terme).
c) Au prix de 2,80 $ et avec une production de Le CTM minimum de chaque entreprise
190 unités, pour chaque entreprise, P> CTM est atteint avec une production de 15 unités
lorsque le CTM = 2,06 $. Donc, toutes les lorsque le CTM = Cm = 19 $, donc le prix
entreprises existantes réalisent des profits à long terme doit être de 19 $. À ce prix,
économiques après avoir couvert tous les coûts les consommateurs demandent 2 400 unités
d’opportunité comprenant un profit normal. (d’après le tableau 11.11). Il s’ensuit que
(Voir le Rappel 3). De nouvelles entreprises lorsque l’industrie est en équilibre à long
seront incitées à entrer dans l’industrie parce terme, il doit y avoir 2 400 unités/15 unités
que celle-ci permet de réaliser des profits par entreprise =160 entreprises dans
plus élevés que les profits normaux. Bien l’industrie. Chacune d’elle produit 15 unités
que l’industrie soit en équilibre à court et réalise un profit économique nul.
terme, elle ne l’est pas à long terme parce
que le nombre d’entreprises ne s’est pas TABLEAU 1 l.l 1
«stabilisé». De nouvelles entreprises entreront
dans l’industrie, la courbe d’offre de l’industrie Quantité Quantité
se déplacera vers la droite et le prix baissera P = Cm offerte par offerte par Quantité
jusqu’à ce qu’aucune entreprise ne réalise des (en dollars) 1 entreprise 100 entreprises demandée
profits économiques.
13 12 1 200 3 000
15 13 1 300 2 800
@9 a) Le seuil de rentabilité est atteint avec
17 14 1 400 2 600
15 unités de production. À ce niveau, le
19 15 1 500 2 400
CTM est au minimum (19 $) et il est égal
21 16 1 600 2 200
au Cm. Comme le Cm du déplacement de
23 17 1 700 2 000
la 14e à la 15e unité est de 18 $, et que le Cm
25 18 1 800 1 800
du déplacement de la 15e à la 16e unité est de
27 19 1 900 1 600
20 $, nous pouvons interpoler exactement
29 20 2 000 1 400
la valeur du Cm avec 15 unités à mi-chemin,
entre 18 $ et 20 $, ou à 19 $. Le seuil de
fermeture est atteint avec la production de
12 unités. À ce niveau, le CVM est au 10 a) La figure 11.4 illustre un équilibre à long
minimum (13 $) et il est égal au Cm. terme dans une industrie parfaitement
■ La valeur interpolée du Cm avec exactement concurrentielle.
12 unités de production se trouve à mi-chemin La partie (a) illustre l’équilibre à
entre 12 $ et 14 $, elle est donc de 13 $. l’intersection de la demande de l’industrie
LA CONCURRENCE 153

(D0) et de l'offre de l’industrie (O0) : point a. production (se déplaçant en montant le long
La quantité d’équilibre de l’industrie est Qo> de la courbe Cm du point b' au point a),
et le prix de marché d’équilibre, P0. réduisant ainsi leurs pertes. Lorsque
La partie (b) illustre la situation d’une suffisamment d’entreprises auront quitté
entreprise unique en équilibre à long terme. l’industrie, le prix se sera rétabli (P0) et les
L’entreprise se situe au point a , le point entreprises seront revenues au point a'de
minimum de la courbe de coût total moyen à la partie (b). À ce point, chaque entreprise
court terme (CTM) et de la courbe de coût réalise de nouveau un profit économique nul
moyen à long terme (CMLT). L’entreprise et la production est de nouveau q0. Toutefois,
produit la quantité q0 et réalise un profit comme il y a moins d’entreprises, la quantité
économique nul. d’équilibre de l’industrie est moins grande.
b) La figure 11.4 illustre également le nouvel
équilibre à court terme. La baisse de la
FIGURE 11.4
demande fait déplacer la courbe de demande
du marché vers la gauche, de Z)0 à Z),. Le a) Industrie
nouvel équilibre du marché se situe au point
b. Le prix a baissé, allant de P0 à P, et la
quantité d’équilibre de l’industrie a diminué,
passant de Qq à La baisse de prix incite
les entreprises à réduire leur production,
comme l’illustre le déplacement du point a
au point b' sur la courbe Cm de la partie (b).
Comme Px est inférieur au CTM minimum,
les entreprises subissent des pertes en
situation de nouvel équilibre à court terme.
La figure 11.4 illustre également
le nouvel équilibre à long terme. Avec des
pertes à court terme, les entreprises quitteront
l’industrie à long terme, ce qui entraîne
le déplacement vers la gauche de la courbe b) Entreprise
d’offre de l’industrie, faisant monter le prix
et, donc, réduire les pertes. Les entreprises
continuent de quitter l’industrie jusqu’à ce
que la courbe d’offre de l’industrie se soit
suffisamment déplacée pour éliminer les
pertes, soit de O0 à Ox. Cela donne un nouvel
équilibre de l’industrie à long terme au point
c. À ce point, le prix se rétablit à son niveau
initial P0, mais la quantité de l’industrie
a baissé, passant à Q-
À mesure que les entreprises sortent
de l’industrie, le prix du marché augmente,
les entreprises restantes augmentent leur
Chapitre

12
Le monopole

• lorsque la RT est maximale, la Rm est égale


CONCEPTS CLÉS à zéro, T) = 1 ;
• lorsque la RT 4-, la Rm est négative, T| < 1.

Les causes du monopole ♦ Les techniques et coûts d’un monopole sont


semblables à ceux d’une entreprise en concurrence
Monopole - industrie qui ne compte qu’un parfaite.
seul fournisseur de produits qui n’ont pas de
♦ Le monopole qui maximise ses profits choisit le
proches substituts. La création des monopoles est
volume de production auquel Rm = Cm et fixe le
entraînée par
prix maximal que les consommateurs sont prêts à
♦ les barrières à l’entrée - obstacles qui protègent payer (sur la courbe de demande).
une entreprise contre l’entrée de concurrents
♦ La production d’un monopole ne se situe jamais
potentiels.
dans la partie où sa courbe de demande est
• Les barrières légales à l’entrée — entraînent inélastique. Le monopole n’a pas de courbe d’offre.
la création d’un monopole légal, l’entrée et
♦ Le monopole peut réaliser des profits économiques
la concurrence étant limitées par une concession,
même à long terme, car les barrières le protègent
un permis, un brevet ou des droits d’auteur.
contre l’entrée sur le marché de nouvelles
• Les barrières naturelles à l’entrée — entraînent
entreprises.
la création d’un monopole naturel,
lorsqu’une entreprise qui réalise des économies
d’échelle peut approvisionner l’ensemble La discrimination par les prix
du marché à un CTM inférieur à celui
que donnerait la coexistence de plusieurs La discrimination par les prix - consiste à fixer un
entreprises. prix moins élevé à certains clients pour un produit
identique-, ou bien à faire payer à un client un prix
moins élevé sur un gros achat que sur un petit achat,
Le monopole non discriminant
même si les coûts de production sont identiques.

Un monopole non discriminant est un monopole La discrimination par les prix peut faire augmenter
qui écoule toute sa production au même prix. les profits en permettant à l’entreprise de s’approprier
une plus grande partie du surplus du consommateur.
♦ La courbe de demande d’un monopole se confond
avec la courbe de demande de l’industrie. ♦ Discrimination par les prix parfaite - prix différent
pour chaque unité vendue ; l’entreprise obtient
♦ La recette marginale (Rm) < prix (P). Pour vendre
le prix maximal que chaque consommateur est
des unités supplémentaires, il faut réduire le P
prêt à payer.
sur l’ensemble de la production.
• La courbe de Rm se confond avec la courbe
♦ En descendant le long de la courbe de demande
de demande. Même production que dans
du monopole
un marché parfaitement concurrentiel.
• lorsque la recette totale (RT) T, la Rm • Cependant, la totalité du surplus
est positive, T| > 1 ; du consommateur tient lieu de profit.
LE MONOPOLE 155

♦ Pour qu’il y ait discrimination par les prix il faut Les avantages du monopole

• que les biens ne puissent être revendus ; ♦ Économies d’échelle (si CMLT lorsque l’échelle T Q).
• que le prix imposé soit moins élevé pour
♦ Économies de gamme (si CTM lorsque T gamme
le groupe ayant la plus grande élasticités;
de biens produits)
• que le prix imposé soit plus élevé pour
le groupe ayant la moins grande élasticité. ♦ T Innovation et progrès technique (dans certains cas).

♦ Le volume de production du monopole qui


pratique la discrimination par les prix est plus
élevé que celui du monopole non discriminant.
RAPPELS

Comparaison entre le monopole I L’extrême opposé de la concurrence parfaite


et la concurrence est le monopole. Alors qu’en situation de
concurrence parfaite de nombreuses entreprises
Le prix et la production peuvent décider de la quantité produite, mais

♦ Dans un monopole non discriminant, le P est pas du prix auquel elles la vendront, un
monopole est une entreprise unique qui a
> au P de l’entreprise concurrentielle.
la capacité de fixer et la quantité et le prix.
♦ Dans un monopole non discriminant, la Q est Ces différences créent des différences relatives
< à la quantité de l’entreprise concurrentielle. à la recette du monopole. On suppose que les
courbes de coût de ces deux types de marché
• Plus la discrimination par les prix que pratique
sont les mêmes.
le monopole est parfaite, plus sa Q se rapproche
Puisqu’il n’y a qu’une seule entreprise, la
de la Q de l’entreprise concurrentielle.
courbe de demande de l’industrie se confond
L’allocation efficiente des ressources avec la courbe de demande de l’entreprise.
Lorsque sa courbe de demande a une pente
♦ L’allocation des ressources dans un monopole
négative, le monopole non discriminant
non discriminant est moins efficiente que celle
qui souhaite vendre une unité supplémentaire
de l’entreprise concurrentielle. Le monopole ne
de production doit baisser son prix. Cette baisse
permet pas de retirer des gains de l’échange :
a deux effets sur la recette. D’abord, la vente
il limite la production, s’approprie une partie
d’une unité supplémentaire fera augmenter la
du surplus du consommateur, mais crée
recette d’un montant égal au prix. Toutefois,
une perte sèche — la perte totale du surplus
comme l’entreprise doit également baisser le prix
du consommateur et du surplus du producteur
des unités précédentes, la recette provenant
(recette — coût d’opportunité de la production)
de ces unités va diminuer. La variation nette
au-dessous des niveaux efficients.
de la recette, la recette marginale, sera inférieure
♦ Le monopole qui pratique une discrimination au prix et la courbe de recette marginale
par les prix parfaite s’approprie la totalité du se situera sous la courbe de demande.
surplus du consommateur sous forme de profit, En combinant cette nouvelle recette aux
mais réalise une allocation efficiente des ressources courbes de coûts du chapitre 10 qui nous sont
comme l’entreprise concurrentielle — pas de perte familières, nous obtenons l’importante figure
sèche. 12.4(b) du manuel, qui est reproduite à la
figure 12.1. Remarquez les points suivants
Les coûts de monopole
de ce graphique.
♦ Perte sèche.

♦ Coût des ressources utilisées pour la recherche de


rentes — la recherche de possibilités de monopoles
rentables.

♦ En l’absence de barrières à l’entrée, le coût des


ressources utilisées pour la recherche de rentes
= la valeur du profit de monopole (lorsqu il y a
recherche de rentes).

♦ Le coût social du monopole = perte seche + profit


économique du monopole (lorsqu il y a recherche
de rentes).
156 CHAPITRE 12

2 On peut facilement tracer la courbe de recette


FIGURE 12.1
marginale correspondant à toute courbe
de demande linéaire de la manière suivante:
la coordonnée à l’origine du prix (où Q = 0)
est la même que pour la courbe de demande,
et l’abscisse à l’origine de la quantité (où P— 0)
est exactement la moitié de la quantité maximale
demandée. La courbe de recette marginale est
donc une ligne droite de pente négative deux
fois plus abrupte que la pente de la courbe de
demande.

3 Pour un monopole, la discrimination par les


prix ne peut être rentable que si les élasticités des
courbes de demande du produit sont différentes
GHjantï^^ pour les différents groupes de consommateurs.
Dans ce cas, le monopole discriminant traite
Pour qu’un monopole non discriminant les groupes comme des marchés différents. Un
puisse maximiser ses profits, il faut qu’il trouve monopole discriminant qui cherche à maximiser
le volume de production auquel Rm = Cm. son profit a pour règle de trouver le volume
Cette règle s’applique également à l’entreprise de production auquel la Rm de chaque marché
en situation de concurrence parfaite. = Cm. Alors, dans chaque marché, il fixe le prix
Pour une entreprise en situation de maximal que le groupe de consommateurs est
concurrence parfaite, la Rm est aussi égale au disposé à payer pour ce volume de production
prix, de sorte que l’intersection de Rm et de Cm (sur la courbe de demande). Lorsque l’élasticité
indique le volume de production et le prix qui de la demande des groupes de consommateurs
permettent de maximiser les profits. Ce n’est pas est différente, selon cette règle, les prix seront
le cas pour le monopole. La Rm n’est pas égale différents dans les deux marchés.
au prix, et lorsqu’on a trouvé le volume de Toutefois, si ces deux marchés présentent
production qui permet de maximiser le profit, une élasticité de la demande égale, les recettes
il reste encore au monopoleur à fixer le prix. marginales seront alors égales et les prix
Pour trouver le prix qui permet de maximiser correspondant aux recettes marginales seront
le profit, tracez une ligne verticale imaginaire aussi égaux. Il est inutile de demander des prix
vers le haut jusqu’à la courbe de demande à partir différents, et la règle de maximisation des profits
de l’intersection de Rm et de Cm. Tracez ensuite du monopole discriminant redevient identique
une ligne horizontale imaginaire jusqu’à l’axe à la règle s’appliquant au monopole non
des prix pour lire le prix. discriminant.
Si vous comprenez ce que représentent les
distances horizontale et verticale du rectangle 4 La prédiction selon laquelle les profits
qui désigne le profit économique, vous risquerez économiques à court terme d’une entreprise
moins de vous tromper lorsque vous tracerez en situation de concurrence parfaite sont
ce rectangle. La distance verticale se situe entre éliminés par la concurrence à long terme est
la courbe de demande (ou recette moyenne) fondée sur l’absence de barrières à l’entrée
et la courbe de coût total moyen. Cette distance d’une industrie parfaitement concurrentielle.
mesure la différence entre la recette moyenne Inversement, la prédiction voulant que les
et le coût total moyen, qui est égale au profit profits du monopole peuvent persister à long
économique moyen ou au profit économique terme est fondée sur la présence de barrières
par unité. La distance horizontale ne représente à l’entrée du monopole. Cependant, lorsqu’on
que le nombre d’unités produites. Par conséquent, prend en compte l’activité de recherche de
le rectangle (distance verticale X distance rentes, il se pourrait qu’il n’y ait pas de profits
horizontale) = profit économique par unité économiques à long terme, même pour un
X nombre d’unités = profit économique total. monopole. La recherche de rentes comprend
Ne faites pas l’erreur de tracer la distance toutes les activités qui visent à obtenir des droits
verticale en descendant vers l’intersection de monopole existants ou à créer de nouveaux
de Cm et de Rm. Cette intersection n’a aucune droits de monopole. La concurrence entre les
signification économique dans le calcul du profit chercheurs de rentes fait monter le coût de la
économique total. recherche de rentes jusqu’à ce qu’il soit égal à la
LE MONOPOLE 157

valeur des profits du monopole potentiel, 8 En raison de l’activité de recherche de rentes,


de sorte que le monopoleur devenu chercheur le coût social du monopole est moins élevé que
de rentes réalise peu de profit économique la perte sèche.
ou n’en réalise pas du tout.

9 Lorsqu’on prend en compte la recherche


AUTOÉVALUATION de rentes, les profits économiques découlant
du monopole sont garantis à long terme.

Vrai/Faux/Incertain
(Justifiez votre réponse.)
10 Une industrie monopolistique qui réalise
I À des niveaux de production supérieurs à ceux d’importantes économies d’échelle et
où la recette totale diminue, la recette marginale de gamme peut produire davantage qu’une
est positive. industrie parfaitement concurrentielle et fixer
un prix inférieur à celui de cette dernière.

2 La courbe d’offre d’un monopole se confond


avec sa courbe de coût marginal. Questions à choix multiple

1 Quelle barrière suivante est une barrière


naturelle à l’entrée de nouvelles entreprises
3 Un monopole fait des profits économiques. dans une industrie?
a) Le permis d’exercer une profession
b) Les économies d’échelle
c) L’émission d’un brevet
4 Pour un monopoleur qui pratique une d) Une concession
discrimination par les prix parfaite, la courbe e) Toutes ces réponses.
de demande est également la courbe de recette
marginale
2 Pour porter les ventes de 7 unités à 8 unités,
un monopoleur non discriminant doit baisser
le prix de 7 $ l’unité à 6 $ l’unité. Quelle est
la recette marginale dans cet intervalle ?

5 La discrimination par les prix ne donne de bons a) 48 $

résultats que dans le cas des biens que l’on peut b) 6$


revendre facilement.
c) 1 $
0-i$
F ej Aucune de ces réponses.

3 Si la recette marginale est négative à


6 Lorsqu’on passe d’une situation de concurrence un certain volume de production, alors
parfaite à celle de monopole non discriminant, a) le prix doit être négatif.
une partie de la perte sèche est causée par une b) le monopoleur devrait augmenter sa
réduction du surplus du producteur. production.
c) l’élasticité de la demande est inférieure
à 1 à ce volume de production.
d) la demande doit être élastique à ce volume
7 Aucune perte sèche n’est attribuable de production.
au monopole, car le monopole gagne e) le monopoleur devrait cesser ses activités.
tout ce que perd le consommateur.
4 Les inconvénients du monopole sont
a) les économies de gamme.
b) les économies d’échelle.
c) la diffusion des progrès techniques.
d) la recherche de rentes.
e) Toutes ces réponses.
58 CHAPITRE 12

5 Lorsqu’on prend en compte l’activité de 9 Le volume de production d’un monopole


recherche de rentes, le coût social du monopole qui pratique la discrimination par les prix
est égal (non parfaite) sera
a) à la perte sèche attribuable au monopole. a) inférieur à celui d’un monopole non
b) au profit de monopole. discriminant.
c) à la perte sèche plus le profit de monopole. b) supérieur à celui d’un monopole non
d) à la perte sèche moins le profit de monopole. discriminant, mais inférieur à celui d’une
e) au surplus du consommateur perdu plus industrie parfaitement concurrentielle.
le surplus du producteur perdu. c) égal à celui d’une industrie parfaitement
concurrentielle.
6 Si un monopole non discriminant cesse ses d) supérieur à celui d’une industrie parfaitement
activités, au volume de production correspondant concurrentielle.
au point auquel la courbe de coût marginal e) Aucune de ces réponses.
croise la courbe de recette marginale, il doit être
vrai que I0 Etudiez la courbe de demande de l’industrie
a) la recette marginale est inférieure au coût de la figure 12.3. Si l’industrie est en situation
variable moyen. de concurrence parfaite, quelle partie du
b) le coût marginal est inférieur au coût variable diagramme indique le surplus du consommateur ?
moyen. a) aek
c) le prix est inférieur au coût variable moyen. b) dhk
d) la recette totale est inférieure au coût total. c) dik
e) Toutes ces réponses. d) dih
e) Aucune de ces réponses.
7 Pour le monopole non discriminant présenté
à la figure 12.2, lorsque le profit est maximisé, FIGURE 12.3
la quantité est
a) 3 et le prix, 3 $.
b) 3 et le prix, 6 $.
c) 4 et le prix, 4 $.
d) 4 et le prix, 5 $.
e) 5 et le prix, 4 $.

FIGURE 12.2

I I Étudiez la courbe de demande de l’industrie


de la figure 12.3. Si l’industrie est en situation
de concurrence parfaite, quelle partie du
diagramme indique le surplus du producteur ?
a) aek
b) dhk
c) dik
d) dih
e) Aucune de ces réponses.

I2 Étudiez la courbe de demande de l’industrie


8 Si le monopole non discriminant présenté à de la figure 12.3. Quelle partie du diagramme
la figure 12.2 maximise son profit, à combien indique la perte sèche attribuable à un monopole
s’élève le profit économique total ? non discriminant?
a) 3$ a) eacf
b) 4$ b) acd
c) 6 $ c) abd
d) 9$ d) bcd
e) Aucune de ces réponses. e) Aucune de ces réponses.
LE MONOPOLE 159

13 Quelle partie de la figure 12.3 indique la perte 0/qu’ il se situe dans la partie inélastique
sèche causée par un monopole qui pratique la de sa courbe de demande,
discrimination par les prix parfaite ? e) qu’il se situe dans la partie inélastique
a) eacf de sa courbe de recette marginale.
b) acd
c) abd 18 Un monopole qui pratique la discrimination
d) bcd par les prix parfaite
e) Aucune de ces réponses. a) a une courbe de demande parfaitement
I4 Quatre monopoleurs discutent dans un inélastique.
restaurant. Quelle déclaration faite par l’un b) maximisera sa recette.
d eux représente une bonne stratégie de c) est assuré de réaliser un profit.
maximisation des profits ? d) choisira un volume de production tel que
a) « Dans mon entreprise, nous n’augmentons la courbe de coût marginal croisera la courbe
pas la production avant de savoir si cette de demande.
augmentation fera augmenter la recette totale. » e) réalisera une allocation non efficiente
b) «Je crois que la minimisation des coûts des ressources.
est la clé de la maximisation des profits. »
c) « Nous nous efforçons d’exploiter au I9 Un monopole devra cesser ses activités
maximum notre équipement en produisant à court terme
à plein régime. » a) s’il subit une perte économique.
d) «Je ne surveille pas les profits totaux à la b) si la Rm est inférieure au CVM.
loupe, mais je n’approuve aucune transaction c) si le prix est inférieur au CVM.
qui ne me permet pas d’augmenter davantage d) si le volume de production qui permet
mes recettes que mes coûts. » de maximiser les profits se situe dans
e) Aucune de ces réponses. la partie élastique de la courbe de demande.
e) si la Rm est inférieure à la RM.
15 Un monopole réalise des économies de gamme si
a) le coût total moyen diminue à mesure
20 Le tableau 12.1 présente les coûts marginaux
que l’échelle de l’entreprise augmente.
de l’entreprise XYZ. Si XYZ vend 3 unités au
b) le coût moyen diminue à mesure que
prix de 6 $ chacune, à combien s’élève le surplus
la production augmente.
du producteur?
c) le profit total diminue à mesure que
a) 2$
la production augmente.
b) 6$
d) le coût total moyen diminue à mesure
c) 7$
que le nombre de biens différents produits
d) 9$
augmente.
e) 12$
e) le profit total diminue à mesure que
le nombre de différents biens produits
TABLEAU 12.1
augmente.
16 Si le volume de production d’un monopole qui
Quantité Coût marginal
maximise ses profits est tel que le coût marginal
est supérieur à la recette marginale, alors le 1 2
monopole 2 3
a) devrait augmenter le prix et diminuer 3 4
la production. 4 5
b) devrait baisser le prix et augmenter
la production.
c) devrait baisser le prix et diminuer 21 Un monopoleur non discriminant maximisera
la production. ses profits si son volume de production est
d) subit des pertes. tel que
e) maximise ses profits. a) le prix est égal au coût marginal.
I7 Un monopole qui maximise ses profits ne choisit b) le prix est égal à la recette marginale.
/cJJla recette marginale est égale au coût
jamais un volume de production tel
mar gin al.
a) qu’il subit des pertes.
b) que la recette marginale est inférieure au prix. d) la recette moyenne est égale au coût marginal.

c) que le coût moyen est supérieur au coût e) la recette moyenne est égale à la recette
marginale.
marginal.
160 CHAPITRE 12

22 De nombreux clubs vidéos demandent un 3 Le prix de la dernière unité vendue et la quantité


prix moins élevé pour la location de vidéos vendue sont exactement les mêmes pour une
le mercredi soir que les fins de semaine. Cette industrie en situation de concurrence parfaite
discrimination par les prix n’est rentable que si et pour un monopole exerçant une discrimination
l’élasticité de la demande de vidéos le mercredi est parfaite par les prix. Les consommateurs sont-ils
a) plus grande que l’élasticité de la demande donc indifférents face à l’un ou l’autre ?
de vidéos en fin de semaine. Expliquez.
b) moins grande que l’élasticité de la demande
4 Dans quelles conditions un monopole serait-il
de vidéos en fin de semaine.
plus efficient qu’un grand nombre d’entreprises
c) positive et l’élasticité de la demande de vidéos
concurrentielles ? Illustrez, à l’aide d’un graphique,
en fin de semaine est négative.
le type de situation où le monopole produit
d) négative et l’élasticité de la demande
davantage et fixe un prix moins élevé qu’une
de vidéos en fin de semaine est positive.
industrie composée de nombreuses entreprises
e) égale à un.
concurrentielles.

23 L’activité menée pour créer un monopole 5 Un monopole non discriminant est le seul
a) est la recherche de rentes. vendeur de crochets du marché canadien. Le
b) est illégale au Canada. coût fixe total de ce monopole s’élève à 112 $
c) est la discrimination par les prix. par jour. Ses coûts variables totaux et ses coûts
d) est un monopole légal. totaux (les deux en dollars par jour) sont
e) ne coûte rien. indiqués au tableau 12.2.
a) Remplissez le tableau en calculant le coût
24 Quel énoncé suivant est vrai pour un monopoleur marginal, le coût variable moyen et le coût
non discriminant mais pas pour une entreprise total moyen. (N’oubliez pas d’inscrire le coût
en concurrence parfaite ? marginal entre les lignes de production.)
a) L’entreprise maximise son profit de telle sorte
que le coût marginal est égal à la recette
marginale.
TABLEAU 12.2
b) L’entreprise est un preneur de prix.
Coût Coût Coût
c) L’entreprise peut vendre n’importe quelle
variable Coût Coût variable total
quantité de produits à n’importe quel prix total total marginal moyen moyen
«. quelle choisira. Quantité (CVT) (CT) (Cm) (CWV1) (CTM)
/a)]Le coût marginal de l’entreprise est inférieur
(en dollars) (en dollars)
'-''à la recette moyenne, par jour) par crochet)
e) Aucune de ces réponses.
9 135 247
25 En présence d’une discrimination par les prix
parfaite, lequel des énoncés suivants est faux? 10 144 256
a) Les acheteurs ne peuvent revendre le produit.
1 1 155 267
b) L’entreprise peut faire des différences entre
les consommateurs.
12 168 280
c) L’entreprise fixe les prix.
d) L’entreprise s’approprie le surplus
13 183 295
du consommateur.
e) L’allocation des ressources est bien
14 200 312
moins efficiente que pour un monopole
non discriminant. 15 219 331

Problèmes à court développement 16 240 352

17 263 375
1 Pourquoi, dans un monopole non discriminant,
la recette marginale est-elle inférieure au prix ?
18 288 400

2 Expliquez pourquoi la production d’une industrie


19 315 427
concurrentielle sera toujours supérieure à la
production de la même industrie en situation
20 344 456
de monopole non discriminant.
LE MONOPOLE 161

b) Le tableau 12.3 présente certains points situés c) Tracez, sur le graphique de la figure 12.4,
sur la courbe de demande de l’entreprise, la courbe de demande et les courbes de Rm,
ainsi que les données de coût total provenant de CVM, de CTM et de Cm correspondant
de la partie a. Remplissez le tableau en aux données des parties a et b. Indiquez
inscrivant les valeurs de coût marginal du sur votre graphique la production d’équilibre
tableau 12.2 et en calculant la recette totale, et la surface qui correspond au profit
la recette marginale et le profit économique. économique.
(.N’oubliez pas d’inscrire les valeurs de la
recette marginale, comme le coût marginal, FIGURE 12.4
entre les lignes de production.)
Quel est le volume de production qui
permet à l’entreprise de maximiser ses profits ?
À quel prix vendra-t-elle ses crochets ?
À combien s’élèvera son profit économique
total ? Expliquez vos réponses.

TABLEAU 12.3

Profit
Quantité Recette Recette Coût Coût écono¬
demandée totale marginale total marginal mique

(Qd) (RT) (Rm) (CT) (Cm) (RT-CT)


(en (en (en (en (en (en
crochets dollars dollars par dollars dollars par dollars
par jour) par jour) crochet) par jour) crochet) par jour)
Quantité

57 9 247
d) Les courbes de coût de l’entreprise ne changent
pas, mais maintenant la courbe de demande
56 10 256
du marché se déplace. Le tableau 12.4 indique
certains points situés sur la nouvelle courbe
55 1 1 267
de demande ainsi que les données de coût
total provenant de la partie a. Remplissez
54 12 280
le tableau en inscrivant vos valeurs du coût
marginal du tableau 12.2 et en calculant
53 13 295
les nouvelles valeurs de la recette totale, de
la recette marginale et du profit économique.
52 14 312

51 15 331

50 16 352

49 17 375

48 18 400

47 19 427

46 20 456
162 CHAPITRE 12

TABLEAU 12.4 FIGURE 12.5

Profit

Quantité Recette Recette Coût Coût écono¬

demandée totale marginale total marginal mique

(Qd) (RT) (Rm) (CT) (Cm) (RT-CT)

(en (en (en (en (en (en


Prix crochets dollars dollars par dollars dollars par dollars

(p> par jour) par jour) crochet) par jour) crochet) par jour)

24,50 9 247

24 10 256

23,50 1 1 267

23 12 280 Quantité

22,50 13 295
f) Supposons que la demande baisse encore,
22 14 312 de sorte que l’équation de la nouvelle courbe
de demande est P— 19 — 1/2 Qo- Sur le
21,50 15 331 graphique de la figure 12.6, tracez cette courbe
de demande et copiez la courbe de CVM
21 16 352 de la figure 12.4. Expliquez pourquoi
le monopoleur cessera ses activités à court
20,50 17 375 terme.

20 18 400 FIGURE 12.6

19,50 19 427

19 20 456

18,50 21 487

Quel est le volume de production qui


permet à l’entreprise de maximiser ses profits ?
À quel prix vendra-t-elle maintenant les
crochets ? À combien s’élèvera son profit
économique total ? Expliquez vos réponses,
e) Sur le graphique de la figure 12.5, tracez
la nouvelle courbe de demande et la courbe
de Rm. Copiez les courbes de CVM, de
CTM et de Cm de la figure 12.4. Indiquez
Quantité
sur le graphique la nouvelle production
d’équilibre et la surface qui représente le
profit économique.
®& 6 On vous donne les informations suivantes
sur une industrie composée de 100 entreprises
identiques. La courbe de demande de cette
industrie est

P= 36 - 0,01Q
La courbe de recette marginale de l’industrie est

Rm = 36 - 0,02Q

La courbe de coût marginal d’une entreprise est


Cm — — 12 + 2 Q
LE MONOPOLE 163

La somme horizontale des courbes de coût Lucien décide de pratiquer la discrimination


marginal de toutes les entreprises de l’industrie par les prix entre ces deux groupes en traitant
est chaque demande séparément et en demandant le
Cm = - 12 + 0,02Q prix qui lui permet de maximiser ses profits dans
chacun de ces deux sous-marchés. Le coût
À l’aide de ces informations, répondez
marginal et le coût total moyen sont égaux et
aux questions suivantes.
constants à 2 $ par repas.
a) Supposons que les 100 entreprises sont toutes
a) Remplissez le tableau 12.5 en calculant la
contrôlées par un monopole non discriminant.
recette totale et la recette marginale associées
Calculez le volume de production qui permet
à la demande des courtiers {RTC et Rmc) ainsi
de maximiser les profits du monopole.
que la recette totale et la recette marginale
Calculez le prix par unité de production
associées à la demande des retraités {RTR et
que demandera le monopoleur.
RmR). {N’oubliez pas d’inscrire la recette
b) Supposons maintenant que les 100 entreprises
marginale entre les lignes.)
soient exploitées indépendamment dans
b) Quel est le volume de production et le prix
le cadre d’une industrie parfaitement
qui permettent de maximiser les profits dans
concurrentielle. Calculez la quantité d’équilibre
le cas des courtiers ?
à court terme pour l’industrie dans
c) Quel est le volume de production et le prix
son ensemble. Calculez le prix d’équilibre
qui permettent de maximiser les profits dans
à court terme de l’industrie.
le cas des retraités?
c) Sans utiliser votre réponse sur la production
d) À combien s’élève le profit économique total ?
de l’industrie de la partie b, calculez
e) Montrez que le profit économique total de la
la production d’équilibre à court terme
partie d se confond avec le profit maximal en
par entreprise.
le comparant avec le profit économique total
d) Comparez les résultats obtenus pour le prix
si Lucien sert : 1 repas supplémentaire à
et la quantité du monopole avec ceux obtenus
chaque courtier et chaque retraité ; 1 repas de
pour le prix et la quantité de l’industrie
moins à chaque courtier et chaque retraité.
parfaitement concurrentielle.
f) Quel est le surplus du consommateur pour les
7 Le Bistro Chez Lucien attire le midi deux types courtiers ? pour les retraités ? pour tous les
de clients : les courtiers et les retraités. Le tableau clients ?
12.5 nous présente les barèmes de demande de
repas du midi de ces deux groupes.

TABLEAU 12.5
Courtiers Retraités

Quantité Recette Recette Quantité Recette Recette


Prix demandée totale marginale demandée totale marginale

(Qd) (RTC) (Rmc) (Qd) (RTr) (RM*)


(P)
8 0 0

7 1 0

6 2 0

5 3 1

4 4 2

3 5 3

2 é 4

1 7 5

8 6
0
64 CHAPITRE 12

8 Les courtiers se plaignent amèrement de c) L’industrie est un monopole qui pratique


la politique de discrimination par les prix de la discrimination par les prix en demandant
Lucien et le menacent d’apporter leurs propres deux prix: Px et Pd.
repas au bureau si Lucien ne demande pas d) L’industrie est un monopole qui pratique
le même prix à tous les clients. Lucien capitule la discrimination par les prix parfaite.
sous la pression et s’installe avec sa calculatrice
pour déterminer le volume de production FIGURE 12.7
et le prix qui lui permettraient de maximiser
ses profits en tant que monopoleur non
discriminant. En utilisant l’information donnée
dans le problème à court développement 7,
pouvez-vous faire ce calcul à sa place sans regarder
les indices ci-après ? Sinon, répondez aux
questions suivantes.
a) Calculez le barème de demande de Lucien
en additionnant la quantité demandée par
les courtiers et les retraités pour chaque prix.
Construisez un tableau similaire au tableau
12.3, avec des colonnes pour le prix,
la quantité demandée, la recette totale et
la recette marginale. (N’oubliez pas d’inscrire
la recette marginale entre les lignes).
b) Le coût marginal et le coût total moyen
restent à 2 $ par repas. À quelle quantité 10 Avant les Jeux Olympiques, le Comité
de repas et à quel prix Lucien peut-il International Olympique (CIO) vend aux
maximiser ses profits ? enchères le droit de diffuser les Jeux pour les
c) À combien s’élève le profit économique total accorder au plus offrant. NBC a gagné les
de Lucien lorsqu’il est monopoleur non enchères pour les Jeux Olympiques de 1996,
discriminant ? Comment ce profit se compare- en payant 456 millions de dollars américains.
t-il avec le profit économique total Les droits de reproduction à la télévision ont
lorsque Lucien pratique la discrimination conféré à NBC un monopole. Pourtant, les
par les prix ? analystes avaient (correctement) prédit que
d) Quel est le surplus du consommateur pour NBC ne ferait pas de profit. Leur prédiction
tous les clients ? Comment se compare-t-il au était fondée, puisque NBC n’avait pas réalisé
surplus du consommateur de tous les clients de profit avec de semblables émissions télévisées
lorsque Lucien pratique la discrimination par des Jeux Olympiques de 1988 et de 1992.
les prix ? Les questions suivantes permettront de
comprendre cet apparent paradoxe.
9 La figure 12.7 nous donne les courbes de a) Quel type de monopole détient NBC ?
demande, de recette marginale et de coût b) NBC réalise une recette en vendant
marginal d’une certaine industrie. Votre tâche du temps d’antenne aux commanditaires
consiste à illustrer comment le surplus du commerciaux. Supposons que NBC
consommateur et le surplus du producteur sont obtienne une courbe de demande normale
répartis dans les quatre types de marché de cette à pente négative en vendant des messages
industrie. Pour chaque cas, retracez toutes les publicitaires de 30 secondes. Ne nous
parties pertinentes de la figure 12.7, puis 1) occupons pas pour le moment du paiement
indiquez la partie du graphique qui correspond quelle devra faire au CIO et supposons
au surplus du consommateur en y traçant des que le coût marginal et le coût total moyen
lignes horizontales ; 2) indiquez la partie qui de la prestation de temps d’antenne soient
correspond au surplus du producteur en y égaux et constants. Tracez un graphique
traçant des lignes verticales ; et 3) indiquez la représentant la décision que prend NBC
partie (le cas échéant) qui correspond à la perte pour maximiser ses profits. Même si vous
sèche en y faisant des pointillés. n’avez pas suffisamment d’informations
a) L’industrie comprend de nombreuses pour obtenir des chiffres précis, indiquez
' entreprises parfaitement concurrentielles. de manière générale le nombre de messages
b) L’industrie est un monopole non publicitaires vendus et le prix par message
discriminant. publicitaire.
LE MONOPOLE 165

c) Indiquez sur votre graphique la partie


©8c {RM- CTM) x Q= (6 $ - 4 $) x 3.
qui représente le profit économique. Si les
9 b c vrai dans le cas de la discrimination
prédictions de l’analyste sont exactes, quelle
par les prix parfaite.
est la valeur correspondant à cette partie ?
10 b Surface au-dessus du prix, mais sous
d) Pourquoi NBC n’a-t-elle pas réalisé de profit la demande (disposition à payer).
économique ?
11 d Surface sous le prix, mais au-dessus de Cm.
12 b Somme des surplus du producteur (bcd)
et du consommateur (abd) perdus par
comparaison avec le résultat de la concurrence.
RÉPONSES 13 e La perte sèche est nulle.
14 d Les comparaisons entre Rm — Cm sont la clé.
La recette a et le coût b sont importants, mais
V rai/Faux/l ncertai n
doivent être comparés sur le plan de la
(Justifiez votre réponse.)
rentabilité.
15 d Définition.
1 F La Rm est négative.
16 a Tracez le graphique. Qw/, > Q
2 F Un monopole n’a pas de courbe d’offre.
correspondant à Rm — Cm.
3 I Rien ne garantit qu’un monopole réalisera
17 d RT serait inutilement -l. Le monopoleur
un profit lorsque la recherche de rentes
pourrait X Q et î P pour T RT.
est suffisamment coûteuse.
© 18 d Même résultat que pour l’industrie en
© 4 V La courbe de demande indique la recette
situation de concurrence parfaite, donc e est
obtenue pour chaque unité successive.
faux. D = Rm donc a est faux. Maximisation
5 F Ne peut pas être rapidement revendu.
des profits, donc b est faux.
6 V La surface située sous le prix de concurrence
19 c Même règle pour le seuil de fermeture que
et au-dessus de la Cm.
pour la concurrence parfaite.
© 7 I Vrai pour le monopole qui pratique
©20 d Somme de (P — Cm) pour chaque unité
la discrimination par les prix parfaite,
de production.
faux pour tous les autres monopoles.
21 c Règle permettant de choisir le volume
©8F Plus élevé. Les ressources utilisées pour
de production.
la recherche de rentes entraînent des coûts
22 a Fixe un prix moins élevé lorsque T) est plus
sociaux. grande et un prix plus élevé lorsque T| est
9 F En situation d’équilibre, les profits
moins grande.
économiques peuvent être totalement
23 a Définition. Cette activité entraîne des coûts,
éliminés par les coûts de recherche de rentes.
©24 d a vrai pour les deux, b vrai pour concurrence
10 V Avantages du monopole. seulement, c faux pour les deux.
25 e L’allocation des ressources est aussi efficiente
Questions à choix multiple que pour la concurrence parfaite.

1 b Les autres sont des barrières légales. Problèmes à court développement


2 d RT (P = 7 $) = 7 $ x 7 = 49 $. RT (P= 6 $)
— 6 $ x 8 = 48 $. Rm = A RT — 48 $ — 49 $■ I Pour pouvoir vendre une unité supplémentaire
3 c Voir le texte du manuel. Devrait -1 production. de production, un monopole doit baisser le
La décision de cesser les activités est prix. Cette baisse a deux effets sur la recette :
prématurée puisque le monopoleur peut l’un positif et égal au prix, et l’autre, négatif. La
recette marginale est l’effet net, qui doit être
changer les prix.
4 d Les autres sont des avantages. La recherche moins grand que le prix. D’abord, l’unité

de rentes utilise des ressources qui auraient supplémentaire vendue au nouveau prix moins

pu être employées de manière productive. élevé augmente la recette d’un montant égal

5 c Profit de monopole = ressources utilisées au prix. Cependant, un monopole non


discriminant doit également baisser le prix
pour la recherche de rentes.
6 c a, b sont des comparaisons entre les valeurs demandé aux clients précédents qui, sinon,

marginales et moyennes non pertinentes, d auraient payé un prix plus élevé. L’effet net sur
la recette marginale est égal au prix moins la
est vrai si RT < CVT.
perte de recette due à la baisse du prix demandé
7 b Q lorsque Rm = Cm. Le P le plus élevé
que les consommateurs sont disposés à payer aux clients précédents. Cette différence doit
forcément être inférieure au prix.
pour 3 unités.
166 CHAPITRE 12

2 Le volume de production d’une industrie


FIGURE 12.8
concurrentielle est choisi de façon à permettre
à la courbe de coût marginal de l’industrie OU
de croiser la courbe de demande de l’industrie. a3 Équilibre
X O
Le volume de production d’un monopole non . dans une industrie
concurrentielle
discriminant est choisi de manière à ce que
rc Le monc pôle réalisé
la courbe de coût marginal de l’industrie croise
-"^des économies d'échelle
la courbe de recette marginale du monopole. PM ^ et de gamme
Comme la courbe de recette marginale
se situe au-dessous de la courbe de demande,
cela signifie que le volume de production
du monopole est moins élevé. ^D

Rm
3 Bien que la quantité vendue et le prix demandé
au dernier client soient les mêmes pour la 0 C M Quantité
concurrence parfaite et le monopole qui pratique
une discrimination par les prix parfaite, la
répartition du surplus du consommateur n’est a) Le tableau 12.2 rempli est présenté au tableau
pas la même. Comme un monopole qui pratique 12.2 Solution.
une discrimination par les prix parfaite demande
à chaque client le prix le plus élevé que celui-ci TABLEAU 12.2 SOLUTION
est disposé à payer, il n’y a pas de surplus du
consommateur. Tout surplus du consommateur Coût Coût Coût
qui aurait été réalisé dans une industrie variable Coût Coût variable total
total total marginal moyen moyen
parfaitement concurrentielle revient maintenant
Quantité (CVT) (CT) (Cm) (CVM) (CTM)
au monopole. Les consommateurs aimeraient
s’approprier un plus grand surplus du (en dollars) (en dollars)

consommateur et, donc, payer moins pour par jour) par crochet)

la même quantité. Les consommateurs préfèrent 9 135 247 15,00 27,44


donc la concurrence parfaite. ... 9
10 144 256 14,40 25,60
4 Un monopole serait plus efficient qu’une ... 1II1
industrie parfaitement concurrentielle s’il réalisait 1 1 155 267 14,09 24,27
suffisamment d’économies d’échelle ou de ... 13
gamme, ou les deux. Ces économies doivent 12 168 280 14,00 23,33
être suffisamment importantes pour que le ... 15
monopole produise davantage que l’industrie 13 183 295 14,08 22,69
concurrentielle et vende à un prix inférieur. ... 17
La figure 12.8 illustre ce type de situation. Ce 14 200 312 14,29 22,29
qui compte, c’est que la courbe de coût marginal ... 19
du monopole soit non seulement plus basse que 15 219 331 14,60 22,07
la courbe d’offre de l’industrie concurrentielle, ... 21
mais aussi suffisamment basse pour croiser la 16 240 352 15,00 22,00
courbe de Rm au point correspondant à un ... 23
volume de production supérieur à C (la 17 263 375 15,47 22,06
production concurrentielle). On rencontre ce ... 25
type de situation lorsque les économies d’échelle 18 288 400 16,00 22,22
ou de gamme sont très importantes. ... 27
19 315 427 16,58 22,47
... 29
20 344 456 17,20 22,80

b) Le tableau 12.3 rempli est présenté au tableau


12.3 Solution.
LE MONOPOLE 167

TABLEAU 12.3 SOLUTION FIGURE 12.4 SOLUTION

Profit
Quantité Recette Recette Coût Coût écono-
demandée totale marginale total marginal mique
(Qd) (RT) (Rm) (CT) (Cm) (RT-CT)
(en (en (en (en (en (en
Prix crochets dollars dollars par dollars dollars par dollars
(p> par jour) par jour) crochet) par jour) crochet) par jour)

57 9 513 247 266


.47 .9
56 10 560 256 304
.45 .Il

55 1 1 605 267 338


.43 .13
54 12 648 280 368
.41 .15 Quantité

53 13 689 295 394


.39 .17
52 14 728 312 416 d) Le tableau 12.4 rempli est présenté au tableau

.37 .19 12.4 Solution.

51 15 765 331 434


.35 .21
TABLEAU 12.4 SOLUTION

50 16 800 352 448


Profit
.33 .23
Quantité Recette Recette Coût Coût écono-
49 17 833 375 458 Prix demandée totale marginale total marginal mique
.31 .25 (P) (Qd) (RT) (Rm) (CT) (Cm) (RT-CT)

48 18 864 400 464 (en (en (en (en (en (en


crochets dollars dollars par dollars dollars par dollars
.29 .27 par jour) par jour) crochet) par jour) crochet) par jour)
147 19 893 427 466 |
24,50 9 220,50 247 -26,50
.27 .29
.19,50 . 9
46 20 920 456 464
24,00 10 240,00 256 -16,00
On atteint la production d’équilibre lorsque Cm = Rm = 28, .18,50 .1 1
Q= 19, P = 47 $, profits économiques = 466 $ par jour. 23,50 1 1 258,50 267 -8,50

.17,50 .13
On atteint le volume de production qui 12 276,00
23,00 280 -4,00
permet de maximiser les profits lorsque le .16,50 .15
coût marginal est égal à la recette marginale, 122,50 13 292,50 295 -2,501
soit 19 unités. Le prix maximal que peut .15,50 .17
demander l’entreprise en vendant toujours 22,00 14 308,00 312 -4,00
19 unités est 47 $■ Cette combinaison .14,50 .19
de quantité et de prix permet de réaliser 21,50 15 322,50 331 -8,50
un profit économique total de 466 $, qui, .13,50 .21
comme on peut le voir au tableau, correspond 21,00 16 336,00 352 -16,00
au profit maximal. .12,50 .23
c) Le graphique requis se trouve à la figure 12.4 20,50 17 348,50 375 -26,50
Solution. .1 1,50 .25

20,00 18 360,00 400 -40,00

.10,50 .27

19,50 19 370,50 427 -56,50

. 9,50 .29

19,00 20 380,00 456 -76,00

On atteint la production d’équilibre lorsque


Cm - Rm — 16, Q = 13, P = 22,50 $, profit
économique - — 2,50 $.
168 CHAPITRE 12'

On atteint le volume de production qui


FIGURE 12.6 SOLUTION
permet de maximiser les profits lorsque le
coût marginal est égal à la recette marginale,
qui est maintenant de 13 unités. Le prix
maximal que l’entreprise peut demander en
vendant toujours 13 unités est 22,50 $. Cette
combinaison de quantité et de prix produit
un profit économique total de — 2,50 $ (une •
perte économique). Comme le montre le
tableau, il s’agit de la perte minimale possible.
L’entreprise continuera de produire à court
terme, car cette perte est inférieure à la perte
de seuil de fermeture, qui serait de 112$,
le montant de son coût fixe total,
e) Le diagramme requis est présenté à la figure
12.5 Solution. Quantité

FIGURE 12.5 SOLUTION


6 a) Le monopoleur choisira le volume de
production auquel la recette marginale
est égale au coût marginal. Pour calculer
cette quantité, rendez l’équation de recette
marginale égale à l’équation de coût marginal
de 1 ’industrie (puisque le monopoleur contrôle
toutes les entreprises).

36 — 0,02Q = -12 + 0,02Q


48 = 0,04Q
1 200 = Q

Pour calculer le prix que demandera le


Quantité monopoleur, remplacez la quantité de 1 200
dans l’équation de demande. De même,
on peut, après avoir trouvé la quantité qui
f) Le graphique requis est présenté à la figure
correspond à l’intersection de Cm et de Rm,
12.6 Solution. Puisque la courbe de demande
regarder vers le haut pour lire le prix sur
est située sous la courbe de CVM, quelle
la courbe de demande.
que soit la quantité que pourrait produire
l’entreprise, le prix sera inférieur au CVM. P = 36-0,01Q
Cela signifie que l’entreprise subira une perte P = 36-0,01 (1 200)
monétaire sur chaque unité produite en plus P = 36 -12
de la perte de son coût fixe total. Dans ce cas, P = 24
le monopoleur réduira sa perte au minimum
b) On atteint la quantité d’équilibre à court terme
en cessant ses activités et en ne perdant que
de l’industrie parfaitement concurrentielle
son coût fixe (112$).
lorsque la courbe de demande de l’industrie
croise la courbe d’offre de l’industrie. Rendez
l’équation de demande de l’industrie égale à
l’équation d’offre de l’industrie, qui est
la somme horizontale des courbes de coût
marginal de toutes les entreprises de l’industrie.

36 - 0,01Q = -12 + 0,02Q


48 = 0,03Q
1 600 = Q
LE MONOPOLE 169

Pour calculer le prix d’équilibre à marginal d’une entreprise, qui est également
court terme pour l’industrie, nous pouvons sa courbe d’offre à court terme.
remplacer la quantité de 1 600 soit dans
l’équation de demande de l’industrie, P = - 12 + 2Q
soit dans l’équation d’offre de l’industrie. 20 = -12 + 2Q
Avec l’équation de demande de l’industrie, 32 = 2Q
on obtient 16 = Q
d) Le prix de monopole (24 $) est plus élevé que
P = 36-0,01Q
le prix de concurrence parfaite (20 $), et le
P = 36 -0,01 (1 600)
volume de production du monopole (1 200)
T3 = 36 —16
est inférieur à celui de l’industrie en situation
P = 20
de concurrence parfaite (1 600).
Avec l’équation d’offre de l’industrie,
on obtient le même résultat, soit 7 a) Le tableau rempli est présenté au tableau
12.5 Solution.
P = -12 + 0,02Q b) On atteint le volume de production qui
P = -12 + 0,02 (1 600)
permet de maximiser les profits réalisés
P = -12+ 32
grâce aux courtiers lorsque Cm = 2 $ = Rmc.
P = 20 Il s’agit de 3 repas du midi, et le prix du repas
c) Si la production de l’industrie est de 1 600 pour les courtiers est de 5 $.
unités et qu’il y a 100 entreprises identiques, c) Le volume de production qui permet de
alors la production par entreprise sera maximiser les profits réalisés grâce aux
évidemment de 1 600 unités/100 entreprises retraités est atteint lorsque Cm = 2 $ = RmR.
= 16 unités/entreprise. Toutefois, dans cette Il s’agit de 2 repas du midi, et le prix du repas
question, on vous demande expressément pour les retraités est de 4 $.
de ne pas utiliser l’information concernant d) La recette totale provenant des courtiers est
la production de l’industrie. de 15 $ et celle provenant des retraités est
L’autre calcul de la quantité d’équilibre à de 8 $, soit 23 $. Puisque le coût total moyen
court terme par entreprise consiste à remplacer est de 2 $ le repas, le coût total est de 2 $
le prix d’équilibre de 20 dans la courbe de coût X 5 repas = 10 $. Le profit économique total
est donc de 13 $.

TABLEAU 12,5 SOLUTION

Courtiers Retraités

Quantité Recette Recette Quantité Recette Recette


Prix demandée totale marginale demandée totale marginale

(P) (Qd) (RTC) (Rmc) (Qd) (RTr) (RAM

8 0 0 0 0
.7 .0
7 1 7 0 0
.5 .0
6 2 12 0 0
. 3 .5
1 5 3 15 1 5
. 1 . 3

4 4 16 1 2 8
1

3 5 15 3 9
.-3 .-1
2 6 12 4 8
.-5 .-3

1 7 7 5 5
.-7 .-5
8 0 6 0
0
170 CHAPITRE 12

e) Si Lucien sert 1 repas supplémentaire aux b) On atteint le volume de production qui


courtiers et aux retraités, cela fera 4 repas pour permet de maximiser les profits lorsque le
les courtiers (à raison de 4 $ le repas) et 3 repas Cm — 2 $ — Rm. Cela se produit à mi-chemin
pour les retraités (à raison de 3 $ le repas). entre une combinaison de 4 repas au coût
Comme le coût total moyen est de 2 $ par de 5 $ chacun, et une combinaison de 6 repas
repas, le coût total est de 2 $ X 7 repas = 14 $. au coût de 4 $ chacun. Lune ou l’autre
La recette totale provenant des courtiers est de combinaison produit le même profit
16 $ et celle provenant des retraités est de 9 $,• économique. Étudions le cas de la combinaison
soit 25 $. Le profit économique total est donc de 6 repas au coût de 4 $ chacun.
de 11 $, ce qui est inférieur aux 13 $ de la c) La recette totale est de 4 $ par repas X 6 repas
partie d. = 24 $. Le coût total moyen est de 2 $
Si Lucien sert 1 repas de moins aux par repas. Le coût total est de 2 $ X 6 repas
courtiers et aux retraités, cela fera 2 repas = 12 $. Le profit économique total est
pour les courtiers (au prix de 6 $ le repas) donc de 12 $.
et 1 repas pour les retraités (au prix de 5 $ Les profits économiques de Lucien en
le repas). Comme le coût total moyen est de tant que monopoleur non discriminant (12 $)
2 $ le repas, le coût total est de 2 $ X 3 repas sont inférieurs à ses profits économiques
= 6 $. La recette totale provenant des en tant que monopoleur pratiquant
courtiers est de 12 $ et celle provenant des la discrimination par les prix (13 $).
retraités est de 5 $, soit 17 $. Le profit d) En observant les courbes de demande, nous
économique est donc de 11 $, ce qui est pouvons dire que un client (courtier) est prêt
inférieur aux 13 $ de la partie d. à payer son repas 7 $, deux clients (courtier et
f) Le surplus du consommateur dans le cas des retraité) sont prêts à payer le repas 5 $ et deux
courtiers est clients (courtier et retraité) sont prêts à payer
(7 $ - 5 $) + (6 $ - 5 $) + (5 $ - 5 $) - 3 $. le repas 4 $. Le surplus du consommateur
pour tous les clients est
Le surplus du consommateur dans le cas des
1 (7 $ - 4 $) + 1 (6 $ - 4 $) + 2 (5 $ - 4 $)
retraités est
+ 2 (4 $ - 4 $) = 7 $.
(5 $ - 4 $) + (4 $ - 4 $) = 1 $.
Ce surplus du consommateur sans
Le surplus du consommateur de tous les
discrimination par les prix (7 $) est plus élevé
clients est
que le surplus du consommateur avec
3 $ + 1 $ = 4 $ discrimination par les prix (4 $).
& 8 a) Le tableau rempli est présenté au tableau 12.6
Solution. a) Dans une situation de concurrence parfaite,
le prix est égal au coût marginal. Le montant
TABLEAU 12.6 SOLUTION
du surplus du consommateur est représenté
Quantité Recette Recette par la surface située sous la courbe de demande
Prix demandée totale marginale mais au-dessus du prix (P0), alors que le
(P) (Qd) (RT) (Rm) montant du surplus du producteur est
représenté par la surface située au-dessus
8 0 0
de la courbe Cm mais au-dessous
. 7
7 du prix; voir la figure 12.7 Solution (a).
1 7
. 5
6 2 12
. 4
5 4 20
. 2 1
4 6 24
. 0
3 8 24
.-2
2 10 20
.-4
1 12 12
.-6
0 14 0
LE MONOPOLE 171

b) S’il s’agit d’un monopole non discriminant,


le prix sera supérieur au Cm et le volume
de production, inférieur à celui d’une industrie
concurrentielle. Le surplus du consommateur
est, là encore, représenté par la surface située
sous la courbe de demande, mais au-dessus
du prix (P2), alors que le surplus du
producteur est représenté par la surface située
au-dessus de la courbe de Cm, mais au-dessous
du prix jusqu’au volume de production de
monopole. La partie restante du grand triangle
représente une perte sèche puisqu’il s’agit
du montant de surplus de l’industrie
concurrentielle qui est perdu dans le cas
du monopole non discriminant ; voir la figure
12.7 Solution (b).
c) Un raisonnement similaire nous permet
d’établir les surfaces indiquées à la figure 12.7
Solution (c), qui correspondent au surplus du
consommateur, au surplus du producteur et
à la perte sèche.
d) Dans le cadre d’une discrimination par les
prix parfaite, le producteur s’approprie tout le
surplus potentiel et il n’y a pas de perte sèche
(ni de surplus du consommateur). Voir la
figure 12.7 Solution (d).

FIGURE 12.7 SOLUTION


10 a) NBC détient un monopole légal, fondé
sur une concession, un droit exclusif accordé
à une entreprise pour quelle fournisse un
bien ou un service.
b) Le graphique requis est présenté à la figure
12.9 Solution. La quantité de messages
publicitaires est Qmp et Ie prix, PMP.

FIGURE 12.9 SOLUTION


172 CHAPITRE 12

c) Le rectangle ombré situé au-dessus du d) NBC n’a pas réalisé de profits économiques
CTM et au-dessous du PMP de la figure 12.9 en raison des coûts de recherche de rentes.
Solution représente le profit économique. NBC a dû se livrer à la concurrence pour
Ce profit, fondé sur les coûts qui excluent détenir le monopole sur la télédiffusion des
le paiement de 456 millions de dollars Jeux. Pour les soumissionnaires, il est logique
américains devant être versé au CIO, s’élèvera de continuer d’offrir un prix élevé pour ces
à 456 millions de dollars américains si les droits jusqu’au point où leur prix est égal
analystes ont raison. à la valeur du profit économique qu’ils
réaliseraient s’ils détenaient ces droits.
La concurrence
monopolistique
et l’oligopole

légèrement différents. Les entreprises jouissent ainsi


CONCEPTS CLÉS d’un certain pouvoir de monopole, et peuvent entrer
librement sur le marché et en sortir.

La diversité des structures de marché ♦ Implications

Les structures de marché de la plupart des • L’entreprise doit faire face à une courbe de

industries se situent quelque part entre deux demande à pente négative.

extrêmes : la concurrence parfaite et le monopole. • La courbe de recette marginale est différente


de la courbe de demande.
♦ Pour évaluer le degré de concurrence d’un marché, • L’entreprise peut choisir le prix et le niveau
les économistes ont recours à deux mesures de production.
de la concentration industrielle:
♦ Résultats
• Le ratio de concentration fondé sur quatre
entreprises (RC4) - pourcentage des ventes • La maximisation des profits : choisir Q de sorte
de l’industrie réalisé par les quatre plus que Cm — Rm ainsi que le prix le plus élevé
importantes entreprises de cette industrie. (sur la courbe de demande).
• L’indice Herfindahl-Hirschman (IHH) — • La perspective de profits économiques à court

somme de la part du marché au carré de terme favorise l’entrée sur le marché de


chacune des 50 plus importantes entreprises nouvelles entreprises, d’où un déplacement vers

de l’industrie. la gauche de la courbe de demande de chaque


entreprise.
♦ Des ratios de concentration élevés entraînent • Les profits économiques à long terme sont nuis,
habituellement un faible niveau de concurrence. P = CTM, CTM pas minimal (capacité
excédentaire).
♦ Limites des ratios de concentration
• Les pertes d’efficience dans l’allocation des
• Ils sont fondés sur la part du marché des ressources et les frais de publicité élevés doivent
entreprises à l’échelle nationale, alors que de être pondérés compte tenu des gains tirés de
nombreuses industries vendent sur le marché l’augmentation de la variété des produits et
régional ou mondial. de leur innovation, ainsi que du surcroît
• Ils ne donnent aucune indication sur d’information dont bénéficient les
l’importance des barrières à 1 entree. consommateurs.
• Les entreprises se diversifient dans d autres
industries. L’oligopole

La concurrence monopolistique L’oligopole est une structure de marché dans laquelle


quelques entreprises se font concurrence de manière

La concurrence monopolistique est une structure de stratégique. L’entreprise doit tenir compte des effets
marché dans laquelle un grand nombre d entreprises de ses décisions sur le comportement des autres et

se font concurrence en pratiquant la différenciation des effets des décisions des autres entreprises sur son
propre profit.
des produits, c’est-à-dire en fabriquant des produits
174 CHAPITRE 13

♦ Selon le modèle de courbe de demande coudée, ♦ Équilibre en stratégie dominante - lorsque


si l’entreprise augmente ses prix, aucune entreprise chaque joueur a une stratégie dominante.
ne suit et si l’entreprise réduit ses prix, toutes
♦ Équilibre de Nash — A prend la meilleure décision
les entreprises suivent. Il en résulte que chaque
possible compte tenu de la décision de B, et B
entreprise a une courbe de demande coudée,
prend la meilleure décision possible compte tenu
le coude de la courbe se situant au prix et à la
de la décision de A.
quantité du marché.

• Prédictions : prix rigides - le coude de la courbe


Le jeu de l’oligopole
crée une discontinuité dans la courbe Rm de
sorte que le Cm peut varier dans cet intervalle
Le duopole est une structure de marché qui comprend
sans faire varier le prix ni la quantité.
deux entreprises.
• Problèmes : aucune indication de la manière
dont P est déterminé ni comment l’entreprise ♦ Dans un jeu de duopole, chaque entreprise peut
réagit lorsqu’elle découvre que ses attentes sur soit se conformer à une entente de collusion
la courbe de demande ne correspondent pas pour limiter la production et augmenter le prix,
à la réalité. soit tricher.

♦ Selon le modèle d’oligopole à entreprise dominante, ♦ Un cartel est un groupe d’entreprises qui ont
on suppose qu’une grande entreprise a un avantage conclu une entente de collusion.
de coût substantiel, et qu’il y a de nombreuses
♦ Dans un jeu unique, il existe une solution de type
petites entreprises.
«dilemme du prisonnier» — chaque entreprise a la
• Prédictions: cette grande entreprise fonctionne stratégie dominante de tricher, même si les deux
comme un monopole ; elle fixe le prix qui lui entreprises ont intérêt à se faire confiance et à
permet de maximiser ses profits. Les petites respecter les termes de leur entente.
entreprises sont des preneurs de prix qui se
♦ Dans un jeu répété, d’autres stratégies peuvent
comportent comme des entreprises en situation
créer un équilibre coopératif, chaque entreprise
de concurrence parfaite.
respectant les termes de l’entente.
• Problèmes: rien n’indique pourquoi la grande
entreprise a un avantage de coût substantiel ni • Une stratégie de coup pour coup consiste à suivre
ce qui se passe lorsque les petites entreprises la stratégie qu’a adopté l’autre joueur durant la
acquièrent une technologie à faible coût. période précédente.
• Une stratégie de déclic consiste à coopérer si
l’autre joueur coopère, sinon à tricher
La théorie des jeux
indéfiniment si l’autre joueur triche.

La théorie des jeux est une méthode d’analyse des La théorie des jeux peut servir à analyser d’autres choix
interactions stratégiques. Les jeux ont en commun : des entreprises — par exemple, le montant à consacrer à
la recherche et au développement ou à la publicité, ou
♦ des règles: elles précisent ce que les joueurs ont
bien la décision d’entrer dans l’industrie ou d’en sortir.
le droit de faire ;
Dans un marché contestable, les entreprises sont peu
♦ des stratégies : il s’agit de mesures concernant la
nombreuses, mais elles sont libres d’entrer et de sortir,
hausse ou la baisse des prix, de la production,
de sorte que les entreprises existantes sont livrées
des frais de publicité ou de la qualité du produit;
à la concurrence des entrants potentiels. Les entreprises
♦ des gains : il s’agit des profits et des pertes des existantes peuvent avoir recours aux stratégies
joueurs. La matrice des gains indique les gains suivantes.
par rapport aux stratégies.
♦ Jeu d’entrée et de représailles - fixer un prix
Le «dilemme du prisonnier» est un jeu entre deux concurrentiel et réaliser un profit normal pour
personnes qui se joue une seule fois. La stratégie évincer un concurrent potentiel.
dominante de chaque joueur est de tricher, c’est-à-dire,
♦ Fixation de prix limite — fixer un prix inférieur et
dans ce cas, d’avouer. Les concepts du jeu sont les
une quantité supérieure au prix et à la quantité de
suivants.
monopole, pour empêcher l’entrée d’une nouvelle
♦ Stratégie dominante — la meilleure stratégie d’un entreprise.
joueur quelle que soit la stratégie adoptée par
l’autre joueur.
LA CONCURRENCE MONOPOLISTIQUE ET L'OLIGOPOLE 175

Dans ce chapitre, nous utilisons la théorie des


RAPPELS jeux élémentaires pour expliquer ce qu’est
l’oligopole. Le jeu du dilemme du prisonnier
1 La plupart des industries ne sont pas de purs illustre les concepts les plus importants de la
monopoles ni en situation de concurrence théorie des jeux (règles, stratégies, gains), qui
parfaite ; elles se situent quelque part entre ces sont ensuite utilisés dans des modèles de théorie
deux extrêmes. Cela ne veut pas dire que les des jeux plus complexes comme ceux des jeux
deux derniers chapitres soient inutiles. En répétés.
examinant les entreprises dans le cadre de Il est important d’apprendre à trouver
structures de marché extrêmes, nous pouvons l’équilibre d’un jeu du type du dilemme du
traiter dans un seul chapitre de tout l’éventail prisonnier. Prenez l’exemple des joueurs A et
des industries qui se situent entre les deux. B, où chacun doit choisir entre deux stratégies :
Les formes intermédiaires de structures avouer ou nier. Tracez d’abord la matrice des
de marché ont en commun de nombreuses gains. Considérez ensuite cette matrice dans la
caractéristiques de la concurrence parfaite perspective de A. Puisque A ne sait pas si B va
et/ou du monopole. Considérons la règle de avouer ou nier, A se pose donc deux questions :
maximisation des profits qui consiste à choisir le 1) en supposant que B avoue, est-ce que j’ai
niveau de production auquel Cm — Rm. Cette intérêt à avouer ou à nier? 2) En supposant que
règle s’applique non seulement à la concurrence B nie, est-ce que j’ai intérêt à avouer ou à nier?
parfaite et au monopole, mais également à la Si la meilleure stratégie de A est d’avouer,
concurrence monopolistique, à la courbe de que B avoue ou nie, la stratégie dominante de A
demande coudée et aux modèles d’oligopoles est d’avouer. Considérons ensuite la matrice des
à entreprise dominante. L’entrée libre entraîne gains dans la perspective de B. Laissons B se
des profits économiques à long terme de zéro poser deux questions équivalentes pour trouver
autant pour la concurrence parfaite que pour la stratégie dominante de B. La combinaison de
la concurrence monopolistique. Les courbes la stratégie dominante de A et de la stratégie
de demande et de recette marginale à pentes dominante de B correspond au résultat
négatives s’appliquent également à la d’équilibre du jeu.
concurrence monopolistique, à la courbe de
demande coudée et aux modèles d’oligopoles Le principal moteur du dilemme du prisonnier
à entreprise dominante. est la tension entre le résultat d’équilibre (lorsque
Les hypothèses extrêmes et non réalistes des la meilleure stratégie des deux joueurs est
modèles de concurrence parfaite et de monopole d’avouer parce qu’ils ne peuvent se faire
nous permettent d’isoler les effets d’importantes confiance) et le fait que les deux joueurs auraient
forces, comme la maximisation des profits, et intérêt à coopérer. Toutes les situations
d’importantes contraintes, comme la d’équilibre que nous avons examinées jusqu’à
concurrence et la demande de marché. Ces présent ont été des résultats stables où tous les
forces et contraintes exercent également des intérêts des agents (utilité et profit) ont été
effets dans des structures de marché plus maximisés. N’oubliez pas, l’équilibre est défini
réalistes. Cependant, il était nécessaire d isoler au comme étant une situation dans laquelle il n’y
préalable ces forces et contraintes plutôt que de a aucune incitation à changer. L’équilibre du
tenter d’analyser immédiatement des structures dilemme du prisonnier est différent. Même si
de marché « réalistes » comme la concurrence les deux joueurs avouent, l’intérêt personnel
monopolistique et l’oligopole, ce qui aurait pu de chacun n’est pas maximisé. C’est la possibilité
donner une foule de données et de résultats supplémentaire d’interaction stratégique (faire
déroutants. Cela aurait été comme conduire confiance ou ne pas faire confiance) qui crée
sans carte routière dans une grande ville l’instabilité des résultats.

inconnue. L’instabilité des résultats dans ce jeu simple


nous aide à comprendre des phénomènes de
2 Lorsque vous tracez une courbe d entreprise marché plus complexes, comme la guerre des
en situation de concurrence monopolistique prix de l’essence. Lorsque les propriétaires de
en équilibre à long terme, assurez-vous que la stations-service se font confiance, les prix
courbe CTM est tangente à la courbe de restent relativement élevés et les profits sont
demande au niveau de production où les courbes maximisés. Toutefois, il est toujours possible
de Cm et de Rm se croisent. Assurez-vous que les propriétaires se sentent incités à ne pas
également que la courbe Cm croise la courbe respecter les termes de leur entente de collusion.
CTM au point minimal de la courbe CTM. Lorsque la tricherie prend place, la confiance
176 CHAPITRE 13

n’est plus de mise et les propriétaires sont attirés 7 Un oligopole tiendra compte de la décision prise
par le résultat d’équilibre où tous les par d’autres entreprises avant de décider de
propriétaires trichent : les prix tombent et les baisser ses prix.
profits sont réduits. Finalement, les profits
réduits amènent les propriétaires à courir le
risque de se refaire confiance, puisqu’ils se
rendent compte qu’il n’est rien de pire que les 8 II y a équilibre de Nash lorsque A prend la
prix et les profits existants. Toutes les stations- meilleure décision possible, compte tenu de la
service augmentent leurs prix et le cycle reprend. décision que prend B, et B prend la meilleure
Cette instabilité du résultat des prix et des décision possible, compte tenu de la décision
profits découle du cycle de confiance et de de A.
méfiance. Autrement dit, l’instabilité découle de
l’interaction stratégique entre les propriétaires de
stations-service.
9 S’ils concluent une entente de collusion, les
duopoleurs peuvent (ensemble) réaliser autant
de profits qu’un seul monopole.

AUTOÉVALUATION

Vrai/Faux/Incertain 10 L’équilibre en stratégies dominantes des


(Justifiez votre réponse.) duopoleurs qui ont conclu une entente de
collusion est que les deux entreprises trichent.

I Un ratio de concentration élevé indique un


faible niveau de concurrence.

Questions à choix multiple

2 La différenciation des produits confère à 1 Le ratio de concentration fondé sur quatre


une entreprise en situation de concurrence entreprises mesure la part
monopolistique un certain pouvoir de a) des profits de l’industrie totale des quatre plus
monopole, s / grandes entreprises.
b) des ventes de l’industrie totale des quatre plus
grandes entreprises.
c) du coût de l’industrie totale des quatre plus
3 L’entrée libre est une caractéristique clé de la grandes entreprises.
concurrence monopolistique qui produit une d) du capital de l’industrie totale des quatre plus
capacité excédentaire. grandes entreprises.
e) Aucune de ces réponses.

2 Pour une entreprise en situation de concurrence


4 Le modèle de la courbe de demande coudée monopolistique en équilibre à long terme,
prédit que le prix et la quantité seront sensibles a) P = Cm.
à de faibles variations des coûts. b) Cm = CTM.
(ç.Y)RM = CTM, mais P > Cm.
a) Cm = RM, mais CTM > RM.
e) Aucune de ces réponses.
5 Lorsqu’une industrie en situation de concurrence
monopolistique est en équilibre à long terme, les 3 La figure 13.1 représente une entreprise en
profits économiques sont nuis et le prix est égal situation de concurrence monopolistique en
au coût total moyen minimal. équilibre à court terme. Quel est le niveau de
production de l’entreprise?

§) Qi n
©>02 ^
6 La publicité faite par des entreprises en situation c) Q
de concurrence monopolistique entraîne une d) Q4
allocation inefficiente. e) zéro
LA CONCURRENCE MONOPOLISTIQUE ET L'OLIGOPOLE 177

FIGURE 13.1 7 Lequel des facteurs suivants n’est pas un facteur


commun à tous les jeux?
a) Les règles
b) La collusion
c) Les stratégies
d) Les gains
e) L’analyse de l’interaction stratégique

8 Dans le jeu du dilemme du prisonnier avec


Alain et Bernard, la meilleure stratégie de
chaque prisonnier serait
a) que les deux prisonniers avouent.
b) que les deux prisonniers nient.
c) qu’Alain nie et que Bernard avoue.
d) que Bernard nie et qu’Alain avoue.
e) Aucune de ces réponses.

9 La fixation de prix limite évoque


4 La figure 13.1 présente une entreprise en a) le prix le plus élevé que peut imposer un
situation de concurrence monopolistique en monopole.
équilibre à court terme. Quel est le profit b) une entreprise qui baisse son prix de
économique par unité de l’entreprise ? monopole pour empêcher l’entrée d’autres
©p4-p2 entreprises.
b )P4~Pl c) une stratégie utilisée par les entreprises qui
c) A entrent sur des marchés contestables.
d )P3 d) le prix déterminé par un modèle de courbe
e) Aucune de ces réponses. de demande coudée.
e) Aucune de ces réponses.
5 La figure 13.1 présente une entreprise en
situation de concurrence monopolistique en 10 Lequel des énoncés suivants ne se rapporte pas
équilibre à court terme. À long terme, à une limite des ratios de concentration en tant
(S>de nouvelles entreprises entreront dans que mesure de compétitivité de l’industrie ?
l’industrie, et la courbe de demande de a) Les ratios de concentration sont des mesures
chaque entreprise existante se déplacera vers nationales, mais les entreprises de certaines
la gauche. industries sont exploitées au niveau régional.
b) de nouvelles entreprises entreront dans b) Les ratios de concentration sont des mesures
l’industrie, et la courbe de demande de nationales, mais les entreprises de certaines
chaque entreprise existante se déplacera vers industries sont exploitées au niveau
la droite. international.
c) les entreprises existantes quitteront l’industrie, c) Les ratios de concentration ne nous
et la courbe de demande de chaque entreprise renseignent pas sur l’importance des barrières
existante se déplacera vers la gauche. à l’entrée dans l’industrie.
d) les entreprises existantes quitteront 1 industrie, d) Les ratios de concentration ne nous
et la courbe de demande de chaque entreprise renseignent pas sur la manière dont les coûts
restante se déplacera vers la droite. varient d’une firme à l’autre dans l’industrie.
e) il n’y aura pas de changement par rapport au e) Les ratios de concentration permettent
court terme. difficilement de classer les entreprises
multiproduits par industrie.

6 Dans le modèle d’oligopole à entreprise


dominante, les entreprises plus petites se I I Dans une situation de concurrence
monopolistique, les profits économiques à long
comportent comme
terme ont tendance à se rapprocher de zéro en
a) des oligopoles.
raison de
b) des monopoles.
a) la différenciation des produits.
c) des concurrents monopolistiques.
(Bpl’absence de barrières à l’entrée,
d) des concurrents parfaits.
c) la capacité excédentaire.
e) un cartel.
178 CHAPITRE 13

d) l’inefficience. 16 Considérons le même cartel composé d’Électruc


e) la courbe de demande à pente négative et d’Apparalec. Maintenant, le jeu est répété
à laquelle chaque entreprise fait face. indéfiniment et chaque entreprise emploie une
stratégie de coup pour coup. Il y a équilibre
I2 Chacune des caractéristiques suivantes lorsque
appartient à la concurrence monopolistique. a) aucune des deux entreprises ne se conforme à
Laquelle n’évoque pas un oligopole ? l’entente.
a) Chaque entreprise fait face à une courbe de b) les deux entreprises se conforment à l’entente.
demande à pente négative. c) Électruc triche alors qu’Apparalec se
b) Les entreprises maximisent les profits. conforme à l’entente.
c) Les ventes d’une entreprise n’auront pas d) Apparalec triche alors qu’Électruc se
d’effet sensible sur d’autres entreprises. conforme à l’entente.
d) Il y a plus d’une entreprise dans l’industrie. e) Réponse indéterminée.
e) Les entreprises fixent les prix.
I7 L’équilibre décrit à la question 16 est
13 Si un duopole ayant conclu une entente de a) un équilibre de stratégie crédible.
collusion maximise son profit, b) un équilibre de joueur dominant.
a) chaque entreprise doit produire la même c) un équilibre de duopole.
quantité. d) un équilibre de stratégie de déclic.
b) chaque entreprise doit produire la quantité e) un équilibre coopératif.
maximale possible.
c) la recette marginale de l’industrie doit être I8 Lequel des éléments suivants se rapporte
égale au coût marginal de l’industrie au à la concurrence parfaite, à la concurrence
niveau de production total. monopolistique et au monopole non
d) la demande de l’industrie doit être égale au discriminant ?
coût marginal de l’industrie au niveau de a) Un produit homogène
production totale. b) Des profits économiques à long terme de zéro
e) la production totale sera supérieure à ce (a) La quantité qui permet de maximiser les
quelle serait s’il n’y avait pas entente de profits à court terme lorsque Cm = Rm
collusion. d) La facilité d’entrée et de sortie
e) Aucune de ces réponses.
14 À long terme, une entreprise en situation de
concurrence monopolistique réalisera un profit 19 Lequel des énoncés suivants sur les segments de
économique égal à celui la courbe de demande coudée de la figure 13.2
a) d’une entreprise en situation de concurrence est correct?
monopolistique à court terme. a) Selon AB, de nouvelles entreprises entreront
b) d’un membre de cartel. dans l’industrie alors que, selon BC, aucune
c) d’un monopole pur qui pratique la nouvelle entreprise n’y entrera.
discrimination par les prix. b) Selon AB, aucune nouvelle entreprise
d) d’une entreprise parfaitement concurrentielle. n’entrera dans l’industrie alors que, selon BC,
e) Aucune de ces réponses. de nouvelles entreprises y entreront.
c) La discontinuité entre les segments reflète les
I5 Les entreprises Électruc et Apparalec concluent imperfections du marché.
une entente de collusion pour maximiser leurs d) Selon AB, d’autres entreprises suivront une
profits. Si ce jeu n’est pas répété, l’équilibre en augmentation de prix alors que, selon BC,
stratégies dominantes est le suivant :
a) les deux entreprises ne respectent pas les
termes de l’entente.
® d’autres entreprises ne suivront pas une baisse
de prix.
Selon AB, d’autres entreprises ne suivront pas
b) les deux entreprises respectent les termes une augmentation de prix alors que, selon
de l’entente. BC, d’autres entreprises suivront une baisse
c) Electruc triche et Apparalec se conforme de prix.
à l’entente.
d) Apparalec triche alors qu’Électruc se
conforme à l’entente.
e) Réponse indéterminée.
LA CONCURRENCE MONOPOLISTIQUE ET L’OLIGOPOLE 179

22 Reportez-vous au jeu non répété du tableau


FIGURE 13.2
13.1. En équilibre, quels sont les profits de
l’entreprise A ?
a) -10 $
b) 2$
c) 10$
d) 20$
e) Réponse indéterminée.

23 Reportez-vous au jeu non répété du tableau


13.1. Si les deux entreprises pouvaient conclure
une entente de collusion, quels seraient les
profits de l’entreprise A ?
a) -10$
b) 2$
c) 10$
20 À long terme, l’entreprise en situation de d) 20$

concurrence monopolistique e) Réponse indéterminée.


a) fera face à une courbe de demande
parfaitement élastique. 24 À long terme, le niveau de production d’une

b) produira au-delà de la quantité qui entreprise en situation de concurrence

correspond au CTM minimum. monopolistique sera tel que le prix équivaudra

c) produira en deçà de la quantité qui a) au coût marginal.

correspond au CTM minimum. b) à la recette marginale.


c) au coût variable moyen.
d) produira la quantité qui correspond au
d) au coût total moyen.
CTM minimum.
e) b et d.
e) réalisera des profits économiques.

25 Laquelle des caractéristiques suivantes «est pas


21 Le tableau 13.1 présente la matrice des gains
commune au monopole non discriminant et
relatifs aux profits des entreprises A et B, lorsque
à la concurrence monopolistique ?
celles-ci peuvent adopter deux stratégies : 1 ) fixer
a) Des entreprises faisant face à une courbe de
un prix peu élevé ou 2) fixer un prix élevé. Dans
demande à pente négative
ce jeu (joué une seule fois), l’équilibre est un
b) La quantité maximisant les profits lorsque
équilibre en stratégies dominantes puisque
Cm = Rm
a) l’entreprise B réduira davantage son profit
c) Le CTM d’équilibre supérieur au CTM
que l’entreprise A si toutes les deux fixent un
minimal
prix moins élevé.
d) Des profits économiques à long terme positifs
b) l’entreprise B est l’entreprise dominante.
e) Des profits normaux à long terme positifs
c) la meilleure stratégie pour chaque entreprise
ne dépend pas de la stratégie adoptée par
l’autre. Problèmes à court développement
d) en ne respectant par son entente, aucune
entreprise ne menace l’autre de maniéré
1 Compte tenu de l’étendue géographique des
crédible.
marchés, comment un ratio de concentration
e) Toutes ces réponses.
peut-il sous-estimer le degré de compétitivité
d’un marché ? Comment peut-il surestimer
TABLEAU 13.1
le degré de compétitivité ?

Entreprise B
2 Comparer les avantages et les inconvénients
Prix bas Prix élevé de la concurrence parfaite et de la concurrence
monopolistique en ce qui concerne l’allocation
Prix A: 2$ A: 20$
efficiente.
bas B: 5$ B: -15$
Entreprise A
Prix A: -10$ A: 10$ 3 Considérez le cas de deux duopoleurs ayant
élevé B: 25$ B: 20$ conclu une entente de collusion et qui sont
engagés dans un jeu répété. En équilibre, les
180 CHAPITRE 13

entreprises se conformeront-elles ou non à leur


FIGURE 13.4
entente? Expliquez pourquoi.

4 Comment peut-on expliquer par la théorie des CD


X
jeux une guerre des prix qui élimine les profits?

5 Considérez une entreprise dans une industrie


en concurrence monopolistique à court terme.
À l’aide des axes de la figure 13.3, tracez un
graphique pour chacune des situations suivantes.

FIGURE 13.3
0 Quantité

CD
X
b) Si l’entreprise réussit à augmenter ses profits à
Q_
court terme, que se passera-t-il à long terme ?

7 Une petite ville des Prairies compte deux


boulangeries - Au Pain toujours frais et Au Pain
jamais rassis. Les coûts de transport sont élevés
relativement au prix du pain, si bien que les
boulangeries ne subissent pas la concurrence
d’autres villes ; l’industrie locale du pain est un
duopole. Au Pain toujours frais et Au Pain jamais
rassis ont les mêmes courbes de coût, et chacune
0 Quantité réalise un profit annuel de 2 000 $.
Supposons qu’un nouveau service de
publicité, Pub Plus s’installe. Si une boulangerie
a) L’entreprise réalise un profit économique. utilise les services de Pub Plus, ses profits
b) L’entreprise subit une perte qui va la forcer annuels augmenteront pour s’élever à 5 000 $,
à cesser ses activités. tandis que l’autre boulangerie perdra 2 000 $.
c) L’entreprise subit une perte, mais elle Si toutes deux utilisent les services de Pub Plus,
continue ses activités. chacune d’elles réalisera un profit de zéro.
d) En partant de la situation décrite en c, Si aucune d’elles n’utilise les services de Pub
expliquez ce qui se produit dans cette Plus, chaque boulangerie continuera de réaliser
industrie et indiquez quels sont les effets un profit annuel de 2 000 $.
sur le graphique que vous avez tracé en c ? a) Représentez ce duopole sous forme de jeu
(Un nouveau graphique n’est pas requis.) en identifiant les joueurs, les stratégies et
e) L’entreprise est en équilibre à long terme. les résultats possibles.
b) Construisez la matrice des gains.
& 6 Considérez une industrie en concurrence c) Quel est le résultat d’équilibre ? Expliquez.

wM monopolistique en équilibre à long terme.


L’entreprise A de cette industrie tente 8 La figure 13.5(a) présente la courbe de coût total

f,ê d’augmenter ses profits par la publicité,


a) Sur le graphique de la figure 13.4, montrez
les effets à court terme de la publicité.
moyen (CTM), qui est identique poux Au Pain
toujours frais et Au Pain jamais rassis. La figure
13-5 (b) présente la courbe de demande de pain
Expliquez brièvement votre graphique. de la ville et la courbe de coût marginal
conjointe des deux entreprises. Supposons que
les deux boulangeries concluent une entente de
collusion pour maximiser leurs profits et qu elles
consentent à se partager également la production
pendant un an.
a) Quel sera le niveau de production de chaque
boulangerie et à quel prix le pain sera-t-il
vendu ?
b) Quels seront le coût total moyen et le profit
de chaque boulangerie ?
LA CONCURRENCE MONOPOLISTIQUE ET L'OLIGOPOLE 181

FIGURE 13.5

a) Courbe CTM de chaque boulangerie b) Courbe de demande de pain et courbe


conjointe Cm des deux entreprises

Quantité (en milliers de pains par an) Quantité (en milliers de pains par an)

9 Supposez maintenant que Au Pain jamais rassis 2 V Crée une courbe de demande à pente

persuade sa concurrente que la demande a baissé négative.

et qu’elles doivent réduire leur prix de 10 <t par 3 F La différenciation des produits entraîne une

pain pour vendre les quantités sur lesquelles elles pente négative de la courbe de demande.

se sont entendues. Bien entendu, la demande n’a 4 F Insensible aux petits changements de coût.

pas diminué, mais Au Pain toujours frais produit 5 F Profits nuis, mais P > CTM minimal.

la quantité convenue et demande 10 «t de moins & 6 I Cela dépend du contenu de la publicité et


des gains découlant de l’accroissement de la
par pain. Au Pain jamais rassis, le tricheur,
diversité des produits.
demande aussi 10 € de moins qu’avant, mais
7 V L’oligopole concerne le comportement
augmente suffisamment sa production pour
stratégique.
satisfaire la demande résiduelle à ce prix.
8 V Définition.
a) Combien de pains la boulangerie Au Pain
9 V Avec une entente de collusion, ils agissent
jamais rassis produit-elle ?
exactement comme un monopole.
b) Quels sont le coût total et le profit de Au
& 10 I Vrai pour le jeu non répété, mais peut être
Pain toujours frais ?
faux pour le jeu répété.
c) Quels sont le coût total moyen et le profit de
Au Pain jamais rassis ?
Questions à choix multiple
I0 Revenez à la situation initiale. Les entreprises se
préparent à conclure une entente de collusion à
long terme. Au Pain toujours frais persuade sa I b Définition.
concurrente que, si celle-ci s’avisait de tricher, @ 2 c Courbe de demande (RM = P) tangente au
elle baisserait ses prix en deçà des siens dès la CTM et au-dessus de Cm.
découverte de la tricherie. La boulangerie Au 3 b Lorsque Cm = Rm.
Pain jamais rassis voudra-t-elle maintenant ne & 4 a Avec la quantité Q, P — CTM.
pas respecter les termes de l’entente ? Pourquoi 5 a Avec de nouvelles entreprises sur le marché,
ou pourquoi pas ? la demande de l’industrie est répartie entre un
plus grande nombre d’entreprises et la courbe
de demande de chaque entreprise se déplace
RÉPONSES donc vers la gauche.
6 d L’entreprise dominante fixe le prix, les

Vrai/Faux/Incertain entreprises plus petites acceptent ce prix


donné.
(Justifiez votre réponse.)
7 b Pas de collusion dans le dilemme du
prisonnier.
I I Souvent vrai, mais dépend de la zone
géographique du marché, des barrières à 8 b Mais cela ne se produit pas parce que les
l’entrée et des entreprises à marches multiples. joueurs ne peuvent se faire suffisamment
182 CHAPITRE 13

confiance les uns les autres pour conclure concurrence monopolistique entraîne un coût
une entente de collusion. total moyen élevé en raison de la production
9 b Définition, c serait vrai pour les entreprises réduite et des dépenses de publicité importantes.
existantes. La concurrence monopolistique entraîne une
10 d Les ratios de concentration ne servent pas grande variété de produits - ce qu’apprécient les
à mesurer les coûts. consommateurs - alors que, dans une industrie
1 1 b a et e —> possibilité de profits, c et d sont des parfaitement concurrentielle, toutes les
résultats, non des forces —> des profits nuis. . entreprises proposent le même produit. La
& 12 c L’oligopole suppose l’interdépendance entre concurrence monopolistique favorise également
les entreprises. une grande innovation des produits et, effet
13 c Voir la figure 13.6(b) du manuel. positif d’une abondante publicité, elle procure
14 d Profits économiques nuis. une information précieuse aux consommateurs.
15 a Semblable au résultat du dilemme du De ce fait, l’efficience réduite de l’allocation
prisonnier. des ressources et les coûts de publicité plus
& 16 b Équilibre coopératif; chaque joueur répond élevés (CTM plus élevé) associés à la
de manière rationnelle à la menace crédible concurrence monopolistique doivent être
de l’autre. pondérés par rapport aux gains qu’entraînent
17 e Définition. une plus grande diversité des produits, une plus
18 c b et d faux pour le monopole, a faux pour grande innovation dans les produits et une
la concurrence monopolistique. information plus utile pour les consommateurs.
19 e AB plus élastique que BC car les hausses de
prix ne sont pas suivies. La meilleure stratégie de chaque entreprise est
20 c Capacité excédentaire au niveau Q lorsque de tricher, peu importe la stratégie qu’adopte
la courbe de demande est tangente à la l’autre entreprise. Appelons les entreprises A
courbe CTM à pente négative. et B. L’entreprise A sait que, si l’entreprise B
21 c Définition de l’équilibre en stratégies se conforme à l’entente de collusion, A peut
dominantes. augmenter son profit en trichant. Si l’entreprise
22 b Les deux entreprises fixent un prix bas. B triche, alors l’entreprise A sait qu elle doit
23 c Les deux entreprises fixent un prix élevé. aussi tricher pour réduire sa perte de profit
24 d Lorsque la courbe de demande est tangente au minimum. De ce fait, pour l’entreprise A,
à la courbe CTM. la stratégie dominante consiste à tricher.
& 25 d Profits économiques à long terme nuis pour C’est également la stratégie dominante de
la concurrence monopolistique. l’entreprise B.

Selon la théorie des jeux, la guerre des prix est,


Problèmes à court développement
dans une industrie où règne la collusion, la
conséquence de la réponse des entreprises à la
1 Les ratios de concentration sont calculés décision de l’une d’elles de ne pas se conformer
dans une perspective géographique nationale. à l’entente. Si une entreprise triche en baissant
Si la zone réelle de marché n’est pas nationale, son prix, alors toutes les autres entreprises
le ratio de concentration estimera probablement baisseront leur prix et une guerre des prix
faussement le degré de compétitivité d’une s’ensuivra. Lorsque le prix est suffisamment bas
industrie. Si le marché réel est global, le ratio (peut-être au niveau de profit nul), les
de concentration sous-estimera le degré de entreprises seront de nouveau incitées à se
compétitivité. L’entreprise peut avoir un ratio conformer à leur entente de collusion.
de concentration de 100 en tant que seul
producteur de la nation, mais elle fait face à une a) La figure 13.3 (a) Solution illustre le cas de
concurrence internationale considérable. Lorsque l’entreprise en concurrence monopolistique
la zone géographique du marché est régionale, qui réalise un profit économique à court
le ratio de concentration surestimera le degré terme. On remarque sur ce graphique que,
de compétitivité. Le ratio de concentration au niveau de production qui permet de
comprend les entreprises d’autres régions du maximiser les profits, le prix est supérieur
pays qui ne sont pas de réels concurrents dans au coût total moyen. Le profit économique
la région. est indiqué par la zone ombrée.

2 La concurrence parfaite entraîne une production


au coût total moyen minimal, alors que la
LA CONCURRENCE MONOPOLISTIQUE ET L’OLIGOPOLE 183

FIGURE 13.3 SOLUTION FIGURE 13.3 SOLUTION


a) c)

d) Étant donné que les entreprises subissent


généralement une perte, elles quitteront
b) La figure 13-3(b) Solution illustre le cas d’une
l’industrie. Cela signifie que les courbes de
entreprise qui va cesser ses activités à court
demande des entreprises restantes se déplacent
terme puisque le prix est inférieur au coût
vers la droite à mesure qu elles attirent les
variable moyen au niveau de production qui
clients des entreprises qui quittent l’industrie.
permet de maximiser les profits (réduire les
À mesure que les courbes de demande des
pertes au minimum).
entreprises se déplacent vers la droite, les
pertes sont réduites. Les entreprises
FIGURE 13.3 SOLUTION
continueront de quitter l’industrie jusqu’à ce
b) que les pertes soient éliminées, et les courbes
de demande des entreprises continueront
donc de se déplacer jusqu’à ce quelles soient
tangentes à la courbe CTM.
e) La figure 13.3 (d) Solution illustre une
situation courante d’entreprise en concurrence
monopolistique en équilibre à long terme. On
note principalement que la courbe de
demande à laquelle fera face l’entreprise est
tangente à la courbe CTM au niveau de
production qui permet de maximiser les
profits. Par conséquent, l’entreprise réalise un
profit économique nul.

FIGURE 13.3 SOLUTION

d)
c) La figure 13.3 (c) Solution illustre le cas
d’une entreprise qui subit une perte mais qui
continue ses activités. La perte est indiquée
par la zone ombrée. Remarquez que, au
niveau de production qui permet de
maximiser les profits, le prix est inférieur au
CTM, mais supérieur au CVM.
184 CHAPITRE 13

©6 a) Les frais de publicité augmenteront le coût toujours frais fait de la publicité, mais pas Au
de l’entreprise A, mais également la demande Pain jamais rassis., 3) Au Pain jamais rassis fait
(ce qu’elle espère). Si l’augmentation de la publicité, mais pas Au Pain toujours frais
de la demande (recette) est supérieure et 4) aucune des deux ne fait de publicité,
à l’augmentation du coût, l’entreprise A b) La matrice des gains est présentée au tableau
aura alors augmenté son profit. La figure 13.2. Les chiffres donnent le profit réalisé par
13.4 Solution illustre cette situation. Les Au Pain toujours frais (TF) et Au Pain jamais
courbes initiales sont D0, CTM0 et Rm0. rassis (JR) pour chaque résultat possible.
Au départ, le niveau de production de
l’entreprise est Qo> elle vend au prix P0 et TABLEAU 13.2
réalise un profit économique nul. Les frais Au Pain jamais rassis
de publicité, qui sont des coûts fixes, font Publicité Pas de publicité
monter la courbe CTM vers CTMX {Cm ne
se déplace pas), mais également les courbes Publicité TF: 0 TF: 5 000$

de demande et de recette marginale vers Dx Au Pain JR: 0 JR: -2 000 $


toujours
et Rmv respectivement. Le déplacement de TF: -2 000 $ TF: 2 000$
frais Pas de
la courbe de demande est suffisamment publicité JR: 5 000$ JR: 2 000$
important pour qu’il y ait maintenant un
profit économique positif au nouveau niveau
de production qui permet de maximiser les c) Considérons d’abord comment Au Pain
profits (Qj). Le profit économique est toujours frais décide de la stratégie à adopter.
indiqué par la zone hachurée. Si Au Pain jamais rassis fait de la publicité. Au
Pain toujours frais peut faire de la publicité et
FIGURE 13.4 SOLUTION réaliser un profit nul ou bien ne pas faire de
publicité et subir une perte de 2 000 $. Par
conséquent, Au Pain toujours frais souhaitera
faire de la publicité si c’est ce que fait Au Pain
jamais rassis.
Si Au Pain jamais rassis ne fait pas de
publicité, Au Pain toujours frais peut faire de
la publicité et réaliser un profit de 5 000 $ ou
bien ne pas faire de publicité et réaliser un
profit de 2 000 $. Par conséquent Au Pain
toujours frais souhaitera faire de la publicité
que Au Pain jamais rassis en fasse ou pas.
Au Pain jamais rassis parviendra à la même
conclusion. L’équilibre en stratégies
dominantes est que les deux entreprises
fassent de la publicité.

b) Nul doute que d’autres entreprises se 8 a) Les entreprises conviendront de produire


mettront aussi à faire de la publicité pour 30 000 pains chacune, quelles vendront
pouvoir augmenter leur profit (ou récupérer 50 <£ l’unité. Nous déterminons cela en
leur profit perdu). Puisque toutes les autres remarquant (figure 13.5 b) que le niveau
entreprises font de la publicité, la courbe de production qui permet de maximiser les
de demande de notre entreprise se déplacera profits (monopole) est de 60 000 pains pour
de nouveau vers la gauche à mesure quelle l’industrie {Rm — Cm avec 60 000) et que le
perd certains des clients quelle avait gagnés. prix de l’industrie est de 50 <t le pain. Puisque
A long terme, toutes les entreprises réaliseront les entreprises ont convenu de se répartir
de nouveau des profits nuis, même après avoir également la production, chacune d’elles
fait de la publicité. produira 30 000 pains,
b) À partir de la figure 13.5 (a), nous
7 a) Les joueurs sont Au Pain toujours frais et déterminons que, avec 30 000 pains, le coût
Au Pain jamais rassis. Chaque entreprise a total moyen de chaque entreprise est de 40 <t
• deux stratégies: faire de la publicité ou non. par pain. Puisque le prix est de 50 <t par pain,
Il y a quatre résultats possibles: 1) les deux chaque entreprise réalisera un profit
entreprises font de la publicité ; 2) Au Pain de 3 000 $.
LA CONCURRENCE MONOPOLISTIQUE ET L'OLIGOPOLE 185

0 9 a) Au nouveau prix de 40 <t le pain, la quantité 10 Étant donné qu’il s’agit d’une entente à long
totale demandée est de 80 000 pains. Comme terme, Au Pain jamais rassis ne devrait pas
Au Pain toujours frais continue de produire chercher à tricher. En effet, bien que Au Pain
30 000 pains, cela signifie que Au Pain jamais jamais rassis puisse augmenter son profit à court
rassis produira les 50 000 pains restants terme en trichant, elle perdrait par la suite
demandés. beaucoup plus si Au Pain toujours frais usait
b) Puisque Au Pain toujours frais continue de de représailles.
produire 30 000 pains, son coût total moyen Ën fait, et c’est le cœur du problème,
demeure à 40 <t le pain. À ce prix, Au Pain si le comportement de Au Pain jamais
toujours frais réalisera un profit économique nul. rassis change, ce n’est pas parce que
c) Au Pain jamais rassis a augmenté la le coût ou la demande a changé, mais
production, qui est maintenant de 50 000 parce que le comportement de Au Pain
pains, ce qui donne un coût total moyen de toujours frais a changé. Dans le duopole
30 é le pain. De ce fait, avec un prix de 40 <t, et l’oligopole, la meilleure stratégie de
le profit économique de Au Pain jamais rassis chaque entreprise dépend du comportement
sera de 5 000 $. des autres entreprises.
Révision
e
2 PARTIE

Chapitres 7 à 13

PROBLÈME

FIGURE R2.I
(a) Industrie (b) Entreprise

Bien que la concurrence parfaite, le monopole de coût de l’une des nombreuses entreprises
et la concurrence monopolistique soient identiques de l’industrie.
des structures de marché qui présentent Supposons d’abord qu’il s’agit d’une
d’importantes différences, les graphiques industrie en situation de concurrence parfaite
de la figure R2.1 peuvent permettre de les (cp).
analyser.
La figure R2.1(a) présente une industrie a) Décrivez la courbe de demande d’une
en équilibre à court terme à l’intersection entreprise ; décrivez la courbe de recette
des courbes de demande (D0) et d’offre (O0). marginale.
Ne tenez pas compte pour le moment b) Sur la figure R2.1, déterminez le prix
de l’autre courbe d’offre (Oj) ni de la courbe d’équilibre à court terme de l’industrie (Py)
de recette marginale (Rm). La demande (on vous donne pratiquement la réponse!),
du marché à long terme se confond avec la production de l’industrie (Q^) et la
la demande du marché à court terme. Il n’y production de l’entreprise («JÜ, et indiquez-les.
a ni économies ni déséconomies externes. Chaque entreprise réalise-t-elle un profit ou
La figure R2.1(b) indique les courbes subit-elle une perte ? Expliquez.
CHAPITRES 7 À 13 187

c) Que se produit-il à long terme ? Sur la figure


R2.1, déterminez le prix d’équilibre à long EXAMEN DE MI-ÉTAPE
terme de l’industrie (PLT), la production de
l’industrie (Q77) et la production d’une Allouez 56 minutes pour cet examen (28 questions,
entreprise (qLTcp), et indiquez-les. 2 minutes par question). Pour chaque question,
Supposons maintenant que le marché choisissez la meilleure réponse.
de la figure R2.1(a) est approvisionné par
un monopole légal non discriminant (m). 1 L’utilité marginale est égale
Oj est la courbe de coût marginal a) à l’utilité totale divisée par le prix.
du monopoleur. Ne tenez pas compte b) à l’utilité totale divisée par le nombre total
de 0Q ni de la partie (b) de la figure. d’unités consommées.
d) Décrivez la courbe de demande ainsi c) à la pente de la courbe d’utilité totale.
que la courbe d’offre du monopoleur. d) à l’inverse de l’utilité totale.
e) Sur la figure R2.1(a), déterminez le prix e) à la partie située sous la courbe de demande
d’équilibre à court terme de l’industrie (P ) mais au-dessus du prix du marché.
et la production (Q^), et indiquez-les.
f) À la quantité Q^, si le coût total moyen 2 Un consommateur maximise son utilité
du monopoleur est inférieur à P , expliquez en achetant 2 unités du bien X au prix de 5 $
ce qui se produit à long terme. Sur la figure l’unité et 3 unités du bien Y au prix de 7 $
R2.1(a), déterminez le prix d’équilibre à long l’unité. Quel est le ratio de l’utilité marginale
terme de l’industrie (P77 ) et la production de X à l’utilité marginale de Y?
(Q77' ), et indiquez-les. a) 5/7
g) Comparez l’efficience de l’allocation b) 7/5
des ressources à long terme de cette industrie c) 2/3
en situation de monopole et en situation d) 3/2
de concurrence parfaite. Indiquez toute perte e) 10/21
sèche sur la figure appropriée.

Supposons enfin que l'industrie 3 La courbe de d’offre d’un monopole


de la figure R2.1(a) est approvisionnée non discriminant est
par de nombreuses entreprises en situation a) sa courbe de coût marginal.
de concurrence monopolistique (cm). b) sa courbe de coût marginal située au-dessus
La courbe d’offre de l’industrie est O0 et, de son coût variable moyen minimal.
au prix P , chaque entreprise est en équilibre c) sa courbe de coût variable moyen.
à long terme. Ne tenez pas compte de Oj d) sa courbe de recette marginale.
ni de Rm. Chaque entreprise a les courbes e) Aucune de ces réponses.
de coût que présente la figure R2.1(b), mais
fabrique un produit légèrement différencié. 4 Une entreprise discriminante prospère
doit pouvoir
h) Décrivez la courbe de demande d une
a) éviter la revente par les consommateurs.
entreprise et tracez-la sur la figure R2.1 (b).
b) différencier les consommateurs ayant une
i) Sur la figure R2.1 (b), déterminez
élasticité-prix de la demande élevée et exiger
la production à long terme (qcm) d une
d’eux des prix peu élevés.
entreprise en situation de concurrence
c) différencier les consommateurs ayant une
monopolistique et indiquez-la. Décrivez
faible élasticité-prix de la demande et exiger
un point que vous pouvez déterminer avec
d’eux des prix élevés.
précision sur la courbe de recette marginale
d) Toutes ces réponses.
d’une entreprise. Expliquez pourquoi il s agit
e) Aucune de ces réponses.
d’un équilibre à long terme.
j) Comparez l’efficience de 1 allocation
des ressources de cette industrie en situation 5 La pente de la courbe de coût marginal

de concurrence monopolistique avec est positive en raison

l’efficience de l’allocation des ressources de a) de l’utilité marginale décroissante.

l’industrie en situation de concurrence b) des rendements marginaux décroissants.

parfaite. De quel élément doit-on tenir c) de l’inefficacité technique.


d) de l inefficience économique.
compte dans ce cas et non dans celui d un
e) Aucune de ces réponses.
monopole ?
188 RÉVISION 2e PARTIE

6 En économie, le long terme est le laps de temps 10 La figure R2.2 présente une entreprise
durant lequel en situation de concurrence monopolistique
a) une année ou plus s’écoule. en équilibre à court terme. À long terme, quel
b) tous les facteurs de production sont variables. est le profit économique de l’entreprise par unités
c) tous les facteurs de production sont fixes. a) P4 - P2
d) il y a au moins un facteur de production fixe et b) P4 - Pi
au moins un facteur de production variable. c) P3
e) tous les facteurs de production sont variables, •' d )P2
mais la taille de l’installation est fixe. Aucune de ces réponses. 2YC-Q
7 Supposons que l’on mesure le bien X en abscisse
et le bien Y, en ordonnée. Le taux marginal I I Pour un monopoleur non discriminant,
de substitution est quel énoncé suivant est faux ?
a) le prix relatif du bien X en unités du bien Y a) Il n’existe pas de relation unique et univoque
b) le prix relatif du bien Y en unités du bien X. entre le prix et la quantité offerte.
c) le degré auquel un consommateur renoncera b) Pour toute production supérieure à zéro,
au bien Y pour obtenir une plus grande Rm < RM.
quantité du bien X et en retirer la même c) La courbe de demande de l’industrie
satisfaction. se confond avec la courbe de demande
d) le degré auquel un consommateur renoncera du monopoleur.
au bien X pour obtenir une plus grande d) Toute l’information nécessaire pour
quantité du bien Y et en retirer la même déterminer la quantité et le prix
satisfaction. qui permettent de maximiser les profits est
e) la pente de la droite de budget. donnée à l’intersection de Rm et de Cm.
e) La recette totale atteint son point maximal
8 Lorsque le prix d’un bien varie, on appelle la lorsque Rm = 0.
variation de la consommation qui laisse
le consommateur indifférent I2 Un monopole non discriminant ne se déplace
a) l’effet d’utilité. jamais
b) l’effet de substitution. a) sur la partie de la courbe de demande
c) l’effet de revenu. où la demande est élastique.
d) l’effet de prix. b) sur la partie de la courbe de demande
e) l’effet inférieur. où la demande est d’élasticité unitaire.
9 La figure R2.2 présente une entreprise f ch sur la partie de la courbe de demande
en situation de concurrence monopolistique où la demande est inélastique.
en équilibre à court terme. Quel est le prix d) sur la partie de la courbe de demande
de l’entreprise ? correspondant à la quantité où la recette
a) Pi marginale est positive, étant donné
b )P2 que la recette totale n’est pas maximale.
in e) Aucune de ces réponses.

e) P5 I3 Suzanne maximise son utilité dans


sa consommation de manteaux de vison
FIGURE R2.2 et de Porsches. Si l’utilité marginale du dernier
manteau de vison quelle a acheté est deux fois
l’utilité marginale de la dernière Porsche quelle
a achetée, nous savons alors avec certitude que
a) Suzanne achète deux fois plus de manteaux
de vison que de Porsches.
b) Suzanne achète deux fois plus de Porsches
que de manteaux de vison.
c) Suzanne achète plus de Porsches que
de manteaux de vison, mais nous ne savons
pas dans quelle proportion.
d) Le prix d’un manteau de vison est deux fois
celui d’une Porsche.
e) Le prix d’une Porsche est deux fois celui
d’un manteau de vison.
CHAPITRES 7 À I 3 189

14 La différence entre la valeur d’un bien et le prix I9 Quel(s) aspect(s) de l’équation de budget
est appelée une variation du revenu modifie-t-elle ?
a) demande excédentaire. a) La pente et l’ordonnée à l’origine
b) offre excédentaire. b) La pente et l’abscisse à l’origine
c) surplus du consommateur. c) L’abscisse et l’ordonnée à l’origine, mais
d) excès du consommateur. pas la pente
e) utilité marginale. d) La pente seulement
e) Aucune de ces réponses.
15 Si la valeur actuelle de 100 $ qui seront versés
dans un an est de 80 $, quel est le taux d’intérét 20 Lorsq ue le prix d’un bien normal augmente,
annuel? l’effet de revenu
a) 50 % a) fait augmenter la consommation du
b) 10% bien et l’effet de substitution fait baisser
c) 20 % la consommation.
d) 25 % b) fait baisser la consommation du bien
e) 8 % et l’effet de substitution fait augmenter
la consommation.
16 Une entreprise a 200 $ de coûts explicites c) et l’effet de substitution font tous deux
et vend la production résultante à 250 $. augmenter la consommation du bien.
Le taux normal de profit est de 10%. Lequel d) et l’effet de substitution font tous deux
des énoncés suivants est vrai? baisser la consommation du bien.
a) Les coûts implicites sont de 25 $. e) est toujours plus grand que l’effet
b) Les profits économiques sont de 20 $. de substitution.
c) Les profits économiques sont de 50 $.
d) Les profits économiques sont supérieurs 21 Si le CFM diminue, alors le Cm doit être
aux profits comptables. a) croissant.
e) Les coûts explicites sont supérieurs b) décroissant.
aux coûts implicites. c) supérieur au CFM.
d) inférieur au CFM.
I7 Dans une industrie parfaitement concurrentielle e) Aucune de ces réponses.
de 100 entreprises, la courbe de demande d’une
entreprise 22 Si tous les facteurs de production augmentent
a) est à élasticité unitaire. de 10% et la production, de plus de 10%, c’est
b) est identique à la courbe de demande donc que
de l’industrie. a) le coût total moyen est décroissant.
c) correspond à 1/100 de la courbe de demande b) le coût total moyen est croissant.

t de l’industrie.
est telle que RM = Rm.
Aucune de ces réponses.
c) la courbe de CMLT a une pente positive.
^J)il y a des rendements d’échelle croissants.
e) il y a des rendements d’échelle décroissants.

I8 Si les entreprises d’une industrie concurrentielle 23 Dans le tableau R2.1, quelle(s) méthode(s) de
réalisent des profits économiques, de nouvelles production d’un hologramme médical est(sont)
entreprises entreront dans l’industrie. Cela fera techniquement efficace(s) ?
déplacer a) 1
a) la courbe de demande de l’industrie vers b) 2
la gauche, d’où la baisse du prix du marché, c) 3
b) la courbe de demande de 1 industrie vers d) Toutes ces réponses
la droite, d’où l’augmentation du prix e) 1 et 3 seulement

du marché.
c) la courbe d’offre de l’industrie vers la gauche,
d’où l’augmentation du prix du marché.
^^)la courbe d’offre de 1 industrie vers la droite,
d’où la baisse du prix du marché,
e) Aucune de ces réponses.
190 RÉVISION 2e PARTIE

28 La courbe d’offre d’une industrie concurrentielle


TABLEAU R2.I TROIS MÉTHODES
à long terme aura une pente positive
DE PRODUCTION D'UN
a) en présence d’économies externes.
HOLOGRAMME MÉDICAL
b) en présence de déséconomies externes.
c) en l’absence d’économies ou de déséconomies
Quantités de facteurs
externes.
de production
d) en présence de coûts externes.
Méthode Travail Capital e) en présence d’avantages externes.

1 5 10
2 10 15
3 15 5
RÉPONSES

Problème
24 Reportez-vous au tableau R2.1. Si le prix
du travail est de 20 $ l’unité et le prix du capital, a) La courbe de demande d’une entreprise et
de 10 $ l’unité, quelle(s) méthode(s) est(sont) la courbe de recette marginale sont une ligne
économiquement efficiente(s) ? horizontale au prix Pcp.
a) 1 b) Voir la figure R2.1 (b) Solution. Chaque
b) 2 entreprise réalise des profits économiques
c) 3 parce que, au volume de production qcp,
d) 2 et 3 seulement le prix (Pcp = recette marginale) est supérieur
e) 1 et 3 seulement au coût total moyen.
c) Voir la figure R2.1 Solution. En réponse
25 Le modèle de la courbe de demande coudée à la réalisation de profits économiques, de
a) indique que le prix reste constant même nouvelles entreprises entrent dans l’industrie,
en présence de fluctuations de la demande. ce qui fait déplacer la courbe d’offre de
b) indique comment le prix courant l’industrie vers la droite jusqu’à ce quelle
est déterminé. atteigne 01. Lorsque le prix est tombé à PLT,
c) suppose que la recette marginale augmente les profits économiques ont été éliminés, et
parfois avec la production. chaque entreprise réalise des profits normaux
d) suppose que les concurrents baisseront seulement.
les prix et ne tiendront pas compte d) La courbe de demande du monopoleur
des augmentations de prix. se confond avec la courbe de demande
e) Aucune de ces réponses. de l’industrie D0. Le monopoleur n’a pas
de courbe d’offre puisqu’il peut choisir
26 Dans le dilemme du prisonnier avec une combinaison de prix et de quantité.
les joueurs Alain et Bernard, l’équilibre e) Voir la figure R2.1 (a) Solution.
en stratégie dominante est le suivant : f) Même si le monopoleur réalise des profits
a) les deux prisonniers avouent. économiques, rien ne se passe à long terme
b) les deux prisonniers nient. parce que les barrières légales empêchent les
c) Alain nie et Bernard avoue. nouvelles entreprises d’entrer dans l’industrie.
d) Bernard nie et Alain avoue. La production et le prix d’équilibre à long
e) Indéterminé. terme sont les mêmes que le prix à court
terme (Pm) et la production (QQ.
27 Dans une industrie parfaitement concurrentielle,
g) Le monopole est moins efficient que la
le prix du marché est de 5 $. Une entreprise
concurrence parfaite, ce qui est représenté par
produit le volume de production auquel le
la partie correspondant à la perte sèche
coût marginal est de 5 $ et croissant, et auquel
indiquée à la figure R2.1(a) Solution.
le coût total moyen est de 25 $. Que devrait
h) Voir la figure R2.1 (b) Solution. Étant donné
faire l’entreprise pour maximiser ses profits
que la production est en équilibre à long
à court terme ?
terme, la courbe de demande à pente
a) Cesser ses activités
descendante doit être tangente à la courbe
b) Augmenter sa production
CTM au prix d’équilibre Pcp.
c) Diminuer sa production
-d) Conserver le même volume de production
e) L’information n’est pas suffisante pour que
l’on puisse répondre à la question.
CHAPITRES 7 À 13 19!

FIGURE R2.I SOLUTION

(a) Industrie (b) Entreprise

Quantité

i) Voir la figure R2.1 Solution. Une entreprise 9 d Le prix le plus élevé possible pour vendre Q.
qui maximise ses profits choisit la production 10 e Profit économique à long terme par unité
{q^) correspondant au point où Cm croise = zéro.
Rm. De ce fait, la courbe Rm doit croiser la @11 d Besoin de la courbe de demande pour
courbe Cm à qm. Il s’agit d’un équilibre à déterminer le prix.
long terme puisque l’entreprise réalise un 12 c Si la demande est inélastique —> Rm < 0.
profit économique nul, et qu’il n’y a donc pas Mais Rm doit toujours être > 0 pour croiser
d’incitation à entrer dans l’industrie ni à en Cm (positive).
sortir. 13 d Selon la condition maximale d’égalité Um/P.
j) Dans une situation de concurrence Il n’est pas nécessaire qu’il y ait de relation
monopolistique, le CTM est plus élevé (P^) entre Um et la quantité.
que dans une situation de concurrence 14 c Définition.
parfaite (PLTcp), ce qui suppose une efficience 15 d 80 $ = 100 $/(l + r). Résoudre pour r.
moins grande. Toutefois, la perte d’efficacité 16 e Coûts implicites = 0,10 (200 $) = 20 $;
dans l’allocation des ressources de la profits économiques = 30 $ ; profits
concurrence monopolistique doit être comptables = 50 $.
pondérée en fonction du gain obtenu sur le 17 d Horizontale au prix du marché. Élasticité
plan de la variété. infinie.
18 d Les profits économiques/pertes amènent
les déplacements de l’offre. Profits —» entrée
de nouvelles entreprises.
EXAMEN DE M I - É TA P E Ô 19 c Voir le point 2 des Rappels du chapitre 8.
A revenu (y) ne change pas la pente mais
1 c Um - A utilité totale/A quantité. fait A abscisse et ordonnée à l’origine
2 a Quantités non pertinentes. Les ratios de (y/Pf) ety/Ps).
l’Um doivent être égaux aux ratios des prix. 20 d Travaillent tous deux dans le même sens.
3 e Le monopole n’a pas de courbe d offre. T P —> si consommation.
4 d Voir le texte du manuel.
21 e CFM toujours -l ; pas nécessairement
5 b i rendements marginaux = -f Pm T Cm. de relation avec Cm. Voir la figure 10.5(b)
6 b Définition. Tous les facteurs de production du manuel.

et la taille de 1 installation sont variables. 22 d Définition. Puisque tous les facteurs de

7 c Définition, a et b se rapportent a la pente production sont variables, a et b ne sont


de la droite de budget. TmS = e au meilleur pas pertinents.

point accessible seulement.


23 e 2 utilise plus de travail et plus de capital que 1.
24 a 1 coûte 200 $ alors que 2 et 3 coûtent 350 $.
8 b Définition.
192 RÉVISION 2e PARTIE

25 d a vrai si Cm fluctue. Rm toujours -f avec Qî,


donc c est faux.
26 a Résultat du jeu.
& 27 e L’entreprise au niveau Q où P = Cm, mais elle
perd de l’argent puisque RM < CTM. Besoin
de plus d’information sur CVM pour
déterminer si les réponses a ou d sont
correctes.
28 b T coûts à mesure que l’industrie T Q.
c 1 a ma

14
La détermination du prix
et l’allocation des facteurs
de production

• VPm = Pm X Rm
CONCEPTS CLÉS • Pour une entreprise en situation
de concurrence parfaite, Rm = P
du produit. La VPm est donc =Pm
Les prix des facteurs de production
(production supplémentaire résultant
et les revenus
d’une unité supplémentaire d’un facteur
de production) X T5 du produit.
Les entreprises emploient les services de facteurs
de production (travail, capital, terre, esprit ♦ L’entreprise qui maximise ses profits embauche des
d’entreprise) pour les besoins de leur production. unités supplémentaires d’un facteur de production
jusqu’au point où la VPm = PF. Pour le travail (T),
♦ La demande et l’offre de travail, de capital
il s’agit du point où la VPmT = taux salarial.
et de terre sur les marchés des facteurs déterminent
les prix de ces facteurs (PF) (salaires, intérêt, ♦ La courbe de demande de l’entreprise pour
rente) et leurs quantités (QF). ce facteur est identique à la courbe de la valeur
du produit marginal de ce facteur.
• Revenu des facteurs = (PF X QF)
• Le prix du facteur de production qu’est l’esprit ♦ La courbe de demande de travail de l’entreprise
d’entreprise est le profit normal. Le profit/la se déplace vers la droite si
perte est un revenu résiduel qui revient aux
• T T3 du produit
propriétaires de l’entreprise.
• T prix des autres facteurs de production
♦ T demande du facteur —> T PF, T QF et T revenu. • changement technique T Pm du travail.
X demande du facteur —> X PF, X QF et X revenu. ♦ La courbe de demande de travail du marché -
♦ T offre du facteur —» -f PF, T QF et T revenu somme des quantités de travail demandées
si T)0 > 1 ; X revenu si < 1. par toutes les entreprises pour chaque taux salarial.

♦ si offre du facteur —» î PF, X QF et -f revenu Élasticité


% A quantité de travail demandée
si T)0 > 1 ; T revenu si Tq^, < 1. de la demande =
de travail % À taux salarial

La demande de facteurs de production La demande de travail est plus élastique

La demande de facteurs est une demande derivee, ♦ lorsque le processus de production est à forte
en ce sens que les facteurs sont demandes pour leur proportion de main-d’œuvre (salaires représentant
contribution à l’objectif de maximisation des profits un pourcentage plus élevé du coût total) ;

de l’entreprise et qu elle dépend des contraintes ♦ plus le Pm du travail diminue lentement ;


techniques et des contraintes du marche.
♦ plus l’élasticité de la demande du produit final
♦ La valeur du produit marginal (VPm) est est grande ;
l’augmentation de la recette totale qui resuite
de l’utilisation d’une unité supplémentaire ♦ plus les possibilités de substitution du capital
au travail sont grandes à long terme.
d’un facteur de production.
194 CHAPITRE 14

L’offre de facteurs de production ♦ Le revenu se compose de la valeur de réserve


(partie située sous la courbe d’offre) et de la rente
L’offre de facteurs est déterminée par les décisions économique (partie située au-dessus de la courbe
des ménages relativement à la répartition de leur temps d’offre, mais au-dessous du prix du facteur).
entre les activités de marché (travail) et les activités
♦ Plus d’offre d’un facteur est inélastique, plus la
hors marché (loisirs, production hors marché).
proportion de son revenu qui représente la rente
♦ Lorsque le salaire est supérieur au salaire économique est grande.
de réserve, les ménages offrent leurs services
♦ Distinction entre rente (prix payé au facteur
sur le marché.
de production, terre) et rente économique
♦ Effet de substitution résultant de T salaire (composante du revenu de presque tous les
—> T quantité de travail offerte. facteurs de production).

♦ Effet de revenu résultant de T salaire


—> -1 quantité de travail offerte.
♦ Lorsque le salaire î, la quantité de travail offerte
RAPPELS
T si l’effet de substitution > l’effet de revenu.

♦ La courbe d’offre de travail d’un ménage


1 Ce chapitre offre un aperçu général des
est atypique lorsque l’effet de revenu est > l’effet
caractéristiques communes à tous les marchés
de substitution.
des facteurs de production. Ainsi, si l’on suppose
♦ La courbe d’offre de travail du marché est la que les entreprises cherchent à maximiser leurs
somme horizontale de toutes les courbes d’offre profits, cela signifie quelles emploieront chaque
des ménages. Sa pente est positive dans l’éventail facteur de production jusqu’au point où la valeur
des salaires normaux. du produit marginal est égale au coût marginal
du facteur, que le facteur de production soit
L’offre de capital est déterminée par les décisions
le travail, la terre ou le capital.
que prennent les ménages relativement à l’épargne.
Les ménages fournissent le capital financier que les
entreprises utilisent pour acheter le capital physique.
2 Veillez à bien faire la distinction entre la valeur
du produit marginal d’un facteur de production
♦ La quantité de capital financier offerte T lorsque et la recette marginale d’une unité de production.
Comme nous l’avons vu dans le manuel, on peut
• le revenu courant > le revenu futur.
calculer la valeur du produit marginal d’un
• les taux d’intérêt T.
facteur en multipliant la recette marginale par
♦ La courbe d’offre de capital du marché est la le produit marginal ( VPm = Rm X Pm). Nous
somme horizontale de toutes les courbes d’offre pouvons le faire intuitivement de la manière
des ménages. L’offre à court terme est inélastique. suivante : le produit marginal nous indique
L’offre à long terme est plus élastique. l’augmentation de la production résultant de
l’utilisation d’une unité supplémentaire d’un
♦ Pour une entreprise, l’offre de capital à court
facteur de production, et la recette marginale
terme est fixe. L’offre à long terme est parfaitement
nous indique le revenu supplémentaire résultant
élastique.
de la vente d’une unité produite supplémentaire.
L’offre de terre est fixe et parfaitement inélastique, Par conséquent, en multipliant Pm par Rm, nous
Cependant, pour une entreprise, l’offre de terre est obtenons le montant de revenu supplémentaire
élastique. résultant de l’utilisation d’une unité
supplémentaire d’un facteur de production

Les revenus, la rente économique (la VPm).


et la valeur de réserve
3 Le graphique le plus important de ce chapitre
La rente économique est le niveau de revenu que se trouve à la figure 14.4 du manuel, et il est
reçoit le détenteur d’un facteur de production au-dessus reproduit à la figure 14.1. En utilisant l’exemple
du montant requis pour offrir ce facteur sur le marché. du Lave-auto Pierrot, la figure montre que la
Le revenu requis pour inciter l’offre d’un facteur courbe de demande de travail d’une entreprise
de production est la valeur de réserve. Elle est se confond avec sa courbe de valeur du produit
le coût d’opportunité de l’utilisation d’un facteur. marginal du travail.
LA DÉTERMINATION DU PRIX
ET L'ALLOCATION DES FACTEURS DE PRODUCTION 195

FIGURE 14.1
AUTOÉVALUATION

Vrai/Faux/Incertain
(Justifiez votre réponse.)

I Le profit économique est le prix du facteur


que constitue l’esprit d’entreprise.

2 Si la production du bien A est à forte proportion


de main-d’œuvre, la demande de travailleurs
utilisés dans la production du bien A a tendance
Quantité de travail (en nombre de travailleurs)
à être inélastique.

3 Une augmentation de l’offre de travail augmente


le revenu du travail.

4 Un ménage n’offre aucun travail lorsque le taux


salarial est inférieur à son salaire de réserve.

Quantité de travail (en nombre de travailleurs) 5 La courbe d’offre de travail sera atypique
si l’effet de substitution est supérieur à l’effet
de revenu.
Le graphique (a) de la figure présente la
courbe de la valeur du produit marginal ( VPm)
fondée sur les chiffres du tableau 14.1 de la
page 324 du manuel. Remarquez que les valeurs
6 Si le revenu courant d’un ménage est faible
du produit marginal sont inscrites à mi-chemin
comparativement à son revenu futur, ce ménage
entre les chiffres utilisés pour les calculer. Ainsi,
épargnera très peu.
en passant de 0 travailleur à 1 travailleur, on
obtient une VPm de 20 et on inscrit cette valeur
de 20 sur la courbe de VPm à mi-chemin entre
0 et 1 travailleur.
Le graphique (b) présente la demande
7 La courbe d’offre de capital est extrêmement
élastique.
de travail de Pierrot en demandant combien de
travailleurs supplémentaires Pierrot serait dispose
à engager à d’autres taux salariaux. Étant donne
que Pierrot cherche à maximiser son profit, il
engagera le nombre de travailleurs qui permettra 8 L’offre du marché d’une parcelle de terrain

à la valeur du produit marginal du travail d être particulière est parfaitement élastique.

égale au taux salarial. Si le taux salarial horaire


est de 10 $, par exemple, Pierrot engagera
3 travailleurs puisque la valeur du produit marginal
du travail est de 10 $ lorsque 3 travailleurs sont 9 Si l’offre d’un facteur de production est
engagés. On peut obtenir d autres points de la parfaitement inélastique, son revenu intégral

courbe de demande de la meme manière. Le correspond à sa valeur de réserve.

résultat est que la courbe de demande de travail


se confond avec la courbe de VPm.
196 CHAPITRE 14

10 Si Stéphan Bureau était disposé à lire les nouvelles 4 Lorsqu’une entreprise est un preneur de prix
pour un salaire de 100 000 $ par an, et s’il gagne sur le marché du travail, la courbe de la valeur
2 100 000 $ par an pour le faire, sa rente du produit marginal du travail est également sa
économique serait de 2 000 000 $. a) courbe de coût marginal du travail.
b) courbe de demande de travail.
c) courbe d’offre de travail.
d) courbe d’offre de production.
e) courbe de recette moyenne.
Questions à choix multiple
5 Une entreprise qui maximise ses profits
1 Les prix des facteurs sont les salaires continuera d’employer des unités d’un facteur
des travailleurs, la rente de la terre, de production variable jusqu’à ce que
a) le profit normal résultant du capital a) le coût marginal du facteur soit
et l’intérêt de l’argent. égal au produit marginal.
b) les dividendes résultant du capital et l’intérêt b) le coût marginal du facteur soit
de l’argent. égal à la valeur de la recette moyenne.
c) l’intérêt résultant du capital et le profit c) le coût moyen du facteur soit égal à la valeur
normal de l’esprit d’entreprise. du produit marginal.
d) l’intérêt résultant du capital et le profit d) le coût marginal du facteur soit
économique de l’esprit d’entreprise. égal à la valeur du produit marginal.
e) le profit économique résultant du e) la valeur du produit marginal de ce facteur
capital et le profit normal de l’esprit soit égale à zéro.
d’entreprise.
6 Si le taux salarial augmente, l’effet de substitution
2 Supposons que l’offre d’un facteur de production incitera le ménage à
est très élastique. Une augmentation a) augmenter son salaire de réserve.
de la demande de ce facteur entraîne b) augmenter son activité hors marché
a) une forte augmentation de la quantité et diminuer son activité de marché.
offerte de ce facteur et une faible augmentation c) augmenter son activité de marché et diminuer
de son prix. son activité hors marché.
b) une faible augmentation de la quantité offerte d) augmenter son activité de marché
de ce facteur et une forte augmentation de et son activité hors marché.
son prix. e) diminuer son activité de marché
c) une forte augmentation de l’offre de ce et son activité hors marché.
facteur et une faible augmentation de son
prix. 7 Si le taux salarial augmente, l’effet de revenu
d) une faible augmentation de l’offre incitera le ménage à
de ce facteur et une forte augmentation a) augmenter son salaire de réserve.
de son prix. b) augmenter son activité hors marché
e) une faible augmentation de l’offre et diminuer son activité de marché.
de ce facteur et une faible augmentation c) augmenter son activité de marché et diminuer
de son prix. son activité hors marché.
d) augmenter son activité de marché
3 Une augmentation de l’offre d’un facteur et son activité hors marché.
de production fera e) diminuer son activité de marché
a) augmenter le revenu de ce facteur et son activité hors marché.
si l’élasticité de la demande du facteur
est inférieure à 1. 8 Si le prix de sa production diminue
b) diminuer le revenu de ce facteur si l’élasticité considérablement, une entreprise en situation
de la demande du facteur est inférieure à 1. de concurrence parfaite emploiera
c) augmenter le revenu de ce facteur a) moins de travailleurs, ce qui entraînera
si l’élasticité de l’offre du facteur est la baisse des salaires.
inférieure à 1. b) moins de travailleurs, ce qui entraînera
d) diminuer le revenu de ce facteur si l’élasticité la hausse du produit marginal du travail.
■ de l’offre du facteur est inférieure à 1. c) moins de travailleurs, ce qui entraînera
e) toujours diminuer le revenu de ce facteur. la baisse du produit marginal du travail.
LA DÉTERMINATION DU PRIX
ET L'ALLOCATION DES FACTEURS DE PRODUCTION 197

d) plus de travailleurs, ce qui entraînera 14 La variation de la recette totale résultant de


la hausse du produit marginal du travail. l’emploi d’une unité supplémentaire de capital est
e) plus de travailleurs, ce qui entraînera la baisse a) le produit marginal du capital.
du produit marginal du travail. b) la recette marginale du capital.
c) le coût de recette marginale du capital.
9 Une innovation technique qui entraîne
d) la valeur du produit marginal du capital.
une hausse du produit marginal du travail e) la valeur du produit moyen du capital.
fera déplacer
a) la courbe de demande de travail I5 Toutes autres choses étant égales, plus
vers la gauche. la proportion du coût total provenant
b) la courbe de demande de travail vers la droite. travail est grande,
c) la courbe d’offre de travail vers la gauche. a) plus la demande de travail est élastique.
d) la courbe d’offre de travail vers la droite. b) moins la demande de travail est élastique.
e) b et d. c) plus l’offre de travail est élastique.
d) moins l’offre de travail est élastique.
10 À court terme, une entreprise fait face
e) moins la demande de travail est grande.
à une offre de capital
a) parfaitement élastique.
16 Lequel des énoncés suivants concernant
b) parfaitement inélastique.
la demande de travail à court terme, ceteris
c) à pente positive.
paribus, est vrai ?
d) à pente négative
a) Plus la demande d’un produit est élastique,
e) atypique.
moins la demande de travail est élastique.

11 Si le désir de loisir augmentait, le taux salarial b) Plus le processus de production est à forte
proportion de main-d’œuvre, moins
a) augmenterait et la quantité de travailleurs
la demande de travail est élastique.
embauchés diminuerait.
c) Plus la courbe du produit marginal du travail
b) augmenterait et la quantité de travailleurs
diminue rapidement, moins la demande
embauchés augmenterait.
de travail est élastique.
c) diminuerait et la quantité de travailleurs
d) Toutes ces réponses.
embauchés diminuerait.
e) Aucune de ces réponses.
d) diminuerait et la quantité de travailleurs
embauchés augmenterait.
e) diminuerait et la quantité demandée de
I7 Soit le barème d’offre d’un facteur de production
présenté au tableau 14.1. Si 4 unités de
travailleurs augmenterait.
ce facteur sont offertes au prix de 8 $ l’unité,
I2 À mesure que le taux salarial augmente, à combien la valeur de réserve s’élèvera-t-elle ?
un ménage aura une courbe d’offre de travail a) 8$
atypique si b) 16$
a) l’effet de revenu se déplace dans le même sens c) 20$
que l’effet de substitution. d) 32$
b) le taux salarial s’élève au-dessus du salaire e) Aucune de ces réponses.
de réserve.
c) l’effet de substitution est supérieur TABLEAU 14.1
à l’effet de revenu.
d) l’effet de revenu est supérieur Prix Quantité offerte
à l’effet de substitution. du facteur ( $) du facteur
e) les loisirs sont un bien inférieur. 0 0
2 1
I3 Un exemple de la demande dérivée
4 2
est la demande
6 3
a) de chandails dérivée par un etudiant
8 4
en économie.
10 5
b) de chandails produits à l’aide du travail
et du capital.
c) de travail utilisé dans la production
de chandails.
d) de brosses pour chandails.
e) Aucune de ces réponses.
198 CHAPITRE 14

18 Soit le barème d’offre d’un facteur de production 23 Quel énoncé suivant ne sera pas vrai pour une
donné au tableau 14.1. Si 4 unités de ce facteur entreprise en situation d’équilibre qui maximise
sont offertes au prix de 8 $ l’unité, à combien ses profits ?
s’élève la rente économique ? a) La recette marginale est égale au produit
a) 8$ marginal.
b) 16$ b) La recette marginale est égale au coût
c) 20$ marginal.
d) 32$ c) La recette marginale multipliée par le produit
e) Aucune de ces réponses. marginal d’un facteur est égale au coût
marginal du facteur.
19 Le fait que le revenu courant d’un ménage soit d) La valeur du produit marginal d’un facteur
faible ou élevé comparativement au revenu futur est égale au coût marginal du facteur.
dépend e) La valeur du produit marginal d’un facteur
a) de l’étape du cycle de vie du ménage. divisée par le coût marginal du facteur
b) du fait que le ménage est riche ou pauvre. est égale au produit marginal d’un facteur.
c) du fait que les taux d’intérêt sont élevés
ou bas. 24 Le revenu d’un facteur dont l’offre est
d) du fait que le montant de l’épargne relativement inélastique sera
est bas ou élevé. a) la valeur de réserve seulement.
e) Toutes ces réponses. b) la rente économique seulement.
c) un plus gros montant de valeur de réserve
que de rente économique.
20 La rente économique est
d) un plus gros montant de rente économique
a) le prix payé pour l’utilisation d’un hectare
que de valeur de réserve.
de terre.
e) des montants égaux de valeur de réserve
b) le prix payé pour l’utilisation d’une unité
et de rente économique.
de capital.
c) le revenu requis pour entraîner l’offre d’une
25 Lequel des facteurs suivants n’influe que sur
quantité donnée d’un facteur de production.
l’élasticité à long terme de la demande de travail ?
d) le montant de revenu au-dessus du montant
a) La forte proportion de main-d’œuvre
requis pour entraîner l’offre d’une quantité
du processus de production.
donnée d’un facteur de production.
b) Les possibilités de substituer du capital
e) la valeur de réserve d’un facteur
au travail.
de production
c) L’élasticité de la demande du produit.
d) Le degré de rapidité avec lequel la courbe
2I La courbe d’offre de travail à laquelle fait face de produit marginal du travail diminue.
une entreprise dans un marché parfaitement e) Aucune de ces réponses.
concurrentiel
a) a une pente positive.
b) est atypique. Problèmes à court développement
c) a une pente d’abord positive qui se replie
ensuite vers l’axe des ordonnées à mesure 1 Pourquoi le prix du facteur de production
que le salaire augmente. qu’est le capital ne correspond-il pas au prix
d) est verticale. de l’équipement qu’achète l’entreprise ?
e) est horizontale.

2 Discutez des effets de substitution et de revenu


22 L’effet de revenu d’une hausse de salaire est sur la quantité de travail offerte lorsque le taux
a) le revenu accru que les travailleurs doivent salarial diminue.
recevoir pour être prêts à travailler davantage.
b) l’augmentation du prix des biens de 3 Pourquoi les jeunes ménages ont-ils tendance
consommation provoquée par l’augmentation à épargner moins que les ménages plus âgés ?
du revenu des consommateurs.
c) la demande de loisirs accrue provoquée par 4 Les prix des articles au détail dans Yorkville,
l’augmentation du salaire des consommateurs. le district à la mode de Toronto, sont-ils élevés
d) le pouvoir d’achat accru des consommateurs parce que les loyers sont élevés ou les loyers
qu’entraîne l’augmentation de leur revenu. sont-ils élevés parce que les prix le sont ?
e) Toutes ces réponses. Expliquez votre réponse.
LA DÉTERMINATION DU PRIX
ET L'ALLOCATION DES FACTEURS DE PRODUCTION 199

5 Le tableau 14.2 présente les barèmes du produit 6 Supposez maintenant que la demande du marché
marginal et du produit total d’une entreprise pour la production de l’entreprise étudiée au
qui vend sa production et achète du travail sur problème à court développement 5 diminue,
les marchés concurrentiels. Au départ, le prix entraînant la diminution du prix de production,
auquel l’entreprise peut vendre n’importe quel qui devient de 3 $ l’unité. Les barèmes de produit
volume de production est de 5 $ l’unité et le marginal et de produit total restent inchangés.
taux salarial auquel elle peut acheter n’importe a) Remplissez les deux dernières colonnes vides
quelle quantité de travail est de 15 $ l’unité. du tableau 14.2 en calculant la RT et la VPmT
correspondant au prix de production de 3 $.
TABLEAU 14.2 b) Si le salaire reste à 15 $ par unité de travail,
quelle quantité de travail l’entreprise utilisera-
P= 5$ P = 3$ t-elle pour maximiser son profit ?
Valeur Valeur Quel sera son volume de production ?
Quantité du du
de Produit Recette produit Recette produit c) Le coût fixe total est toujours de 125 $.
travail Production marginal totale marginal totale marginal À combien s’élève le profit ?
(J) (Q) (PmT) (RT) (VPmT) (RT) (VPmT)
d) L’entreprise cessera-t-elle ses activités à court
0 0 terme ? Expliquez votre réponse.
. . .12 e) Tracez un graphique du nouvel équilibre
1 12 du marché du travail.
. . .10
2 22 7 Le prix de production de l’entreprise étudiée
. . . .8 au problème à court développement 6
3 30 est toujours de 3 $, mais le salaire s’élève
maintenant à 21 $ par unité de travail. Les
4 36 barèmes de produit total et de produit marginal
. . . .4 restent les mêmes.
5 40 a) Comment évolue la courbe de demande
...2 de travail (la courbe de VPm du travail) ?
6 42 b) Dans ces conditions, quelle quantité de travail
l’entreprise utilisera-t-elle pour maximiser son
a) Remplissez les deux premières colonnes vides profit ? Quel sera son volume de production ?
du tableau 14.2 en calculant la RT et la VPmT c) Le coût fixe total est toujours de 125 $.
correspondant à un prix de production de 5 $■ À combien s’élève le profit?
b) On peut calculer la valeur du produit d) Tracez un graphique de l’équilibre du marché
marginal du travail (VPmT) en utilisant 1 une du travail.
ou l’autre des formules suivantes :
®& 8 Une entreprise en situation de concurrence
VPmT = ART/AT parfaite et en équilibre à long terme produit des
VPmT = Rm X PmT pastilles à la menthe en utilisant deux facteurs
où A RT = la variation de la recette totale, de production seulement - le travail et le capital.
AT= la variation du travail, Rm = la recette Chaque facteur est vendu dans un marché des
marginale et Pm-j- = le produit marginal du facteurs parfaitement concurrentiel. Le travail
travail. Montrez que ces deux formules sont coûte 30 $ l’unité et le capital, 50 $ l’unité.
équivalentes lorsque la quantité de travail a) En supposant que l’entreprise ait utilisé

varie de 1 à 2 unités. la quantité de capital qui lui permet


c) Si l’entreprise maximise son profit, combien de maximiser son profit, tel qu’indiqué
de travailleurs embauchera-t-elle ? Quel sera à la partie b de cette question, la courbe

son volume de production ? de la valeur du produit marginal du travail est

d) Si le coût fixe total est de 125 $, a combien VPmT = 110 - 8/5 Qr


s’élèvera le profit?
e) Quel est le profit si l’entreprise emploie une où VPmTcst la valeur du produit marginal

unité de travail de plus que la quantité qui lui du travail et Q^, la quantité (en unités)

permet de maximiser son profit ? une unité de travailleurs engagés. Combien d’unités
de travail l’entreprise embauche-t-elle ?
de moins?
b) En supposant que l’entreprise ait utilisé
f) Tracez le graphique de la demande et
de l’offre de travail et illustrez 1 équilibré la quantité de travail qui lui permet
de maximiser son profit, tel qu’indiqué
du marché du travail.
200 CHAPITRE 14 '

à la partie a de cette question, la valeur <s) | q (Jne industrie en situation de concurrence


du produit marginal du capital est parfaite au coût constant est d’abord en équilibre

VPmK = 125 -75/40 Qx à long terme avec PQ et Q, comme le montre la


figure 14.2. Chaque entreprise de l’industrie fait
où VPmK représente la valeur du produit face à la courbe de valeur du produit marginal
marginal du capital et QK, la quantité ( VPmTo ) et à la courbe de salaires représentées
(en unités) de capital utilisé. Combien à la figure 14.3 et elle emploie initialement
d’unités de capital l’entreprise emploie-t-elle? ,~ QT0 unités de travail.
c) Le prix d’une pastille à la menthe est de 10 $.
i) Combien de pastilles à la menthe FIGURE 14.2 INDUSTRIE
l’entreprise produit-elle? (Indice:
n’oubliez pas que l’entreprise est en
équilibre à long terme.)
ii) Quel est le produit marginal (physique)
de la 25e unité de travail employée ?

9 Le tableau 14.3 présente les barèmes d’offre


et de demande de travail du marché.

TABLEAU 14.3

Quantité Quantité
Taux salarial de travail de travail Quantité

(en dollars offerte demandée


à l’heure) (en heures) (en heures)

0 0 240 FIGURE 14.3 ENTREPRISE


1 20 200
2 40 160
3 60 120
4 80 80
5 100 40
6 120 0

a) Quels sont le salaire d’équilibre et le nombre


de travailleurs engagés ?
b) Quel est le montant total de revenu reçu
par les travailleurs de ce marché ?
c) Représentez le marché sous forme graphique.
Déterminez le salaire d’équilibre et le nombre
de travailleurs.
d) En vous aidant de votre graphique de la
partie c, trouvez le montant du revenu du a) Un déclin des préférences entraîne le
travail qui correspond à la valeur de réserve ? déplacement de la demande de l’industrie
À combien s’élève la rente économique ? vers Dx. Tracez sur la figure 14.3 les nouvelles
e) Supposez maintenant que la demande courbes (le cas échéant) résultant de cette
de travail augmente de 60 heures pour baisse de la demande et expliquez tout
chaque taux salarial. déplacement des courbes. Comment évolue
i) Quels sont le nouveau taux salarial la quantité de travail à court terme employée
d’équilibre et la nouvelle quantité par l’entreprise ?
de travail ? b) Il n’y a aucun autre déplacement de la
ii) Quel est le nouveau montant total demande. Expliquez ce qui se passe à long
de revenu du travail ? terme. Pourquoi ? Tracez toute courbe
iii) Quel pourcentage de ce revenu représente supplémentaire nécessaire sur les figures
la valeur de réserve ? la rente 14.2 et 14.3 en indiquant l’équilibre définitif
économique ? à long terme.
LA DÉTERMINATION DU PRIX
ET L'ALLOCATION DES FACTEURS DE PRODUCTION 201

15 a T salaire —> plus grande T coûts totaux


RÉPONSES et T P du produit —> plus grande -l ventes
et i- nombre de travailleurs embauchés,

Vrai/Faux/Incertain 0 16 c L’inverse de a, b est vrai,

(Justifiez votre réponse.) 17 b (b x h) 12 = (4x8$) 12= 16$.


0 18 b Revenu total (32 $ = 4x8$) moins la valeur
de réserve (16 $ ou (b X h) 12 = (4 X 8 $) 12).
1 F Profit normal. Le profit économique 0 19 a Les ménages plus jeunes ont un revenu
est le revenu résiduel des propriétaires courant relativement bas, alors que le revenu
de l’entreprise, qui peuvent comprendre courant des ménages plus âgés est
l’entrepreneur. relativement élevé.
2 F A tendance à être élastique. 20 d Définition. Le prix payé pour l’utilisation
3 I Vrai si l’élasticité de la demande de travail de la terre est la rente.
est > 1 ; faux si elle est < 1. 21 e Parce que l’entreprise représente une petite
4 V Le salaire de réserve est minimal pour l’offre partie du marché du travail, elle peut acheter
de travail. autant de travail qu’elle le désire au taux
5 F L’effet de revenu est supérieur à l’effet salarial du marché.
de substitution. 22 c T salaire —> T revenu —» T demande de biens
6 V Il n’est pas judicieux d’épargner afin normaux, y compris des loisirs,
de pouvoir consommer davantage à l’avenir 0 23 a Rm mesurée en dollars, Prn mesuré en unités
si on espère pouvoir consommer davantage physiques. Les autres réponses sont des
à l’avenir de toute manière. définitions ou des conditions de maximisation
0 7 I Vrai pour le long terme, faux pour le court du profit.
terme. 24 d Tracez le graphique d’une courbe d’offre
8 F Parfaitement inélastique. abrupte et comparez les parties situées
9 F Le revenu intégral est la rente économique. au-dessus et au-dessous de la courbe d’offre
10 V Le revenu supplémentaire par rapport au PF et à la QF d’équilibre,
à la valeur de réserve. 25 b a, c, d influent à la fois sur le court terme
et le long terme.

Questions à choix multiple Problèmes à court développement

1 En général, le prix d’un facteur représente


1 c Définition.
le coût d’opportunité d’utilisation de ce facteur
2 a Mouvement ascendant le long d’une courbe
de production par l’entreprise. Les salaires sont
d’offre aplatie.
le coût d’utilisation du travail et la rente est
3 b -l PF avec demande inélastique —> (PF X QF). le coût d’utilisation de la terre. Le coût d’utilisation
4 b Montre la quantité de travailleurs embauchés
d’une unité de biens de production achetée
à chaque taux salarial. par l’entreprise correspond à l’intérêt qui doit
5 d Lorsque la courbe d’offre du facteur de
être payé (explicitement ou implicitement) sur
production de l’entreprise (Cm) croise la
les fonds immobilisés pour l’achat de ce bien.
courbe de demande de ce facteur (VPm-j). Les biens de production durent longtemps.
6 c Remplacez les loisirs par du travail. Les coûts d’immobilisation pendant n’importe
7 b Consomme plus de biens normaux, y compris
quelle période ne correspondent pas au prix
des loisirs, ce qui incite à moins travailler. d’achat du bien de production ; ils correspondent
0 8b La courbe de demande de travail se déplace au coût des fonds immobilisés pour l’achat
vers la gauche. Offre horizontale. En raison du bien durant cette période.
du Pm décroissant, -i F —> T Pm.
9 b Voir le texte d’introduction du manuel. 2 Si le taux salarial diminue, le coût d’opportunité
10 b Fixe dans le cas de biens de production précis, des loisirs diminue, et les ménages auront
lia La courbe d’offre de travail se déplacerait vers tendance à se déplacer du travail vers les loisirs
(effet de substitution), ce qui réduit la quantité
la gauche.
I2 d Les effets de revenu et de substitution sont de travail offerte. Le salaire moins élevé fait
également baisser le revenu des ménages et les
opposés pour le travail.
13 c Facteur de production utilisé comme intrant. amène donc à réduire leur demande de loisirs

14 d Recette provenant de la vente du produit et d’autres biens normaux (effet de revenu),


ce qui augmente la quantité de travail offerte.
marginal du capital.
202 CHAPITRE 14

3 Les ménages ont tendance à moins épargner travail passe de 1 à 2 unités, utilisez les valeurs
lorsque le revenu courant est bas par rapport de Rm (5 $, le prix d’une unité supplémentaire
au revenu futur et à épargner davantage lorsque de production) et de PmT (10 unités de
le revenu courant est élevé par rapport au revenu production). Cela donne la même valeur
futur. Les ménages plus jeunes se trouvent de produit marginal du travail que
le plus souvent dans la première situation, précédemment: 5 $ X 10 unités = 50 $.
et les ménages plus âgés, dans la seconde. c) L’entreprise maximise son profit en
embauchant des travailleurs jusqu’au point
4 Les loyers sont élevés parce que les prix où la VPm du travail est égale au coût
sont élevés. À Yorkville, l’offre de terrain marginal du travail (taux salarial). Ce point
est parfaitement inélastique. Le prix du terrain se produit avec 5 unités de travail. La VPm
(son loyer) est donc entièrement déterminé du passage de 4 à 5 unités de travail est
par la demande de terrain. de 20, et la VPm du passage de 5 à 6 unités
La demande est élevée parce que les de travail est de 10. D’où, par interpolation,
marchands savent que cet emplacement de choix avec exactement 5 unités de travail, la VPm
leur permettra de demander des prix plus élevés est de 15 (à mi-chemin entre 20 et 10).
et de réaliser de plus gros profits qu’ils le Donc, lorsque l’entreprise emploie 5 unités
pourraient dans d’autres emplacements. de travail, la VPm du travail est égale au taux
salarial (15 $). Avec 5 unités de travail,
5 a) Les colonnes remplies pour la RT et la VPmT la production qui permet de maximiser les
correspondant au prix de production de 5 $ profits sera de 40 unités (d’après le tableau
sont présentées au tableau 14.2 Solution. 14.2 Solution).
On obtient les valeurs de la RT en multipliant d) Pour calculer le profit, nous devons d’abord
le volume de production par le prix de calculer la recette totale puis soustraire le coût
production (5 $). On obtient la VPmT total. La recette totale est de 200 $ (40 unités
entre deux quantités de travail en divisant de production X 5 $ par unité) et le coût total
la variation de la RT par la variation est également de 200 $ — la somme du coût
de la quantité de travail, ou en multipliant variable total (travail) de 75 $ (5 unités de
la Rm (5 $) par le PmT . travail X 15 $ par unité) et du coût fixe total
de 125 $. Par conséquent, le profit est nul.
TABLEAU 14.2 SOLUTION e) Si l’entreprise emploie une unité
supplémentaire de travail (6 unités), la recette
P = 5$ P = 3$
totale sera de 210 $ (42 unités de production
Valeur Valeur X le prix de 5 $). Le coût total sera le coût
Quantité du du
de Produit Recette produit Recette produit fixe de 125 $ plus 90 $ de coût total variable
travail Production marginal totale marginal totale marginal (6 unités de travail X le taux salarial de 15 $),
(T) (Q) (PmT) (RT) (VPmT) (RT) (VPmT)
soit 215 $. Par conséquent, le profit sera de
0 0 0 0 moins 5 $ (une perte de 5 $).
, . .12 . .60 . .36 Si l’entreprise emploie une unité de travail
1 12 60 36 de moins (4 unités), la recette totale sera
. .10 . .50 . .30 de 180 $ (36 unités de production X le prix
2 22 1 10 66 de 5 $). Le coût total sera le coût fixe
. . . .8 . .40 . .24 de 125 $ plus 60 $ de coût total variable
3 30 150 90 (4 unités de travail X le taux salarial de 15 $),
. . . .6 . .30 . .18 soit 185 $. Par conséquent, le profit sera
4 36 180 108 de moins 5 $ (une perte de 5 $).
. . . .4 . .20 . .12 f) Le graphique de l’équilibre du marché
5 40 200 120 du travail est représenté à la figure 14.4.
...2 . .10 . . .6 La demande de travail est donnée par
6 42 210 126 la courbe de VPmT de l’entreprise, appelée
D0 (nous parlerons de Dx au problème
b) D’après la partie a, la formule VPmT à court développement 6).
= ART/AT donne une valeur du produit Remarquez que les valeurs de VPm sont
marginal du travail de 50 $ lorsque la quantité indiquées à mi-chemin entre les quantités
- de travail passe de 1 à 2 unités. Pour confirmer correspondantes de travail. Par exemple,
que la seconde formule ( VPmT = Rm X Pmr) la VPm de 60 est indiquée entre 0 et 1 unité
donne la même réponse lorsque la quantité de de travail.
LA DÉTERMINATION DU PRIX
ET L'ALLOCATION DES FACTEURS DE PRODUCTION 203

Étant donné que l’entreprise achète le travail et le coût total, de 185 $ - la somme du coût
dans un marché du travail parfaitement variable total (travail) de 60 $ (4 unités de
concurrentiel, l’offre de travail à l’entreprise travail X 15 $ par unité) et du coût fixe total
est parfaitement élastique au taux salarial de 125 $. Par conséquent, le profit est de -77 $,
du marché. La courbe d’offre de travail est T soit une perte de 77 $.
= 15 $. L’équilibre est atteint à l’intersection d) L’entreprise ne cessera pas ses activités, puisque
de ces courbes, et il correspond à un taux salarial la recette totale (108 $) suffit à couvrir le coût
de 15 $ et à l’utilisation de 5 unités de travail. variable total (60 $) et une partie du coût
fixe. Si l’entreprise décidait de cesser ses
FIGURE 14.4 activités, elle perdrait les 125 $ de coût fixe
plutôt que simplement 77 $.
e) Le graphique de l’équilibre du marché du
travail est présenté à la figure 14.4. La nouvelle
demande de travail est donnée par la nouvelle
courbe de VPmT de l’entreprise, Dx. L’offre de
travail n’a pas changé ; elle continue d’être
horizontale au salaire du marché concurrentiel
de 15 $. Le nouvel équilibre est atteint à
l’intersection de ces courbes et il correspond
à un taux salarial de 15 $ et à l’utilisation
de 4 unités de travail.

a) Comme la recette marginale et le produit


marginal du travail ne sont pas touchés
par une variation du taux salarial, la courbe
de demande de travail (la VPm du travail)
restera à Dl.
b) Si le taux salarial augmente pour s’élever
a) Les colonnes remplies pour la RT et la VPmT à 21 $, la quantité de travail qui permet de
correspondant au prix de production de 3 $ maximiser le profit baissera pour être de
sont présentées au tableau 14.2 Solution. 3 unités, puisque la VPmTcst égale au taux
On obtient les valeurs de la RT en multipliant salarial avec 3 unités de travail. Avec
le volume de production par le prix de 3 unités de travail, le volume de production
production (3 $). On obtient la VPmT entre qui permet de maximiser le profit
deux quantités de travail quelconques en sera de 30 unités (d’après le tableau
divisant la variation de la RT par la variation 14.2 Solution).
de la quantité de travail, ou en multipliant c) Le profit est égal à la recette totale moins
la Rm (3 $) par le PmT. le coût total. La recette totale est de 90 $
b) Si le taux salarial est toujours de 15 $> la (30 unités de production X 3 $ par unité)
quantité de travail qui permet de maximiser et le coût total est de 188 $ - la somme du
le profit baissera jusqu’à 4 unités, puisque la coût variable total (travail) de 63 $
VPmT est égale au taux salarial avec 4 unités (3 unités de travail X 21 $ par unité) et du
de travail. La VPm du passage de 3 à 4 unités coût fixe total de 125 $. Par conséquent,
de travail est de 18, et la VPm du passage de le profit est de —98 $, soit une perte
4 à 5 unités de travail est de 12. D où, par de 98 $.
interpolation, avec exactement 4 unités de d) Voir la figure 14.4. La courbe de demande
travail, la VPm est de 15 (à mi-chemin entre de travail pertinente continue d’être D,, mais
18 et 12). Puisque 4 unités de travail sont la courbe d’offre de travail reflète la hausse du

utilisées, la production qui permet taux salarial concurrentiel ; elle est maintenant

de maximiser le profit sera de 36 unités horizontale lorsque le taux salarial est de 21 $

(d’après le tableau 14.2 Solution). (appelée T’ = 21 $). L’équilibre est atteint

c) Le profit est égal à la recette totale moins à l’intersection de ces courbes et correspond

le coût total. La recette totale est de 108 $ à un taux salarial de 21 $ et à l’utilisation

(36 unités de production X 3 $ par unité) de 3 unités de travail.


204 CHAPITRE 14

D@ 8 a) En supposant que l’entreprise ait employé La valeur du produit marginal est également

son capital, nous connaissons (en principe) définie comme étant le produit marginal du

les coûts fixes et pouvons calculer la quantité travail (PmT) multiplié par la recette marginale
de facteur variable (travail) qui permet de (.Rm), ou VPmT= Pmr X Rm. En réarrangeant

maximiser les profits en choisissant la VPmT les valeurs pour résoudre l’équation pour Pm-p,
qui est égale au taux salarial (30 $). nous obtenons:

30 = 110- 8/5 Qr PmT = VPmT /Rm


8/5 Qt = 80
En utilisant les valeurs de la valeur du
Qr =50 produit marginal de la 25e unité de travail (70)
b) En supposant que l’entreprise ait utilisé son et de la Rm (le prix de production de 10 $),
travail, nous pouvons traiter le travail en tant nous obtenons:
que coût fixe et le capital en tant que facteur PmT = 70/10
de production variable. Calculez la quantité
PmT = 7
de capital immobilisé qui permet de maximiser
le profit en établissant VPmK = le coût d’une
9 a) Le taux de salaire d’équilibre est de 4 $
unité de capital (50 $).
l’heure et la quantité de travail employée
50 = 125-75/40 QK est de 80 heures.
b) Le revenu total est de 320 $ (4 $ l’heure
75/40 Qk = 75
X 80 heures).
Qk = 40 c) Les courbes d’offre et de demande de travail
c) i) Le fait que l’entreprise est en équilibre sont représentées à la figure 14.5. L’offre
à long terme permet de calculer la de travail est représentée par O0 et la demande
quantité de pastilles à la menthe produite. initiale de travail par D0. L’équilibre est
En situation d’équilibre à long terme, atteint à l’intersection de ces deux courbes.
l’entreprise réalise un profit économique Vous pouvez voir que le salaire d’équilibre
nul, si bien que la recette totale est est de 4 $ l’heure et la quantité de travail
exactement égale au coût total. employée, de 80 heures.
Nous pouvons calculer le coût total
en additionnant les coûts des deux (seuls) FIGURE 14.5
facteurs de production. Le coût du travail est
de 50 unités de travail X 30 $ par unité, soit
1 500 $. Les coûts d’immobilisation sont de
40 unités de capital X 50 $ par unité, soit
2 000 $. Le coût total est donc de 3 500 $.
La recette totale doit aussi être égale
à 3 500 $. Puisque la recette totale (RT) est
simplement le prix de production (P )
multiplié par la quantité vendue (Q), nous
pouvons calculer Q en utilisant les valeurs de
RT (3 500 $) et de P (10$).

RT = P xQ
3 500$ = 10$ xQ

Q = 350
d) Étant donné que la courbe d’offre de travail
ii) Pour calculer le produit marginal de la est une ligne à 45° partant de l’origine, la
25e unité de travail, nous commençons moitié du revenu est la valeur de réserve (la
par calculer la valeur du produit marginal partie située sous la courbe d’offre) et l’autre
de la 25e unité de travail. En utilisant moitié, la rente économique (la partie située
Qj =25 dans l’équation de la VPmT, au-dessus de la courbe d’offre). Puisque
nous obtenons le revenu est de 320 $, l’une et l’autre sont
de 160 $.
VPmT =110-8/5 (25)
e) i) Chaque entrée dans la dernière colonne
= 110-40
du tableau 14.3 augmentera de 60 unités.
= 70 Le nouveau salaire d’équilibre est de 5 $
LA DÉTERMINATION DU PRIX
ET L'ALLOCATION DES FACTEURS DE PRODUCTION 205

l’heure et la nouvelle quantité de travail Lorsque des entreprises quittent l’industrie,


d’équilibre est de 100 heures. La nouvelle l’offre de l’industrie diminue, le prix augmente
courbe de demande de travail est et la quantité offerte par chaque entreprise
représentée à la figure 14.5 sous le nom restante augmente. Dans le chapitre 10,
de Dx. nous avons vu que cela signifie que chaque
ii) Le revenu total est maintenant de 500 $ entreprise restante se déplace vers le haut le
(5 $ l’heure X 100 heures). long de Cm. Nous pouvons maintenant voir
iii) Là encore, la pente de la courbe d’offre de pourquoi. Chaque entreprise restante
travail (qui n’a pas changé) implique que augmente son utilisation du facteur de
le revenu est réparti également entre la production variable, le travail. En raison de
valeur de réserve et la rente économique. rendements décroissants, le produit marginal
Chacune d’elle sera égale à 250 $. (physique) du travail diminue. Avec un salaire

& 10 a) Voir la figure 14.2 Solution et la figure fixe, le coût marginal (Cm = S/Pmr)
14.3 Solution. La baisse de la demande fait augmente.
descendre le prix de production de l’industrie
à Px, ce qui fait déplacer vers la gauche la FIGURE 14.2 SOLUTION INDUSTRIE

courbe de la valeur du produit marginal du


travail pour les entreprises. Il n’y a pas de
variation de salaire, donc la quantité de travail
utilisée par chaque entreprise diminue, pour
passer à QTX.
b) Le fait qu’il existe une industrie à coût constant
initialement en situation d’équilibre à long
terme permet de comprendre ce qui se produit
à long terme. Avec P0 et Q0, les entreprises
réalisent des profits économiques nuis. Lorsque
le prix baisse à Px, les entreprises subissent
des pertes économiques. Cela pousse certaines
Quantité
entreprises à sortir de l’industrie, ce qui
entraîne le déplacement vers la gauche de
la courbe d’offre de l’industrie. Étant donné
qu’il s’agit d’une industrie à coût constant,
nous savons que le nouvel équilibre à long
FIGURE 14.3 SOLUTION ENTREPRISE
terme, même avec des volumes de production
réduits, doit être atteint au même prix que
l’équilibre initial. Par conséquent, la courbe
d’offre de l’industrie se déplace vers Ox.
L’équilibre à long terme définitif de 1 industrie
est de nouveau atteint au prix original P0
et à la nouvelle quantité Qj.
Comme le prix de production redevient
P0, la courbe de la valeur du produit marginal
du travail de chaque entreprise restante
se déplace pour revenir vers la droite et finir
à sa position originale. La quantité de travail
utilisée par chaque entreprise restante
augmente, passant de QTX à QT0.
Le marché du travail

Les syndicats ont pour objectifs l’augmentation


CONCEPTS CLÉS des salaires ainsi que l’amélioration des conditions de
travail et des perspectives d’emploi. Ils peuvent atteindre
ces objectifs au moyen des méthodes suivantes:
Les différences de qualification
♦ en limitant l’offre de travail.
Les différences de qualification se produisent en raison
de différences dans ♦ en T la demande de travailleurs syndiqués ou
en rendant la demande plus inélastique par l’appui
♦ la demande - la valeur du produit marginal
aux restrictions à l’importation, aux lois sur le
( VPm) des travailleurs qualifiés est supérieure
salaire minimum et à la limitation de l’immigration,
à celle des travailleurs non qualifiés. Line plus
en T la demande du produit, en T le produit
grande VPm —> une plus forte demande.
marginal des travailleurs syndiqués.
♦ l’offre — l’acquisition de compétences est coûteuse.
Les êtres humains, de par leur expérience et leurs Monopsone - structure de marché qui ne compte
connaissances, constituent le capital humain. Les qu’un seul acheteur. L’entreprise qui est l’unique
coûts d’acquisition des connaissances —» courbe employeur dans une ville est un monopsone sur le
d’offre de la main d’œuvre qualifiée au-dessus de marché du travail.
la courbe d’offre de la main-d’œuvre non qualifiée.
♦ Pour embaucher plus de main-d’œuvre,
Les écarts de salaire entre syndiqués le monopsone doit payer un salaire plus élevé.
et non-syndiqués La courbe du coût marginal du travail (Cm-f
du monopsone a une pente positive et elle est plus
Les syndicats sont des associations de travailleurs. abrupte que la courbe d’offre du marché du travail
♦ Syndicat de métier - regroupement au-dessus de laquelle elle se situe.
de travailleurs dont les compétences sont
♦ Pour le monopsone, la règle de maximisation des
sensiblement les mêmes, mais qui travaillent
profits consiste à employer une quantité de travail
dans des entreprises ou des industries différentes.
telle que la courbe de CmT croise la courbe de VPm,
♦ Syndicat industriel - regroupement de travailleurs puis à offrir aux travailleurs le salaire le plus bas
d’une même entreprise ou d’une même industrie auquel ils sont prêts à travailler (sur la courbe d’offre).
qui ont des compétences diverses et qui exercent
divers métiers. ♦ Dans une situation de monopsone, les emplois
sont moins nombreux et les salaires moins élevés
♦ Formule Rand - requiert que tous les travailleurs
que dans un marché du travail concurrentiel.
d’un atelier fermé versent des cotisations au
Une courbe d’offre de travail plus élastique
syndicat, qu’ils en soient membres ou non.
—> si possibilités pour le monopole de
♦ Négociations collectives - processus de négociation ■1 les salaires, de l’emploi ou de î le profit
entre les syndicats et les employeurs. économique.
LE MARCHÉ DU TRAVAIL 207

♦ Lorsque les travailleurs forment un syndicat dans


un marché du travail en situation de monopsone RAPPELS
cela —> un monopole bilatéral, où le taux salarial
est déterminé par voie de négociations entre I Ce chapitre présente le concept de monopsone,
le syndicat et l’entreprise. Le salaire le plus élevé une entreprise qui est le seul acheteur d’un
se trouve au point où la courbe de CmT croise la marché, par exemple le marché du travail. Le
courbe de VPm. Le salaire le plus bas est un salaire monopsone fait face à une courbe d’offre de
de monopsone pur. travail à pente positive. Il s’ensuit que sa courbe

♦ L’adoption d’une loi sur le salaire minimum dans de coût marginal du travail (Cm-j-) est différente

un marché en situation de monopsone —» î des de sa courbe d’offre de travail.

salaires et T de l’emploi. Il existe un étroit parallèle entre a) la relation


entre la courbe d’offre de travail et la courbe de
CmTd\i monopsone et b) la relation, que vous
Les écarts de salaire entre hommes connaissez maintenant (chapitre 12), entre la
et femmes courbe de demande et la courbe de recette
marginale (Rm) du monopole. Ces deux types
Les écarts salariaux entre hommes et femmes sont liés
de relation découlent de l’hypothèse qu’il existe
♦ aux types d’emploi - les hommes et les femmes un prix unique pour le travail ou la production
remplissent des tâches différentes; les emplois dans le marché pertinent.
occupés par les hommes sont souvent mieux Le monopole étant le seul vendeur d’un
rémunérés. produit sur un marché, il fait face à une courbe
♦ à la discrimination - résultant en des taux salarial de demande à pente négative. La recette
et d’emploi plus faibles pour les travailleurs marginale provenant de la vente d’une unité
victimes de discrimination. supplémentaire de production est inférieure au
prix de vente parce que le monopole doit
♦ aux différences de capital humain - différences
en matière d’années de scolarité (en grande
également réduire le prix de toutes les unités

partie éliminées), d’expérience professionnelle et


précédentes. La courbe de Rm du monopole

d’interruptions de carrière (ont tendance à diminuer).


non discriminant se situe donc au-dessous de
la courbe de demande.
♦ aux différents niveaux de spécialisation -
Pour le monopsone dans un marché du
en raison de conventions sociales, les hommes
travail, le coût marginal de l’emploi d’une unité
se sont spécialisés dans les activités de marché
supplémentaire de travail est supérieur au salaire
alors que les femmes ont réparti leur temps entre
parce que le monopsone doit également augmenter
les activités de marché et les activités hors marché.
le salaire de toutes les unités de travail précédentes.
• La production domestique est la production La courbe de CmT du monopsone se situe donc
de biens et de services destinés aux ménages au-dessus de la courbe d’offre de travail.
et non au marché.
• La spécialisation dans certaines activités
—» "f de la productivité et des salaires.
AUTOÉVALUATION
La législation sur l’équité salariale
Vrai/Faux/Incertain
Les lois sur l’équité salariale fixent les salaires (Justifiez votre réponse.)
en évaluant la valeur des emplois d’après
des caractéristiques objectives plutôt que d après
I La valeur du produit marginal des travailleurs
ce que le marché payera.
non qualifiés est inférieure à celle des travailleurs
♦ Équité salariale - verser un salaire égal pour
qualifiés.
différents emplois objectivement similaires.

♦ Les lois sur l’équité salariale —> T de 1 équité


en présence de discrimination.
♦ Les lois sur l’équité salariale —> conséquences
imprévues de T du chômage pour les femmes
lorsque les différences salariales sur le marche
sont fondées sur la productivité.
208 CHAPITRE 15

2 La distance verticale entre la courbe d’offre Questions à choix multiple


de travail des travailleurs qualifiés et celle
des travailleurs non qualifiés représente
1 Laquelle des raisons suivantes m’explique
la valeur du produit marginal de la qualification.
pas pourquoi le salaire des travailleurs qualifiés
est supérieur à celui des travailleurs non qualifiés ?
a) Le marché des travailleurs qualifiés est plus
3 Plus la valeur du produit marginal de la concurrentiel que le marché des travailleurs
qualification est élevée et plus cette qualification non qualifiés.
est coûteuse à acquérir, plus les différences b) La valeur du produit marginal des travailleurs
salariales entre les travailleurs qualifiés qualifiés est supérieure à celle des travailleurs
et les travailleurs non qualifiés sont faibles. non qualifiés.
c) Le coût de la formation des travailleurs
qualifiés est supérieur au coût de formation
des travailleurs non qualifiés.
4 Les syndicats appuient les lois sur le salaire d) Les travailleurs qualifiés ont acquis
minimum en partie parce qu elles augmentent plus de capital humain que les travailleurs
le coût de la main-d’œuvre non qualifiée, non qualifiés.
un substitut de la main-d’œuvre qualifiée. e) La courbe de demande des travailleurs
qualifiés se situe à droite de la courbe de
demande des travailleurs non qualifiés.

5 Pour une entreprise en situation de monopsone


2 Toutes autres choses étant égales, si les coûts
sur le marché du travail, la courbe d’offre
de formation augmentent substantiellement,
de travail se confond avec la courbe du coût
nous pouvons nous attendre à voir
marginal du travail.
a) une diminution de la valeur du produit
marginal des travailleurs qualifiés.
b) une diminution de la demande de travailleurs
6 Plus l’offre de travail est élastique, moins le qualifiés.
monopsone a de possibilités de réaliser un profit c) une augmentation de l’offre de travailleurs
économique. qualifiés.
d) une augmentation des salaires des travailleurs
qualifiés.
e) une augmentation du nombre de travailleurs
7 L’adoption d’un salaire minimum supérieur qualifiés employés.
au salaire courant fera augmenter le salaire mais
diminuer l’emploi. 3 Un syndicat qui regroupe des travailleurs
possédant les mêmes compétences, quelle que
soit l’industrie ou l’entreprise qui les emploie, est
a) un syndicat industriel.
8 Dans le cas d’un monopole bilatéral sur un
b) un syndicat de métier.
marché du travail, le salaire dépend du pouvoir
c) un syndicat local.
de négociation des deux parties.
d) un syndicat national.
e) un syndicat de la fonction publique.

9 Selon la théorie économique, la discrimination 4 La figure 15.1 illustre un monopsone sur


le marché du travail (CmT = coût marginal
dans l’emploi entraîne des différences salariales.
du travail). Le taux salarial et la quantité
de main-d’œuvre engagée qui permettent
de maximiser le profit seront
10 Lorsque les hommes gagnent, en moyenne, a) 4 $ l’heure et 800 heures de travail.
plus que les femmes, on est en présence b) 4 $ l’heure et 400 heures de travail.
de discrimination. c) 7 $ l’heure et 600 heures de travail.
d) 9 $ l’heure et 400 heures de travail.
e) Aucune de ces réponses.
LE MARCHÉ DU TRAVAIL 109

FIGURE 15.1 9 Lorsque, à cause de la discrimination, un groupe


ne peut accéder à l’éducation et à la formation,
l’effet sur ce groupe de travailleurs sera un
déplacement
a) de la courbe de VPm vers la droite
et une augmentation du salaire.
b) de la courbe de VPm vers la gauche
et une diminution du salaire.
c) de la courbe de VPm vers la droite
et une diminution du salaire.
d) de la courbe d’offre de travail vers
le haut et une diminution du salaire.
e) de la courbe d’offre de travail vers le
bas et une diminution du salaire.

10 La formule Rand rend


a) la négociation collective obligatoire.
b) obligatoire la cotisation de tous les travailleurs
5 Si le marché du travail illustré à la figure 15.1 au syndicat d’une entreprise partiellement
devient concurrentiel, le salaire d’équilibre et la syndiquée.
quantité de travail employée d’équilibre seront c) légal l’arbitrage exécutoire.
a) 4 $ l’heure et 800 heures de travail. d) illégaux le lock-out et la grève dans certaines
b) 4 $ l’heure et 400 heures de travail. conditions.
c) 7 $ l’heure et 600 heures de travail. e) Aucune de ces réponses.
d) 9 $ l’heure et 400 heures de travail.
e) Aucune de ces réponses. I I L’équité salariale est un moyen de verser
le même salaire aux hommes et aux femmes
6 Si un syndicat se forme face au monopsone a) pour l’exécution d’un même travail.
de la figure 15.1, on se trouve en situation b) pour la production domestique comme
a) d’arbitrage exécutoire. pour l’activité de marché.
b) de demande dérivée. c) pour différentes tâches qui sont jugées
c) de duopole. comparables.
d) d’oligopole de collusion. d) quel que soit le travail.
e) de monopole bilatéral. e) quel que soit le sexe.

7 Si un syndicat et le monopsone de la figure 15.1 I2 Reportez-vous à la figure 15.2. Pour toute


concluent une négociation collective, le résultat quantité donnée de travailleurs employés,
sera un salaire a) l’élasticité de la demande de travailleurs
a) de 7 $. qualifiés est moins grande que
b) de 4 $ à 7 $. l’élasticité de la demande de travailleurs
c) de 4 $ à 9 $. non qualifiés.
d) de 7 $ à 9 $. b) les travailleurs qualifiés recevront un
e) qui ne peut être déterminé sans informations salaire inférieur à celui des travailleurs
supplémentaires. non qualifiés.
c) les travailleurs qualifiés recevront un
8 Les écarts salariaux entre hommes et femmes salaire supérieur à celui des travailleurs
peuvent s’expliquer par non qualifiés.
a) le type d’emploi. d) la distance verticale entre les courbes
b) les différences de capital humain. représente la compensation pour le coût
c) les différences de niveau de spécialisation. de l’acquisition de compétences.

d) la discrimination. e) la distance verticale entre les courbes

e) Toutes ces réponses. est la valeur actuelle du capital humain.


210 CHAPITRE 15

15 Dans une situation de monopole bilatéral, une


FIGURE 15.2
grève ou un lock-out se produit habituellement
parce que
a) la demande de travail est relativement
inélastique.
b) la demande de travail est relativement
élastique.
c) l’offre de travail est relativement inélastique.
d) l’offre de travail est relativement élastique.
e) le syndicat ou l’entreprise ont mal évalué
l’état des négociations.

16 Laquelle des mesures suivantes est la moins


susceptible d’être appuyée par les syndicats ?

13 La distance verticale entre les deux courbes a) L’augmentation du salaire minimum.

d’offre de la figure 15.3 b) Les restrictions à l’immigration.


a) représente la compensation pour le coût c) L’accroissement des importations.

d’acquisition d’une qualification. d) L’augmentation de la demande des biens

b) est la VPm de la qualification. produits par des syndiqués.

c) est le résultat de la discrimination à l’égard e) L’augmentation du produit marginal

des travailleurs non qualifiés. du travail syndiqué.

d) disparaîtra avec l’entrée libre sur le marché


des compétences. I7 Les différences de productivité sur les marchés

e) disparaîtra avec un salaire égal pour un travail concurrentiels du travail font que le salaire versé

d’égale valeur. aux secrétaires est substantiellement plus élevé


que le salaire versé aux nettoyeurs. Une nouvelle

FIGURE 15.3 loi sur l’équité salariale estime que les emplois
de secrétaires et de nettoyeurs sont comparables.
Pour égaliser les salaires, le taux salarial des
nettoyeurs est augmenté et celui des secrétaires,
diminué. L’emploi des secrétaires
a) augmentera, alors que celui des nettoyeurs
diminuera.
b) diminuera, alors que celui des nettoyeurs
augmentera.
c) et des nettoyeurs augmentera.
d) et des nettoyeurs diminuera.
e) et des nettoyeurs ne changera pas.

18 Reportez-vous de nouveau à la situation décrite


14 Reportez-vous seulement à l’information à la question 17. Conformément à la loi sur
présentée dans la figure 15.3. Pour l’équité salariale, le chômage des
tout taux salarial donné, plus d’heures a) secrétaires va augmenter.
de travail b) nettoyeurs va augmenter.
a) seront demandées aux travailleurs non c) secrétaires et des nettoyeurs va augmenter.
qualifiés qu’aux travailleurs qualifiés. d) secrétaires et des nettoyeurs va diminuer.
b) seront demandées aux travailleurs e) secrétaires et des nettoyeurs restera le même.
qualifiés qu’aux travailleurs non
qualifiés. I9 Lequel des énoncés suivants est faux ?
c) seront offertes par les travailleurs Le capital humain
non qualifiés que par les travailleurs a) représente l’accumulation des compétences
qualifiés. et des connaissances des êtres humains.
d) seront offertes par les travailleurs qualifiés b) est coûteux à acquérir.
que par les travailleurs non qualifiés. c) augmente avec la formation en cours
e) seront offertes par les travailleurs non d’emploi.
qualifiés, si la VPm de la qualification d) augmente grâce aux salaires plus élevés.
augmente. e) augmente grâce à l’éducation.
LE MARCHÉ DU TRAVAIL 21

20 Comparativement à un marché du travail


concurrentiel ayant les mêmes courbes de valeur
FIGURE 15.4
du produit marginal et d’offre de travail, un
marché en situation de monopsone a
a) un taux salarial et un taux d’emploi
moins élevés.
b) un taux salarial plus bas et un taux d’emploi
plus élevé.
c) un taux salarial plus élevé et un taux d’emploi
plus bas.
d) un taux salarial et un taux d’emploi
plus élevés.
e) un résultat indéterminé.

21 Un syndicat se forme pour limiter l’offre de


travail dans un marché du travail parfaitement
concurrentiel. Si le syndicat réussit à faire
augmenter le salaire,
a) l’emploi diminuera. 24 Supposons que le gouvernement adopte

b) l’emploi augmentera. une loi sur le salaire minimum qui interdit

c) l’emploi restera le même. d’engager de la main-d’œuvre à un taux salarial

d) la masse salariale totale augmentera. horaire inférieur à 8 $. Dans la figure 15.4, le

e) la masse salariale totale baissera. coût marginal du travail (Cm j-) du monopsone
a) n’est pas touché.

22 Les lois sur l’équité salariale b) est égal à 8 $ seulement de 0 à 250 heures

a) ne peuvent atteindre leurs objectifs lorsque de travail.

les différences salariales sont fondées c) est égal à 8 $ seulement de 0 à 350 heures

sur la discrimination. de travail.

b) peuvent atteindre leurs objectifs lorsque d) est égal à 8 $ seulement de 0 à 500 heures
de travail.
les différences salariales sont fondées
e) se déplace vers le haut d’une distance verticale
sur la productivité.
correspondant à 8 $.
c) peuvent atteindre leurs objectifs lorsque
les différences salariales sont fondées 25 Lorsque tous les travailleurs doivent être
sur la discrimination. membres du syndicat avant que l’entreprise
d) analysent les caractéristiques des emplois puisse les engager, on est en présence
et déterminent leur valeur en fonction a) d’un atelier ouvert.
du marché. b) d’un atelier fermé.
e) comparent la valeur marchande des c) d’un atelier syndical.
caractéristiques objectives des emplois. d) d’un syndicat de métier.
e) d’une association professionnelle.
23 Dans le cas du monopsone illustré à la figure
15.4, le taux salarial et la quantité de main-d œuvre
Problèmes à court développement
engagée qui permettent de maximiser les profits
seront 1 Les travailleurs syndiqués gagnent des salaires
a) 9 $ l’heure et 300 heures de travail. bien supérieurs au salaire minimum. Alors
b) 8 $ l’heure et 350 heures de travail. pourquoi est-il dans l’intérêt des syndicats de
c) 8 $ l’heure et 500 heures de travail. favoriser les augmentations du salaire minimum ?
d) 7 $ l’heure et 400 heures de travail.
e) 6 $ l’heure et 300 heures de travail.
2 Robert et Suzanne forment un ménage. Ils ont
décidé que Suzanne se spécialiserait entièrement
dans les activités de marché et que Robert
se livrerait à des activités domestiques et à des
activités de marché. Si la majorité des ménages
ressemblent à celui de Robert et Suzanne,
pourquoi y aura-t-il une différence salariale
entre les hommes et les femmes, même
en l’absence de discrimination ?
212 CHAPITRE 15

Supposons que le marché du travail pour les n’entraîne pas d’autres avantages. Le coût
conseillers financiers soit exactement le même du cours est de 1 200 $ et la meilleure autre
que celui que décrit la figure 15.8 (à la page possibilité, l’investissement de cette somme par
356 du manuel), où est pratiquée à l’égard des Julien, rapporte 10% d’intérêt par an pour le
conseillères financières une discrimination qui reste de sa vie. Julien devrait-il payer les 1 200 $
favorise les conseillers financiers. Expliquez les et suivre le cours ? Expliquez votre réponse.
effets de l’adoption d’un barème de rémunération
équitable qui égalise les salaires à 40 000 $ par an. - Dans un premier temps, nous observons une
industrie faisant face à un marché du travail
Dans quelles situations les lois sur l’équité concurrentiel où l’offre de travail vient de deux
salariale peuvent-elles atteindre leur objectif de sources : les travailleurs du pays et les travailleurs
plus grande équité ? Dans quelles situations ces étrangers. Supposons également que la production
lois risquent-elles de ne pas atteindre leur objectif? de l’industrie fait face à la concurrence de
produits importés.
La figure 15.5 présente l’offre et la demande de a) Dans la figure 15.6, représentez graphiquement
travailleurs non qualifiés et qualifiés. 0Nq et Oq le marché du travail concurrentiel initial.
sont les courbes d’offre de travailleurs non Tracez les courbes de demande et d’offre
qualifiés et qualifiés, respectivement, et DNq et de travail et déterminez le taux salarial
Dq sont les courbes de demande de travailleurs et le niveau d’emploi d’équilibre.
non qualifiés et qualifiés, respectivement.
FIGURE 15.6
FIGURE 15.5

_o
O

X
D
O

0 Quantité de travail

b) Supposons maintenant qu’un syndicat


composé de travailleurs du pays se forme.
Grâce à son appui, on adopte une loi qui
a) Quelle est la VPm de la qualification avec
interdit aux entreprises d’engager des
5 000 heures de travail demandées par chaque
travailleurs étrangers. Quel effet cette loi
type de travailleurs ?
aura-t-elle sur le niveau d’emploi et le taux
b) Quel montant de rémunération horaire
salarial ? Illustrez graphiquement votre réponse
supplémentaire est requis pour inciter
en utilisant le graphique de la partie a.
à l’acquisition d’une qualification pour
c) Enfin, l’industrie et le syndicat appuient
un même niveau d’emploi ?
l’adoption d’une loi qui limite l’importation
c) Quels sont le salaire et la quantité de travail
de produits qui font concurrence aux produits
d’équilibre dans le marché de la main-d’œuvre
de l’industrie nationale. Sur le même
qualifiée ?
graphique, montrez les effets de cette loi
d) Quels sont le salaire et la quantité de travail
sur le taux salarial et l’emploi.
d’équilibre dans le marché de la main-d’œuvre
non qualifiée ?

6 Julien a la possibilité d’augmenter son capital


humain en suivant un cours de formation qui
fera augmenter son revenu de 100 $ par an pour
le reste de sa vie. Supposons que cette formation
LE MARCHÉ DU TRAVAIL 213

Papier Pollunet est une fabrique de pâte et 10 Soit les négociations entre Papier Pollunet
papier qui emploie presque tous les habitants et le syndicat indiqué dans le problème à court
d’une petite ville du Nouveau-Brunswick. développement 9b.
Le marché du travail de la ville, qui se rapproche a) Qu’est-ce qui détermine le taux salarial
du monopsone, est présenté à la figure 15.7, qui sera effectivement payé ?
où O est la courbe d’offre de travail et VPm b) Si Papier Pollunet et le syndicat ont
et CmT, les courbes de valeur du produit un pouvoir de négociation égal et le savent,
marginal du travail et du coût marginal quel sera probablement le taux salarial ?
du travail, respectivement. c) Si Papier Pollunet et le syndicat ont
un pouvoir de négociation égal, mais que
FIGURE 15.7 le syndicat pense, à tort, être le plus fort,
quel sera probablement le résultat des
négociations ?
d) Si Papier Pollunet et le syndicat ont un
pouvoir de négociation égal, mais que
Papier Pollunet pense, à tort, être le plus
fort, quel sera probablement le résultat
des négociations ?

Vrai/Faux/Incertain
(Justifiez votre réponse.)

Travail (en milliers d'heures)


1 V Conséquences —» demande de travailleurs
non qualifiés à gauche de la demande
de travailleurs qualifiés.
a) Si Papier Pollunet est un monopsone
2 F La distance verticale est la compensation
qui maximise ses profits, quel taux salarial
pour le coût de l’acquisition de la
offrira-t-il et combien de travail emploiera-t-il ?
qualification.
Quelle est la valeur du produit marginal
3 F Les différences salariales seront plus grandes.
du travail à ce niveau d’emploi ? 4 V T du prix du substitut —> T de la demande
b) Si la ville comprenait un marché du travail de travail syndiqué.
concurrentiel ayant la même courbe de 5 F La courbe de CmTtst située au-dessus
VPm, quels seraient le taux salarial et le de la courbe d’offre de travail.
niveau d’emploi d’équilibre ? Comparez ^ 6 V T T| offre de travail —> X des différences
ces résultats à ceux du monopsone de entre le CmTç.t le salaire.
la partie a. g) 7 I Vrai pour le marché du travail concurrentiel,
faux pour le marché du travail en situation
Soit les possibilités suivantes du marché de monopsone (où salaire T et emploi T).
du travail de la figure 15.7.
8 V Voir le texte du manuel.
a) Supposons que le gouvernement impose 9 V -i des salaires pour les travailleurs qui
un salaire minimum de 4 $ 1 heure. Quel taux subissent la discrimination et T des salaires
salarial l’entreprise Papier Pollunet offrira-t-elle pour les travailleurs favorisés par la
et combien de travailleurs embauchera-t-elle ? discrimination.
Comparez ces résultats à ceux du problème 10 I Les différences peuvent être attribuables au
type d’emploi, à la discrimination, au capital
à court développement 8a.
humain et/ou à la qualification.
b) Supposons qu’il n’y a pas de salaire
minimum, mais que les travailleurs forment
un syndicat. Si le syndicat tente de négocier Questions à choix multiple
un taux salarial plus éleve tout en maintenant
l’emploi au niveau de monopsone, quel est I a Les différences salariales ne sont pas dues
le taux salarial maximum que Papier Pollunet à des différences dans la concurrence. Dans
sera disposé à payer ? Quel taux salarial tous les cas, T de la concurrence —> -t des
minimum acceptera le syndicat ? salaires des travailleurs qualifiés.
214 CHAPITRE 15

2 d T coût d’acquisition d’une qualification Problèmes à court développement


—» déplacement vers le haut (la gauche)
de la courbe d’offre de travailleurs 1 Une augmentation du salaire minimum fera
qualifiés. monter le coût de l’embauche de main-d’œuvre
3 b Définition. non qualifiée, ce qui a tendance à faire augmenter
4 b Qrau point où la CmT croise la VPm. la demande de main-d’œuvre qualifiée, qui devient
Ensuite le plus bas salaire requis pour que un substitut de la main-d’œuvre non qualifiée.
les travailleurs offrent Qr (sur la courbe
d’offre). 2 Si Suzanne se spécialise dans l’activité de marché
5 c Au point où la courbe O croise la VPm. alors que Robert diversifie ses activités, il est
6 e Voir le texte du manuel. probable que le potentiel de revenu de Suzanne
7 c Entre le minimum que l’entreprise peut sera supérieur à celui de Robert en raison de
obtenir (4 $) et le maximum que le syndicat la spécialisation de celle-ci. Si la majorité des
peut obtenir (9 $). ménages suivaient ce processus de spécialisation,
8 e Toutes ces raisons peuvent contribuer aux le revenu des femmes serait supérieur à celui des
différences salariales. hommes, même en l’absence de discrimination.
9 e L’éducation et la formation feraient
T le capital humain et déplacer la courbe 3 Le salaire des conseillères financières augmentera
d’offre de travail vers le haut. (il passera de 20 000 $ à 40 000 $ par an) et
10 b Voir le texte du manuel. l’emploi de conseillères augmentera (il passera
11 c Voir le texte du manuel. de 1 000 à 2 000). Le salaire des conseillers
012 c L’inverse de a est vrai; d s’applique à l’offre; financiers diminuera (il passera de 60 000 $
e n’a pas de sens. à 40 000 $ par an) et l’emploi de conseillers
13 a Voir la figure 15.1 du manuel. diminuera (il passera de 3 000 à 2 000). Cette
14 c Aucune courbe de demande sur le graphique. évolution produira le résultat qui aurait prévalu
T de la VPm de la qualification fait déplacer en l’absence de discrimination. Il n’y a ni
la demande et non l’offre, demande ni offre excédentaire de conseillers
15 e a- d peuvent avoir une incidence sur financiers ou de conseillères financières.
le pouvoir de négociation relatif mais ne
provoquent pas les grèves ni les lock-out. 4 Les lois sur l’équité salariale ayant force exécutoire
16 c T des importations —> X des produits peuvent atteindre leur objectif d’améliorer l’équité
fabriqués au pays —> -l de la demande salariale dans les situations où la discrimination
de travailleurs syndiqués du pays. est la source des différences salariales. Il est
17 d Mais -l de l’emploi de secrétaires en raison possible d’augmenter les salaires du groupe
de pénuries et -l de l’emploi de nettoyeurs le moins bien payé, qui subit la discrimination,
à cause du chômage. pour tenir compte de sa productivité
18 b Le marché du travail des secrétaires connaît véritable et ce, sans augmenter le chômage
une pénurie et non le chômage. dans ce groupe.
19 d T des salaires est souvent un effet, mais Les lois sur l’équité salariale ne peuvent
non une cause de T du capital humain. atteindre leurs objectifs dans les situations
20 a Voir la figure 15.5 du manuel. où les différences salariales sont dues aux
21 a Déplacement vers la gauche de la courbe différences de productivité. Lorsqu’il existe
d’offre —> si de 1 emploi. L’effet sur la masse des différences de productivité, les lois sur
salariale dépend de l’élasticité de la demande l’équité salariale peuvent avoir comme
de travail. conséquence imprévue d’augmenter
22 c L’inverse de a et b est vrai. Evaluent les le chômage dans le groupe de travailleurs
caractéristiques des emplois en se fondant le moins bien payé.
sur des facteurs objectifs, et non sur la valeur
du marché. 5 a) La VPm de la qualification est la différence
23 e Qrau point où le CmT croise la VPm. entre la VPm des travailleurs qualifiés par
Représente le salaire le plus bas requis pour rapport à celle des travailleurs non qualifiés ;
que les travailleurs offrent Qr (sur la courbe la distance verticale entre les courbes
d’offre). de demande de travailleurs qualifiés et de
0 24 d Cmr horizontale au salaire minimum jusqu’à travailleurs non qualifiés. Dans la figure 15.5,
ce qu’il croise O. L’entreprise doit alors T les la VPm de la qualification est de 3 $ l’heure
salaires pour que la quantité de travail offerte T. pour 5 000 heures de travail des deux types
25 b Définition. de travailleurs.
LE MARCHÉ DU TRAVAIL 215

b) Puisque les courbes d’offre de travail donnent b) Lorsqu’une loi qui interdit aux entreprises
la compensation minimum que les travailleurs d’engager des travailleurs étrangers est adoptée,
sont prêts à accepter pour offrir une la courbe d’offre de travail (maintenant
quantité donnée de travail, la compensation syndiqué) se déplacera vers la gauche (vers Os
supplémentaire pour la qualification dans la figure 15.6 Solution). Cela aura pour
est représentée par la distance verticale entre effet de faire monter le taux salarial à 5! et de
les courbes d’offre de travailleurs qualifiés baisser la quantité de travail employée à Q7J.
et de travailleurs non qualifiés. Avec c) Les restrictions de l’importation feront monter
5 000 heures de travail des deux types de la demande du bien produit dans l’industrie
travailleurs, elle est de 3 $ l’heure. et donc augmenter la demande dérivée
c) Dans le marché de la main-d’œuvre qualifiée (VPm) de travail de cette industrie, ce
en situation d’équilibre, le taux salarial sera qu’illustre le déplacement de Dc à Ds dans
de 6 $ l’heure et l’emploi, de 4 000 heures de la figure 15.6 Solution. Le résultat sera une
travail. Cela se produit au point d’intersection autre augmentation du taux salarial vers S2
de Dq et de Oq. et une augmentation de l’emploi vers QT2.
d) Dans le marché de la main-d’œuvre non Le fait que la quantité de travail employée
qualifiée en situation d’équilibre, le taux maintenant soit ou non supérieure
salarial sera de 3 $ l’heure et l’emploi, à la quantité concurrentielle dépend
de 4 000 heures de travail. Cela se produit de l’importance du déplacement
au point où DNq croise 0Nq. de la courbe de demande de travail.

Julien ne devrait suivre le cours que si sa valeur


8 a) Un monopsone qui maximise son profit
est supérieure à son coût. Le cours coûte 1 200 $
emploiera une quantité supplémentaire de
alors que la valeur du cours correspond à la
travail jusqu’au point où le CmTtst égal
valeur actuelle des 100 $ supplémentaires que
à la VPm. Par rapport à la figure 15.7,
Julien recevra chaque année pour le reste de sa
cela signifie que Papier Pollunet emploiera
vie. La valeur actuelle de ce revenu correspond
6 000 heures de travail. Pour que l’entreprise
au flux de revenu qui, s’il est investi aujourd’hui
emploie cette quantité, la courbe d’offre de
à 10% (la meilleure autre possibilité de Julien),
travail O nous indique que le taux salarial
produirait un flux de revenu équivalent. La
doit être de 3 $ l’heure, un montant inférieur
valeur actuelle du revenu supplémentaire est de
à la valeur du produit marginal du travail de
1 000 $ (100 $/0,l = 1 000 $). Ce montant étant
6 $ l’heure.
inférieur au coût du cours de formation, Julien
b) Dans un marché du travail concurrentiel,
ne devrait pas suivre celui-ci.
le taux salarial serait de 4 $ l’heure pour
a) Les courbes initiales de demande et d’offre 8 000 heures de travail. Le marché de
de travail concurrentielles sont représentées travail concurrentiel entraîne un taux
par Dc et Oc dans la figure 15.6 Solution salarial et un niveau d’emploi plus élevés
(ne tenez pas compte des autres courbes que le marché de monopsone.
pour l’instant). Le taux salarial d’équilibre
est Sc et le niveau d’emploi concurrentiel 9 a) Si le gouvernement fixe un salaire minimum
d’équilibre, QTC. à 4 $ l’heure, pour Papier Pollunet, le coût
marginal du travail devient constant à 4 $
l’heure (jusqu’à 8 000 heures de travail). Par
FIGURE 15.6 SOLUTION
conséquent, en égalisant le coût marginal
du travail et la valeur du produit marginal
du travail, on obtient un taux salarial de 4 $
et 8 000 heures de travail. L’adoption d’un
salaire minimum pour le marché du travail
de monopsone résulte en un taux salarial plus
élevé et un plus fort niveau d’emploi.
b) Le taux salarial maximum que Papier Pollunet
est prêt à payer pour 6 000 heures de travail
est de 6 $ l’heure (la VPm de cette quantité
de travail). Le taux salarial minimum que
le syndicat acceptera pour 6 000 heures
de travail est de 3 $ l’heure (le prix d’offre
de cette quantité de travail).
216 CHAPITRE 15

10 a) Le taux salarial qui sera réellement versé c) Si le syndicat demande un taux salarial
à la conclusion de la négociation dépend des supérieur à 4,50 $, Papier Pollunet
pouvoirs de négociation relatifs de l’entreprise mettra probablement les travailleurs en
et du syndicat. Le pouvoir de négociation lock-out.
dépend des coûts (des lock-out et des grèves) d) Si Papier Pollunet offre un taux salarial
que chaque partie peut faire subir à l’autre inférieur à 4,50 $ l’heure, le syndicat
en cas d’échec des négociations, déclenchera probablement une grève. En
b) Papier Pollunet et le syndicat se partageront général, lorsque des lock-out ou des grèves
la différence entre 6 $ et 3 $ et se mettront se produisent, c’est parce que l’une des parties
d’accord sur un taux salarial de 4,50 $. a mal évalué la situation.
C h a

I
Les marchés financier
et les marchés des
ressources naturelles

♦ Pour maximiser ses profits, l’entreprise a recours


CONCEPTS CLÉS à la règle qui lui dicte d'acheter du capital - elle
achète du capital supplémentaire jusqu’à ce que
la valeur actuelle des futurs flux des valeurs
La structure des marchés financiers
du produit marginal soit égale au prix d’achat.
Ou bien, elle achète du capital supplémentaire
Les marchés financiers coordonnent les plans d’épargne
lorsque la valeur actuelle nette {VAN) est positive.
des ménages (offre de capital financier) et les décisions
d’investissement des entreprises (demande de capital • VAN = valeur actuelle du flux futur des valeurs
financier pour acheter des biens de production). du produit marginal — prix d’achat.

♦ Le prix du capital est le taux d’intérêt. La courbe de demande du marché du capital


est la somme horizontale des courbes de demande
♦ Les intermédiaires financiers facilitent les
des entreprises individuelles.
transactions dans les trois marchés des capitaux:
♦ T r —> 4- VAN —> f quantité de capital demandée.
• les marchés boursiers - marchés
où se négocient les actions des entreprises. ♦ Déplacement de la courbe de demande de capital
• les marchés obligataires — marchés vers la droite résultant de
où se négocient les obligations émises
• progrès techniques.
par les entreprises et par le gouvernement.
• croissance démographique.
• les marchés de prêts - marchés où les ménages,
les entreprises et les intermédiaires financiers
prêtent et empruntent. L’offre de capital

L’offre de capital est déterminée par les décisions


La demande de capital d’épargne des ménages.

Le capital est un stock (variable exprimée en dollars ♦ L’épargne (revenu — consommation) est un flux
à un certain moment). qui vient s’ajouter au patrimoine du ménage
(stock des épargnes passées accumulées).
Investissement net = A stock de capital
= investissement brut {flux de ♦ On appelle choix de portefeuille la décision
l’achat de nouveau capital) que prend le ménage quant à la répartition
— dépréciation {flux de la quantité de sa richesse entre divers actifs financiers.
de capital qui devient obsolète) L’épargne est déterminée par
La demande de capital est déterminée par les choix • le revenu — T revenu —> T épargne.
que font les entreprises pour maximiser leurs profits. • le revenu futur anticipé — lorsque le revenu
♦ Pour maximiser ses profits, l’entreprise a recours courant > revenu futur prévu —> T épargne.
à la règle qui lui dicte de louer la quantité de capital • le taux d’intérêt - effet de substitution
telle que la valeur du produit marginal du capital de T r —> T épargne. L’effet de revenu de
est égale au prix de l’utilisation du capital. Ce prix T r —> T épargne pour le prêteur net et
■l épargne pour l’emprunteur net.
correspond au taux d’intérêt.
218 CHAPITRE 16

La courbe d'offre du marché du capital est la somme ♦ La demande de stock d’une ressource naturelle
horizontale de toutes les courbes d’offre des ménages. est déterminée par le taux d’intérêt anticipé de la
L’offre à court terme est inélastique. L’offre à long détention du stock. Le taux d’intérêt anticipé du
terme est plus élastique. stock d’une ressource naturelle = taux de profit
économique de l’utilisation de la ressource
♦ T r —> T quantité de capital offerte.
+ taux anticipé de l’augmentation du prix
♦ La courbe d’offre de capital se déplace vers la droite de la ressource.
en raison de
Le marché d’une ressource naturelle est en équilibre
• T revenu ou T de la population. de stock lorsque le taux de rendement du marché pour
• répartition du revenu plus inégale ou plus d’autres actions et obligations présentant un risque
forte proportion de ménages d’âge moyen dans comparable est égal au taux d’intérêt anticipé du prix
la population. de la ressource.

♦ Principe de Hotelling — le prix d’une ressource


Les taux d’intérêt et le cours des actions naturelle augmente au même rythme que le taux
d’intérêt d’autres actions et obligations.
Les taux d’intérêt et le prix des actifs s’ajustent de
♦ Prix de demande maximal - prix auquel
manière à ce que la quantité demandée de capital
la demande de consommation de la
soit égale à la quantité offerte de capital. Le taux
ressource naturelle (le flux de la demande)
d’intérêt d’une action est le rendement de l’action
est nulle.
(dividende de l’action/cours de l’action). Le taux
d’intérêt d’une obligation est le rendement de ♦ Le prix d’une ressource naturelle est fixé de manière
l’obligation (paiement de coupon/cours de que son prix anticipé augmente au même rythme
l’obligation). que le taux d’intérêt et atteigne le prix de demande
maximal au moment où le stock de la ressource
♦ Le prix de l’action T si
sera épuisé. Le prix et le taux d’utilisation
• le dividende futur anticipé T. dépendent
• le taux d’intérêt -l.
du taux d’intérêt - T r —» -l prix courant
♦ Ratio cours-bénéfice - ratio du cours actuel et T consommation courante.
de l’action au profit actuel réalisé par action. de la demande de flux - T demande
—» T prix courant et T consommation courante.
• Si le profit futur prévu > profit actuel —»? ratio
du stock de la ressource qui reste - T stock
cours-bénéfice.
actuel —> ? prix courant et T consommation
• Le volume de transactions T avec des attentes
courante.
divergentes quant aux profits futurs.

♦ Il y a prise de contrôle lorsque la valeur boursière


d’une entreprise est < à la valeur actuelle de son
bénéfice anticipé.
RAPPELS
♦ Il y a fusion lorsque deux entreprises sont
convaincues de mettre en commun leurs actifs I Une entreprise qui maximise ses profits utilisera
—> T leur valeur boursière combinée. un facteur supplémentaire aussi longtemps
que l’utilisation du facteur permettra d’augmenter
davantage les recettes que les coûts ; autrement
Les marchés des ressources naturelles
dit, aussi longtemps que la valeur du produit
marginal de la ressource naturelle ( VPm)
Les ressources naturelles sont les moyens de production
sera supérieure à son coût marginal (Cm). La
qu’offre le milieu physique (dons de la nature). Les
quantité qui permet de maximiser les profits
ressources naturelles non renouvelables ne peuvent
sera la quantité à laquelle la valeur du produit
être utilisées qu’une seule fois et elles ne peuvent pas
marginal d’un facteur est égale à son coût
être remplacées. Les ressources naturelles renouvelables
marginal. Étant donné que, dans un marché
peuvent être utilisées indéfiniment (grâce à une saine
concurrentiel, le coût marginal est le prix
gestion) et ne peuvent s’épuiser.
du facteur (PF), cette condition de maximisation
♦ Pour une ressource naturelle, le stock des profits devient VPm = PF dans un marché
est la quantité totale existante de la des facteurs concurrentiel. Cela suppose
ressource ; le flux est le taux d’utilisation que la courbe de demande d’un facteur
de la ressource. est donnée par sa courbe de VPm.
LES MARCHÉS FINANCIERS
ET LES MARCHÉS DES RESSOURCES NATURELLES 219

La condition de maximisation des profits 10 $/100 $ = 0,1 ou 10%. Une baisse du cours
indiquée au rappel 1 s’applique généralement à de l’obligation à 91 $ fera monter le rendement
tous les marchés des facteurs concurrentiels. de l’obligation à 10 $/91 $ = 0,11, soit 11%.
Cependant, dans le cas des marchés financiers, Par conséquent, une baisse du cours des
cette condition doit être adaptée selon que obligations équivaut à une augmentation du
l’entreprise loue ou achète du capital. taux d’intérêt des obligations, et vice versa.
L’entreprise qui maximise ses profits louera
des biens de production jusquà ce que la valeur du 4 La relation inverse entre le taux d’intérêt
produit marginal par période de location soit égale et le prix des actifs s’applique également aux
à la location par période. Il s’agit de la « version cours des actions. Cette relation est légèrement
flux » de la condition de maximisation des plus compliquée parce que le revenu annuel
profits en ce qui concerne l’apport de capital. d’une action — son dividende — est variable
Si, toutefois, une entreprise envisage d’acheter et non fixe. Néanmoins, ceteris paribus,
plutôt que de louer du capital, il faudra adapter une hausse des taux d’intérêt entraînera
la condition de maximisation des profits, car une baisse du cours de l’action, et vice versa.
le capital est généralement exploité pendant plus
d’une période, et les valeurs de produit marginal
qu’il générera seront réparties au fil du temps.
Le prix d’achat, cependant, doit être payé au AUTOÉVALUATION
moment présent. Pour comparer le prix d’achat
(PF) au flux des valeurs du produit marginal,
Vrai/Faux/Incertain
il faudra calculer la valeur actuelle de ce flux.
(Justifiez votre réponse.)
Dans le cas des biens de production qui
seront utilisés au cours de plusieurs périodes,
la condition de maximisation des profits est I Les ressources naturelles ne peuvent être utilisées
la suivante : acheter une unité supplémentaire qu’une seule fois et ne peuvent être remplacées.
de capital jusqu’à ce que la valeur actuelle du
flux des valeurs du produit marginal soit égale
au prix de l’unité de capital. Étant donné que la
valeur actuelle nette ( VAN) d’un investissement
2 Si le prix d’une unité de capital est supérieur
est définie en tant que valeur actuelle du flux
à la valeur actuelle de la valeur de son produit
des valeurs du produit marginal moins le prix de
marginal, une entreprise qui maximise son profit
l’unité de capital, une condition équivalente est :
devrait l’acheter.
acheter des biens d’équipement jusqu’à ce que la
valeur actuelle nette de l’investissement soit de zéro.
Il s’agit de la « version stock » de la condition de
maximisation des profits en ce qui concerne
3 Une nouvelle machine d’une durée d’utilisation
l’apport de capital de l’entreprise. (Si vous devez
d’un an et qui fait augmenter, à la fin de
vous rafraîchir la mémoire au sujet de la valeur
l’exercice, la recette de l’entreprise de 1 050 $
actuelle, reportez-vous au chapitre 9).
se vend au prix de 1 000 $. L’entreprise devrait
Peut-être vous êtes-vous demandé pourquoi les acheter la machine si le taux d’intérêt est de 6%.
analystes financiers associent une augmentation
du prix des obligations à une baisse du taux
d’intérêt. Ce chapitre explique pourquoi les taux
d’intérêt et tous les prix des actifs (y compris 4 Si les actifs financiers nets d’un ménage
le cours des obligations) sont inversement relies. sont négatifs, une baisse du taux d’intérêt
fera augmenter l’épargne de ce ménage.
m ,,. , revenu fixe
Taux d’intérêt = —;—;—- rr
prix de 1 actir

Étant donné un revenu annuel fixe provenant


de l’actif, plus le prix de l’actif est eleve, moins le 5 Selon le principe de Hotelling, le marché

taux d’intérêt ou rendement est élevé. Lorsqu une des actions des hôtels est en équilibre lorsqu’on
obligation de Nortel, par exemple, rapporte un prévoit que le cours de l’action augmentera
revenu annuel fixe de 10 $ et que le cours d une au même rythme que le taux d’intérêt.

obligation de Nortel est de 100 $, le taux


d’intérêt ou rendement de 1 obligation est de
220 CHAPITRE 16'

6 Dire que le cours d’une obligation a augmenté 3 La baisse de la valeur du capital résultant
revient à dire que le rendement de l’obligation de son utilisation au fil du temps est donnée par
a baissé. a) le niveau d’épargne.
b) l’investissement.
c) l’investissement net.
d) l’investissement brut moins
l’investissement net.
7 L’entreprise B prendra le contrôle de l’entreprise
e) la valeur actuelle nette.
A si elle estime que la valeur boursière de
l’entreprise A est supérieure à la valeur actuelle 4 Une entreprise prévoit payer un dividende
des profits anticipés de l’exploitation de de 8 $ par action chaque année, indéfiniment.
l’entreprise A. Si le taux d’intérêt du marché est de 8 %, quel
sera le prix d’une action de cette entreprise ?
a) 8 $
b) 8,64 $
8 Bien que les marchés puissent répartir de manière c) 64$
optimale les ressources qui ne peuvent être d) 80$
remplacées ni reproduites, ils ne peuvent répartir e) 100$
les ressources naturelles non renouvelables de
5 Le rendement d’une action augmente lorsque
manière optimale.
a) le dividende augmente ou que le cours
de l’action augmente.
b) le dividende augmente ou que le cours
de l’action diminue.
9 Si un substitut du pétrole peu onéreux était c) le dividende diminue ou que le cours
mis au point, nous pourrions prévoir une baisse de l’action augmente.
du prix de demande maximal du pétrole. d) le dividende diminue ou que le cours
de l’action diminue.
e) Aucune de ces réponses.

6 L’obligation A est plus risquée que l’obligation B.


I0 Le modèle économique des ressources naturelles
Dans ce cas, en situation d’équilibre,
non renouvelables suppose que le marché fournira
a) le taux d’intérêt de A doit être supérieur
une incitation automatique à préserver ces
à celui de B.
ressources au fur et à mesure de l’épuisement
b) le taux d’intérêt de A doit être inférieur
de leur stock.
à celui de B.
c) le taux d’intérêt de A doit être égal
à celui de B.
d) personne ne souhaite acheter l’obligation A.
e) seuls les acheteurs qui préfèrent le risque
Questions à choix multiple
achèteront l’obligation A.

7 Laquelle des ressources suivantes est


1 Lequel des éléments suivants est un exemple une ressource naturelle non renouvelable ?
de capital physique ? a) Le charbon.
a) Une pelle mécanique. b) La terre.
b) Une action de IBM. c) L’eau.
c) La monnaie. d) Les arbres.
d) Une obligation de General Motors. e) Aucune de ces réponses.
e) Toutes ces réponses.
8 Le rendement du stock d’une ressource naturelle est
a) le taux d’intérêt sur le prêt utilisé pour
2 Lequel des éléments suivants est un exemple acheter la ressource.
de stock ? b) la valeur du produit marginal de la ressource.
a) Un investissement. c) la valeur du produit marginal de la ressource
b) La dépréciation. divisée par son prix.
c) Le capital. d) la valeur du produit marginal de la ressource
d) Le revenu. multipliée par le taux d’intérêt du marché.
e) Aucune de ces réponses. e) le taux de variation du prix de la ressource.
LES MARCHÉS FINANCIERS
ET LES MARCHÉS DES RESSOURCES NATURELLES 221

9 Quelle entreprise du tableau 16.1 a le ratio 13 Si le taux d’intérêt augmente,


cours-bénéfice le plus élevé ? a) l’effet de substitution encourage le ménage
a) L’entreprise A. à épargner davantage seulement s’il est
b) L’entreprise B. un emprunteur net.
c) L’entreprise C. b) l’effet de substitution encourage le ménage
d) Toutes ont le même ratio. à épargner davantage seulement s’il est un
e) L’information est insuffisante pour faire prêteur net.
le calcul. c) l’effet de revenu encourage le ménage
à épargner davantage seulement s’il est
un emprunteur net.
TABLEAU 16.1 d) l’effet de revenu encourage le ménage
à épargner davantage seulement s’il est
Données Entreprise Entreprise Entreprise un prêteur net.
A B C e) Les énoncés b et d sont vrais.

Profits de l’exercice
précédent par action 8$ 10$ 5$ 14 On appelle la décision que prennent les ménages
quant à la répartition de leur richesse entre
Dividende divers actifs financiers et quant à leur capacité
par action 4$ 2$ 5$ d’endettement
Valeur de l'action 80$ 20$ 25$ a) un investissement.
b) une décision de bilan.
Nombre d’actions
c) un choix de portefeuille.
en circulation 1 000 10 000 3 000
d) le problème cours-bénéfice.
Valeur actuelle e) le problème de la valeur actuelle nette.
du profit anticipé 200 000 $ 200 000 $ 50 000 $
15 Si le marché du stock d’une ressource naturelle
est en équilibre, le prix de cette ressource
10 Quelle entreprise du tableau 16.1 a le rendement a) augmentera à un rythme égal au taux
le plus élevé ? d’intérêt.
a) L’entreprise A. b) diminuera à un rythme égal au taux d’intérêt.
b) L’entreprise B. c) est égal au prix de demande maximal
c) L’entreprise C. de la ressource.
d) Toutes ont le même rendement. d) est égal à la valeur du produit marginal
e) L’information est insuffisante pour faire de la ressource.
le calcul. e) est égal au prix de Hotelling de la ressource.

I I Quelle entreprise du tableau 16.1 risque le plus I6 Le prix courant d’une ressource naturelle
de faire l’objet d’une prise de contrôle? est plus élevé lorsque
a) L’entreprise A. a) la valeur du produit marginal est plus faible.
b) L’entreprise B. b) le stock restant de la ressource est plus élevé.
c) L’entreprise C. c) le taux d’intérêt est moins élevé.
d) Toutes courent le même risque. d) le prix de demande maximal est moins élevé.
e) L’information est insuffisante pour faire e) Aucune de ces réponses.
le calcul.
I7 Une obligation a une valeur de rachat de 100 $
I2 Plus le taux d’intérêt est élevé, et rapporte 10 $ d’intérêt chaque année. Si vous
a) plus la valeur actuelle nette avez payé l’obligation 100 $ et que son prix
d’un investissement est élevée. courant est de 98 $, quel est le rendement
b) moins la valeur actuelle du flux des valeurs actuel ?
du produit marginal d’un investissement a) 2%

est élevée. b) 8,2%


c) plus la quantité de capital demandée c) 10%
d) 10,2%
est élevée.
d) plus la valeur du produit marginal du capital e) 12,2%

est élevée.
e) moins la valeur du produit marginal
du capital est élevée.
222 CHAPITRE 16

18 Le cours d’une action de la société XYZ b) le coût du capital est supérieur à la valeur
est de 80 $, et le paiement de dividende courant actuelle du flux de la valeur du produit
est de 4 $. Quel est le rendement de l’action? marginal.
a) 3,2% c) le coût du capital est égal à la valeur actuelle
b) 4% du flux de la valeur du produit marginal.
c) 5% d) la valeur actuelle nette est supérieure à zéro.
d) 8% e) Aucune de ces réponses.
e) 20%
23 Les entreprises investiront aussi longtemps
19 II se produit un événement qui, de l’avis de que la valeur actuelle nette
tous, va entraîner la baisse du profit anticipé de a) sera supérieure au taux d’intérêt.
la société XYZ. Dès que l’événement est connu, b) sera égale au taux d’intérêt.
le cours des actions de XYZ c) sera inférieure au taux d’intérêt.
a) fluctue à la hausse ou à la baisse avec d) sera positive.
un volume élevé de transactions. e) sera nulle.
b) fluctue à la hausse ou à la baisse avec
un faible volume de transactions. 24 Lequel des événements suivants ferait déplacer
c) baisse graduellement avec un volume élevé la courbe d’offre de capital vers la droite ?
de transactions. a) Une augmentation de la proportion
d) baisse rapidement avec un faible volume des jeunes ménages dans la population.
de transactions. b) Une augmentation du taux d’intérêt.
e) baisse rapidement avec un volume élevé c) Une diminution du taux d’intérêt.
de transactions. d) Une augmentation du revenu moyen
des ménages.
e) Une augmentation de la valeur du produit
20 La prise de contrôle d’une entreprise se produira
marginal du capital.
probablement si
a) la valeur boursière de l’entreprise est
25 Une machine qui coûte 2 000 $ générera une
supérieure à la valeur actuelle du profit futur
valeur de produit marginal de 1 100 $ après un
anticipé provenant de l’exploitation de
an et le même montant après deux ans. Quelle
l’entreprise.
est la valeur actuelle nette de la machine si le
b) la valeur boursière de l’entreprise est inférieure
taux d’intérêt est de 10% ?
à la valeur actuelle du profit futur anticipé
a) -90,91 $
provenant de l’exploitation de l’entreprise.
b) -49,90 $
c) le profit actuel de l’entreprise est supérieur
c) 0$
à la valeur actuelle du profit futur anticipé
d) 90,91 $
provenant de l’exploitation de l’entreprise.
e) 1 909,09 $
d) le profit actuel de l’entreprise est inférieur
à la valeur actuelle du profit futur anticipé
provenant de l’exploitation de l’entreprise. Problèmes à court développement
e) les taux d’intérêt sont élevés.
1 Pourquoi la quantité de capital demandée
augmente-t-elle lorsque le taux d’intérêt
21 On appelle les institutions qui ont pour
diminue?
principales activités d’accepter des dépôts,
de consentir des prêts et d’acheter des valeurs
2 Supposons que les entreprises A et B aient réalisé
mobilières
le même montant de profit par action durant
a) des courtiers.
le dernier exercice, mais que le cours d’une action
b) des intermédiaires financiers.
de l’entreprise A soit plus élevé que celui de
c) des compagnies d’assurance.
l’entreprise B. Que peut-on alors dire sur les
d) des monopsones.
ratios cours-bénéfice des deux entreprises ? sur
e) des caisses de retraite.
les profits futurs anticipés des deux entreprises ?

22 Une entreprise qui maximise ses profits choisira 3 Pétrole Exxoff cause, dimanche soir, un
d’acheter une unité supplémentaire de capital déversement accidentel de pétrole qui risque
chaque fois que d’être très coûteux. Que pouvons-nous déduire
a) la valeur actuelle du flux de la valeur du consensus des actionnaires participants
du produit marginal est supérieure à zéro. si lundi: le cours de l’action d’Exxoff diminue
LES MARCHES FINANCIERS
ET LES MARCHÉS DES RESSOURCES NATURELLES 223

sensiblement à l’ouverture de la bourse et ne varie anticipé de Bogazon inc., qui est maintenant de
plus ? le cours de l’action d’Exxoff fluctue 600 000 $.
légèrement toute la journée avec un volume a) Si le cours des actions de Bogazon inc.
élevé de transactions ? ne varie pas, une prise de contrôle est-elle
probable ? Si la réponse est oui, quel sera le
4 Pourquoi un taux d’intérêt élevé implique-t-il montant le plus élevé qu’un acheteur sera prêt
un prix courant plus bas pour une ressource non à offrir pour une action ?
renouvelable ? b) Si le cours des actions s’aligne maintenant sur
le cours du marché et que le dividende reste
5 L’entreprise Bogazon inc. a commencé l’année de 7,50 par action, quel est le rendement des
avec un stock de capital de 100 000 $. Ce stock actions ?
de capital s’est déprécié de 12% durant l’année.
9 Le zapton est une ressource naturelle non
Bogazon inc. a également acheté de nouvelles
renouvelable qui est presque épuisée. Il ne reste
tondeuses à gazon d’une valeur de 10 000 $.
plus que 1215 barils de zapton. Le tableau
Quel a été l’investissement brut de Bogazon
16.3 indique le barème des valeurs du produit
durant l’année? son investissement net?
marginal du zapton.

<3> 6 Bogazon inc. envisage l’achat de tondeuses TABLEAU 16.3


à gazon supplémentaires. Ces tondeuses ont
une durée d’utilisation de deux ans et coûtent Barils Valeur du

chacune 120 $. Le tableau 16.2 présente les de zapton produit marginal

valeurs du produit marginal pour chaque année. par an (en dollars)

0 14,64
TABLEAU 16.2 133 13,31
254 12,10
Nombre de VPm la VPm la
tondeuses première seconde VAN VAN VAN 364 1 1,00
à gazon année année (r= 0,05) (r= 0,10) (r = 0,15) 464 10,00
555 9,09
ire 100 80
2e 80 64
a) Tracez un graphique de la courbe de demande
3e 72 62
de zapton (en tant que flux).
b) Quel est le prix de demande maximal
a) Remplissez le tableau 16.2 en calculant les
du zapton ?
valeurs actuelles nettes ( VAN) lorsque le taux
d’intérêt est de 5% (r = 0,05), 10% (r = 0,10) & 10 Poursuivons avec le problème du zapton.
et 15% (r - 0,15). Supposons que le taux d’intérêt est de 10%.
b) Combien de tondeuses Bogazon inc. achètera- a) Quel est le prix courant d’équilibre d’un
t-elle si le taux d’intérêt est de 15%? de baril de zapton ? Comment avez-vous obtenu
10%? de 5%? ce résultat ?
c) Construisez la courbe de demande de b) En supposant que l’année courante est l’année
tondeuses à gazon approximative de Bogazon 1, remplissez le tableau 16.4 pour chaque
inc. en représentant graphiquement les trois année jusqu’à ce que le stock de zapton soit
points indiqués dans la partie b et en les épuisé.
reliant par une courbe.
TABLEAU 16.4
7 La valeur actuelle des profits futurs anticipes Stock initial Stock final
de Bogazon inc. est de 400 000 $. Il y a Année Prix de zapton de zapton
8 000 actions de Bogazon inc. en circulation.
1 1 215
Le dividende payé pour l’année en cours a ete
de 7,50 $ l’action.
2
a) Quel est le cours de l’action de Bogazon inc. ?
b) Quel est le rendement des actions ?
3

8 La ville de Métropolis (où se trouve Bogazon 4


inc.) lance le concours «Les plus belles pelouses»
et offre un prix en argent. Cela a pour effet de
faire augmenter la valeur actuelle du profit futur
224 CHAPITRE 16

13 d Parce que le revenu des ménages T grâce


F
1
RÉPONSES aux prêts. L’effet de substitution —> toujours
T épargne.
14 C Définition.
Vrai/Faux/Incertain
15 a Il s’agit du principe de Hotelling.
(Justifiez votre réponse.)
& 16 c L’inverse de a et b est vrai. Aucune relation
directe avec le prix courant et le prix
1 I Vrai pour les ressources naturelles non de demande maximal.
renouvelables, faux pour les ressources 17 d Rendement = gains annuels/cours
naturelles renouvelables. des obligations.
2 F L’inverse est vrai. 18 c Rendement = dividende/cours de l’action.
<*> 3 F VA = 1 $/ (1 + 0,06) = 990,56 $.
050 19 d si profit futur anticipé —» 4- prix. Entente
VAN = 990,56 $ - 1 000 $ = -9,44 $. —> faible volume.
La VAN étant négative, ne pas acheter 20 b Coût de l’achat < valeur de l’entreprise.
la machine. 21 b Définition, c est un exemple d’intermédiaire
& 4 I Dépend de la force relative de l’effet financier.
de revenu (T épargne) et de l’effet 22 d Lorsque la valeur actuelle du flux de la valeur
de substitution (X épargne). du produit marginal > coût du capital.
5 F Vrai pour le marché des ressources naturelles. 23 d a, b et c sont des comparaisons illogiques.
6 V Rendement de l’obligation = 24 d a—> déplacement vers la gauche, b et c
(gain fixe)/(cours de l’obligation). —> mouvement le long de la courbe d’offre.
7 F L’inverse est vrai - si la valeur boursière e fait déplacer la courbe de demande.
de l’entreprise A est inférieure à ... 0 25 a VAN= [ 1 100 $/(l,l) + 1 100 $/l,l)2]
8 F Les marchés peuvent être efficients en -2 000 $.
l’absence d’effets externes. Au fur et
à mesure que la ressource est épuisée,
le prix T —> f quantité demandée et Problèmes à court développement
T utilisation de substituts.
9 V Une plus faible demande de pétrole fait 1 Les entreprises qui maximisent leurs profits
que les consommateurs n’achèteront plus demanderont du capital aussi longtemps que
de pétrole même à un prix peu élevé. la valeur actuelle du flux de la valeur future du
10 V Parce que le prix des ressources va T. produit marginal découlant du nouveau capital
sera supérieure au prix du nouveau capital ;
autrement dit, aussi longtemps que sa valeur
Questions à choix multiple actuelle nette sera positive. Étant donné qu’un
plus faible taux d'intérêt suppose que la valeur
1 a Les autres sont du capital financier. actuelle de tout flux futur donné de la valeur
2 c Les autres sont des flux. du produit marginal restera supérieure, la valeur
3 d Est égal à la dépréciation. actuelle nette sera positive pour un plus grand
4 e r - dividende/cours de l’action. 0,08 = 8 $/ nombre de biens de production supplémentaires
cours de l’action. Cours de l’action = 8 $/ 0,08. et, par conséquent, les entreprises achèteront
5 b Rendement = dividende/cours de l’action. plus de capital. Ainsi, la quantité de capital
6 a II est nécessaire que le rendement soit plus demandée augmente à mesure que baisse le taux
élevé pour compenser le risque plus grand d’intérêt.
si l’investisseur souhaite être porteur
d’obligations. 2 Le ratio cours-bénéfice est le prix courant d’une
7 a Les autres sont des ressources naturelles action divisé par le montant des profits réalisés
renouvelables. par action durant l’exercice précédent. Par
8 e Voir le texte du manuel. conséquent, le ratio cours-bénéfice de l’action
9 a Cours de l’action/profit le plus récent de l’entreprise A est supérieur au ratio
par action = 80 $/8 $. cours-bénéfice de l’action de l’entreprise B.
10 c Dividende/cours de l’action = 5 $/25 $. Cela suppose que, bien que les profits (par
Ô I I a Valeur boursière de l’entreprise (80 $/action action) réalisés durant l’exercice précédent
X 1 000 actions = 80 000 $) < valeur actuelle par les deux entreprises puissent être les mêmes,
• du profit anticipé (200 000 $). on anticipe que les futurs profits (par action)
I 2 b L’inverse de a et c est vrai, r n’a pas d’effet de l’entreprise A seront supérieurs à ceux
sur la VPm du capital. de l’entreprise B.
LES MARCHÉS FINANCIERS
ET LES MARCHÉS DES RESSOURCES NATURELLES 125

Le cours d’une action représente la valeur valeurs des VPmx et VPm2 sont présentées
actuelle des profits futurs anticipés, et un cours au tableau 16.2 pour la première, la seconde
plus élevé signifie donc que les profits futurs et la troisième tondeuse à gazon, et PT est de
anticipés seront plus élevés. 120 $. On obtient les valeurs des VAN données
au tableau 16.2 Solution en utilisant ces valeurs
Une baisse sensible du prix de l’action dans l’équation qui précède et en évaluant
d’Exxoff avec un faible volume de transactions l’expression des autres valeurs de r, le taux
suppose qu’il est évident pour tout le monde d’intérêt.
que le déversement accidentel de pétrole
entraînera une importante baisse de la valeur TABLEAU 16.2 SOLUTION
actuelle des profits futurs d’Exxoff.
Si, en revanche, le stock d’Exxoff donne Nombre de YPm la VPm la
tondeuses première seconde VAN VAN VAN
lieu à un grand volume de transactions avec à gazon année année (r = 0,05) (r = 0,10) (r = 0,15)
une faible variation du prix net, cela suppose
|re 100 80 47,80 37,02 27,45
que les avis divergent concernant les conséquences
du déversement de pétrole sur la rentabilité 2e 80 64 14,24 5,62 -2,04
future d’Exxoff. 3e 72 62 4,81 -3,31 - 10,51

4 En situation d’équilibre, le prix courant b) Si le taux d’intérêt est de 15 %, Bogazon


de la ressource sera le prix qui, s’il continue achètera une seule tondeuse à gazon

d’augmenter à un taux égal au taux d’intérêt, supplémentaire, puisque la seconde tondeuse

atteindra le prix de demande maximal au à gazon a une valeur actuelle nette négative.

moment où le stock de la ressource sera épuisé. Si le taux d’intérêt est de 10%, Bogazon

Un taux d’intérêt plus élevé suppose que l’on achètera deux tondeuses à gazon, et si le taux

prévoit une augmentation du prix plus rapide. d’intérêt est de 5 %, Bogazon achètera trois

Par conséquent, si le prix courant doit atteindre tondeuses à gazon.

le prix de demande maximal au moment c) La courbe de demande approximative des

approprié, il devra partir d’un prix inférieur tondeuses à gazon est présentée à la figure
16.1. La courbe indique que, à un taux
au prix courant.
Autre réponse dans la perspective du choix d’intérêt de 15 %, la quantité demandée sera
de une tondeuse à gazon. À un taux d’intérêt
de portefeuille : plus le taux d’intérêt augmente,
de 10%, la quantité demandée sera de deux
plus l’attrait relatif de l’actif financier augmente,
tondeuses à gazon, et à un taux d’intérêt de
ce qui réduit la demande de la ressource et fait
5 %, la quantité demandée sera de trois
donc baisser son prix.
tondeuses à gazon.

5 L’investissement brut est de 10 000 $, soit


FIGURE 16.1
le montant de l’achat de nouveau capital.
L’investissement net est de -2 000 $, soit
le montant de l’investissement brut moins
la dépréciation de 12 000 $.

<3> 6 a) Le tableau 16.2 rempli est présenté au tableau


16.2 Solution. On calcule la VAN en tant que
valeur actuelle du flux des valeurs du produit
marginal résultant d’un investissement moins
le coût de l’investissement. Pour les tondeuses
à gazon d’une durée d’utilisation de deux ans,
on calcule la VAN à l’aide de 1 équation
suivante :

Quantité de tondeuses à gazon


VPmx VPm2 _p
VAN =
1+r (1+r)2 7
7 a) Le cours d’une action de Bogazon
Dans cette équation, la VPmx et la VPm2 correspondra à la valeur actuelle des profits

sont les valeurs du produit marginal de la futurs anticipés pour l’entreprise divisée
première et de la deuxième annee, respectivement, par le nombre d’actions en circulation :

et PT est le prix d’une tondeuse a gazon. Les 50 $ = 400 000 $/8 000.
226 CHAPITRE 16

b) Le rendement des actions est de 15 %. & 10 a) Si le taux d’intérêt est de 10%, le prix courant
Il s’agit du dividende (7,50 $) exprimé en d’un baril de zapton est de 10 $. Nous savons
pourcentage du prix d’une action (50 $). que le prix courant (d’équilibre) doit être
tel que, si le prix du zapton augmente
8 a) Oui, il y aura prise de contrôle, parce que au rythme de 10 % par an (égal au taux
la valeur boursière (400 000 $) est inférieure d’intérêt), le stock sera épuisé au moment
à la valeur actuelle des profits futurs anticipés où le prix de demande maximal sera atteint.
(600 000 $). Le prix le plus élevé qu’un Nous pouvons déterminer le prix courant
acheteur sera prêt à payer pour une action en procédant à rebours à partir des éléments
est de 75 $, soit la valeur actuelle des profits suivants: 1) le prix de demande maximal
futurs anticipés divisée par le nombre est de 14,64 $ et 2) le stock restant est de
d’actions (600 000 $/8 000). 1215 barils.
b) Le rendement des actions sera maintenant Étant donné que le prix du zapton
de 10%. Il s’agit du dividende (7,50 $) augmente au rythme de 10 % par an,
exprimé en pourcentage du nouveau cours le prix de l’année précédant celle où le prix
d’une action (75 $). de demande maximal sera atteint doit être
de 14,64 $ (prix de demande maximal) divisé
9 a) La courbe de demande de zapton est donnée par 1,10 (1 + le taux de croissance du prix de
par la courbe de la valeur du produit marginal. 10%), soit 13,31 $■ D’après le tableau 16.3
Elle est présentée à la figure 16.2. (et sachant que le stock de zapton sera épuisé
lorsqu’on atteindra le prix de demande
FIGURE 16.2 maximal de 14,64 $), nous pouvons déduire
que 133 barils de zapton seraient achetés cette
année-là. De la même manière, en procédant
à rebours jusqu’à ce que 1215 barils de zapton
aient été achetés, nous découvrons que
le prix d’équilibre courant doit être de 10 $.
b) En suivant la procédure indiquée dans la
partie a, nous pouvons remplir le tableau
16.4, comme le montre le tableau 16.4 Solution.

TABLEAU 16.4 SOLUTION

Stock initial Stock final


Année Prix de zapton de zapton

1 10,00 1 215 751


2 1 1,00 751 387
b) Le prix de demande maximal du zapton
3 12,10 387 133
est de 14,64 $. Ce prix est suffisamment
4 13,31 133 0
élevé pour que la ressource ne soit plus
5 14,64 0
du tout utilisée.
L’incertitude
et l’information

L’assurance
CONCEPTS CLÉS
L’achat d’une police d’assurance permet de réduire
le risque.
L’incertitude et le risque
♦ L’assurance fonctionne en regroupant les risques
Incertitude — plusieurs événements peuvent se produire, de manière que l’ensemble des résultats défavorables
mais nous ne savons pas lesquels. Les concepts soit relativement certain. Les compagnies d’assurance
connexes sont : perçoivent des primes auprès de tous pour couvrir
le coût total des accidents, mais elles ne versent
♦ probabilité - chiffre compris entre 0 et 1 qui
des indemnisations qu’aux victimes de pertes.
mesure le nombre de chances qu’un événement
donné se produise ; ♦ Les personnes qui éprouvent une aversion pour
le risque trouvent rentable d’acheter une police
♦ risque - permet d’estimer la probabilité
d’assurance et elles sont disposées à payer pour
des événements incertains.
-l le risque. Elles paient une prime peu élevée pour
Face à l’incertitude, les individus maximisent s’assurer quelles ne subiront pas de grosse perte.
leur utilité escomptée — l’utilité moyenne de tous
les événements possibles. L’utilité escomptée peut
L’information
différer de Futilité réelle (utilité qu’obtient réellement
l’individu lorsqu’un événement se produit).
L’information économique permet de réduire
♦ Utilité du patrimoine - utilité qu’accorde l’incertitude, mais la collecte de cette information
une personne à un certain niveau de richesse. a un coût d’opportunité — le coût de l’information.
T patrimoine —» T utilité, mais X utilité marginale L’information étant coûteuse, les individus
du patrimoine La courbe d’utilité du patrimoine l’économisent.
mesure le coût du risque.
♦ Les consommateurs qui cherchent à payer le plus
♦ Coût du risque = montant dont le patrimoine bas prix possible appliquent la règle de la recherche
escompté doit T pour —> la même utilité optimale:
escomptée qu’une situation sans risque.
• poursuivre la recherche du plus bas prix jusqu’à
♦ Plus grande est l’aversion pour le risque (preference ce que l’avantage marginal que l’on prévoit tirer
pour les situations les moins incertaines), plus de cette recherche = coût marginal de la recherche,
l’utilité marginale du patrimoine -l rapidement puis acheter; ou
(plus la courbe de l’utilité du patrimoine est • poursuivre la recherche jusqu’à ce que l’on
abrupte) et plus le risque est coûteux. trouve un prix < prix de réserve (le prix le plus
élevé que le consommateur est disposé à payer),
♦ Pour une personne neutre face au risque, le coût
puis acheter.
du risque est nul et la courbe d utilité du patrimoine
est une droite.
228 CHAPITRE 17'

Les vendeurs ont recours à la publicité pour persuader La gestion du risque


et informer. sur les marchés financiers
♦ La qualité des biens d’expérience ne peut être
Il est possible de réduire le risque sur les marchés
évaluée qu’une fois que ces biens ont été achetés
financiers en diversifiant le portefeuille d’actions
—> la publicité sur les biens d’expérience vise
(ne pas mettre tous ses œufs dans le même panier).
habituellement à persuader.
Dans un marché efficient, s’il est possible de faire
♦ La qualité des biens de recherche peut être évaluée *
des prévisions, elles seront faites, et elles influeront
avant l’achat de ces biens —A la publicité sur les
sur le prix courant du marché.
biens de recherche vise habituellement à informer.
♦ Prix courant = cours futur prévu; englobe toute
La publicité —> T coûts, T concurrence et
l’information disponible ;
—> probablement -l coûts si elle permet de réaliser
des économies d’échelle. ♦ Aucune prévision disponible sur les possibilités
de profits.

L’information privée Le marché boursier est un marché efficient. Le cours


des actions fluctue lorsque de l’information nouvelle
L’information privée est à la disposition d’une personne, devient disponible.
mais elle est trop coûteuse pour que d’autres puissent
se la procurer —A deux problèmes :

♦ aléa moral — une fois l’entente conclue, l’une


des parties peut agir de manière à en retirer RAPPELS
T d’avantages que l’autre partie, moins bien
I L’incertitude joue un rôle dans presque toutes
informée.
les décisions économiques. Pourtant, dans les
♦ anti-sélection — tendance des gens à utiliser chapitres précédents, nous n’en avons pas tenu
des renseignements exclusifs qui leur permettent compte dans notre analyse des choix des
de conclure des ententes qui sont à leur avantage consommateurs et des entreprises. Pourquoi ?
et qui défavorisent la partie moins bien informée. Pour simplifier et parce que, habituellement,
l’incertitude n’invalide pas les règles économiques
On peut surmonter les problèmes liés à l’information
au moyen de signaux, qui transmettent l’information importantes. Par exemple, la règle clé pour
comprendre la maximisation des profits est
que le marché peut utiliser. Les garanties sur les voitures
d’occasion, les renseignements sur le crédit et que l’entreprise choisit le volume de production
auquel Cm = Rm. L’entreprise peut avoir
les limites de prêts sur le marché des prêts ainsi que
de l’incertitude quant au Cm et à la Rm, mais
le dossier de conduite et les franchises des compagnies
elle tente toujours de trouver le volume de
d’assurance automobile constituent des exemples
de signaux. production auquel ils seront égaux. Si nous
avions intégré dès le début l’incertitude dans
notre analyse, la règle de maximisation des
L’information asymétrique profits aurait été bien plus compliquée, et il
sur les marchés du travail aurait été plus difficile de voir la règle Cm = Rm.
Toute stratégie correcte de construction de
L’information asymétrique sur les marchés du travail modèle ne tient pas compte initialement de
—> difficulté pour l’employeur de surveiller les efforts l’incertitude. Souvenez-vous, nous avons vu au
de ses employés. chapitre 1 que les modèles sont des représentations

♦ L’employeur a recours à des modes de très simplifiées du monde réel axées sur des

rémunération (commissions de ventes, forces essentielles (comme la maximisation des

rémunération selon le classement au tournoi, profits) et qui font abstraction de certains détails

participation aux bénéfices) pour inciter les moins importants du monde réel (comme
l’incertitude). Avant de tirer des conclusions
employés à maximiser les profits sans surveillance.
sur le monde réel à partir de modèles simples,
il importe de voir si, en y intégrant des facteurs
complexes du monde réel, comme l’incertitude,
nous obtiendrons les mêmes résultats. Le présent
chapitre sur l’incertitude fournit quelques outils
L'INCERTITUDE ET L'INFORMATION 229

de base qui peuvent être utilisés dans des cours L’utilité escomptée est l’utilité moyenne
plus avancés pour « compliquer » ou enrichir les de tous les résultats possibles. On la calcule de la
modèles simples des choix des entreprises et des même manière. Si les utilités associées à chaque
consommateurs. possibilité de patrimoine sont Uv U2 et U5,
L’existence du risque et de l’incertitude est une respectivement, alors l’utilité escomptée est égale à
caractéristique objective de la réalité. La manière (U\ Xpi) + (U2 xp2) + (f/3 X^>3).
dont les responsables des décisions y font face
dépend, toutefois, de leurs attitudes face au risque. Lorsqu’on analyse les choix faits dans
Certaines personnes éprouvent plus d’aversion l’incertitude, on doit d’abord calculer le
pour le risque que d’autres. patrimoine escompté. Cependant, en fin
Le barème futilité du patrimoine d’une de compte, la décision se fonde sur l’utilité
personne reflète son niveau d’aversion pour escomptée associée au patrimoine escompté.
le risque. Selon les économistes, les barèmes
d’utilité du patrimoine de la plupart des gens
présentent une utilité marginale décroissante
au fur et à mesure que leur richesse s’accroît. AUTOÉVALUATION
Toutefois, dans le cas de deux personnes,
Alexandre et Madeleine, par exemple, l’utilité
Vrai/faux/lncertain
marginale du patrimoine d’Alexandre peut
(Justifiez votre réponse.)
diminuer plus rapidement que l’utilité marginale
du patrimoine de Madeleine. Alexandre aura
une plus grande aversion pour le risque que I Le risque est une situation où plusieurs
Madeleine et il hésitera davantage à se lancer événements peuvent se produire, sans que
dans des entreprises risquées qui offrent la nous sachions lesquels se produiront.
possibilité d’un rendement élevé au coût d’une
plus grande probabilité de perte. Ceteris paribus,
Alexandre sera moins enclin à se lancer dans
une entreprise hasardeuse parce qu’il accordera 2 Plus l’utilité marginale de votre patrimoine
une importance relativement plus grande diminue rapidement, moins vous avez d’aversion
à la diminution probable du patrimoine pour le risque.
due à une perte qu’à l’augmentation probable
du patrimoine due à un gain. À cause de ces
pondérations, il est probable qu’une personne
qui a de l’aversion pour le risque sera également 3 L’utilité marginale du patrimoine d’une
plus susceptible de s’assurer contre les pertes. personne neutre face au risque est constante.
De nombreux marchés autres que celui
de l’assurance (p. ex. le marché boursier) existent
en partie pour permettre aux personnes qui
ont de l’aversion pour le risque de se décharger 4 Si Petro-Canada effectue des forages dans une

du risque sur les personnes qui ont moins région non cartographiée des Territoires du
Nord-Ouest sans aucune idée de la probabilité
d’aversion pour le risque.
d’y trouver du pétrole, Petro-Canada fait face
Le patrimoine escompté et 1 ’utilité escomptée
à l’incertitude plutôt qu’au risque.
sont deux importants concepts pour 1 analyse
des choix faits dans l’incertitude. Le patrimoine
escompté (appelé parfois valeur esperee)
est le patrimoine moyen provenant de toutes
5 La publicité des biens de recherche vise plutôt
les sources possibles. Il correspond a la moyenne
à informer qu’à persuader.
pondérée du patrimoine associe a chaque
résultat possible, les facteurs de pondération
étant les probabilités de chaque résultat.
Par exemple, s’il existe trois possibilités de
6 Si vous jouez avec des allumettes parce que vous
produire du patrimoine Ap Pi et A3, et que les
avez une assurance contre les incendies, il y a un
probabilités qui leur sont associées sont p\, pi et
problème d’anti-sélection.
p3, respectivement, alors le patrimoine escompté
est égal à
(A, X px) + (A2 X p2) + (A3 X p3).
230 CHAPITRE 17

7 Un mode de rémunéradon efficient permet 4 On appelle les biens dont il n’est possible
de maximiser les profits de l’entreprise ainsi d’évaluer la qualité qu’après les avoir achetés
que la rémunération de l’employé. a) des biens d’information privée.
b) des biens de recherche.
c) des biens d’expérience
d) des biens inférieurs.
8 Les portefeuilles d’actions diversifiés sont préférables e) des citrons.
aux portefeuilles d’actions non diversifiés.
5 Le prix de réserve d’un acheteur est
a) le prix le plus bas que l’acheteur est disposé
à payer.
9 Dans un marché efficient, le prix courant b) le prix le plus élevé que l’acheteur est disposé
est rarement égal au prix futur prévu. à payer.
c) le prix égal à l’avantage marginal escompté
et au coût marginal de la recherche.
d) a et c
10 Lorsque les prix d’un marché efficient sont e) b et c.
instables, les prévisions quant aux prix futurs
sont aussi instables. 6 Les garanties permettent de distinguer les
voitures en bon état de marche des citrons
seulement si
a) la garantie ne coûte rien au concessionnaire.
Questions à choix multiple b) le prix de la voiture n’est pas touché, que
le concessionnaire donne une garantie ou non.
1 Plus l’utilité marginale du patrimoine c) il est profitable pour les vendeurs de voitures
d’une personne diminue rapidement, en bon état d’offrir une garantie et si offrir
a) plus cette personne est prête à prendre une garantie est trop coûteux pour les vendeurs
des risques. de citrons.
b) plus cette personne est neutre face aux risques. d) il est profitable pour les vendeurs de voitures
c) plus cette personne a de l’aversion pour en bon état et de citrons d’offrir une garantie.
le risque. e) a et b.
d) plus il est probable que cette personne
ait un problème d’aléa moral. 7 Selon les barèmes d’utilité du patrimoine
e) moins il est probable que cette personne du tableau 17.1,
achète une police d’assurance. a) Chloé a plus d’aversion pour le risque
qu’Éliane.
2 La valeur espérée d’un jeu qui offre 50 %
b) Éliane a plus d’aversion pour le risque
de chances de gagner 60 $ et 50% de chances
que Chloé.
de ne rien gagner est de
c) Chloé est neutre face au risque alors qu’Eliane
a) 10$.
a de l’aversion pour le risque.
b) 20 $.
d) Eliane est neutre face au risque alors
c) 30 $.
que Chloé a de l’aversion pour le risque.
d) 60 $.
e) il est impossible de calculer l’aversion
e) Aucune de ces réponses.
pour le risque et la neutralité face au risque.
3 Sur le graphique d’une courbe d’utilité du
patrimoine normale, le patrimoine étant porté TABLEAU 17.1 UTILITÉ DU PATRIMOINE
sur l’axe horizontal et l’utilité sur l’axe vertical,
l’utilité marginale du patrimoine est Utilité (en unités)
a) un point sur l’axe horizontal.
Patrimoine
b) un point sur l’axe vertical.
(en dollars) Chloé Éliane
c) une zone située sous la courbe d’utilité
du patrimoine. 0 0 0
d) la pente d’un rayon partant de l’origine 20 45 60
vers un point situé sur la courbe d’utilité 40 80 90
-du patrimoine. 60 1 10 100
e) la pente de la courbe de l’utilité 80 130 105
du patrimoine.
L'INCERTITUDE ET L'INFORMATION 231

8 Le patrimoine escompté de Chloé découlant I3 Quel mode de rémunération du directeur


d’un investissement qui pourrait rapporter soit général d’une entreprise est le plus susceptible
40 $, soit 80 $, à probabilité égale, est de d’être efficient?
a) 60 $. a) Une commission de vente.
b) 105 $. b) Une participation aux bénéfices totaux
c) 120$. de l’entreprise.
d) 210 $. c) Un salaire fondé sur les années d’emploi.
e) Aucune de ces réponses. d) Une rémunération selon le classement
au tournoi.
9 Reportez-vous au tableau 17.1 et au fait e) b et d.
17.1. Si Éliane n’a pas d’assurance-maladie,
son patrimoine escompté est de 14 Quel mode de rémunération d’un vendeur

a) 30 $. est le plus susceptible d’être efficient ?

b) 48 $. a) Un taux salarial horaire.

c) 60 $. b) Une commission de vente.


c) Une participation aux bénéfices de l’entreprise.
d) 80 $.
d) Un salaire fondé sur les années d’emploi.
e) 100$.
e) Aucune de ces réponses.

FAIT 17.1 I5 Monique doit choisir entre l’option A ou


l’option B. L’option A lui garantit 10 000 $.
Eliane gagne 60 $ si elle reste en bonne santé L’option B lui donne 5 000 $ avec une
et travaille, mais elle ne gagne rien si elle tombe
probabilité de 0,5 et 15 000 $ avec une
malade et ne peut travailler Éliane a 80% probabilité de 0,5. Puisqu’elle a une courbe
de chances de rester en bonne santé et 20% d’utilité du patrimoine normale, Monique
de chances de tomber malade. a) préférera et choisira A.
b) préférera et choisira B.
c) préférera A mais choisira B.
10 Reportez-vous au tableau 17.1 et au fait 17.1. d) préférera B mais choisira A.
Si Éliane n’a pas d’assurance-maladie, son utilité e) sera neutre face à A et B.
escomptée est de
a) 0. 16 S’il existe trois possibilités et que chacune d’elles
b) 50. a une probabilité d’un tiers de se produire, alors
c) 80. a) il n’y a ni incertitude, ni risque.
d) 100. b) l’incertitude existe, mais il n’y a pas de risque.
e) Aucune de ces réponses. c) le risque existe, mais pas l’incertitude.
d) l'incertitude et le risque existent.
I I Reportez-vous au tableau 17.1 et au fait 17.1. e) il n’y a pas suffisamment d’information
Éliane paie 20 $ pour une assurance-maladie pour faire la distinction entre l’incertitude
qui versera 60 $ si elle tombe malade et rien et le risque.
si elle est en bonne santé. Le patrimoine
I7 Dans un marché efficient, lequel des énoncés
escompté d’Éliane est maintenant de
suivants est faux ?
a) 36 $.
a) Le prix courant est égal au prix futur prévu.
b) 40 $.
b) Le prix courant englobe toute l’information
c) 56 $.
disponible.
d) 100$.
c) Il n’existe aucune possibilité de profits
e) Aucune de ces réponses.
prévisible.
d) Les prix sont stables.
12 Reportez-vous au tableau 17.1 et au fait
e) Les prévisions dépendent des fluctuations.
17.1. Éliane paie 20 $ pour une assurance-maladie
qui versera 60 $ si elle est malade et rien si elle 18 Un mode de rémunération efficient
reste en bonne santé. L’utilité escomptée d Éliane a) ne peut tenir compte des effets du hasard.
est maintenant de b) ne sera pas accepté par l’employé.
a) 40. c) répartit également les recettes entre
b) 90. l’employeur et l’employé.
c) 92. d) maximise le revenu prévu de l’employé.
d) 100. e) maximise le profit prévu de l’entreprise.
e) Aucune de ces réponses.
232 CHAPITRE 17

19 Les acheteurs à la recherche du prix 24 La publicité


le moins élevé consacreront des ressources a) des biens de recherche vise à persuader.
supplémentaires à l’information lorsque b) des biens d’expérience vise à informer.
a) le profit marginal anticipé est positif. c) qui est persuasive fait augmenter
b) le profit marginal anticipé est inférieur la concurrence.
au coût marginal de la recherche. d) est coûteuse.
c) le profit marginal anticipé est égal au coût e) Toutes ces réponses.
marginal de la recherche.
d) le coût marginal de la recherche est positif. 25 Le cours des actions boursières est instable parce que
e) Aucune de ces réponses. a) les prévisions sont irrationnelles.
b) les prévisions changent fréquemment
20 Si les acheteurs ne peuvent évaluer la qualité des en raison de nouvelles informations.
voitures d’occasion et qu’il n’y a pas de garantie, c) le marché boursier n’est pas efficient.
a) seules des voitures d’occasion en mauvais état d) les actionnaires ne se comportent pas comme
seront vendues. des détenteurs de biens.
b) seules des voitures d’occasion en bon e) Toutes ces réponses.
état seront vendues.
c) les voitures en bon état seront vendues à prix
Problèmes à court développement
plus élevé que les voitures en mauvais état.
d) il existe un problème d’aléa moral.
1 Si vous acceptez de payer les premiers 200 $
e) il n’y a pas de problème d’anti-sélection.
de dommages (franchise de 200 $), votre prime
d’assurance automobile pourrait être de 1 000 $
2I La richesse de Romane passe de 3 000 à 6 000 $,
par an. Par contre, si vous acceptez une franchise
ce qui fait monter son utilité de 80 à 100 unités.
de 500 $, votre prime d’assurance pourrait
Si sa courbe d’utilité du patrimoine est normale,
être de 800 $ par an. Pourquoi les compagnies
avec un patrimoine de 9 000 $, son utilité
d’assurance imposent-elles des primes qui
pourrait être de
sont inversement liées à la perte totale que
a) 100 unités.
le consommateur accepte de subir ?
b) 115 unités.
c) 120 unités.
2 Pourquoi les banques prêtent-elles plus
d) 125 unités.
facilement aux personnes qui possèdent des
e) 180 unités.
cartes de crédit et qui ont déjà emprunté à une
banque qu’aux personnes qui ont toujours payé
22 Le coût du risque est le montant duquel
comptant et n’ont jamais fait d’emprunt?
le patrimoine escompté doit augmenter pour
donner la (le) même
3 Les propriétaires de beaucoup de grandes
a) utilité marginale que dans une situation
entreprises sont des groupes d’actionnaires,
sans risque.
qui engagent des cadres pour les diriger.
b) utilité escomptée que dans une situation
Pourquoi la participation aux bénéfices
sans risque.
est-elle un bon mode de rémunération pour
c) patrimoine escompté que dans une situation
les cadres supérieurs de ce genre d’entreprises ?
sans risque.
d) probabilité que dans une situation sans
4 Qu’entend-on par marché efficient? Expliquez
risque.
pourquoi, dans un marché efficient, le prix
e) assurance que dans une situation sans risque.
courant du marché sera toujours égal au prix
futur prévu.
23 Après l’achat d’une police d’assurance contre
le vol, vous décidez de dépenser moins d’argent
en dispositifs de sécurité domiciliaire. Votre
comportement est un exemple
a) d’anti-sélection.
b) de resquilleur.
c) d’émission de signaux au marché d’assurance.
d) d’information asymétrique.
e) d’aléa moral.
L'INCERTITUDE ET L’INFORMATION 233

5 La figure 17.1 présente la courbe d’utilité 7 Supposons que René et Simon disposent,
du patrimoine de Catherine. Celle-ci envisage initialement, de 40 $ chacun. On leur offre
de faire un investissement qui rapportera soit Px l’occasion de jouer à un jeu qui coûte 20 $
avec une probabilité de 50 %, soit P2 avec une la partie. Ils ont une chance sur deux de gagner
probabilité de 50 %. 60 $ et une chance sur deux de ne rien gagner.
a) Nommez Ux l’utilité que retire Catherine de I\. a) Quelle est la valeur escomptée {patrimoine
Nommez U2 l’utilité que retire Catherine de P2. escompté) du jeu?
Nommez PE le patrimoine escompté b) Quelle est l’utilité escomptée du jeu pour
de Catherine et U l’utilité quelle espère René ? pour Simon ?
en retirer. À combien s’élèvera c) Si René et Simon ont un comportement de
son patrimoine escompté ? maximisation, lequel des deux sera disposé à
b) Quel est le coût du risque pour Catherine ? jouer au jeu et lequel des deux ne le sera pas
Expliquez votre réponse. et préférera ne pas risquer de perdre les
40 $ initiaux ? Justifiez votre réponse.
FIGURE 17.1
8 Le tableau 17.3 présente le barème d’utilité du
patrimoine de Léonard. Celui-ci envisage un
projet d’investissement qui rapportera soit 0 $,
soit 20 000 $, à probabilité égale.

TABLEAU 17.3 UTILITÉ


DU PATRIMOINE
DE LÉONARD

Patrimoine
(en milliers Utilité
de dollars) (en unités)

0 0
4 52
5 60
8 79
10 87
Patrimoine (en milliers de dollars)
15 98
20 104

6 Le tableau 17.2 présente les barèmes d’utilité a) Quel est le patrimoine que Léonard espère
du patrimoine de René et de Simon. Lequel retirer du projet?
éprouve le plus d’aversion pour le risque, Rene b) Quelle est l’utilité escomptée de Léonard?
ou Simon ? Expliquez votre réponse. c) Quel est le coût du risque de Léonard ?
d) Léonard est-il prêt à entreprendre le projet
TABLEAU 17.2 UTILITÉ DU PATRIMOINE si cela lui coûte 5 000$ ?

Utilité (en unités) G) 9 Supposons qu’il existe plusieurs projets


d’investissements identiques à celui que décrit
Patrimoine
le problème à court développement 8, et que
(en dollars) René Simon
de nombreuses personnes ont le même barème
0 0 0 d’utilité du patrimoine que Léonard. Léonard
20 100 60 souhaite acheter une assurance en cas d’échec
40 150 1 10 de son projet (possibilité de résultat nul),
60 175 150 a) Quel est le montant le plus élevé que Léonard
80 187 180 sera prêt à payer pour une police d’assurance
100 193 200 qui lui versera 20 000 $ en cas d’échec de
son projet? (Indice: vous n’avez pas à tenir
compte du coût du projet.)
234 CHAPITRE 17

b) Quel est le montant le moins élevé


que pourrait demander une compagnie RÉPONSES
d’assurance si son coût d’exploitation,
sans compter les montants versés en
Vrai/Faux/Incertain
indemnisations, est de 1 000 $ par police?
(Justifiez votre réponse.)
c) Existe-t-il une possibilité de marché pour
l’assurance ?
1 F Définition de l’incertitude.
& 10 Mathieu possède un terrain sur lequel il cultive
2 F Plus grande aversion pour le risque.
des légumes. Cette année, il a accepté un emploi
3 V Voir le texte du manuel. Courbe d’utilité
de garçon de café et ne pourra s’occuper de son
marginale du patrimoine linéaire.
jardin. Il envisage d’engager Thomas, qui a un
4 V Le risque signifie que l’on peut estimer
peu d’expérience en jardinage, pour s’en occuper
les probabilités.
à sa place.
5 V La qualité peut être évaluée avant l’achat.
Thomas trait des vaches pour un exploitant
6 F Problème de l’aléa moral.
de ferme laitière et il travaille très fort pour 30 $
7 F Maximise les profits et est acceptable
par jour. En discutant avec Thomas, Mathieu
pour l’employé.
se rend compte que Thomas attribue à la détente
& 8 I Vrai si l’investisseur a de l’aversion pour
une valeur de 10 $ — Thomas préfère se détendre
le risque, mais sinon peut être faux.
plutôt que de travailler, mais pour lui un jour
9 F Prix courant = prix futur prévu.
de travail de 10 $ ou un jour de détente sont
10 V Voir le texte du manuel.
également acceptables.
Mathieu sait que le revenu provenant
de la culture des légumes dépend des efforts Questions à choix multiple
du cultivateur et des conditions climatiques.
Le tableau 17.4 donne les recettes selon
1 c La question de l’aléa moral ne se pose pas
les combinaisons possibles d’efforts et
dans ce cas. Plus probable que la personne
de conditions climatiques. Les chances que
achète une police d’assurance.
les conditions climatiques soient bonnes
2 c (60 $ x 0,5) + (0 $ x 0,5).
ou mauvaises sont égales.
3 e A utilité/A patrimoine.
4 c Par définition.
TABLEAU 17.4 RECETTE PROVENANT
<3> 5 e Voir le texte du manuel.
DE LA CULTURE
6 c Les acheteurs se fient au signal que donne la
DES LÉGUMES
garantie car le coût d’un faux signal est élevé.
(EN DOLLARS PAR JOUR)
7 b L’utilité marginale du patrimoine diminue
Efforts du travailleur plus rapidement.
8 a (40 $ x 0,5) + (80 $ x 0,5).
Temps Travaille dur Se détend
9 b (60 $ x 0,8) + (0 $ X 0,2).
Beau 160 $ 80 $ Ô 10 c (100 x 0,8) + (0 x 0,2).
Mauvais 80 $ 80 $ I I b ((60 $ - 20 $) x 0,8) + ((60 $ - 20 $) x 0,2).
& I2 b Patrimoine dans les deux cas = 40 $ (60 $
Mathieu envisage deux modes de - 20 $). Utilité escomptée = (90 X 0,8)
rémunération distincts. + (90 X 0,2).
13 e Etroite relation entre efiforts/concurrence
Mode 1 Mathieu paie Thomas 31 $ par jour.
et profit.
Mode 2 Mathieu paie Thomas 10 $ par jour
14 b Lie directement l’effort fourni sans
plus 26% de la recette provenant
surveillance au résultat.
de la culture des légumes.
& 15 a Les valeurs anticipées de A et B sont égales,
À l’aide de cette information, déterminez mais l’option A est moins risquée et donc
quel mode de rémunération Mathieu devrait préférable.
adopter. [N’oubliez pas qu’un mode de 16 d Définitions. Le risque est un sous-ensemble
rémunération efficace a deux caractéristiques. de l’incertitude.
Il doit maximiser le profit pour le principal I7 d Les prix fluctuent selon les changements
et être acceptable pour l’agent (supposez que dans les prévisions.
lés conditions offertes à l’agent sont au moins I8 e Est établi par le principal, mais doit
aussi bonnes que celles que lui offre l’autre être acceptable pour l’agent.
meilleure possibilité d’emploi).]
L'INCERTITUDE ET L'INFORMATION 235

19 e Lorsque l’avantage marginal anticipé > coût prévu parce que toute déviation, qui fournit
marginal. une possibilité de profit prévisible, sera éliminée
20 a L’inverse de d et e est vrai. immédiatement.
Ô 21 b Avec l’utilité marginale du patrimoine
5 a) La figure 17.1 Solution illustre les utilités
décroissante, doit être <120.
de Catherine si celle-ci reçoit Px et P2. Son
22 b Voir le texte du manuel.
patrimoine escompté (PE) est égal à la
23 e Voir le texte du manuel.
probabilité de recevoir Px, multipliée par Px,
24 d a, b et c sont vrais si on intervertit persuader plus la probabilité de recevoir P2, multipliée
et informer.
par P2. Parce que chaque probabilité est
25 b Seule source de changement dans le marché de 0,5, le PE est simplement la moyenne
efficient. de (à mi-chemin entre) Px et P2. D’après un
calcul semblable, l’utilité escomptée (UE) est
Problèmes à court développement la moyenne pondérée de (à mi-chemin entre)
1 En permettant aux clients de payer des primes Ux et U2.
moins élevées s’ils consentent à payer un plus b) Le coût du risque correspond à l’augmentation

grand pourcentage du coût des dommages, les du patrimoine escompté qui permet d’obtenir

compagnies d’assurance réduisent le problème la même utilité escomptée que dans une

de l’anti-sélection. Les conducteurs à risque élevé, situation sans risque. Si Catherine reçoit le

sachant qu’ils sont prédisposés aux accidents, patrimoine P dans une situation sans risque,

sont prêts à payer de plus fortes primes pour le niveau d’utilité sera le même que pour PE.

bénéficier d’une couverture presque complète, Le patrimoine escompté de Catherine (PE)

alors que les conducteurs à faible risque, sachant doit alors être supérieur au patrimoine de la

qu’ils ont rarement des accidents, choisiront des situation sans risque (P) du montant
(PE — P) afin quelle puisse être neutre face
primes moins élevées et une couverture moins
aux deux résultats. Par conséquent (PE - P)
complète. Grâce aux franchises, le problème
est le coût du risque.
d’anti-sélection, les conducteurs à risque élevé
évinçant du marché les conducteurs à faible
FIGURE 17.1 SOLUTION
risque, risque moins de se produire. La
compagnie d’assurance doit offrir des primes
différentes en fonction des différents risques
assumés.

2 Les banques ont plus d’information sur la


capacité et la volonté de payer des personnes
auxquelles elles, ou d’autres institutions
financières, ont déjà accordé des prêts. Un bon
dossier de remboursement de prêt est une preuve
(un signal) que le client est un emprunteur
à faible risque. Si un client n’a jamais fait
d’emprunt, la banque doit trouver d’autres
moyens d’évaluer s’il s’agit d’un emprunteur
à risque élevé ou à faible risque.

3 Les actionnaires ne peuvent surveiller facilement


les efforts ou le rendement des cadres. Toutefois,
comme les décisions des cadres ont un effet
direct et important sur les profits de 1 entreprise,
ceux-ci, lorsqu’ils partagent les bénéfices dans 6 Pour trouver qui a le plus d’aversion pour
le cadre d’un programme de participation aux le risque, nous devons déterminer quelle utilité
bénéfices, sont motivés à prendre des decisions marginale du patrimoine diminue le plus
qui maximisent les profits des actionnaires. rapidement à mesure que le patrimoine augmente.
4 Un marché efficient est un marche où le prix Le tableau 17.6 présente l’utilité totale et l’utilité
courant englobe toute l’information pertinente marginale du patrimoine dans le cas de René
disponible. Le prix sera donc égal au prix futur et de Simon. Puisque l’utilité marginale du
prévu et il n’y aura pas de possibilités de profits patrimoine de René diminue plus rapidement
prévisibles. Dans un marché efficient, le prix de que celle de Simon, René a le plus d’aversion

marché courant sera toujours égal au prix futur pour le risque.


236 CHAPITRE 17

jouera donc. Ces résultats ne sont pas


TABLEAU 17.6 UTILITÉ TOTALE/MARGINALE
surprenants, puisque René a plus d’aversion
DU PATRIMOINE
pour le risque que Simon et que, ceteris
paribus, il est moins enclin à jouer à un jeu de
René Simon
hasard et davantage enclin à conserver ses 40 $.
Utilité Utilité Utilité Utilité
totale marginale totale marginale & 8 a) Le patrimoine escompté de Léonard est de
Patrimoine (en (en (en (en (0 $ x 0,5) + (20 000 $ x 0,5) = 10 000 $.
(en dollars) unités) unités) unités) unités) b) L’utilité escomptée de Léonard est de (0 X 0,5)
+ (104 X 0,5) = 52 unités.
0 0 0
c) Un patrimoine incertain de 0 $ ou de
.., 100 . . 60
20 000 $ produit un patrimoine escompté
20 100 60
de 10 000 $ et une utilité escomptée de
. . . 50 . . 50
52 unités. Dans le tableau 17.3, nous pouvons
40 150 1 10
voir qu’un patrimoine certain (dans une
. . . 25 . . 40
situation sans risque) de 4 000 $ donne aussi
60 175 150
une utilité de 52 unités. Le coût du risque
. . . 12 . . 30
est de 6 000 $, soit le montant duquel le
80 187 180
patrimoine escompté doit augmenter par
, . . 6 . . 20
rapport au patrimoine dans une situation sans
100 193 200
risque pour donner la même utilité que dans
la situation sans risque (10 000 $ - 4 000 $).
a) On peut calculer la valeur escomptée du jeu, d) Pour Léonard, futilité de 5 000 $ à laquelle
ou le patrimoine que l’on espère retirer du il renoncerait en investissant dans le projet
jeu, en utilisant la méthode indiquée dans (60 unités) est supérieure à l'utilité escomptée
le rappel 3. Multipliez chaque possibilité du projet risqué (52 unités). Léonard n’est
de patrimoine par la probabilité de l’obtenir pas prêt à se lancer dans ce projet.
et faites la somme des résultats. Gagner au jeu
produit un patrimoine de 80 $ (40 $ pour & 9 a) Léonard est disposé à acheter une police
commencer - 20 $ pour jouer + 60 $ de d’assurance pourvu que l’utilité que l’achat
prix). Perdre au jeu produit un patrimoine de l’assurance lui permettrait d’obtenir soit
de 20 $ (40 $ pour commencer - 20 $ pour au moins égale à l’utilité escomptée du projet
jouer). La valeur ou le patrimoine que l’on risqué. Etant donné que l’utilité escomptée
espère retirer du jeu est de (80 $ X 0,5) du projet (52 unités) est la même que l’utilité
+ (20 $ X 0,5) = 50 $. escomptée d’un revenu certain de 4 000 $,
b) On peut aussi calculer l’utilité escomptée Léonard sera prêt à payer jusqu’à 16 000 $
du jeu en utilisant la méthode indiquée dans pour une police d’assurance qui lui versera
le rappel 3. Multipliez l’utilité de chaque 20 000 $ en cas d’échec du projet. Avec une
possibilité de patrimoine par la probabilité assurance, Léonard recevra 20 000 $, que
de l’obtenir et additionnez les résultats. le projet réussisse ou non. En soustrayant le
Pour René, l’utilité escomptée est de montant de la prime d’assurance de 16 000 $,
(187 x 0,5) + (100 x 0,5) = 143,5 unités. il lui restera un revenu net certain de 4 000 $.
Pour Simon, l’utilité escomptée est de Acheter une police d’assurance et prendre
(180 X 0,5) + (60 X 0,5) = 120 unités. le risque de se lancer dans le projet ne ferait
c) Pour décider s’il doit ou non jouer au jeu, aucune différence pour Léonard. Avec
chaque joueur compare l’utilité escomptée n’importe quelle prime inférieure à 16 000 $,
qu’il retirera du jeu avec l’utilité escomptée Léonard ferait mieux d’acheter une assurance.
qu’il retirera en ne jouant pas et en Il aurait un revenu net certain supérieur à
conservant ses 40 $. 4 000 $ et une utilité supérieure à 52 unités.
Pour René, l’utilité escomptée du jeu
est de 143,5 unités, ce qui est inférieur
aux 150 unités d’utilité que procure le fait
de conserver le patrimoine de 40 $. René ne
jouera pas. Pour Simon, l’utilité escomptée
du jeu est de 120 unités, ce qui est supérieur
aux 110 unités d’utilité qu’il retirerait en
conservant son patrimoine de 40 $. Simon
L'INCERTITUDE ET L'INFORMATION 237

b) Une compagnie d’assurance qui maximise Mathieu et Thomas seront tous deux dans
ses profits demanderait un montant de prime une situation plus avantageuse avec le mode
qui lui permettrait au moins de couvrir ses de rémunération 2.
coûts prévus par projet - le paiement prévu Mais Thomas est-il dans une situation plus
plus les coûts d’exploitation. Étant donné que avantageuse s’il cultive des légumes dans le cadre
la moitié des projets échouent, le paiement du mode 2 que s’il trait des vaches ? Si Thomas
prévu est de 10 000 $ par projet (20 000 $ X trait des vaches, il gagne 30 $, mais comme
0,5). Avec des coûts d’exploitation de 1 000 $ il doit fournir beaucoup d’efforts, la valeur
par projet, le montant minimum qu’une de l’effort est pour lui de 30 $ — 10 $ = 20 $.
compagnie d’assurance pourrait demander Il s’agit d’une valeur inférieure à celle qu’il
est de 11 000 $ (10 000 $ + 1 000 $). recevrait s’il cultivait des légumes dans le cadre
c) Étant donné que les gens sont prêts du deuxième mode de rémunération. Par
à payer 16 000 $, ce qui est supérieur au prix conséquent, ce mode de rémunération est plus
minimum que les compagnies d’assurance avantageux pour lui.
doivent demander pour réaliser un profit
(11 000 $), il existe un marché pour
l’assurance. Comparativement à une situation
où il n’y aurait pas d’assurance, les particuliers
et les assureurs seraient dans une situation
plus avantageuse avec des primes allant
de 11 000$ à 16 000$.

& 10 Le revenu varie selon que Thomas fait des efforts


ou se détend et selon qu’il fait beau ou mauvais
temps. La probabilité de beau ou de mauvais
temps est de 0,5.
Si Thomas fait des efforts, le revenu total
prévu est de (160 $ x 0,5) + (80 $ x 0,5) = 120 $.
Si Thomas se détend, le revenu total prévu
est de (80 $ X 0,5) + ((80 $ X 0,5) = 80 $.
Pour connaître le meilleur mode de
rémunération, calculez le revenu de chaque
mode pour le principal et pour l’agent.
Mode 1 : Mathieu (principal) paie Thomas
(agent) 31 $ par jour. Thomas va-t-il choisir
de faire des efforts ou de se détendre ? Si Thomas
se détend, il reçoit 31 $. Étant donné quil na
pas à fournir de gros efforts, la valeur de l’effort
est de 31 $ - 10 $ = 21 $. Thomas choisira de se
détendre. Le revenu prévu sera de 80 $ par jour,
31 $ allant à Thomas et 49 $ à Mathieu.
Mode 2: Mathieu (principal) paie Thomas
(agent) 10 $ plus 26% du revenu provenant
de la culture des légumes. Si Thomas se détend,
il reçoit 10 $ + (0,26 X 80 $) = 30,80 $. Comme
il n’a pas à fournir de gros efforts, la valeur de la
détente est également de 30,80 $. Si Thomas fait
des efforts, il reçoit 10 $ + (0,26 X 120 $) = 41,20 $.
Comme il doit fournir un gros effort, la valeur
de l’effort est de 41,20 $ — 10 $ = 31,20 $.
Thomas choisira de faire des efforts. Le revenu
prévu sera de 120 $ par jour, 41,20 $ allant
à Thomas et 78,80 $ à Mathieu.
• • 0
e v i s i o n

Chapitres 14 à 17

PROBLÈME

FIGURE R3.I LES MARCHÉS DU TRAVAIL DES PROGRAMMEURS


(a) Gauchers (b) Droitiers

Programmeurs (en milliers)

Supposons que l’industrie du logiciel n’utilise Les entreprises de services informatiques


que deux facteurs de production - les pratiquent également une discrimination en
programmeurs (travail) et les ordinateurs faveur des programmeurs droitiers, car elles
(capital). Tous les programmeurs, qu’ils pensent (à tort) que les droitiers sont plus
soient gauchers ou droitiers, ont des diplômes productifs que les gauchers. La figure R3.1 (b)
en informatique, sont également qualifiés présente la courbe d’offre de programmeurs
et productifs ; leurs courbes de la valeur droitiers et la courbe de demande de
du produit marginal (VPm) sont illustrées programmeurs droitiers des entreprises.
à la figure R3.1(a) et (b). a) Quelle est la relation entre le salaire
Les entreprises de services informatiques et la valeur du produit marginal pour
(exploitées le plus souvent par des droitiers) les programmeurs gauchers ? pour
pratiquent une discrimination contre les les programmeurs droitiers ?
programmeurs gauchers, car elles pensent b) Indiquez quelles forces de la concurrence
(à tort) que ceux-ci sont moins productifs que auraient tendance à éliminer cette forme de
les droitiers. La figure R3.1 (a) montre la courbe discrimination, même en l’absence de
d’offre de programmeurs gauchers et la courbe législation.
de demande de programmeurs gauchers des
entreprises.
CHAPITRES 14 À 17 239

c) Si les propriétaires d’entreprise ont des


FIGURE R3.2
préjugés tellement tenaces contre les
programmeurs gauchers qu’ils sont prêts
à réaliser de moins gros profits plutôt que
d’employer des programmeurs gauchers,
les forces de la concurrence pourront-elles
éliminer cette discrimination ?
d) Le gouvernement adopte une loi sur l’équité
salariale, conçue pour égaliser les salaires des
droitiers et des gauchers. Cette loi comprend
un programme d’éducation visant à informer
les entreprises de la productivité réelle et égale
de tous les programmeurs, quelle que soit leur
main dominante. Si ce programme donne les
résultats escomptés, quel sera le salaire des
programmeurs ? l’emploi de programmeurs
gauchers ? de programmeurs droitiers ?
e) Maintenant que les salaires des programmeurs
sont égaux, quelle est la relation entre le salaire Les ménages épargnent davantage lorsque leur
et la valeur du produit marginal pour les revenu courant
programmeurs gauchers ? pour les programmeurs a) est faible comparé au revenu futur prévu
droitiers ? Que signifie Y équité salariale à part ou que les taux d’intérêt sont élevés.
le fait que différents groupes reçoivent des b) est faible comparé au revenu futur prévu
salaires égaux ? ou que les taux d’intérêt sont faibles.
f) Avec l’adoption de la loi sur l’équité salariale, c) est élevé comparé au revenu futur prévu
que peut-on dire du salaire moyen des ou que les taux d’intérêt sont élevés.
programmeurs ? En supposant que les d) est élevé comparé au revenu futur prévu
programmeurs et le capital soient des facteurs ou que les taux d’intérêt sont faibles.
de production substituts, expliquez, en vous e) est supérieur ou inférieur au salaire de réserve.
aidant d’un graphique, l’effet exercé sur Une augmentation du patrimoine de Catherine,
la demande de capital de chaque entreprise. qui passe de 3 000 $ à 6 000 $, augmente
Comment évoluent le prix et l’emploi de son utilité, qui passe de 80 unités à 100 unités.
capital dans l’industrie du logiciel à court Si Catherine est neutre face au risque, avec un
terme ? à long terme ? patrimoine de 9 000 $, son utilité pourrait être de
g) Avec l’adoption de la loi sur l’équité salariale, a) 100 unités.
comment évolue le salaire moyen des b) 115 unités.
programmeurs gauchers ? Expliquez l’effet c) 120 unités.
exercé sur les décisions des gauchers d investir d) 125 unités.
dans le capital humain. e) 180 unités.

Un grand magasin qui paie à ses employés


WMWTOiHIMWMTOWMM Pli MIHIillllil I iWffllIPIW^ un salaire fixe indépendant du volume de ventes
EXAMEN DE MI-ÉTAPE connaîtra probablement des problèmes
a) d’aléa moral.
b) d’anti-sélection.
Allouez 32 minutes à cet examen (16 questions,
c) d’information privée.
2 minutes par question). Choisissez la meilleure
d) d’incitation au travail.
réponse pour chaque question.
e) Toutes ces réponses.

I Reportez-vous à la figure R3.2. Lorsque La société XYZ prévoit verser un dividende de


la demande d’un facteur de production passe 5 $ par action chaque année. Si le taux d’intérêt
de D0 à Dp la variation de la valeur de réserve annuel est de 5 %, quel est le prix du marché
correspond à d’une action de XYZ ?
a) la partie ihfc. a) 50$
b) la partie jcfk. b) 5$
c) la partie Ofk. c) 5,25 $
d) la partie Ofh. d) 4,76 $
e) Aucune de ces réponses. e) 100$
240 RÉVISION DE LA 3E PARTIE

6 Le prix de demande maximal d’une ressource


FIGURE R3.3
naturelle est le prix auquel
a) on se mettra à utiliser cette ressource.
b) personne n’utilise cette ressource.
c) la ressource sera utilisée comme substitut
d’autres ressources rares.
d) on se mettra à conserver cette ressource.
e) les économistes adoptent le principe
de Hotelling.

7 La courbe de demande d’un facteur de


production se déplacera vers la droite à la suite de
a) la diminution du prix du facteur.
b) l’augmentation du prix du facteur.
c) la diminution du prix d’un facteur substitut. II Un ratio cours-bénéfice élevé signifie
d) l’augmentation du prix d’un facteur substitut. a) que le cours actuel des actions est plus élevé
e) la diminution du prix de production. que le prix futur prévu des actions.
b) que les investisseurs souhaitent vendre
8 Supposons qu’une entreprise qui maximise ses les actions pour réaliser des profits.
profits engage des travailleurs dans un marché c) que l’on prévoit que les profits futurs
du travail concurrentiel. Si la valeur du produit seront élevés par rapport aux profits actuels.
marginal du travail est supérieure au salaire, d) que l’on prévoit que les profits futurs
l’entreprise devrait seront faibles par rapport aux profits actuels.
a) augmenter le taux salarial. e) Aucune de ces réponses.
b) diminuer le taux salarial.
c) augmenter le nombre de travailleurs 12 Si la valeur actuelle nette d’une nouvelle
quelle engage. machine est positive, alors
d) réduire le nombre de travailleurs a) l’entreprise ne devrait pas acheter la machine.
quelle engage. b) la valeur actuelle du flux des valeurs du produit
e) adopter un processus de production marginal de la machine est supérieure au prix
à plus forte proportion de main-d’œuvre. d’achat.
c) la valeur actuelle du flux des valeurs
9 Un groupe de travailleurs syndiqués dotés du produit marginal de la machine est
de diverses compétences qui travaillent dans inférieure au prix d’achat.
la même entreprise ou industrie est d) a et b
a) un syndicat industriel. e) a et c.
b) un syndicat de métier.
c) un syndicat local. I3 Quel(s) énoncé(s) suivant(s) est (sont) vraiis) ?
d) un syndicat national. 1 On ne peut évaluer la qualité des biens
e) un syndicat de la fonction publique. de recherche qu’une fois qu’on les a achetés.
2 On peut évaluer la qualité des biens
10 Reportez-vous à la figure R3.3. Pour tout salaire de recherche avant de les acheter.
donné, 3 La publicité relative aux biens d’expérience
a) l’élasticité de la demande de travailleurs vise habituellement à persuader.
qualifiés est plus grande que l’élasticité 4 La publicité relative aux biens d’expérience
de la demande de travailleurs non qualifiés. vise habituellement à informer.
b) on embauchera plus de travailleurs qualifiés a) 1 et 3
que de travailleurs non qualifiés. b) 1 et 4
c) on embauchera plus de travailleurs
c) 2 et 3
non qualifiés que de travailleurs qualifiés. d) 2 et 4
d) la distance horizontale entre les courbes e) Aucune de ces réponses.
représente la compensation pour le coût
de l’acquisition des compétences.
e) la distance horizontale entre les courbes
• représente la VPm de la compétence.
CHAPITRES 14 À 17 241

14 Une personne qui achètera le plus probablement


une police d’assurance est une personne dont RÉPONSES
a) l’utilité totale du patrimoine diminue
rapidement. Problème
b) l’utilité totale du patrimoine diminue a) Pour les 2 000 programmeurs gauchers
lentement. employés, le salaire (20 000 $ par an) est bien
c) l’utilité marginale du patrimoine diminue inférieur à la valeur du produit marginal de
rapidement. 60 000 $ par an. Pour les 6 000 programmeurs
d) l’utilité marginale du patrimoine diminue droitiers employés, le salaire (60 000 $ par an)
lentement. est bien supérieur à la valeur du produit
e) l’utilité marginale du patrimoine marginal de 20 000 $ par an.
est constante. b) Une entreprise non discriminante
pourrait réaliser des profits bien plus
I5 La courbe de coût marginal du travail (CmT) élevés en employant un plus grand nombre
d’un monopsone qui fait face à une courbe de programmeurs gauchers pourvu que
d’offre de travail (O) à pente positive la valeur de leur produit marginal soit
a) croise la courbe de VPm du travail au salaire supérieure au salaire. De même, ses profits
d’équilibre. augmenteront si elle emploie moins de
b) est située au-dessous de O et lui est parallèle. programmeurs droitiers pourvu que leur
c) est identique à O. salaire soit supérieur à la valeur de leur
d) est située au-dessus de O et lui est parallèle. produit marginal. Le résultat concurrentiel
e) Aucune de ces réponses. serait un salaire de 40 000 $ par an pour
tous les programmeurs, et l’emploi
16 Supposons que le gouvernement adopte une loi de 4 000 programmeurs gauchers et
sur le salaire minimum qui interdit à quiconque de 4 000 programmeurs droitiers.
d’engager des travailleurs à un salaire horaire c) Non. Les entreprises continuent la
inférieur à 8 $. Dans la figure R3.4, le discrimination si elles « paient » pour la
monopsone emploiera discrimination sous forme de profits réduits.
a) 250 heures de travail. d) Un programme d’éducation efficace modifie
b) 300 heures de travail. les croyances des entreprises concernant les
c) 350 heures de travail. programmeurs gauchers et droitiers et fait
d) 400 heures de travail. déplacer les courbes de demande vers les
e) 500 heures de travail. vraies courbes de VPm. Le résultat sera
le même que le résultat concurrentiel -
FIGURE R3.4 un salaire de 40 000 $ par an pour
tous les programmeurs, et l’emploi
de 4 000 programmeurs gauchers
et de 4 000 programmeurs droitiers.
e) Le salaire est égal à la valeur du produit
marginal pour chaque programmeur. L’équité
salariale signifie également que chaque travailleur
reçoit un montant égal à la valeur de son
produit marginal.
f) Avant l’adoption de la loi, le salaire annuel
moyen des programmeurs est une moyenne
pondérée (par l’emploi) des salaires
des programmeurs gauchers et droitiers.

120 000 S x 2 000) + (60 000 $ x 6 000) = 50 000 $


8 000
Après l’adoption de la loi, le salaire moyen
annuel est tombé à 40 000 $. La baisse du salaire
entraîne une baisse de la demande de capital,
étant donné que les entreprises cherchent
à économiser sur l’apport de capitaux,
qui est maintenant relativement plus coûteux,
(voir la figure R3.5).
242 RÉVISION DE LA 3E PARTIE

7 a L’entreprise demande davantage de son


FIGURE R3.5 MARCHÉ FINANCIER
facteur de production, qui est maintenant
DES ORDINATEURS
relativement moins coûteux pour chaque prix
du facteur, a et b représentent un mouvement
le long de la courbe, e correspond à un
déplacement de la courbe vers la gauche.
8 c L’embauche de T de travail —> T du profit
puisque la VPm > le Cm. L’entreprise ne peut
modifier le salaire.
9 a Par définition.
0 10 h L’inverse de a est vrai, d est vrai pour
la distance verticale entre les courbes d’offre,
e est vrai pour la distance verticale entre les
courbes de VPm.
0 11 c L’inverse de a et b est vrai.
12 b Par définition. L’entreprise devrait acheter
la machine.
13 c Définition des biens de recherche. Puisqu’on
À court terme, l’offre de capital (OCT0)
ne peut évaluer à l’avance la qualité des biens
est parfaitement inélastique. La baisse de la
d’expérience, la publicité informative est
demande de D0 à T>, provoque la baisse du
inutile.
prix du capital vers 77,, alors que la quantité
14 c Plus l’utilité marginale du patrimoine
employée reste constante. À long terme, la
1 rapidement, plus la personne éprouve
courbe d’offre de capital à l’industrie (OZJ) est
de l’aversion pour le risque. L’utilité totale
parfaitement élastique au taux d’intérêt général
du patrimoine T toujours.
TI. La baisse de prix finira par provoquer
0 15 e La courbe CmT est située au-dessus de
le déplacement des ressources en capital
la courbe O, mais ne lui est pas parallèle.
de l’industrie vers d’autres industries où le taux
La courbe CmT croise la courbe VPm
d’intérêt est plus élevé. La courbe d’offre à court
à la Qt d’équilibre (300 heures), non
terme se déplace vers 0C7,, le prix du capital
au salaire d’équilibre.
retourne à TI, et la quantité de capital employée
0 16 c La nouvelle CmT est horizontale lorsque
diminue.
le salaire horaire est de 8 $ jusqu’à ce qu elle
g) Le salaire annuel moyen des programmeurs
croise la courbe O —> Qj jusqu’à l’endroit
gauchers a augmenté — il est passé de
où la nouvelle CmT croise la VPm.
20 000 $ à 40 000 $ -, ce qui incite les
gauchers à investir dans le capital humain
nécessaire (diplôme en informatique)
pour devenir programmeur. L’offre de
programmeurs gauchers a donc augmenté.

EXAMEN DE MI-ÉTAPE

1 b T dans la partie située sous la courbe d’offre.


2 c Voir le texte du manuel.
3 c Neutre face au risque —> utilité marginale
du patrimoine constante. L’utilité marginale
des 3 000 $ supplémentaires est de 20 unités
d’utilité.
4 e Tous sont des problèmes créés
par l’information privée.
5 e Pour un avenir incertain, VA = montant
annuel payé/r = 5 $/0,05.
6 b ■ Par définition.
18
Les lacunes du marché
et les choix publics

Les choix publics et le marché politique


CONCEPTS CLÉS
La théorie des choix publics analyse le secteur public
en tant que marché politique analogue au marché
Le secteur public
économique. Elle comprend les intervenants suivants.

L’ensemble des dépenses de tous les paliers de ♦ Les électeurs — consommateurs des résultats
gouvernement (fédéral, provincial, local) représente du processus politique. Ils expriment leurs
46% de l’ensemble du revenu au Canada. Le exigences en votant, en exerçant des pressions
gouvernement emploie 11 % de la main-d’œuvre et en participant financièrement aux campagnes
canadienne. Les dépenses publiques ont culminé politiques.
en 1992, alors quelles représentaient 50%
♦ Les politiciens — responsables élus par les électeurs.
du revenu total, et elles ont ensuite décliné.
Leur objectif est de se faire élire et de rester en
poste.
La théorie économique du secteur public
♦ Les fonctionnaires - recrutés par les politiciens
élus. Leur objectif est de maximiser le budget
L’activité économique du secteur public découle
de leur organisme.
des efforts faits pour corriger les facteurs suivants.
L’équilibre politique correspond à une situation où les
♦ Les lacunes du marché, qui représentent l’incapacité
choix des électeurs, des politiciens et des fonctionnaires
d’un marché non réglementé à répartir les ressources
sont compatibles et où aucun des trois groupes ne peut
de manière optimale. Il en existe trois cas:
améliorer son sort en prenant des décisions différentes.
• les biens collectifs — biens et services qui
peuvent être consommés simultanément
Les biens collectifs
par tous, et dont personne ne peut être exclu ;
• le monopole — la limitation de la production
Un bien collectif pur est un bien que tout le monde
par les monopoles et les cartels et la recherche
peut consommer et dont on ne peut priver personne.
de rentes empêchent une répartition optimale
Les biens collectifs ont deux caractéristiques.
des ressources ;
• les effets externes — la production de biens ♦ La non-rivalité d’usage - le fait qu’une personne
et de services crée des coûts externes ou des consomme ce bien n’entraîne pas une -l des
avantages externes qui se répercutent sur quantités disponibles de ce bien pour une autre
des personnes étrangères à la transaction. personne.

♦ L’inégalité économique — l’économie d un marché • À l’opposé il y a les biens rivaux


non réglementé produit ce que la plupart des gens — la consommation de ces biens par une
considèrent comme une répartition inégalé du personne oblige d’autres personnes à leur

revenu. consommation de ces biens, (p. ex. des hot dogs).

♦ L’impossibilité d’exclusion d usage — il est


impossible d’empêcher quiconque de consommer
ce bien.
244 CHAPITRE 18

• L’opposé est le bien d’usage exclusif ♦ La politique fiscale est le résultat du théorème
— il est possible d’empêcher d’autres de l’électeur médian — les partis politiques
personnes de consommer ce bien préconisent les politiques qui maximisent
(p. ex. la télévision par câble). l’avantage net de l’électeur médian.

Les biens collectifs —> le problème du resquilleur Le droit d’accise est une taxe sur la vente d’un bien
(personne qui consomme sans payer), qui correspond particulier.
à une production insuffisante de biens collectifs lorsque •
♦ La taxe sur un bien —> déplacement de la courbe
ceux-ci sont produits et vendus par le secteur privé.
d’offre vers le haut, T P d’équilibre, -l Q
♦ L’échelle de prestation de biens collectifs d’équilibre et T perte sèche.
est efficace lorsque l’avantage net est maximisé
♦ L’importance de la perte sèche dépend de l’élasticité
ou lorsque l’avantage marginal est égal au coût
de la demande. Pour réduire la perte sèche, les
marginal du bien public.
gouvernements imposent des taux de taxation plus
• L’avantage total est la valeur monétaire totale élevés sur les biens dont la demande présente une
accordée au bien collectif concerné. élasticité plus faible.
• davantage net est l’avantage total - le coût total.
• L’avantage marginal est T de l’avantage total
à partir d’une unité T du bien collectif.

♦ La fourniture privée de biens collectifs crée


RAPPELS
le problème du resquilleur et entraîne une offre
1 Pour déterminer l’efficience d’un marché
inférieure à la quantité efficiente de ces biens.
(ou de toute autre institution), les économistes
♦ La fourniture publique de biens collectifs peut être utilisent comme critère l’allocation optimale des
efficiente lorsque les politiciens rivalisent entre eux ressources. L’allocation optimale des ressources
pour obtenir les votes des électeurs bien informés. signifie que l’économie produit tous les biens
et services jusqu’au point où le coût marginal est
La fourniture publique de biens collectifs dépend
égal à l’avantage marginal. Dans cette situation,
généralement du marché politique et des actions des
électeurs, des politiciens et des fonctionnaires. personne ne peut être avantagé sans que
quelqu’un d’autre ne soit désavantagé.
♦ Les politiciens ont tendance à suivre le principe Lorsque le marché ne réussit pas à réaliser
de la différenciation minimale — la tendance des cette situation d’efficience «idéale», il y a lacune
concurrents à s’imiter les uns les autres pour du marché. Le marché présente une lacune
obtenir le plus de votes des électeurs/clients. lorsque la production est trop faible et

♦ Les électeurs peuvent choisir de pratiquer que l’avantage marginal de la dernière unité
est supérieur au coût marginal. Par ailleurs,
l’ignorance rationnelle — ils décident
le marché présente une lacune lorsque la
de ne pas acquérir l’information parce que le coût
production est trop forte et que le coût marginal
d’acquisition > avantage anticipé. Les politiciens,
de la dernière unité est supérieur à l’avantage
influencés par les fonctionnaires et les groupes
de pression, peuvent alors permettre une marginal.
Dans ce chapitre (et dans le chapitre 20),
fourniture excédentaire d’un bien collectif
nous apprenons que les gouvernements peuvent
par rapport à la quantité efficiente de ce bien.
pallier les lacunes du marché, mais qu’ils ne
parviennent pas toujours à obtenir une allocation
Les impôts efficiente des ressources. Étant donné que les
marchés et les États ont des lacunes, la question
Les impôts sur les revenus génèrent un important économique pertinente qui se pose est la
pourcentage des recettes publiques. suivante : dans quel cas les lacunes sont-elles
les moins importantes ?
♦ Les électeurs à revenu élevé préfèrent payer
moins d’impôt et recevoir moins d’avantages
du gouvernement. 2 La théorie des choix publics offre une théorie
du marché politique qui est parallèle à la théorie
♦ Les électeurs à faible revenu préfèrent payer économique des marchés des biens et services.
plus d’impôt et recevoir plus d’avantages Il est utile de faire des analogies entre
du- gouvernement. le fonctionnement des marchés politiques
et celui des marchés ordinaires.
LES LACUNES DU MARCHÉ ET LES CHOIX PUBLICS 245

Sur les marchés politiques, les demandeurs et l’impossibilité d’exclusion d’usage, et non
sont les électeurs alors que, sur les marchés par le fait qu’il soit fourni ou non par le secteur
ordinaires, les demandeurs sont les public. De nombreuses villes et collectivités
consommateurs. Dans les deux cas, les fournissent, par exemple, des piscines et des
demandeurs sont soucieux de leurs coûts services de ramassage des ordures ménagères.
et de leurs avantages. Il ne s’agit pas, dans l’un ou l’autre cas, de biens
Sur les marchés politiques, les fournisseurs publics purs malgré le fait qu’ils soient offerts
sont les politiciens et les fonctionnaires, alors par le secteur public. En fait, dans de nombreuses
que sur les marchés ordinaires, les fournisseurs autres collectivités, ces mêmes services sont
de biens et de services sont les entreprises. offerts par le secteur privé.
Sur les marchés politiques, les électeurs
expriment leurs exigences en votant, en 6 II importe de comprendre pourquoi la non-
contribuant financièrement aux campagnes rivalité et l’impossibilité d’exclusion d’usage
électorales et par le lobbying. Il en est ainsi parce associées aux biens publics purs font que nous
que, sur ce marché, les fournisseurs (politiciens) n’obtenons pas la courbe d avantage marginal
sont motivés par le désir de conserver leur poste. de l’ensemble de l’économie de la même manière
Sur les marchés ordinaires, les consommateurs que la courbe des biens privés.
expriment leurs demandes en achetant ou en Un bien privé est caractérisé par la rivalité
n’achetant pas un produit ou un service, puisque de consommation. Par conséquent, pour obtenir
les fournisseurs sont motivés par le désir de la courbe de demande pour l’ensemble de
maximiser leurs profits. l’économie, nous devons faire la somme
Sur ces deux marchés, l’équilibre est un horizontale des courbes d’avantage marginal
état de repos. En situation d’équilibre, il n’existe (demande) individuelles. Par contre, nous
aucune tendance au changement puisque les obtenons la courbe d’avantage marginal de
participants ne peuvent être avantagés en faisant l’économie pour un bien public en faisant la
un choix différent ou en effectuant une somme verticale des courbes d avantage marginal
transaction supplémentaire. individuelles. Il s’agit de la courbe d’avantage
marginal qui permet d’évaluer le niveau de
3 Comprenez la logique sur laquelle repose le fourniture efficient de biens collectifs.
théorème de l’électeur médian. Les politiciens
souhaitent être élus et rester au pouvoir. Pour
cela, ils doivent recevoir une majorité de votes ;
ils doivent gagner au moins 50% des votes, AUTOÉVALUATION
plus un. Les politiciens le savent bien, et c’est
pourquoi l’électeur «du milieu» devient
Vrai/Faux/Incertain
l’électeur clé. Un politicien qui présente un
(Justifiez votre réponse.)
programme ne tenant pas compte des préférences
de l’électeur médian perdra les élections au profit
du politicien dont le programme en tient I La limitation de la production par les
compte. monopoles illustre une lacune du marché.

4 Les groupes d’intérêts bien informés peuvent


inciter le gouvernement à offrir des programmes
qui ne maximisent pas les avantages nets, 2 Selon la théorie des choix publics, non
car la plupart des électeurs pratiquent 1 ignorance seulement il existe une possibilité de lacune
rationnelle. La plupart des électeurs estiment du marché, mais aussi une possibilité de
qu’il leur faudrait énormément de temps «lacune d’État».
et d’efforts pour s’informer sur une question
particulière. Il en résulte qu’un petit groupe
bien informé, axé sur la promotion de ses
intérêts, pourra exercer sur les programmes 3 L’existence de biens collectifs pose le problème
gouvernementaux une influence bien du resquilleur.
supérieure à son poids électoral.

5 Tous les biens fournis par 1 État ne sont pas


nécessairement des biens collectifs. Un bien
collectif est caractérisé par la non-rivalité d usage
246 CHAPITRE 18

4 On obtient la courbe d’avantage marginal On obtient la courbe de demande totale


de l’économie pour un bien collectif en de l’économie pour un bien collectif en
additionnant les avantages marginaux que a) additionnant horizontalement les courbes
retire chaque individu de chaque quantité de coût marginal individuelles.
de biens fournie. b) additionnant verticalement les courbes
de coût marginal individuelles.
c) additionnant horizontalement les courbes
d’avantage marginal individuelles.
5 Le secteur privé produira une quantité inférieure d) additionnant verticalement les courbes
à la quantité efficiente de biens collectifs purs. d’avantage marginal individuelles.
e) Aucune de ces réponses.

On obtient le niveau de fourniture efficient


d’un bien collectif lorsque
6 L’avantage net est nul lorsque le niveau
a) l’avantage net est maximum.
d’allocation des ressources est efficient.
b) l’avantage marginal est maximum.
c) la différence entre l’avantage marginal
et le coût marginal est nulle.
d) le coût marginal est minimum.
7 Selon la théorie des choix publics, les politiciens
e) a et c sont vrais.
et les fonctionnaires sont surtout motivés par
l’intérêt général. Lorsqu’une rue d’une ville n’est pas
congestionnée, il s’agit
a) d’un bien externe.
b) d’un bien interne.
8 Les partis politiques auront tendance à proposer c) d’un bien rival.
des politiques fondamentalement différentes afin d) d’un bien privé.
d’offrir aux électeurs un choix très clair. e) d’un bien public.

Les gouvernements offrent des biens collectifs


purs comme la défense nationale
a) parce que les gouvernements sont plus
9 Si les politiciens répondent aux exigences efficients que les entreprises privées en
des électeurs, les politiques gouvernementales ce qui concerne la production de ces biens.
maximiseront l’avantage net. b) en raison du problème du resquilleur qui
entraîne la sous-production des marchés privés,
c) parce que les individus n’accordent pas une
très grande valeur à la défense nationale.
10 11 n’est pas rationnel que les électeurs ne soient d) en raison de la possibilité des entreprises
pas informés sur une question aussi importante privées de réaliser des profits excédentaires.
que la défense nationale. e) en raison de coûts externes.

Lequel des biens suivants est caractérisé


par l’impossibilité d’exclusion d’usage?
a) Un autobus urbain.
Questions à choix multiple b) Un pont à péage.
c) Un phare.
d) Un musée d’art.
I On obtient la courbe de demande du marché
pour un bien privé en e) Toutes ces réponses.

a) additionnant horizontalement les courbes La quantité de biens collectifs produite sur


de coût marginal individuelles. un marché non réglementé a tendance à être
b) additionnant verticalement les courbes a) inférieure à la quantité qui permet une
de coût marginal individuelles. allocation efficiente.
c) additionnant horizontalement les courbes b) égale à la quantité qui permet une allocation
d’avantage marginal individuelles. efficiente.
d) additionnant verticalement les courbes c) supérieure à la quantité qui permet une
d’avantage marginal individuelles. allocation efficiente.
e) Aucune de ces réponses. d) celle qui maximise l’avantage public total.
e) celle qui maximise l’avantage public net.
LES LACUNES DU MARCHÉ ET LES CHOIX PUBLICS 247

8 Selon la théorie des choix publics,


a) le gouvernement a tendance à appliquer
FIGURE 18.1
les politiques qui permettent une allocation
des ressources efficiente.
b) les politiciens et les fonctionnaires ont
tendance à être plus motivés par l’intérêt
public que les individus travaillant dans
le secteur privé.
c) les choix publics du gouvernement
maximisent les avantages nets.
d) les outils économiques utilisés pour analyser
les marchés s’appliquent à l’analyse du
comportement du gouvernement.
e) les outils de l’analyse politique s’appliquent
à l’analyse des marchés économiques.

9 Selon la théorie des choix publics, un électeur Quantité de fromage importé


(en milliers de kilogrammes par semaine)
votera en faveur d’un candidat dont le
programme politique
a) semble lui offrir le plus grand avantage 13 Les concurrents qui s’imitent les uns les autres
personnel. pour attirer le plus grand nombre possible
b) est le meilleur pour la majorité des gens. d’électeurs illustrent
c) permet le mieux d’obtenir une allocation a) le principe de la différenciation maximale.
efficiente des ressources. b) le principe de la différenciation minimale.
d) favorise l’électeur médian. c) le principe de l’ignorance rationnelle.
e) Toutes ces réponses. d) le principe de la non-rivalité.
e) la théorie du comportement
10 En général, une taxe de 3 $ par unité du bien A
des fonctionnaires de Niskanen.
fera déplacer la courbe d’offre de A vers le haut
d’un montant 14 Reportez-vous à la figure 18.2, qui indique
a) de 3 $ et augmenter le prix de A de 3 $. le coût total et l’avantage total des projets
b) de 3 $ et augmenter le prix de A d’un de construction de quatre écoles secondaires
montant inférieur à 3 $. de différentes tailles. Le projet offrant l’avantage
c) de 3 $ et augmenter le prix de A d’un net le plus élevé est
montant supérieur à 3 $. a) A.
d) supérieur à 3 $ et augmenter le prix b) B.
de A de 3 $. c) C.
e) inférieur à 3 $ et augmenter le prix de A de 3 $. d) D.
e) impossible à déterminer sans information
I I La perte sèche due à la hausse de prix résultant
supplémentaire.
de l’imposition d’une taxe représente la perte
a) du surplus du consommateur.
FIGURE 18.2
b) du surplus du producteur.
c) du surplus du consommateur plus la perte
du surplus du producteur.
d) du surplus du consommateur moins la perte
du surplus du producteur.
e) de la recette fiscale due à la baisse
de la quantité d’équilibre.

I2 La figure 18.1 indique l’offre et la demande


du fromage importé. Si le gouvernement impose
une taxe de 3 $ par kilogramme de fromage
importé, le prix augmentera de
a) 3 $ à 7 $.
b) 3 $ à 6 $.
c) 2 $ à 6 $.
d) 2 $ à 5 $•
e) Aucune de ces réponses.
248 CHAPITRE 18

15 Reportez-vous à la figure 18.2, qui indique 20 Reportez-vous à la figure 18.3. Le surplus


le coût total et l’avantage total de projets du consommateur avant taxe est égal au triangle
de construction de quatre écoles secondaires a) gmc.
de différentes tailles. Le projet le plus susceptible b) gkc.
d’intéresser les électeurs est c) mkc.
a) A. d) dac.
b) B.
c) C.
d) D.
e) impossible à déterminer sans information
supplémentaire.

16 Reportez-vous à la figure 18.2, qui indique


le coût total et l’avantage total de projets
de construction de quatre écoles secondaires
de différentes tailles. Le projet le plus susceptible
de répondre aux objectifs des fonctionnaires
de l’arrondissement scolaire est
a) A.
b) B.
c) C.
d) D.
e) impossible à déterminer sans information 0 Q, Q2 Q3 Q4 Quantité
supplémentaire.

I7 Si les électeurs partagent les mêmes opinions


et sont bien informés, la quantité de services de
21 Reportez-vous à la figure 18.3. Le surplus
défense nationale que fournit le gouvernement
du producteur avant taxe est égal au triangle
aura tendance à
a) gmc.
a) être supérieure à la quantité permettant
b) gkc.
une allocation efficiente.
c) mkc.
b) être inférieure à la quantité permettant
d) dac.
une allocation efficiente.
e) abc.
c) être la quantité la moins coûteuse.
d) être la quantité qui maximise l’avantage net.
22 Reportez-vous à la figure 18.3. La perte sèche
e) être la quantité qui maximise le budget
due à l’impôt est égale au triangle
du ministère de la Défense.
a) abc.
18 La concurrence que se livrent deux partis politiques b) dbc.
entraînera ces partis à proposer des politiques c) dac.
a) qui sont très différentes. d) edc.
b) qui sont très similaires. e) gdj.
c) d’ignorance rationnelle.
d) qui réduisent le bien-être des familles 23 Selon la théorie des choix publics, un électeur
à revenu moyen et augmentent le bien-être aura tendance à être bien informé si la question
des familles riches et des familles pauvres. qui se pose
e) qui permettent aux coûts totaux d’égaler a) est compliquée et difficile à comprendre.
les avantages totaux. b) touche un peu tout le monde.
19 L’ignorance rationnelle c) présente un intérêt particulier à un petit
a) apparaît lorsque le coût de l’information groupe auquel l’électeur n’appartient pas.
est supérieur aux avantages que procure d) a un effet direct important sur l’électeur.
l’information. e) est importante même si elle ne touche pas
b) permet aux groupes d’intérêts spéciaux directement l’électeur.
d’exercer une influence politique.
c) combinée aux pressions des groupes d’intérêts
spéciaux peut entraîner une répartition des
biens collectifs qui n’est pas efficiente.
d) Toutes ces réponses.
e) Aucune de ces réponses.
LES LACUNES DU MARCHÉ ET LES CHOIX PUBLICS 249

24 Le budget d’un service gouvernemental


TABLEAU 18.1
augmentera probablement au-delà de la quantité
qui permet de maximiser l’avantage net de
Avantage
l’économie
Avantage marginal
a) si les électeurs sont bien informés. Nombre Coût total marginal de tous
b) si l’ignorance rationnelle de l’électeur d’agents par jour par résident les résidents
se combine au lobbying des groupes d’intérêts de sécurité (en dollars) (en dollars) (en dollars)

spéciaux. 1 300 10
c) si cela permet de répartir les ressources 2 600 4
de manière efficiente. 3 900 2
d) si les fonctionnaires pratiquent l’ignorance 4 1 200 1
rationnelle.
e) s’il y a des effets externes négatifs. a) Pourquoi un agent de sécurité représente-t-il
un bien collectif pour les résidents
25 Pour n’importe quelle question sur les dépenses d’Appartements Héritage ?
faisant l’objet d’un vote, l’électeur médian est b) Pourquoi aucun gardien ne sera-t-il embauché
celui qui favorise si chaque résident doit agir individuellement?
a) les dépenses les moins élevées. c) Remplissez la dernière colonne du tableau 18.1
b) les dépenses les plus élevées. en calculant l’avantage marginal que procurent
c) le niveau de dépenses efficient. les agents de sécurité à tous les résidents.
d) le niveau de dépenses moyen.
e) des dépenses supérieures au niveau que ® 7 Supposez maintenant que les résidents forment
préfère la moitié des électeurs et inférieures un Conseil des résidents, qui tient lieu d’organe
au niveau que préfère la moitié des électeurs. administratif chargé de la question de la sécurité.
a) Quel est le nombre optimal d’agents
(allocation efficiente) ? Quel est l’avantage
Problèmes à court développement
net que procure le nombre optimal d’agents ?
b) Montrez que, lorsqu’il y a un agent de moins
1 Expliquez les caractéristiques de non-rivalité et ou un agent de plus que le nombre optimal,
& impossibilité d’exclusion d’un bien collectif pur. l’avantage net est inférieur à l’avantage net
que procure le nombre optimal d’agents.
2 Qu’est-ce que le problème du resquilleur ? c) Comment le Conseil des résidents payera-t-il
pour le nombre optimal d’agents ?
3 Comparez brièvement un équilibre dans un
marché politique à un équilibre dans un marché 8 Deux candidats se présentent à l’élection pour
de biens et de services. le poste de président du Club économique. La
seule question qui les sépare porte sur le budget
4 Expliquez pourquoi il peut être rationnel pour à consacrer à la fête annuelle du Club
les électeurs d’être ignorants. économique. Il est bien connu que les sept
membres votants du club (de A à G) ont
5 Pourquoi l’imposition d’un droit d’accise les préférences illustrées au tableau 18.2. Ils
peut-elle faire déplacer vers le haut la courbe ont des idées très arrêtées quant au montant
d’offre du bien imposé? à dépenser pour la fête.

6 Appartements Héritage compte 100 résidents TABLEAU 18.2


qui se préoccupent de leur sécurité. Le tableau
18.1 donne le coût total d’un service de securité Membre Montant proposé
de 24 heures ainsi que l’avantage marginal votant (en dollars)
qu’en retire chaque résident.
A 10
B 20
C 30
D 40
£ 50
F 60
G 70
250 ÇHAPITRE 18

a) Quel montant proposera chaque candidat ? b) Si le mode de répartition 3 est proposé en


b) Pour voir si votre réponse en a est correcte, remplacement du mode 1, obtiendra-t-il
considérez les résultats des deux élections l’appui de la majorité ? Pourquoi ?
suivantes.
i) Le candidat 1 propose le montant que 10 La figure 18.4 présente l’offre et la demande
vous avez donné en a et le candidat de places de cinéma à Notreville. Le marché est
2 propose 1 $ de moins. Qui sera d’abord en équilibre (point a) au prix de 5 $
le candidat gagnant ? Pourquoi ? la place et de 5 000 places vendues par semaine.
ii) Le candidat 1 propose le montant que Supposez que la municipalité impose
vous avez donné en a et le candidat une nouvelle taxe de 2 $ par place de cinéma.
2 propose 1 $ de plus. Qui sera
le candidat gagnant ? Pourquoi ? FIGURE 18.4
c) Supposons que le Club de sociologie élise
également un président et que la même
question prévale. Il est bien connu que les
sept membres votants du Club de sociologie
ont des idées très arrêtées quant au montant
à dépenser pour leur fête annuelle (voir le
tableau 18.3)

TABLEAU 18.3

Membre Montant proposé


votant (en dollars)

T 0
U 0
Quantité de places de cinéma
V 0 (en milliers de places par semaine)
w 40
X 41
Y 42 a) Illustrez graphiquement les effets de la nouvelle
Z 43 taxe. Quels seront le nouveau prix et la
nouvelle quantité d’équilibre ?
Quel montant les deux candidats proposeront-ils b) Quel pourcentage de la taxe de 2 $ sera payé
dans ce cas ? par les consommateurs sous forme de prix
plus élevé et quel pourcentage sera payé par

9 Le tableau 18.4 présente les trois possibilités les vendeurs de places de cinéma sous forme
de répartition du revenu de cinq personnes, de recettes réduites par billet ?
de A à E. Actuellement, le revenu est réparti c) Quel montant de recettes fiscales sera perçu
suivant le mode 1 (donné dans la première chaque semaine ?
colonne du tableau). Examinez les possibilités d) À combien s’élèvera la perte du surplus du

de répartition 2 et 3, l’une après l’autre. consommateur à cause de la taxe ? du surplus


du producteur ? Quelle est la perte sèche
TABLEAU 18.4 de la taxe ?

Répartition 1 Répartition 2 Répartition 3


Personne (en dollars) (en dollars) (en dollars)
RÉPONSES
A 0 200 150
6 200 300 250
C 400 350 450 Vrai/Faux/Incertain
D 700 600 600 (Justifiez votre réponse.)
£ 1 000 850 850

1 V Production non efficiente en ce qui concerne


a) Si le mode de répartition 2 est proposé en
l’allocation des ressources.
- remplacement du mode 1, obtiendra-t-il
2 V On suppose que les agents du gouvernement
l’appui de la majorité ? Pourquoi ?
agissent dans leur propre intérêt, pas
forcément dans l’intérêt public.
LES LACUNES DU MARCHÉ ET LES CHOIX PUBLICS 251

3 V Non-exclusion —> pas d’incitation à payer. 23 d Information acquise seulement si la valeur


4 V Voir le texte du manuel. est > au coût d’acquisition.
5 V Parce qu’on ne tient pas compte des avantages 24 b Production excédentaire des fonctionnaires.
externes. 25 e Médian = exactement entre les deux.
6 F L’avantage net est maximisé.
7 F Préoccupation de son propre intérêt.
Problèmes à court développement
8 F Proposent des politiques similaires, car la
stratégie qui permet d’obtenir le plus de votes 1 Un bien est caractérisé par la non-rivalité d’usage
possibles est celle qui plaît à l’électeur médian. si la consommation de ce bien par une personne
0 9 I Vrai si les électeurs sont bien informés, faux ne réduit pas la consommation de ce bien
s’ils pratiquent l’ignorance rationnelle. par d’autres. La caractéristique d’impossibilité
10 F L’ignorance rationnelle peut être profitable. d’exclusion signifie que, si le bien est produit
et consommé par une personne, on ne peut
en empêcher d’autres de consommer ce bien.
Questions à choix multiple
2 Le problème du resquilleur est le problème
1 c Voir le texte du manuel. des marchés non réglementés qui produisent
2 d Voir la figure 18.4 du manuel. une trop faible quantité d’un bien collectif
3 e Conditions équivalentes. parce que les individus sont peu enclins à payer
4 e Caractéristiques de non-rivalité pour ce bien. La raison est que le paiement par
et d’impossibilité d’exclusion. Encombrement une personne n’aura probablement pas d’effet
—> rivalité. perceptible sur la quantité que cette personne
5 b Ne serait pas profitable pour les entreprises pourra consommer.
privées.
6 c On ne peut empêcher les bateaux 3 Dans les deux cas, l’équilibre est un état de
de voir le phare. repos, en ce sens qu’aucun groupe n’est incité
7 a La production privée n’est pas profitable à modifier son choix. Dans le cas de l’équilibre
à cause du problème du resquilleur. dans un marché ordinaire de biens et de services,
8 d Marché politique. ni les demandeurs ni les fournisseurs ne peuvent
9 a On suppose que l’électeur n’est préoccupé faire d’échanges qui les avantageront davantage.
que par son propre intérêt. De même, lorsqu’un marché politique est en
10 b Tracez le graphique, a n’est vrai que si l’offre équilibre, ni les demandeurs (électeurs) ni les
est parfaitement élastique. Les autres réponses fournisseurs (politiciens et fonctionnaires) ne
sont toujours fausses. peuvent faire un autre choix qui les avantagerait
I I c Voir le texte du manuel. davantage.
12 c Tracez le graphique. Fait déplacer la courbe
4 La plupart des grandes questions n’ont
d’offre vers le haut d’une distance équivalant
qu’un effet faible et indirect sur la plupart des
à 3$.
électeurs. Il ne serait donc pas rationnel pour
13 b Par définition.
un électeur de consacrer beaucoup de temps et
14 b Plus grande différence entre AT et CT.
d’effort pour devenir bien informé, car les coûts
15 b Maximise l’avantage net.
supplémentaires qui en découleraient seraient
16 d Selon la théorie des choix publics,
rapidement supérieurs à tout avantage
les fonctionnaires maximisent le budget
additionnel. C’est seulement lorsque l’électeur
(coût total).
est touché directement et de manière importante
17 d Le gouvernement répondra aux demandes
par une question qu’il vaut la peine pour lui
des électeurs.
d’être informé. Par conséquent, la majorité des
18 b Maximisation des votes suivant le principe
électeurs seront rationnellement ignorants sur
de la différenciation minimale,
une question quelconque.
Ô 19 d a définition, b et c résultats de 1 ignorance
rationnelle. 5 Lorsqu’un droit d’accise est imposé à un
20 b Partie située au-dessous de la demande, fournisseur, il fait monter le prix minimum
au-dessus du prix du marché. que le fournisseur doit recevoir pour être disposé
21 c Partie située au-dessus de 1 offre, au-dessous à en mettre en vente une quelconque quantité.
du prix du marché. Il s’agit tout simplement d’un déplacement vers
22 c Perte du surplus du consommateur + perte le haut de la courbe d’offre. On peut considérer
du surplus du producteur. un nouveau droit d’accise comme une
augmentation du coût de production.
252 CHAPITRE 18

6 a) Un agent de sécurité est un bien collectif b) Pour un agent, l’avantage net est de 700$:
parce que, dans ce cas, il présente les avantage total (1 000 $) moins le coût total
caractéristiques de non-rivalité et (300 $). Pour trois agents, l’avantage net est
d’impossibilité d’exclusion d’usage. Non- également de 700$ : avantage total (1 600 $)
rivalité parce que la consommation par un moins le coût total (900 $). Par conséquent,
résident de la sécurité offerte par un agent ne l’avantage net de 800 $ pour deux agents
réduit pas la sécurité des autres ; impossibilité est le plus élevé.
d’exclusion parce que, lorsqu’un agent de c) Le Conseil des résidents pourrait engager
sécurité est à son poste, tous les résidents le nombre optimal d’agents de sécurité, soit
bénéficient d’une sécurité accrue ; personne deux agents, en demandant à chacun des
n’est exclu. 100 résidents un montant de 6 $ par jour.
b) Si chaque résident doit engager
individuellement un agent de sécurité, 8 a) Chaque candidat proposera de dépenser 40 $,
personne ne sera engagé parce que l’avantage puisqu’il s’agit de ce que préfère l’électeur
que retire chaque résident du premier agent, médian (électeur D).
qui coûte 300 $ par jour, n’est que de 10 $. b) i) Le candidat 1 gagnera puisque D, E, F
c) On obtient les résultats de la dernière colonne et G voteront pour lui parce que le
présentés au tableau 18.1 Solution en montant de 40 $ se rapproche davantage
multipliant l’avantage marginal par résident de leurs préférences que celui de 39 $
par le nombre de résidents, soit 100. Cette que propose le candidat 2. Seulement
multiplication est l’équivalent numérique A, B et C voteront pour le candidat 2.
de la somme verticale des courbes d’avantage ii) Le candidat 1 puisque que A, B, C et D
marginal individuelles pour chaque nombre voteront pour lui alors que seuls E, F
d’agents de sécurité. et G voteront pour le candidat 2.
c) Là encore, les candidats proposeront tous
TABLEAU 18.1 SOLUTION deux de dépenser 40 $ pour la fête puisqu’il
s’agit du montant que préfère l’électeur
Avantage médian. Remarquez que, dans ce cas, la
Avantage marginal
préférence de l’électeur médian n’est pas
Nombre Coût total marginal de tous
d’agents
la moyenne.
par jour par résident les résidents
de sécurité (en dollars) (en dollars) (en dollars)
9 a) A et B seulement seront avantagés par
1 300 10 1 000 la répartition 2. Celle-ci ne recevra l’appui
2 600 4 400 que de A et B, les personnes C, D et E y étant
3 900 2 200 opposées. Remarquez que la répartition 2
4 1 200 1 100 est défavorable à l’électeur médian (C).
b) La répartition 3 recevra l’appui de la majorité
puisqu’elle avantage l’électeur médian. Elle
Ô 7 a) Si le Conseil des résidents engage chaque
recevra l’appui de A, B et C, les personnes
agent pour lequel l’avantage marginal est
D et E s’y opposant.
supérieur au coût marginal, il engagera le
nombre optimal d’agents. Le coût marginal
10 a) La nouvelle taxe fait déplacer la courbe d’offre
de chaque agent supplémentaire est de 300 $.
de places de cinéma vers le haut de 2 $, soit le
L’avantage marginal associé au premier agent
montant de la taxe. À la figure 18.4 Solution,
est de 1 000 $, donc cet agent sera engagé.
la nouvelle courbe est Ol. Le nouveau prix et
De même, l’avantage marginal associé au
la nouvelle quantité d’équilibre (point b) sont
second agent est de 400$ ; celui-ci sera donc
de 6 $ par place et de 4 000 places vendues
aussi engagé.
par semaine, respectivement.
L’avantage marginal associé au troisième
agent n’est toutefois que de 200 $, ce qui
est inférieur au coût marginal. Nous en
concluons donc que la répartition optimale
du nombre d’agents est de deux. Pour deux
agents, l’avantage net est de 800$ : avantage
• total (1 400 $) moins le coût total (600 $).
LES LACUNES DU MARCHÉ ET LES CHOIX PUBLICS 253

b) Sur la taxe de 2 $, 1 $ est payé par les


FIGURE 18.4 SOLUTION
consommateurs, puisque le prix d’une place
passe de 5 $ à 6 $, et 1 $ est payé par les
vendeurs, puisque la recette par place qu’ils
reçoivent diminue, passant de 5 $ à 4 $.
c) La recette fiscale totale sera de 8 000 $
par semaine : 2 $ par place, multiplié par
4 000 places vendues.
d) La perte du surplus du consommateur
est représentée par le triangle abc de la figure
18.4 Solution et elle est égale à 500 $ (selon
la formule pour la surface d’un triangle
rectangle — surface = 1/2 X base (ac) X
hauteur (bc) = 1/2 X 1 000 X 1 $). De même,
la perte du surplus du producteur est
représentée par la surface du triangle adc,
Quantité de places de cinéma
(en milliers de places par semaine) et elle est égale à 500 $. Par conséquent,
la perte sèche est de 1 000 $, soit la somme
du surplus du consommateur et du surplus
du producteur, perdus à cause de la taxe.
L’inégalité
et la redistribution

Comparer ce qui est comparable


CONCEPTS CLÉS
Le patrimoine est le stock des actifs matériels et
financiers que possède une personne. Le patrimoine
L’inégalité de la répartition du revenu
comprend le capital humain ainsi que les biens tangibles.
et du patrimoine au Canada
Le revenu est le flux des gains générés par le stock
de richesse.
Il existe une grande inégalité de la répartition
du revenu er du patrimoine. La courbe de Lorenz ♦ Les données employées pour construire les
illustre le degré d’inégalité. répartitions du patrimoine ne comprennent
pas le capital humain et elles surestiment
♦ La courbe de Lorenz du revenu (patrimoine)
donc les inégalités du patrimoine.
représente graphiquement le pourcentage
cumulé du revenu (patrimoine) par rapport ♦ Les répartitions annuelles du revenu et du
au pourcentage cumulé des familles. patrimoine sont plus inégales que les répartitions
du revenu et du patrimoine sur la totalité du cycle
♦ La «ligne d’égalité» à 45° représente une
de vie.
répartition hypothétiquement égale du revenu
(patrimoine). ♦ Même en tenant compte de ces facteurs, il existe
une grande inégalité dans la répartition du revenu
♦ Plus la courbe de Lorenz est éloignée de la
et du patrimoine au Canada.
ligne d’égalité, plus la distribution est inégale.

♦ La répartition du patrimoine au Canada est


encore plus inégale que la répartition du revenu. Le prix des facteurs, les dotations
La pauvreté existe lorsque les familles ne peuvent
en facteurs et les choix
acheter la nourriture, le logement et les vêtements
La répartition inégale du patrimoine est due
dont elles ont besoin.
à l’inégalité des salaires, à la répartition inégale
♦ On utilise le seuil de faible revenu (atteint des dotations en facteurs et aux choix personnels.
lorsque les familles consacrent 54,7 % ou plus
♦ Les inégalités peuvent être transmises
de leur revenu à la nourriture, au logement
à la génération suivante au moyen de legs
et aux vêtements) pour mesurer la pauvreté.
(dons faits à la génération suivante) et par les
Au Canada, 13,5% des familles ont des
transferts intragénérationnels (mariages entre
revenus inférieurs au seuil de faible revenu.
personnes de la même classe socio-économique).
♦ Les facteurs les plus importants qui influent
sur la pauvreté sont la source du revenu,
le type de famille et l’âge du chef de famille. La redistribution du revenu

Le gouvernement redistribue le revenu par le biais


de l’impôt sur le revenu, des programmes de sécurité
sociale et de la fourniture de biens et de services
à un prix inférieur au coût d’opportunité.
L'INÉGALITÉ ET LA REDISTRIBUTION 255

♦ L’impôt sur le revenu peut être: ♦ La théorie des processus de motivation est axée
sur l’équité des processus ou moyens par lesquels
• progressif - le taux d’imposition marginal
on obtient les résultats (égalité des possibilités).
T avec T du niveau de revenu ;
Le libéralisme classique souligne l égalité des
• régressif - le taux d’imposition marginal
possibilités et de l’échange volontaire, et tout
si avec T du niveau de revenu;
résultat découlant de ce processus est équitable.
• proportionnel (impôt à taux unique)
- le taux d’imposition marginal est constant
quel que soit le niveau de revenu.

♦ Les programmes de sécurité sociale sont les


programmes de soutien du revenu, d’assurance-
emploi et d’aide sociale. 1 Les statistiques utilisées pour construire les
courbes de Lorenz ne donnent pas toujours un
♦ La fourniture de services d’éducation et de santé
tableau exact de l’inégalité. Il est important de
à un prix inférieur au coût d’opportunité (souvent
comprendre pourquoi. Vous devez comprendre
gratuitement) réduit l’inégalité.
pourquoi, par exemple, la répartition du
La redistribution du revenu engendre le dilemme patrimoine qui exclut la valeur du capital
équité-efficience, qui découle du fait que la humain donnera un tableau déformé de la
redistribution utilise les ressources rares et affaiblit répartition du revenu. Vous devez également
les incitatifs, de sorte qu’un partage plus égal des comprendre pourquoi la répartition d’un revenu
richesses entraîne un montant de richesses moins annuel (statique) donnera un tableau déformé
élevé. Il y a deux manières de s’attaquer au dilemme de la répartition du revenu sur le cycle de vie
équité-efficience. (dynamique). Enfin, vous devez comprendre
pourquoi la répartition du revenu avant
♦ Les réformes à la pièce —> une réponse limitée
déductions fiscales et avant paiements de transfert
à la plupart des problèmes immédiats.
donnera un tableau déformé de la répartition
♦ Les réformes radicales comprenant l’impôt négatif du revenu par rapport à la situation après impôt
sur le revenu, qui assure à chaque famille un revenu et après transfert.
annuel garanti et diminue les avantages quelle
reçoit conformément à un taux avantage-perte 2 La question de l’équité dont on traite dans ce
à mesure que son revenu T. L’impôt négatif sur chapitre est une question normative. Remarquez
le revenu permet d’éviter la plupart des problèmes toutefois que le dilemme entre l’équité et
que connaissent les programmes existants, mais l’efficience économique est une question positive.
il est coûteux.

Quelques notions d’équité


AUTOÉVALUATION
La théorie de la justice distributive comprend un
ensemble de principes permettant de tester 1 équité
de la répartition du bien-être économique. Il existe
Vrai/Faux/Incertain
deux catégories de théories de ce type.
(Justifiez votre réponse.)

♦ La théorie des résultats, qui est axée sur 1 équité


I Au Canada, le revenu est plus inégalement
des résultats ou fins (égalité des résultats).
réparti que la richesse.
• La théorie utilitaire — le résultat doit maximiser
la somme des utilités de tous les individus.
Redistribue le revenu jusqu’à ce que 1 utilité
marginale du dernier dollar dépense par chaque 2 Plus la courbe de Lorenz est éloignée de la ligne
individu soit égale. Égalité complété du revenu à 45°, plus la répartition du revenu est égale.
—» utilité totale maximum.
• La théorie du maximin — le résultat doit
procurer la plus grande utilité possible (revenu)
à la personne ayant l’utilité minimum (revenu). 3 Dans le cadre d’un système d’impôt
La répartition la plus équitable n est pas 1 égalité proportionnel, le montant total de l’impôt
complète mais la redistribution jusqu à ce que augmente à mesure qu’augmente le revenu.
la personne dans la plus mauvaise situation
ne puisse plus retirer aucun avantage.
156 .CHAPITRE 19

4 Le taux d’imposition marginal ne change 3 Par transfert intragénérationnel, on veut dire


pas à mesure que le revenu augmente. a) que les femmes pauvres ont tendance
à épouser des hommes riches.
b) que les femmes riches ont tendance à épouser
des hommes riches.
5 Un taux d’imposition régressif redistribue c) que les femmes riches ont tendance à épouser
le revenu des riches aux pauvres. des hommes pauvres.
d) qu’il se produit des mariages entre personnes
de même sexe, car «qui se ressemble
6 Comparativement à la répartition des revenus du s’assemble».
marché, les avantages et impôts du gouvernement e) les mariages entre personnes de même sexe
réduisent l’inégalité de la répartition du revenu. sont interdits.

4 Reportez-vous aux courbes de Lorenz de


la figure 19.1. Quelle est celle qui correspond
7 La répartition des richesses financières ne tient à la plus grande inégalité de revenu ?
pas compte de la répartition du capital humain a) A
et, de ce fait, elle surestime l’inégalité entre les b) B
individus. c) C
à) D
e) Impossible à dire sans information
supplémentaire.
8 Si les dettes pouvaient être transmises, la
répartition du patrimoine deviendrait plus
égale au fil du temps.

9 Un système d’impôt négatif sur le revenu


engendre le cercle vicieux de l’aide sociale
en décourageant la recherche d’un emploi.

10 En général, lorsqu’on réduit l’inégalité du revenu


par une redistribution qui consiste à prendre
aux nantis pour donner aux démunis, on obtient
une plus grande production de biens et de 0 20 40 60 80 100
services. Pourcentage cumulé des familles

5 Dans la figure 19.1, comment appelle-t-on


Questions à choix multiple
la courbe A (ligne droite) ?
1 Les différences entre les salaires reçus par a) Une courbe de répartition du marché.
différentes personnes sont dues à des différences b) Une ligne d’égalité.
a) de produit marginal du travail. c) Une ligne d’équité.
b) de compétences naturelles. d) La courbe du dilemme équité-efficience d’Okun.
c) de capital humain. e) Aucune de ces réponses.
d) Toutes ces réponses.
e) Aucune de ces réponses. 6 Quel point de la figure 19.1 indique que le
quintile de familles les plus riches gagne 40 %
2 Un legs est du revenu ?
a) une erreur. a) a
b) une taxe sur la richesse héritée. b) b
c) un don transmis par une génération à une autre. c) c
d) un incitatif offert à une génération pour d) d
quelle travaille davantage. e) Aucune de ces réponses.
e) un héritage conditionnel.
L'INÉGALITÉ ET LA REDISTRIBUTION 257

7 Même si la répartition des salaires est symétrique, 12 Dans le tableau 19.1, quel est le régime d’impôt
la répartition du revenu sera asymétrique, car progressif?
a) les capacités sont réparties de manière a) Le régime A.
symétrique. b) Le régime B.
b) les capacités sont réparties de manière c) Le régime C.
asymétrique. d) Le régime D.
c) les individus ont tendance à offrir plus e) c et d.
de travail à des salaires plus élevés.
d) les individus ont tendance à offrir moins I3 La répartition du revenu annuel
de travail à des salaires plus élevés. a) sous-estime le degré d’inégalité parce quelle
e) b et d sont vrais. ne tient pas compte de l’étape du cycle de vie
de la famille.
8 Quelle action suivante permet de réduire
b) sous-estime le degré d’inégalité parce quelle
l’inégalité du revenu ou du patrimoine par
ne tient pas compte de la répartition du
rapport à la répartition du marché ?
capital humain.
a) Les paiements de transfert du gouvernement
c) surestime le degré d’inégalité parce quelle ne
aux démunis.
tient pas compte de l’étape du cycle de vie de
b) Un système d’impôt négatif sur le revenu.
la famille.
c) Des legs importants.
d) surestime le degré d’inégalité parce quelle ne
d) L’association intragénérationnelle.
tient pas compte de la répartition du capital
e) Toutes ces réponses.
humain.
9 Une théorie de justice distributive qui met e) est une mesure exacte du degré d’inégalité.
l’accent sur le mécanisme de répartition est
a) une théorie utilitaire. 14 La répartition du patrimoine
b) une théorie des fins. a) sous-estime le degré d’inégalité parce quelle
c) une théorie du maximin. ne tient pas compte de l’étape du cycle de vie
d) une théorie du processus de motivation. de la famille.
e) une théorie des résultats. b) sous-estime le degré d’inégalité parce quelle
ne tient pas compte de la répartition du
10 La richesse diffère du revenu en ce sens que
capital humain.
a) le revenu est un stock, la richesse est un flux.
c) surestime le degré d’inégalité parce quelle
b) la richesse dérive du revenu. ne tient pas compte de l’étape du cycle de vie
c) le revenu est ce que l’on gagne, la richesse
de la famille.
est ce que l’on possède. d) surestime le degré d’inégalité parce quelle ne
d) le revenu est ce que l’on possède, la richesse
tient pas compte de la répartition du capital
est ce que l’on gagne. humain.
e) la richesse est préférable au revenu. e) est une mesure exacte du degré d’inégalité.
I I Reportez-vous au tableau 19.1. Quel
est le régime d’imposition proportionnel? ® I5 D’après la théorie du maximin, le revenu doit
a) Le régime A. être redistribué si
b) Le régime B. a) cela peut améliorer le sort de la personne

c) Le régime C. moyenne.
d) Le régime D.
b) cela peut améliorer le sort de la personne
e) Impossible à déterminer sans information la plus pauvre.
supplémentaire. c) cela peut désavantager la personne la plus riche.
d) le taux salarial est supérieur au produit

TABLEAU 19.1 marginal du travail.


e) le taux salarial est inférieur au produit
Revenu Impôts Impôts Impôts Impôts marginal du travail.
payés - payés - payés - payés -
brut
courant Régime A Régime B Régime C Régime D ® 16 Quel exemple suivant illustre une théorie des
résultats dans le cadre de la justice distributive?
0 0 0 0 200
a) La théorie du produit marginal.
1 000 100 100 200 200
b) La théorie du libéralisme classique.
2 000 200 400 200 200
c) La théorie utilitaire.
400 1 600 200 200
4 000 d) La théorie du processus de motivation.
e) Aucune de ces réponses.
258 CHAPITRE 19

17 Dans la figure 19.2, quel pourcentage 21 Supposez qu’une famille gagne un revenu
de l’ensemble du revenu reçoit le quintile de 0 $ et reçoit du gouvernement un paiement
des familles les plus riches ? de transfert mensuel de 1 000 $. Si cette famille
a) 10% gagne 400 $ durant un mois, le gouvernement
b) 20% réduit son paiement à 700 $. Quel est le taux
c) 30% d’imposition marginal dans ce cas?
d) 40% a) 0%
e) Aucune de ces réponses. b) 10%
c) 40%
FIGURE 19.2 d) 50%
e) 75%

22 Le facteur le plus important qui influe


sur l’incidence de faible revenu est
a) la source de revenu.
b) le sexe du chef de ménage.
c) l’âge du chef de ménage.
d) le type de famille.
e) la région géographique.

23 Si le taux d’imposition marginal augmente


à mesure qu’augmente le revenu, l’impôt sur
le revenu est défini comme étant
a) progressif.
b) proportionnel.
c) négatif.
18 Dans la figure 19.2, quel pourcentage du revenu d) régressif.
reçoit le quintile des familles les plus pauvres ? e) excessif.
a) 10%
b) 20% 24 La redistribution du revenu qui consiste à prendre
c) 30% aux nantis pour donner aux démunis entraînera
d) 40% une réduction de la production totale. On
e) Aucune de ces réponses. appelle ce phénomène
a) la répartition du marché.
19 Dans la figure 19.2, quel pourcentage du revenu b) la théorie des processus de motivation dans
reçoit le troisième quintile des familles ? le cadre de la justice distributive.
a) 10% c) la théorie des résultats de justice distributive.
b) 20% d) un dilemme équité-efficience.
c) 30% e) un dilemme capitaliste.
d) 40%
e) Aucune de ces réponses. 25 Quel diagramme utilisent les économistes pour
illustrer la répartition du revenu ou du patrimoine ?
20 La courbe de la figure 19.2 représente a) La courbe de Lorenz.
a) la ligne de répartition du revenu de marché. b) La distribution normale.
b) la ligne de l’égalité parfaite. c) La courbe de Rawls.
c) la courbe du dilemme équité-efficience d’Okun. d) La courbe du dilemme équité-efficience d’Okun.
d) la courbe de Nozick. e) Aucune de ces réponses.
e) la courbe de Lorenz.

Problèmes à court développement

1 Expliquez les différences et les liens entre


les concepts de patrimoine et de revenu.

2 Quels sont les deux facteurs qui déterminent


le revenu d’une personne? Dans quelle mesure
ces facteurs sont-ils le résultat de forces sur
L'INÉGALITÉ ET LA REDISTRIBUTION 259

lesquelles l’individu n’a aucun contrôle et dans


quelle mesure sont-ils le résultat d’un choix
TABLEAU 19.3 REVENU FAMILIAL APRÈS
individuel ? DÉDUCTIONS FISCALES

Part du
3 Dans le cadre de la justice distributive, les Revenu revenu Pourcentage
deux catégories de théories sont les théories après après cumulé
des processus de motivation et les théories des déductions déductions du revenu
fiscales (en fiscales Pourcentage après
résultats. Quelle est la principale caractéristique
Pourcentage milliards (en cumulé déductions
d’une théorie des processus? Pourquoi la théorie
des familles de dollars) pourcentage) des familles fiscales
utilitaire est-elle une théorie des résultats ?
Quintile
4 Pourquoi existe-t-il un dilemme équité-efficience ? des familles
les plus
5 Le tableau 19.2 donne l’information sur la pauvres 5
répartition du revenu dans une économie qui
Second
génère un revenu annuel total de 100 milliards
quintile 9
de dollars.
Troisième
TABLEAU 19.2 REVENU FAMILIAL TOTAL quintile 12

Revenu total Part du Pourcentage Pourcentage Quatrième


Pourcentage (en milliards revenu (en cumulé des cumulé du
quintile 15
des familles de dollars) pourcentage) familles revenu

Quintile Quintile

des familles des familles


les plus
les plus
riches 30
pauvres 5

Second a) Remplissez le tableau 19.3.


quintile 10 b) Tracez la courbe de Lorenz pour le revenu
après déductions fiscales sur le graphique
Troisième que vous avez utilisé pour le problème à court
quintile 15 développement 5 b et nommez-la B.
c) Quel effet a eu le système d’impôt progressif
Quatrième
sur l’inégalité ?
quintile 20

Quintile des 7 Enfin, supposez qu’en plus le gouvernement


familles les redistribue toutes les recettes fiscales de manière
plus riches 50 que la répartition du revenu après transfert
(après déductions fiscales) soit telle qu’indiquée
a) Remplissez le tableau 19.2 en calculant par le tableau 19.4. Par exemple, les membres
les chiffres des trois dernières colonnes. du groupe le plus pauvre reçoivent un revenu
b) Tracez la courbe de Lorenz du revenu
de transfert de 10 milliards de dollars, de sorte
dans cette économie et nommez-la A. que leur revenu après transfert est de 15 milliards
de dollars.
6 Supposez maintenant qu’un système d impôt
progressif s’applique à cette économie. Le
tableau 19.3 présente la répartition du revenu
après déductions fiscales. Nous avons supposé
qu’aucun revenu n’est redistribué aux familles de
cette économie. Remarquez que le revenu après
déductions fiscales est de 71 milliards de dollars.
260 CHAPITRE 19

TABLEAU 19.4 REVENU FAMILIAL TABLEAU 19.5 ÉVOLUTION DU REVENU


APRÈS PAIEMENTS DURANT TOUT
DE TRANSFERT LE CYCLE DE VIE

Pourcentage Part du Nombre Revenu


du revenu revenu
Groupe de personnes annuel
après après Pourcentage
d’âge dans le groupe individuel
paiements paiements cumulé du

de transfert de transfert Pourcentage revenu après (en années) d’âge (en dollars)
Pourcentage (en milliards (en cumulé paiements
0-20 20 0
des familles de dollars) pourcentage) des familles de transfert
21-35 20 30 000
Quintile 36-50 20 40 000
des familles 51-65 20 60 000
les plus 66-80 20 20 000
pauvres 15
a) Tracez la courbe de Lorenz pour le revenu
Second
durant tout le cycle de vie dans cette
quintile 16
économie et appelez-la A.
Troisième b) Tracez la courbe de Lorenz pour le revenu
quintile 18 annuel dans cette économie et appelez-la B.
c) Laquelle des deux courbes mesure le mieux
Quatrième l’inégalité entre les individus dans cette
quintile 20 économie? Pourquoi?

Quintile
9 Considérez les mondes de Vulcan et de Klingon.
des familles
La figure 19.3 illustre la courbe de Lorenz
les plus
de Vulcan et le tableau 19.6 présente les données
riches 31
de la répartition du revenu de Klingon.

a) Remplissez le tableau 19.4.


FIGURE 19.3
b) Tracez la courbe de Lorenz pour le revenu
après paiements de transfert sur le graphique
que vous avez utilisé pour les problèmes à
court développement 5b et 6b et appelez-la C.
c) Quel est l’effet de la redistribution du revenu
sur l’inégalité grâce aux paiements de transfert ?

S> 8 Considérez une économie composée de


100 personnes identiques sous tous les angles.
Chacune d’elles vit jusqu’à 80 ans et pas plus.
De la naissance à l’âge de 20 ans, elles ne gagnent
pas de revenu; de 21 ans à 35 ans, chacune d’elles
gagne un revenu annuel de 30 000$ ; de 36 ans
à 50 ans, chacune d’elles gagne un revenu
de 40 000$ ; de 51 ans à 65 ans chacune d’elles
reçoit un revenu annuel de 60 000$ ; et de
66 ans à 80 ans, chacune d’elles reçoit un revenu TABLEAU 19.6 RÉPARTITION DU
annuel de 20 000 $. À n’importe quel moment, REVENU DE KLINGON
il y a 20 personnes dans chaque groupe d’âge.
Pour simplifier, nous supposerons qu’il n’y a pas Classement Part du revenu total
de legs. Le tableau 19.5 résume cette information. du revenu familial (en pourcentage)

Quintile inférieur 10
Second quintile 10
Troisième quintile 20
Quatrième quintile 30
Quintile supérieur 30
L’INÉGALITÉ ET LA REDISTRIBUTION 26

a) Nommez les axes de la figure 19.3 et 8 a Les autres augmentent l’inégalité.


expliquez ce que représente la ligne diagonale. ® 9 d Par définition.
b) En utilisant les données du tableau 19.6, 10 c La richesse est un stock, le revenu, un flux
tracez la courbe de Lorenz de Klingon engendré par le patrimoine.
sur la figure 19.3. 11 a Les impôts sont toujours de 10% du revenu.
12 b Avec T revenu, le taux d’imposition moyen
@10 II existe deux principales théories de la justice T de 0% à 10%, 20%, 40%.
distributive. 0 13 c Le capital humain influe sur la répartition
a) Énoncez et définissez une théorie. du patrimoine, non sur le revenu.
b) Énoncez et définissez l’autre théorie. 0 14 d Le cycle de vie influe sur la répartition
c) En utilisant seulement l’information du du revenu, non sur celle du patrimoine.
problème à court développement 9, dites ® 15 b Par définition.
quelle(s) théorie(s) de la justice distributive on ® 16 c Maximise la somme des utilités (résultats)
peut utiliser pour évaluer Vulcan et Klingon ? des individus.
Quel monde est le plus juste suivant cette (ces) 17 d 100% — 60% de revenu cumulé.
théorie(s) ? 18 a 10 % — 0% de revenu cumulé.
19 b 40 % — 20 % de revenu cumulé.
20 e Par définition.
021 e AT/AY= 300 $/400$.
RÉPONSES 22 a Voir les notes de la figure 19.3 du manuel.
23 a Par définition.
24 d Voir le texte du manuel.
Vrai/Faux/Incertain 25 a Par définition.
(Justifiez votre réponse.)
Problèmes à court développement
1 F Voir la figure 19.1 du manuel.
2 F Plus inégale. 1 Le patrimoine est le stock des actifs que possède
3 V Le taux d’impôt marginal est constant, un individu, alors que le revenu est le flux des
mais le total des impôts î. gains que reçoit un individu. Les concepts sont
4 I Selon que l’impôt est régressif (F), reliés en ce sens que le revenu d’un individu
proportionnel (V) ou progressif (F). correspond aux gains générés par son stock de
5 F Des démunis aux nantis. richesse.
6 V Voir la figure 19.5 du manuel. 2 Le revenu d’une personne est déterminé
7 V Le patrimoine que procure le capital humain par les prix du marché des services de ressources
est réparti plus également que la richesse productives et la quantité de services de ressources
financière. que la personne peut et est disposée à vendre à
8 V Seuls les avoirs peuvent être légués. ces prix. Ces deux facteurs dépendent de plusieurs
9 F Élimine le cercle vicieux de l’aide sociale choses, dont certaines sont (au moins partiellement)
et encourage la recherche d’un emploi. sous son contrôle et d’autres ne le sont pas.
10 F Production plus faible à cause du dilemme Le prix des services de travail, le taux salarial,
équité-efficience. est déterminé sur le marché du travail. Toutefois,
comme nous l’avons appris aux chapitres 14 et
15, le taux salarial dépendra du produit marginal
Questions à choix multiple
du travail, qui évolue en fonction des choix des
individus en matière de formation et d’éducation
1 d Toutes influent sur la valeur du produit
ainsi que de leurs aptitudes personnelles.
du travail.
La quantité des services de travail offerts
2 c Par définition.
dépendra aussi des choix personnels quant à la
3 b Mariage au sein de sa propre classe socio¬
manière de passer son temps. La quantité d’autres
économique.
ressources offertes dépendra également des choix
4 d La courbe la plus éloignée de la courbe de 45°.
personnels ainsi que de la dotation en ce facteur.
5 b Par définition.
6 c En passant de 80% à 100% des familles ® 3 Dans le cadre de la justice distributive, une
(quintile supérieur) le revenu passe de 60 % théorie du processus de motivation est axée
à 100% du total (40%). sur l’équité du processus ou des mécanismes
7 c Les salaires plus élevés associes à des revenus permettant d’obtenir les résultats plutôt
encore plus élevés (salaire X quantité de travail). que sur les résultats eux-mêmes. Selon la théorie
262 CHAPITRE 19

utilitaire, le système le plus juste est celui où la b) La courbe appelée A dans la figure 19.4 est
somme des utilités est maximale. Étant donné la courbe de Lorenz du revenu familial total.
que cette théorie est axée sur les résultats ou fins, Elle permet simplement d’inscrire les valeurs
il s’agit d’une théorie des résultats de la justice des deux dernières colonnes du tableau 19.2
distributive. Solution.

4 Si elle signifie augmentation de l’égalité du


FIGURE 19.4
revenu, une « plus grande équité » ne peut être
réalisée que par la redistribution du revenu ;
il est nécessaire d’imposer le revenu de certains
pour pouvoir verser des paiements de transfert
aux autres. Toutefois, il existe des effets incitatifs
qui réduisent le montant total du revenu
pouvant être distribué.
Si des activités productives comme le travail
sont assujetties à l’impôt, la tendance sera de
réduire le temps passé à ces activités. En outre,
tout programme de redistribution nécessiterait
l’utilisation de ressources pour l’administrer,
de sorte que moins de ressources pourraient être
0 20 40 60 80 100
consacrées à d’autres activités productives, ce qui
Pourcentage cumulé des familles
nous permet de constater qu’une tarte que l’on
divise plus également donne une plus petite tarte.

5 a) Le tableau 19.2 rempli est présenté au tableau 6 a) Le tableau 19.3 rempli est présenté
19.2 Solution. La part du revenu de chaque au tableau 19.3 Solution.
groupe de familles est le revenu total de ce
groupe en pourcentage du revenu total dans TABLEAU 19.3 SOLUTION
l’économie (100 milliards de dollars). On REVENU FAMILIAL
obtient le pourcentage cumulé du revenu APRÈS DÉDUCTIONS FISCALES
(dernière colonne) en additionnant la part du
pourcentage du revenu du groupe (à partir de Part du

Revenu revenu Pourcentage


la troisième colonne) à la part du pourcentage
après après cumulé
du revenu total de tous les groupes de familles déductions déductions du revenu
les plus pauvres. fiscales (en fiscales Pourcentage après

Pourcentage milliards (en cumulé déductions

TABLEAU 19.2 SOLUTION des familles de dollars) pourcentage) des familles fiscales

REVENU FAMILIAL TOTAL


Quintile
des familles
Revenu total Part du Pourcentage Pourcentage
Pourcentage (en milliards revenu (en cumulé des cumulé du les plus
des familles de dollars) pourcentage) familles revenu pauvres 5 7 20 7

Quintile Second
des familles quintile 9 13 40 20
les plus
pauvres 5 5 20 5 Troisième
quintile 12 17 60 37
Second
quintile 10 10 40 15 Quatrième
quintile 15 21 80 58
Troisième
quintile 15 15 60 30 Quintile
Quatrième des familles
quintile 20 20 80 50 les plus
riches 30 42 100 100
Quintile des
familles les b) La courbe appelée B à la figure 19.4
plus riches 50 50 100 100 est la courbe de Lorenz du revenu familial
après déductions fiscales.
L'INÉGALITÉ ET LA REDISTRIBUTION 263

c) L’impôt sur le revenu progressif a réduit


FIGURE 19.5
l’inégalité en prenant un plus grand
pourcentage du revenu des groupes de revenu
plus élevés.

7 a) Le tableau 19.4 rempli est présenté


au tableau 19.4 Solution.

TABLEAU 19.4 SOLUTION


REVENU FAMILIAL
APRÈS PAIEMENTS DE TRANSFERT

Pourcentage Part du
du revenu revenu
après après Pourcentage 0 20 40 60 80 100
paiements paiements cumulé du Pourcentage cumulé des individus
de transfert de transfert Pourcentage revenu après
Pourcentage (en milliards (en cumulé paiements
des familles de dollars) pourcentage) des familles de transfert b) La courbe de Lorenz du revenu annuel est
appelée B à la figure 19.5. Elle reflète le fait
Quintile que le quintile (20 %) d’individus les plus
des familles pauvres (0-20 ans) reçoit 0 % du revenu
les plus annuel ; le deuxième quintile d’individus les
pauvres 15 15 20 15
plus pauvres, ceux de 66-80 ans, reçoit
13% du revenu annuel; le troisième quintile
Second
16 40 31 d’individus les plus pauvres, ceux de 21-35 ans,
quintile 16
reçoit 20 % du revenu annuel; ceux de 36-50
Troisième ans reçoivent 27 % du revenu annuel ; et le
quintile 18 18 60 49 quintile d’individus les plus riches, le groupe
des 51-65 ans, reçoit 40% du revenu annuel.
Quatrième c) La répartition du revenu au cours du cycle
quintile 20 20 80 69
de vie est une meilleure mesure du degré
d’inégalité. Dans cette économie fictive, tous
Quintile
les individus sont identiques (égaux), un fait
des familles
que reflète l’égalité des revenus durant la
les plus
100 totalité du cycle de vie. La seule raison pour
riches 31 31 100
laquelle la répartition du revenu annuel dans
cette économie n’est pas égale est que les
b) La courbe nommée C à la figure 19.4 est
individus se trouvent à des étapes différentes
la courbe de Lorenz du revenu familial apres
d’un cycle de vie identique.
déductions fiscales et après paiements de
transfert.
9 a) Voir figure 19.3 Solution. La ligne diagonale
c) La redistribution du revenu au moyen
représente hypothétiquement l’égalité du
de paiements de transfert a réduit 1 inégalité.
revenu.
b) Voir la figure 19.3 Solution.
0 8 a) Puisque, dans ce cas, chaque personne gagne
exactement le même revenu durant la totalité
de son cycle de vie, la courbe de Lorenz du
revenu du cycle de vie coïncide avec la ligne
d’égalité et est nommée d à la figure 19.5.
264 CHAPITRE 19

a) Dans le cadre de la justice distributive, une


FIGURE 19.3 SOLUTION
théorie des résultats est axée sur l’équité des
résultats (égalité des résultats).
b) Une théorie des processus de la motivation est
axée sur l’équité des processus qui permettent
d’obtenir les résultats (égalité des possibilités).
c) Puisque nous n’avons d’information que sur
les résultats de revenu et non sur le processus
par lequel ces résultats ont été obtenus, nous
ne pouvons utiliser que la théorie des résultats
pour porter un jugement sur Klingon et
Vulcan. Comme la courbe de Lorenz de
Klingon est plus près de la ligne d’égalité, le
monde de Klingon est plus juste, selon la
théorie des résultats.
Pourcentage cumulé des familles
La politique
de concurrence

♦ Les pratiques monopolistiques créent une


CONCEPTS CLÉS tension entre les intérêts des consommateurs
et ceux des producteurs. Cette tension est
à la base de la théorie économique de
L’intervention des administrations
la réglementation.
publiques dans les marchés

Les formes d’intervention gouvernementale dans La théorie économique


les marchés monopolistiques et oligopolistiques sont : de l’intervention publique
♦ La réglementation - l’ensemble des règles
La théorie économique de l’intervention publique
concernant la fixation des prix, les normes de
relève de la théorie, plus générale, des choix
production et les conditions d’entrée des nouvelles
publics (chapitre 18), mais elle met l’accent sur
entreprises sur les marchés. La déréglementation
la réglementation gouvernementale.
est le processus d’abolition de ces règles.

♦ Les entreprises publiques - communément


♦ La demande d’interventions T avec T
appelées sociétés d’État. La privatisation est • du surplus du consommateur par acheteur.
la vente de sociétés d’État à des intérêts privés. • du nombre d’acheteurs.
• du surplus du producteur par entreprise.
♦ La législation antimonopole - interdit certaines
• du nombre d’entreprises.
pratiques commerciales spécifiques (pratiques
monopolistiques). ♦ L’offre d’interventions T avec T
L’intervention gouvernementale a pour but d’influer • du surplus du consommateur par acheteur.
sur les surplus. Le surplus total représente les gains • du surplus du producteur par entreprise.
combinés des consommateurs et des producteurs • du nombre de personnes touchées.
à la suite de l’échange ; la somme du surplus du
En situation d’équilibre politique, aucun groupe
consommateur et du surplus du producteur.
d’intérêt ne souhaite faire modifier la réglementation
♦ Surplus du consommateur — valeur que représentent existante, et aucun politicien ne propose donc
les biens pour les consommateurs — prix payé d’interventions. Il existe deux théories de l’équilibre
= surface située sous la courbe de demande politique.
et au-dessus du prix du marché.
♦ La théorie de l’intérêt public - fait des prévisions
♦ Surplus du producteur — recette totale — coût concernant les règlements qui permettent de
d’opportunité = surface située sous le prix maximiser le surplus total et d’éliminer la perte
du marché et au-dessus de la courbe d offre. sèche. L’appareil politique agira dans l’intérêt
public pour éliminer le gaspillage et réaliser une
Le surplus total est maximisé en situation
allocation efficiente des ressources.
de concurrence lorsque la perte sèche - 0.
♦ La théorie de la capture des interventions
♦ En situation de monopole, T surplus publiques - fait des prévisions concernant les
du producteur, -I- surplus du consommateur
règlements qui permettent de maximiser le surplus
et création d’une perte sèche.
266 CHAPITRE 2 0

du producteur et le profit économique. L’appareil ♦ Les résultats sont mitigés, mais les cartels
politique est «capturé» par les intérêts des réglementés semblent réaliser des profits égaux
producteurs. ou supérieurs à la moyenne, ce qui confirme
plutôt la théorie de la capture que la théorie
de l’intérêt public.
La réglementation
et la déréglementation
L’entreprise publique
Les organismes de réglementation du gouvernement
établissent des règlements sur les prix, les volumes Les entreprises publiques peuvent être exploitées
de production et l’accès aux marchés. en fonction des objectifs suivants :

Le monopole naturel (en raison des économies d’échelle, ♦ Allocation efficiente des ressources — le prix
une entreprise peut satisfaire la demande totale est fixé de manière à être égal au coût marginal
à un CTM inférieur à celui de plusieurs entreprises et à couvrir la perte économique au moyen
concurrentes) peut être réglementé selon les règles de subventions. Maximise le surplus
suivantes. du consommateur.

♦ Tarification au coût marginal - selon laquelle ♦ Maximisation du budget en respectant la règle


le prix est fixé de manière à être égal au coût de la tarification au coût marginal — le prix est fixé
marginal. Maximise le surplus total du secteur de manière à être égal au coût marginal, mais
réglementé, mais n’est pas viable parce que l’entreprise gonfle ses coûts de production
les entreprises subissent une perte économique. au maximum. L’allocation est efficiente, mais
les avantages du producteur sont maximisés.
♦ Tarification au coût moyen - selon laquelle le
prix est fixé de sorte qu’il soit égal au coût total ♦ Maximisation du budget avec gratuité de service
moyen. Peut être plus efficace même si elle -T production jusqu’à ce que le prix soit nul.
engendre une perte sèche. Inefficace (perte sèche), cause une surproduction
et maximise les avantages du producteur.
♦ Tarification en fonction du taux de rendement —
consiste à fixer les prix de manière que l’entreprise ♦ Selon les études, les entreprises publiques ont
réglementée puisse obtenir un taux de rendement tendance à produire trop et à être moins efficaces
donné sur le capital. que les entreprises privées. Récente tendance
à privatiser les entreprises publiques.
• Idéalement, ne donne qu’un profit normal ;
même résultat qu’avec la tarification au coût
moyen. La législation antimonopole
• Toutefois, l’entreprise peut se sentir incitée
à gonfler ses coûts pour obtenir plus de profit La Loi sur la concurrence de 1986 fait une
économique. distinction entre

♦ Les résultats sont mitigés, mais les monopoles ♦ Les actes criminels (conspiration pour fixer
naturels réglementés semblent réaliser des profits les prix, truquage des offres, publicité trompeuse),
égaux ou supérieurs à la moyenne, ce qui confirme qui relèvent des tribunaux.
plutôt la théorie de la capture que la théorie de
♦ Les actes non criminels (fusions, abus de position
l’intérêt public.
dominante sur le marché, transaction exclusive),
Le cartel (entente de collusion entre les entreprises qui font l’objet d’un examen par le Tribunal
d’un oligopole visant à limiter la production afin de de la concurrence.
T les prix) peut être réglementé selon les règles suivantes.
Selon toute évidence, la législation antimonopole
♦ Fixer le prix et la quantité aux niveaux sert l’intérêt public.
concurrentiels (intersection des courbes
de demande de l’industrie et de Cm). Résultat
conforme à la théorie de l’intérêt public.

♦ Fixer le prix er la quantité pour maximiser le profit


RAPPELS
(prix le plus élevé associé à la quantité à l’intersection
I La figure 20.1 présente les courbes de recette
des courbes de Cm et de Rm). Résultat conforme
à la théorie de la capture. et de coût marginal d’une industrie. Cette
figure montre que la réglementation permet
de déterminer la répartition du surplus total
LA POLITIQUE DE CONCURRENCE 267

potentiel (le triangle abc), entre le surplus


du consommateur, le surplus du producteur AUTOÉVALUATION
et la perte sèche.

Vrai/Faux/1 ncertain
FIGURE 20.1
(Justifiez votre réponse.)

I Dans un monopole, le surplus du producteur


est maximisé lorsque le volume de production
permet de maximiser les profits.

2 Dans un monopole, le surplus total est


maximisé lorsque le volume de production
permet de maximiser les profits.

3 Selon la théorie de la capture, les taux de


Si l’industrie est parfaitement concurrentielle, rendement sont supérieurs à la normale dans
alors la production sera Q. et le prix du marché, les monopoles naturels réglementés.
Pc. Le surplus total est maximisé et représenté
par le triangle abc. Le surplus total est égal à la
somme du surplus du consommateur (triangle
dbc) et du surplus du producteur (triangle adc). 4 Selon la théorie de la capture, les producteurs
Il n’y a pas de perte sèche. sont favorables à la déréglementation.
Si l’industrie est un monopole qui maximise
ses profits, la production sera et le prix,
Pm. Dans ce cas, le surplus total est représenté
par le trapèze abfg. À cause de la limitation 5 L’intervention gouvernementale vise à conduire
de la production du monopole, le surplus total l’économie vers une allocation des ressources
en situation de monopole est plus petit qu’en plus efficiente.
situation de concurrence. La différence est
la perte sèche engendrée par le monopole, qui
correspond au montant du surplus total perdu
lors du passage de la concurrence au monopole. 6 Selon la théorie de la capture, les organismes
La perte sèche est illustrée par le triangle gfc. de réglementation du gouvernement finiront
Le surplus total peut se répartir en surplus par s’approprier les profits des industries qu’ils
du consommateur (triangle ebf) et surplus réglementent.
du producteur (trapèze aefg). Le surplus du
consommateur est assez faible, mais le surplus
du producteur est maximal.
La production réelle se situera probablement 7 Dans la pratique, un taux de rendement
entre les deux. Comme la production se déplace réglementé équivaut à la tarification au coût
de Qc à CL, le surplus du consommateur diminue, marginal.
alors que le surplus du producteur et la perte
sèche augmentent tous les deux. Si cette industrie
est réglementée, le volume de production se
rapprochera de Q. selon la théorie de 1 intérêt 8 Lorsque le taux de rendement est réglementé,
public, alors que le volume de production se les entreprises peuvent maximiser le surplus du
rapprochera de selon la théorie de la capture producteur en gonflant leurs coûts.
des interventions publiques.
268 CHAPITRE 20

9 Si les prix et les profits de l’industrie baissent 6 Dans une situation de monopole, le surplus
après la déréglementation, la réglementation du producteur est maximisé lorsque
servait probablement les intérêts des a) le coût marginal est égal à la recette
consommateurs. marginale.
b) le coût marginal est égal au coût total moyen
c) le prix est égal au coût marginal.
d) le prix est égal au coût total moyen.
10 La législation antimonopole vise généralement e) le prix est égal au coût variable moyen.
la réalisation d’une allocation efficiente des
ressources et l’intérêt public. 7 La demande d’interventions par les producteurs
sera élevée lorsque
a) le surplus du consommateur par acheteur
est peu important.
Questions à choix multiple b) le surplus du consommateur par acheteur
est important.
1 Quel organisme suivant ritst pas un organisme c) il y a un grand nombre d’acheteurs.
de réglementation fédéral ? d) le surplus du consommateur par entreprise
a) La Commission de contrôle de l’énergie est peu important.
atomique. e) le surplus du producteur par entreprise
b) Le Conseil de la radiodiffusion et est important.
des télécommunications canadiennes.
c) Petro-Canada. 8 L’offre d’ interventions augmente lorsque
d) L’Office national des transports. a) le surplus du consommateur par acheteur
e) Le Conseil national de commercialisation est plus faible.
des produits agricoles. b) le surplus du producteur par entreprise
est plus élevé.
2 La différence entre le montant maximal que c) le nombre de personnes touchées est
les consommateurs sont disposés à payer et le plus faible.
montant qu’ils paient réellement pour une d) Toutes ces réponses.
quantité donnée d’un bien est e) Aucune de ces réponses.
a) le surplus gouvernemental.
b) le surplus du consommateur. 9 Le nombre total de sociétés d’État au Canada
c) le surplus du producteur. est d’environ
d) le surplus total. a) 25.
e) la perte sèche.
b) 35.
c) 90.
3 Parmi les services suivants, lequel est le moins
d) 125.
susceptible de correspondre à un monopole naturel?
e) 1 000.
a) Le métro.
b) L’électricité.
10 La figure 20.2 présente les courbes de recette
c) L’eau et les égouts.
et de coût d’une industrie. Cette industrie
d) Le taxi.
deviendra un monopole naturel parce que
e) La câblodistribution.
a) une entreprise peut satisfaire intégralement
4 L’offre de réglementation économique provient la demande à un prix inférieur à celui que
a) des monopoleurs. pourraient fixer plusieurs entreprises.
b) des syndicats. b) il y a des rendements d’échelle décroissants
c) des associations commerciales. pour tout segment pertinent de la demande.
d) des électeurs. c) il y a des déséconomies d’échelle pour tout
e) des politiciens et des fonctionnaires. segment pertinent de la demande.
d) même une entreprise unique ne pourra
5 Le surplus total est maximisé lorsque réaliser de profit positif dans cette industrie.
a) le coût marginal est égal à la recette e) Toutes ces réponses.
marginale.
b) le coût marginal est égal au coût total moyen.
c) le prix est égal au coût marginal.
d) le prix est égal au coût total moyen.
e) le prix est égal au coût variable moyen.
LA POLITIQUE DE CONCURRENCE 269

FIGURE 20.2 14 Reportez-vous au monopole naturel de la figure


20.2. Si un organisme de réglementation suit
la règle de la tarification au coût marginal, quel
segment de ligne donne le montant de subvention
(par unité de production) requis pour permettre
au monopoleur de poursuivre ses activités ?
a) ba
b) ea
c) fc
d )gd
e) eb

I5 Reportez-vous au monopole naturel de la figure


20.2. Quelle partie du graphique représente la
perte sèche associée à la tarification au coût moyen ?
a) abc
b) cdg
c) cfg
I I Soit le monopole naturel de la figure 20.2. d) aeg

Si l’entreprise n’est pas soumise à une e) Aucune de ces réponses.

réglementation et fonctionne comme une


entreprise privée qui maximise ses profits,
16 Un monopoleur régi par la tarification en
fonction du taux de rendement est incité à
quel sera son volume de production ?
a) 0, parce que l’entreprise subit une perte a) gonfler ses coûts.
b) produire davantage que le volume
économique lorsque P = Cm.
de production efficient.
b) Qo
c) imposer un prix égal au coût marginal.
c) Qi
d) maximiser le surplus du consommateur.
<1)02
e) faire a et b.
e) Q3

I2 Reportez-vous au monopole naturel de la figure


I7 Parmi les actions suivantes, laquelle est conforme
à la théorie de l’intérêt public de T intervention?
20.2. Lorsqu’un organisme de réglementation
a) La réglementation d’un monopole naturel
fixe un prix suffisant pour que l’entreprise
en fixant le prix de manière à ce qu’il soit égal
réalise des profits normaux, quel sera le niveau
au coût marginal.
de production ?
b) La réglementation d’une industrie
a) 0, parce que l’entreprise subit une perte
concurrentielle afin d’augmenter la production.
économique lorsque P = Cm.
c) La réglementation de l’industrie du transport
b) Qo
aérien en établissant des tarifs aériens minimaux.
c) Q,
d) La réglementation de l’agriculture en
d) Qa
établissant des barrières à la sortie de l’industrie.
e) Q3
e) Aucune de ces réponses.

I3 Reportez-vous au monopole naturel de la figure


18 Parmi les actions suivantes, laquelle est conforme
20.2. Le surplus total est maximal lorsque la
à la théorie de la capture de l'intervention?
quantité est
a) La réglementation du monopole naturel
a) Qo et le prix, A
en fixant le prix de manière à ce qu’il soit égal
b) Q. et le prix, A
au coût marginal.
c) Qi et le prix, p4.
b) La réglementation de l’industrie
d) Qz et le prix, A
concurrentielle afin d’augmenter la production.
e) 03 et le prix, A
c) La réglementation de l’industrie du transport
aérien en établissant des tarifs aériens minimaux.
d) La réglementation de l’agriculture
en établissant des barrières à la sortie
de l’industrie.
e) Aucune de ces réponses.
270 CHAPITRE 20

19 En situation d’équilibre politique 25 Un monopole naturel aura probablement


a) l’allocation efficiente des ressources a) un faible coût fixe et un faible coût marginal.
doit être réalisée. b) un faible coût fixe et un coût marginal élevé.
b) personne ne souhaite modifier sa proposition. c) un coût fixe élevé et un faible coût marginal.
c) les entreprises réaliseront un profit d) un coût fixe élevé et un coût marginal élevé.
économique nul. e) un coût fixe élevé et un coût marginal
d) toutes les parties conviendront que l’on a croissant.
obtenu le niveau de réglementation adéquat.
e) Toutes ces réponses. Problèmes à court développement

20 La législation antimonopole vise à 1 II est indispensable de réglementer les


a) soutenir les prix. monopoles à cause des tensions entre l’intérêt
b) établir des sociétés d’État. public et l’intérêt des producteurs. Expliquez.
c) empêcher les pratiques monopolistiques.
d) établir des lois commerciales équitables. 2 Dans le cas d’un monopole naturel réglementé,

e) réglementer les monopoles. quand une tarification au coût moyen serait-elle


meilleure qu’une tarification au coût marginal ?

21 Une société d’Etat qui maximise son budget


3 Pourquoi la tarification en fonction du taux
en appliquant la règle de la tarification au coût
de rendement est-elle équivalente à la tarification
marginal
au coût moyen ?
a) produira le volume de production efficace.
b) produira au-delà du volume de production 4 Expliquez le problème que pose le récent
efficace. processus de déréglementation pour la théorie
c) maximisera le surplus du consommateur. de la capture de l’intervention.
d) maximisera le surplus du producteur.
e) fera a et d. 5 Supposez que le gouvernement tente d’éliminer
le profit de monopole en taxant chaque unité
22 Selon la théorie de l’intérêt public de de production d’un monopole.
l’intervention, la réglementation gouvernementale a) Quel effet ce genre de politique aura-t-il
a) sert les intérêts des fonctionnaires. sur le volume de production d’un monopole
b) vise à maximiser le surplus total. et sur le prix que celui-ci impose ?
c) vise à maximiser le surplus du producteur. b) Quel sera l’effet sur l’efficience économique ?
d) vise à réduire au minimum le surplus
du producteur. @0 6 La demande d’Aérodisques, un disque fait d’une
e) vise à maximiser le surplus du consommateur. matière unique qui lui permet de voler sur une
grande distance, nous est donnée par l’équation
23 On réalise une allocation efficiente des suivante
ressources lorsque P= 10-0,01 Qd.
a) le surplus du consommateur est maximisé.
b) le surplus du producteur est réduit L’équation de la recette marginale
au minimum. correspondante (Rm) est
c) le surplus total est maximisé. Rm = 10 - 0,02Q.
d) le surplus total est réduit au minimum.
e) Aucune de ces réponses. L’entreprise Aérodisques est un monopole
naturel. Le coût fixe total de l’entreprise est
24 La réglementation concerne de 700 $ et le coût marginal est constant à 2 $
a) la discipline du marché. le disque. [Remarque: Cela signifie que le coût
b) les règles administrées par un organisme variable moyen est constant à 2 $ par disque.]
gouvernemental. Supposez que l’entreprise Aérodisques ne soit
c) la vente d’une société d’état à des actionnaires pas réglementée.
privés. a) Quelle sera la quantité vendue et le prix
d) la cartélisation d’une industrie d’un Aérodisque?
concurrentielle. b) À combien s’élève le profit ou la perte
e) la formation de monopoles. économique ?
c) À combien s’élève le surplus du producteur?
d) À combien s’élève le surplus du consommateur ?
e) À combien s’élève le surplus total ?
LA POLITIQUE DE CONCURRENCE 271

7 Supposez maintenant que l’entreprise Aérodisques FIGURE 20.3


devienne réglementée et que l’organisme de
réglementation applique une tarification au coût
marginal.
a) Quelle sera la quantité vendue et le prix
d’un Aérodisque ?
b) À combien s’élève le profit ou la perte
économique ?
c) À combien s’élève le surplus du producteur ?
d) À combien s’élève le surplus du
consommateur ?
e) À combien s’élève le surplus total ?

8 Supposez que l’organisme de réglementation de


l’entreprise Aérodisques applique une tarification
au coût moyen.
a) Quel sera le prix d’un Aérodisque et combien
seront vendus ?
b) À combien s’élève le profit ou la perte
économique ? RÉPONSES
c) À combien s’élève le surplus du producteur?
d) À combien s’élève le surplus
du consommateur?
Vrai/Faux/1 ncertain
e) À combien s’élève le surplus total ?
(Justifiez votre réponse.)

9 Puisque l’entreprise Aérodisques va subir une 1 V Voir le texte du manuel.


perte associée à la tarification au coût marginal, 2 F Vrai pour la concurrence.
le gouvernement doit subventionner 3 V Les organismes de réglementation agissent
la production d’Aérodisques pour inciter dans l’intérêt du monopole.
l’entreprise à produire. 4 F Correspond à la théorie de l’intérêt public.
a) Quel est le montant de la subvention, en 5 I Vrai selon la théorie de l’intérêt public, faux
appliquant une tarification au coût marginal, selon la théorie de la capture.
qui permettra à l’entreprise de réaliser un 6 F L’industrie s’approprie une plus grande part
profit économique de zéro ? du surplus total.
b) Quel est le montant de la perte sèche associée 7 F Ressemble davantage à la tarification au coût
à une tarification au coût moyen ? moyen.
c) Pour pouvoir verser la subvention nécessaire, 8 V î coûts —> T profits.
en appliquant une tarification au coût 9 F Favorisait les producteurs.
marginal, le gouvernement doit augmenter 10 V Voir le texte du manuel.
les recettes fiscales d’un montant égal à celui
de la subvention. Si la perte sèche associée
Questions à choix multiple
à la taxe est de 100 $, quelle est la meilleure
tarification ?
1 c Société d’État.
2 b Par définition.
10 La figure 20.3 illustre les courbes de demande,
3 d Pas d’importantes économies d’échelle.
de recette marginale {Rm) et de coût marginal
4 e Créent la législation.
{Cm) d’une industrie oligopolistique réglementée.
5 c Règle de tarification permettant de réaliser
a) Selon la théorie de l’intérêt public
l’allocation efficiente des ressources.
de l’intervention, quels seront les prix
6 a Règle de tarification permettant au monopole
et la quantité? Pourquoi?
de maximiser ses profits.
b) Selon la théorie de la capture de
7 e b, c —»î demande par les acheteurs.
l’intervention, quels seront les prix
8 b L’inverse de a et c est vrai.
et la quantité? Pourquoi?
9 d Voir le tableau 20.2 du manuel.
c) Expliquer pourquoi les entreprises de
10 a Définition du monopole naturel.
cette industrie pourraient demander une
I I c Lorsque Cm = Rm.
réglementation ?
12 d Lorsque P = CTM.
272 CHAPITRE 20

13 e Lorsque P = Cm. 4 Selon la théorie de la capture, les producteurs


14 d Pour couvrir la perte par unité de production. détourneront la réglementation à leur avantage
15 c Voir la figure 20.3 du manuel. de manière à maximiser le surplus du producteur.
16 a Pour augmenter les profits. Toutefois, si le producteur peut obtenir une
17 a Seule option —»?efficience. réglementation en faisant du lobbying, pourquoi
18 c Aide les compagnies aériennes ne peut-il pas faire cesser la déréglementation ?
et non les consommateurs. On peut aussi se demander pourquoi de nombreux
19 b Voir le texte du manuel. producteurs favorisent la déréglementation. La
20 c Voir le texte du manuel. théorie de la capture répond mal à ces questions.
& 21 e Voir la figure 20.7 (b) du manuel.
22 b Effort pour réaliser l’allocation efficiente 5 a) En imposant une taxe sur chaque unité vendue
des ressources. par un monopole, on augmentera le coût
23 c Voir le texte du manuel. marginal, de sorte que le monopole, qui
24 b Voir le texte du manuel. cherche à maximiser ses profits, augmentera
25 c Produit toujours une courbe CTM le prix et réduira la production,
à pente négative. b) La taxe réduira certainement les profits du
monopole, et pourrait même les éliminer, mais
Problèmes à court développement cela ne ferait qu’augmenter finefficience causée
par le monopole. C’est ce qu’illustre la figure
1 La réalisation de l’allocation efficiente des 20.4. La courbe Cm est la courbe de coût
ressources est dans l’intérêt public; autrement marginal avant imposition de la taxe. Un
dit, il faut élever le volume de production au monopole non réglementé produira la quantité
niveau où il permet de maximiser le surplus Q2 alors que la production économiquement
total. En revanche, il est dans l’intérêt du efficiente est Q3. Cependant, la taxe incite
producteur en situation de monopole de limiter le monopoleur à réduire sa production de
la production pour maximiser le surplus du à Qj, ce qui réduit le gain d’efficience dans
producteur et, donc, le profit de monopole. l’allocation des ressources.
Comme ces intérêts sont contradictoires,
il faut, pour réaliser une allocation des ressources FIGURE 20.4
efficiente, que le monopole soit réglementé.
Selon la théorie de l’intérêt public, il s’agit
du principe de base de la réglementation
des monopoles.

2 La tarification au coût moyen créera une perte


sèche, tout comme la tarification au coût marginal,
en raison de la nécessité d’imposer une taxe.
Puisque le monopole naturel a un coût
marginal inférieur au coût total moyen,
le gouvernement devra, pour appliquer une
tarification au coût marginal, octroyer une
subvention à l’entreprise afin d’inciter celle-ci
à produire.
Pour pouvoir offrir cette subvention, le
gouvernement prélèvera une taxe qui engendrera
une perte sèche. Si cette perte sèche, associée
à la taxe (par exemple, la perte sèche associée à ©Ô 6 La figure 20.5 sera utile pour répondre
la tarification au coût marginal et la subvention aux questions sur le marché des Aérodisques.
concomitante), est supérieure à la perte sèche Elle présente les courbes de recette et de coût
associée à la tarification au coût moyen, la pertinentes pour l’entreprise Aérodisques.
tarification au coût moyen est plus indiquée.

3 II s’agit ici de ne pas oublier que le coût


économique comprend un rendement normal.
Par conséquent, comme la réglementation en
fonction du rendement fixe un prix qui permet à
l’entreprise d’obtenir un taux de rendement
normal, le prix fixé sera égal au coût total moyen.
LA POLITIQUE DE CONCURRENCE 273

c) Le surplus du producteur est la différence


FIGURE 20.5
entre la recette du producteur et le coût
d’opportunité de la production. La recette
totale est de 2 400 $ (6 $ X 400 unités)
et le coût d’opportunité total est de 800 $
(2 $ X 400 unités). Le surplus du producteur
est de 1 600 $. Graphiquement, le surplus
du producteur est illustré par le rectangle
abde de la figure 20.5.
d) Graphiquement, le surplus du consommateur
est illustré par le triangle bcd de la figure
20.5, qui correspond à 800 $.
e) Le surplus total est de 2 400 $, soit la somme
du surplus du producteur et du surplus
du consommateur.

7 a) Avec la tarification au coût marginal, le prix


d’un Aérodisque sera égal au coût marginal,
soit 2 $. Pour calculer la quantité vendue,
a) Dans un marché non réglementé, l’entreprise
utilisez le prix dans l’équation de demande :
Aérodisques choisira la production qui lui
permet de maximiser ses profits, lorsque P= 10-0, OlQo
Rm = Cm. Pour calculer cette production, 2 10 0,01
= - Qd
supposez que Rm = Cm = 2 et trouvez 0,01Qo = 8
la solution pour Q: Qn = 800

Rm = Cm b) Pour déterminer le montant du profit ou


10 - 0,02Q = 2 de la perte économique, nous devons d’abord
8 = 0,02Q déterminer le CTM avec une production
400 = Q de 800 unités. En utilisant la même méthode
que pour le problème précédent, nous trouvons
Pour calculer le prix, utilisez Q = 400 que lorsque Q = 800, le CTM est de 2,875 $,
dans l’équation de demande : ce qui est supérieur au prix d’un montant égal
P= 10-0,01Qd à 0,875 $ (87,5 <f). Par conséquent, l’entreprise
= 10-0,01 (400) Aérodisques subira une perte de 700 $
= 10-4 (0,875 $ X 800). Ou bien, puisque le Cm est
= 6 constant, si le prix est fixé de manière à être
égal au Cm, lui-même égal au CVM, la perte
400 Aérodisques seront produits et vendus totale ne sera que le CFT, soit 700 $.
au prix de 6 $ chacun. c) Le surplus du producteur est nul.
b) Pour déterminer le profit économique, d) Le surplus du consommateur est illustré
il faut d’abord déterminer le coût total par le triangle ach de la figure 20.5, qui est
moyen (CTM) lorsque la production (Q) de 3 200 $.
est 400 unités. e) Le surplus total est de 3 200 $ (un maximum).

CTM= CFM + CVM 8 a) Le calcul du CTM pour divers volumes de


= (CFTIQ) + CVM production nous permet de déterminer que
= (700/400) + 2 la courbe CTM croise la courbe de demande
= 3,75 lorsque Q = 700 et CTM = 3 $. Par
conséquent, avec une tarification au coût
(P - CTM) X Q = Profit moyen X Q moyen, le prix d’un Aérodisque sera de 3 $
= Profit économique
et 700 unités seront vendues.
(6,00 $ - 3,75 $) X Q = 2,25 $/unité X 400 unités vendues b) Puisque le prix est égal au coût total moyen,
= 900 $/an le profit économique est nul.
Par conséquent, le profit économique est c) Le surplus du producteur est de 700 $,
la différence entre le prix (recette moyenne) la surface aijk de la figure 20.5.
et le CTM fois la quantité vendue. Il est égal d) Le surplus du consommateur est de 2 450 $,
à 900 $ et est représenté à la figure 20.5 par le triangle ijc de la figure 20.5.

la surface fbdg. e) Le surplus total est de 3 150 $.


274 CHAPITRE 20

9 a) La subvention totale est égale à la perte b) Selon la théorie de la capture, l’organisme de


associée à la tarification au coût marginal. réglementation choisira la quantité et le prix
Dans le problème à court développement qui lui permettent de maximiser le profit de
7b, elle s’élève à 700 $. l’industrie. Il s’agit de la quantité que
b) La perte sèche associée à la tarification choisirait le monopoleur qui cherche à
au coût moyen est la perte du surplus total maximiser ses profits, soit 300 unités, lorsque
par rapport à la tarification au coût marginal. Cm = Rm. On peut obtenir le prix le plus
Nous avons calculé le surplus total dans élevé qui pourrait être fixé et qui pourrait
le cadre de la tarification au coût marginal toujours permettre de vendre cette quantité à
(3 200 $) au problème à court développement partir de la courbe de demande : il est de 6 $
7e et le surplus total dans le cadre de l’unité.
la tarification au coût moyen (3 150 $) au c) Les entreprises de l’industrie demanderaient
problème à court développement 8e. La perte une réglementation si celle-ci avait pour effet
sèche équivaut à la différence, soit à 50 $. d’augmenter les profits de l’industrie. Comme
c) La tarification au coût moyen est supérieure, nous l’avons vu au chapitre 13, les cartels
car elle est associée à la plus petite perte sont instables parce qu’il existe toujours une
sèche. incitation à ne pas respecter les ententes de
limite de la production et que celles-ci sont
10 a) Selon la théorie de l’intérêt public, les très difficiles à faire respecter. Si, toutefois,
organismes de réglementation fixeront le prix les entreprises d’une industrie peuvent
et la quantité qui permettent de maximiser le inciter le gouvernement, par le biais de la
surplus total. Autrement dit, ils choisiront la réglementation, à faire respecter une entente
quantité et le prix auxquels le Cm est égal à de cartel, elles le feront.
la demande. Cela correspond à la quantité de
500 unités et au prix de 4 $ l’unité.
Les effets externes,
l’environnement
et le savoir

normal) et de T des connaissances sur les problèmes


CONCEPTS CLÉS environnementaux; ceux-ci comprennent la
pollution atmosphérique, la pollution de l’eau
et la pollution des sols.
Les effets externes
Il y a des effets externes en Xabsence de droits
Les effets externes sont les coûts et les avantages de propriété.
d’une activité de production ou de consommation
♦ Les droits de propriété sont des ententes sociales
que subissent les personnes autres que celles qui
qui régissent la propriété, futilisation et la vente
ont décidé de s’engager dans cette activité. Les
de facteurs de production et de biens et services.
effets externes peuvent être négatifs (les effets de
Au Canada, un droit de propriété est un titre légal
la pollution) ou positifs (les effets de la scolarisation).
qu’on peut faire respecter en s’adressant aux
♦ Les coûts externes (effets externes négatifs) sont tribunaux.
les coûts de production d’un bien ou d’un service
♦ Sans droits de propriété, par exemple, sur une
que subissent des personnes autres que
rivière, le coût marginal (Cm) de la pollution de
les consommateurs de ce bien ou service.
cette rivière est de zéro. Les entreprises pollueront
♦ Les avantages externes (effets externes positifs) aussi longtemps que l’avantage marginal {Am)
sont les avantages que retirent de la consommation qu elles retirent de la pollution de la rivière est
d’un bien ou d’un service les personnes autres supérieur à zéro. Il est souvent possible de corriger
que les consommateurs de ce bien ou service. les effets externes négatifs, comme la pollution,
en établissant des droits de propriété.
Les effets externes engendrent des lacunes de marché
(allocation des ressources non efficiente). Les marchés ♦ Le théorème de Coase - lorsqu’il existe des droits
produisent trop de biens et de services accompagnés de propriété et que les coûts de transaction sont
de coûts externes et trop peu de biens ou services bas, il n’y a pas d’effets externes et les transactions
accompagnés d’avantages externes. privées sont efficientes. La manière dont les droits
de propriété sont attribués n a pas d’effet sur les
résultats, mais elle en a sur la répartition des coûts
L’économique de l’environnement
et des avantages.

Beaucoup de personnes croient que tous les dommages Lorsqu’il n’est pas possible d’établir de droits de
causés à l’environnement doivent cesser. Toutefois, propriété ou que les coûts de transaction sont élevés,
l’analyse économique de l’environnement étudie les responsables gouvernementaux de la réglementation
les coûts et les avantages des actions afin de découvrir peuvent adopter d’autres politiques pour obtenir
les niveaux efficients de pollution ou de dommages l’efficience, même en présence d’effets externes.
environnementaux.
♦ Pour atteindre l’efficience, les mesures politiques
♦ La demande d’un environnement plus propre doivent viser le volume de production auquel
T au fil du temps en raison de î des revenus le coût marginal social égale l’avantage marginal
(un environnement de grande qualité est un bien social.
276 CHAPITRE 2 1

• Le coût marginal social (CmS) est égal ♦ Les pays industrialisés et les pays en développement
au coût marginal privé directement subi se trouvent face au dilemme du prisonnier
par le producteur plus le coût marginal imposé où personne ne réduit ses émissions de dioxyde
aux autres sous forme d’effets externes. de carbone parce qu’il est dans l’intérêt de chaque
• L’avantage marginal social {AmS) est égal pays de laisser les autres assumer les coûts des
à l’avantage marginal privé directement reçu mesures environnementales.
par le consommateur plus l’avantage marginal
reçu par d’autres consommateurs sous forme
L’économique du savoir
d’effets externes.

♦ Les redevances sur les émissions sont l’imposition Le savoir crée des avantages externes importants
d’un prix à payer par unité de pollution tels que l’avantage marginal social {AmS) est supérieur
de manière que CmS = AmS. Dans la pratique, à l’avantage marginal privé {AmP). Les résultats
du marché dus au choix de quantités auxquelles
• il est difficile de calculer l’avantage marginal
AmP = CmS sont inférieurs à ceux de la quantité
de la pollution;
efficiente de savoir, ce qui entraîne une lacune
• les pollueurs sont incités à tromper les
du marché.
responsables de la réglementation quant
à l’importance de cet avantage. Pour atteindre l’efficience, les politiques doivent
produire une quantité de savoir telle que AmS = CmS.
♦ Les permis négociables sont des limites quantitatives
qui permettent de contourner la nécessité pour ♦ Les subventions sont des montants que les
l’organisme de réglementation de déterminer administrations publiques versent aux producteurs
l’avantage marginal de la pollution. Chaque de services ou de biens axés sur les connaissances.
entreprise reçoit un permis l’autorisant à émettre
• Pour encourager la production des biens ou des
une certaine quantité de pollution ; ces permis
services accompagnés d’effets externes positifs,
peuvent être achetés et vendus.
on accorde aux producteurs une subvention
• Les entreprises qui réduisent leur pollution égale à l’avantage marginal externe.
en deçà de leur limite permise peuvent vendre • La courbe AmS se confond avec la courbe Am
la réduction «excédentaire» à d’autres entreprises pertinente de la décision du producteur sur
qui peuvent alors polluer davantage. l’acquisition de connaissances —> production
• Dans un marché concurrentiel des permis, à laquelle AmS = CmS.
le prix des permis produira un résultat efficient
♦ La prestation de services à un coût inférieur au coût
tel que CmS = AmS.
de production - les gouvernements offrent les biens
♦ Une taxe égale au coût marginal externe peut être et les services (éducation publique) à un prix
imposée afin de réduire les coûts externes. inférieur (souvent zéro) au coût de production.

• La courbe CmS est la courbe Cm pertinente ♦ Les brevets et les droits d’auteur confèrent aux
de la décision du producteur pollueur innovateurs un droit de propriété intellectuelle
—> production à laquelle CmS = AmS. sur leurs découvertes pour leur permettre d’en
retirer des bénéfices.
Une taxe sur le carburant permettrait de limiter
les émissions de dioxyde de carbone, voire de ralentir • Le savoir ne semblant pas entraîner de
le réchauffement de la planète. rendements marginaux décroissants, il faut
donc offrir des incitatifs pour encourager
♦ On ne peut imposer ce type de taxe, car le rôle
les innovations.
que jouent les émissions de dioxyde de carbone
• Les brevets et les droits d’auteur stimulent
dans le réchauffement de la planète ne fait pas
l’émergence de nouvelles connaissances,
l’unanimité chez les scientifiques, et les économistes
mais ils créent un monopole temporaire,
ne savent pas très bien comment une augmentation
ce qui oblige à comparer l’avantage découlant
de la température se traduit en coûts ; les coûts
de l’augmentation des connaissances avec
seraient subis au moment présent alors que
la perte de monopole.
les avantages incertains ne seraient acquis que
plus tard ; de plus, il y aurait des problèmes sur
le plan de la coopération internationale.
LES EFFETS EXTERNES, L'ENVIRONNEMENT ET LE SAVOIR 277

La figure 21.1 (tirée du manuel) montre que


RAPPELS l’imposition de la production d’un bien ou d’un
service peut entraîner une quantité d’allocation
1 Un marché concurrentiel atteindra le volume efficiente. Le service des transports qui produit
de production auquel le coût marginal privé des coûts externes de pollution et d’encombrement
est égal à l’avantage marginal privé. La quantité en est un exemple.
efficiente est la quantité à laquelle le coût
marginal social est égal à l’avantage marginal
FIGURE 21.1
social.
La différence entre le coût marginal social
et le coût marginal privé est un coût externe,
et la différence entre Favantage marginal social et
l’avantage marginal privé est un avantage externe.
Dans le cas de la plupart des activités
économiques, les gens qui ne participent pas
à l’activité originale ne sont pas touchés, et
il n’y a donc pas de coûts ni d’avantages
externes. Autrement dit, les coûts et avantages
sociaux privés coïncident et les marchés
concurrentiels sont efficients. Toutefois, lorsque
des tiers sont touchés, il y a des coûts ou des
avantages externes et les marchés concurrentiels
ne sont pas efficients. Les marchés produisent
trop de biens et services accompagnés de coûts
externes et pas assez de biens et services
accompagnés d’avantages externes. La courbe de demande de services de
transport se confond avec la courbe d’avantage
2 Les marchés concurrentiels produisant des biens
marginal. Toutefois, le coût marginal privé
et services accompagnés d’effets externes ne
(CmP) et le coût marginal social (CmS) ne sont
sont pas efficients parce que certains de ces effets
pas les mêmes. La courbe CmP reflète les coûts
sont des coûts ou avantages externes. S’il était
qui reviennent au producteur des services de
possible d ’internaliser ces coûts ou avantages,
transport. Il existe toutefois d’autres coûts qui
alors les marchés pourraient être efficients.
se manifestent sous la forme de pollution et
Dans ce chapitre, nous étudierons deux méthodes
d’encombrement imposés aux autres. La courbe
principales qui permettent d’internaliser ces effets
CmS reflète à la fois le coût marginal supporté
externes.
par le producteur et ces coûts externes.
La première consiste à définir clairement
La quantité d’allocation efficiente des services
les droits de propriété et à les appliquer
de transport est atteinte au point Qj,
rigoureusement. Il sera alors possible
soit l’intersection de la courbe D = Am
de récupérer les coûts imposés à ceux qui
et de la courbe CmS. Cependant, un marché
ne participent pas à l’activité initiale par le biais
concurrentiel atteindra la quantité à laquelle
du processus juridique et de payer ceux qui y
les courbes D et CmP se croisent puisque
participent : les coûts deviendront des coûts
les producteurs ne prennent en compte que
internes (privés).
les coûts privés ; le prix du marché sera P0 et
La seconde méthode d’internalisation des
la quantité, Qq, qui est supérieure à la quantité
effets externes consiste à imposer les activités
d’allocation efficiente.
qui génèrent des coûts externes et à subventionner
Toutefois, lorsqu’on impose une taxe au
celles qui génèrent des avantages externes. En
montant égal aux coûts externes, la courbe CmS
imposant une taxe égale au coût externe, on
devient la nouvelle courbe de coût marginal
transforme le coût total en coût interne. De
des producteurs. Par conséquent, le prix du
même, en accordant une subvention d un
marché s’élèvera à P,, la quantité tombera à Qj
montant égal aux avantages externes, on
et l’allocation des ressources sera efficiente.
transforme l’avantage intégral en avantage interne.
Remarquez que le volume de production efficient
3 Si la production d’un bien ou d’un service n’est pas nul, ce qui signifie qu’il y aura un
engendre des coûts externes, un marche certain niveau de pollution et d’encombrement.
concurrentiel atteindra la quantité qui est Le niveau de pollution/encombrement efficient
supérieure au niveau d’allocation efficient. (optimal) n’est pas de zéro.
278 CHAPITRE 2 1

Généralement, les coûts économisés 8 Les brevets encouragent l’invention


en ramenant la pollution au point zéro sont et l’innovation.
inférieurs aux avantages perdus (avantages
provenant des biens de consommation dont
la production a créé la pollution comme
sous-produit). 9 Les permis de polluer négociables exigent
des responsables de la réglementation qu’ils
connaissent les barèmes des avantages marginaux
des entreprises.
AUTOÉVALUATION

Vrai/Faux/Incertain 10 Le résultat du dilemme du réchauffement


(Justifiez votre réponse.) de la planète est le suivant : les pays industrialisés
limitent le niveau de pollution tandis que les
I En présence d’effets externes négatifs, pays en développement polluent.
le coût marginal social et le coût externe
sont équivalents.

Questions à choix multiple


2 Si la production d’un bien n’entraîne pas de
coût externe, le coût marginal social est alors 1 Un effet externe est un coût ou un avantage
égal au coût marginal privé. découlant d’une activité économique qui touche
a) les consommateurs, mais pas les producteurs.
b) les producteurs, mais par les consommateurs.
c) les resquilleurs.
3 Les effets externes sont souvent dus à l’absence d) les rivaux.
de droits de propriété privés. e) Aucune de ces réponses.

2 La figure 21.2 présente la demande du bien A


ainsi que le coût marginal privé (CmP) associé
4 L’existence d’avantages externes signifie que à la production de ce bien. La production
le coût marginal social est inférieur au coût de la sixième unité de production génère
marginal privé. a) un coût externe de 1,50 $.
b) un coût externe de 3 $.
c) un coût externe de 6 $.
d) un avantage externe de 3 $.
5 En présence de coûts externes, le marché privé e) un avantage externe de 6 $.
aura tendance à avoir une production supérieure
au niveau de production efficient. FIGURE 21.2

6 L’attribution de droits de propriété résout


le problème des effets externes négatifs.

7 Le niveau de pollution efficient est toujours


égal à zéro.
LES EFFETS EXTERNES, L’ENVIRONNEMENT ET LE SAVOIR 279

3 Dans la figure 21.2, combien d’unités du bien A 8 La figure 21.3 présente la courbe de demande
seront produites dans un marché déréglementé ? du bien B, la courbe d’avantage marginal social
a) 0 unité (AmS) et la courbe de coût marginal social et
b) 5 unités privé (CmP = CmS). Combien d’unités du bien
c) 6 unités B seront-elles produites et consommées dans
d) 8 unités un marché non réglementé ?
e) Impossible à calculer sans information a) 0 unité
supplémentaire. b) 3 unités
c) 5 unités
4 Dans la figure 21.2, quelle est la quantité d) 6 unités
efficiente du bien A ? e) 9 unités
a) 0 unité
b) 5 unités FIGURE 21.3
c) 6 unités
d) 8 unités
e) Impossible à calculer sans information
supplémentaire.

5 Au niveau actuel de production de boulamites,


l’avantage marginal social est inférieur au coût
marginal social. Pour réaliser une allocation
efficiente,
a) il faut imposer une taxe sur les boulamites.
b) il ne faut pas produire de boulamites.
c) il faut augmenter la production de
boulamites.
d) il faut réduire la production de boulamites.
e) il faut établir des droits de propriété sur les
boulamites.
9 Dans la figure 21.3, quelle est la quantité
qui permet de réaliser une allocation efficiente
6 Lequel des concepts suivants illustre le concept
du bien 5?
de coûts externes ?
a) 0 unité
a) Le mauvais temps réduit la taille des récoltes
b) 3 unités
de blé.
c) 5 unités
b) Une réduction de la taille des récoltes de blé
d) 6 unités
fait baisser la recette des cultivateurs de blé.
e) 9 unités
c) Le tabagisme nuit à la santé du cultivateur.
d) Le tabagisme nuit à la santé des non-fumeurs
10 Examinez la figure 21.3 et indiquez laquelle
à proximité.
des mesures suivantes pourrait inciter le marché
e) Les services de santé publics réduisent
à réaliser une allocation efficiente ?
la propagation de la maladie.
a) Imposer une taxe sur la production
de A de 3 $ l’unité.
7 La production d’un trop grand nombre de
b) Imposer une taxe sur la production
produits associés à des effets externes négatifs
de A de 4 $ l’unité.
est un exemple de
c) Accorder une subvention de 1 $ pour chaque
a) redistribution.
consommation d’une unité de B.
b) souveraineté du consommateur.
d) Accorder une subvention de 3 $ pour chaque
c) souveraineté du producteur.
consommation d’une unité de B.
d) lacune du secteur public.
e) Accorder une subvention de 4 $ pour chaque
e) lacune du marché.
consommation d’une unité de B.
280 CHAPITRE 2 1

I I En présence d’une lacune du marché, I6 Les mesures qui permettent de réaliser une
le gouvernement prendra des mesures pour allocation efficiente en présence d’effets externes
réduire le niveau de non-efficience. Il s’agit comprennent
d’une prédiction de a) des droits de propriété intellectuelle.
a) la théorie des résultats du comportement b) des subventions.
du gouvernement. c) une fourniture de services à un coût inférieur
b) la théorie des processus du comportement au coût de production.
du gouvernement. d) Toutes ces réponses.
c) la théorie de l intérêt public e) Aucune de ces réponses.
du comportement du gouvernement.
d) la théorie des choix publics du comportement I7 Un producteur de piles d’acide sulfurique pollue
du gouvernement. la rivière en amont du Club de natation des
e) la théorie de recherche de rentes nudistes. Si les coûts de transaction sont faibles,
du comportement du gouvernement. le niveau de pollution sera efficient
I2 Un effet externe est a) seulement si Ronald Coase est membre
a) la différence entre le prix et le coût du club de natation.
marginal privé. b) seulement s’il y a une répartition égale
b) la différence entre le prix et le coût des coûts et des avantages.
marginal social. c) seulement si l’on attribue des droits
c) l’effet de la réglementation gouvernementale de propriété de l’eau au producteur.
sur le prix du marché et la production. d) seulement si l’on attribue des droits
d) une personne qui consomme un bien de propriété de l’eau au Club de natation.
sans le payer. e) si l’on attribue des droits de propriété de
e) un coût ou un avantage découlant d’une l’eau au producteur ou au Club de natation.
décision mais ne touchant pas la personne
qui a pris cette décision. 18 Le savoir, en tant que facteur de production,
a) fait face à une productivité marginale
13 La courbe de coût marginal privé (CmP) a
décroissante.
une pente positive à partir de l’origine. Lorsque
b) crée des coûts externes.
les coûts externes par unité de production sont
c) a des coûts égaux à ceux qui sont payés
constants, la courbe de coût marginal social
a alors une pente positive au détenteur de permis.

a) et elle est parallèle à la CmP et située d) est encouragé par les droits de propriété

au-dessus. intellectuelle.

b) et elle est parallèle à la CmP et située e) Toutes ces réponses.

au-dessous.
c) à partir de l’origine et elle est située au-dessus 19 L’élasticité du revenu par rapport à la demande
de la CmP. d’un meilleur environnement est

d) à partir de l’origine et elle est située a) négative.

au-dessous de la CmP b) zéro.


e) identique à la CmP. c) positive.
d) variable selon les tendances.
14 Une économie de marché a tendance à_
e) impossible à déterminer sans information
de(s) biens associés à des effets externes négatifs
supplémentaire.
et à-de(s) biens associés à des effets
externes positifs.
20 La production de trop peu de biens associés
a) trop produire; trop produire
à des effets externes positifs est un exemple de
b) trop produire ; ne pas assez produire
a) lacune du marché.
c) ne pas assez produire ; trop produire
b) lacune du secteur public
d) ne pas assez produire; ne pas assez produire
c) souveraineté du producteur.
e) produire; consommer
d) souveraineté du consommateur.
I5 Les mesures visant à pallier les effets externes
e) coûts externes.
négatifs comprennent tous les éléments suivants sauf
a) des redevances sur les émissions.
b) -des brevets.
c) des limites quantitatives.
d) des taxes.
e) des permis négociables.
LES EFFETS EXTERNES, L'ENVIRONNEMENT ET LE SAVOIR 281

21 Les pollueurs sont mieux informés sur 25 Par le passé, les taux de chlorofluorocarbures
les avantages marginaux de la pollution (CFC) dans les réfrigérateurs et les aérosols
et ils sont incités à tromper les responsables a) n’ont pas été acceptables sur le plan social
de la réglementation au sujet de l’importance en raison des coûts externes.
de ces avantages. Une redevance sur les émissions b) n’ont pas été acceptables sur le plan social
imposée par ces responsables sera probablement en raison des avantages externes.
a) trop élevée, ce qui aggravera la pollution. c) ont été plus qu’acceptables sur le plan social
b) trop élevée, ce qui entraînera un nettoyage en raison des coûts externes.
colossal. d) ont été plus qu’acceptables sur le plan social
c) trop faible, ce qui aggravera la pollution. en raison des avantages externes.
d) trop faible, ce qui entraînera un nettoyage e) ont fait baisser la température moyenne
colossal. de la planète.
e) trop belle pour être vraie.

Problèmes à court développement


22 Reportez-vous au tableau 21.1. Si le marché
des engrais est parfaitement concurrentiel et non
réglementé, la production est égale à 1 Les gouvernements fournissent gratuitement
a) 1. les services d’éducation ou du moins à un
b) 2. prix (droits de scolarité) très inférieur au
c) 3. coût. Sur quel argument économique repose
d) 4. cette politique ?
e) 5.
2 Expliquez comment une taxe peut permettre de
TABLEAU 21.1 MARCHÉ DES ENGRAIS réaliser l’efficience en présence de coûts externes.
CHIMIQUES
3 La production d’acier est polluante et entraîne
Avantage Avantage Coût Coût donc des coûts externes. Supposons que le
marginal marginal marginal marginal gouvernement tente de résoudre ce problème
Production privé (en social (en privé (en social (en en imposant une taxe aux producteurs d’acier
(en tonnes) dollars) dollars) dollars) dollars) (qui sont aussi des pollueurs). Au niveau de
production après taxe, nous voyons que la courbe
1 140 140 50 80
de coût marginal social se situe au-dessous de
2 120 120 60 90
la courbe de demande. Le volume de production
3 100 100 70 100
après taxe est-il efficient ? Si ce n’est pas le cas,
4 80 80 80 1 10
devrait-on augmenter ou réduire la production
5 60 60 90 120
d’acier, et donc la production de pollution ?

4 Votre colocataire, une environnementaliste


23 Reportez-vous au tableau 21.1. Les engrais ont
convaincue, est scandalisée par le concept
a) un coût marginal externe de 100 $.
économique de niveau de pollution «efficient».
b) un avantage marginal externe de 100 $.
À son avis, puisque tout le monde convient
c) un coût marginal externe de 30 $.
que la pollution est « mauvaise », la société doit
d) un avantage marginal externe de 30 $.
s’employer à l’éliminer. Comment, vous qui
e) un coût marginal externe de 0 $.
étudiez en économie, lui ferez-vous comprendre
qu’il n’est pas dans les intérêts fondamentaux
24 Reportez-vous au tableau 21.1. La production
de la société d’éliminer toute la pollution ?
d’engrais efficiente est égale à
a) 1. 5 Quels sont les avantages et les coûts des droits
b) 2. de propriété intellectuelle sur les innovations ?
c) 3.
d) 4. 6 Deux pionniers des prairies, Jérémie et Hortense,
e) 5. ont des champs adjacents. Puisqu’ils ont de très
bons rapports et qu’ils règlent toujours leurs
problèmes, ils ne se sont pas cassé la tête à
installer une barrière. Mais un beau jour, Jérémie
achète un nouveau porc, Babe. Babe pénètre
dans le champ d’Hortense et dévore son maïs.
282 CHAPITRE 21

Si Babe se contentait de rester dans le champ 8 Afin de pallier la non-efficience du niveau


de Jérémie, il y mangerait des déchets sans de services d’éducation à Hicksville, le conseil
valeur. Supposons que Babe mange pour 500 $ municipal a décidé de subventionner les études.
de maïs par an dans le champ d’Hortense (un Le conseil offre 200 $ à chaque étudiant qui
effet externe négatif imposé à Hortense) et que achète une année de services d’éducation.
l’installation d’une barrière entre les fermes coûte a) Tracez sur votre graphique la nouvelle courbe
300 $. Précisons qu’aucun droit de propriété d’avantage marginal privé qui comprend
visant à éloigner les animaux des champs la subvention, et nommez-la AmPv
n’a encore été établi. b) Quels sont approximativement les nouveaux
a) Si un droit de propriété est attribué à Jérémie prix et quantité d’équilibre ?
pour que Babe puisse continuer de pénétrer
dans le champ d’Hortense, celle-ci aura-t-elle 9 Le conseil municipal d’Hicksville porte
intérêt à installer une barrière ? Expliquez. la subvention à 400 $.
b) Si le droit de propriété est plutôt attribué a) Tracez sur votre graphique une autre courbe
à Hortense, afin quelle puisse se faire d’avantage marginal privé qui comprend
rembourser le mais que mange Babe, Jérémie la subvention et nommez-la AmP2.
aura-t-il intérêt à installer une barrière ? b) Quels sont approximativement les nouveaux
Expliquez. prix et quantité d’équilibre ?
c) Expliquez en quoi cet exemple illustre c) A combien devra s’élever la subvention pour
le théorème de Coase. que l’on puisse obtenir la quantité efficiente
de services d’éducation ?
7 Les deux premières colonnes du tableau 21.2
indiquent le barème de demande de services @ 10 La compagnie Odeur Plus produit Llatulot, un
d’éducation à Hicksville, alors que la troisième désodorisant ménager populaire. Malheureusement,
colonne donne le coût marginal privé. À le processus de production libère du dioxyde de
Hicksville, à la différence de l’exemple sur soufre dans l’atmosphère. Le coût marginal privé
les services d’éducation présenté dans le manuel, (CmP) de la production de Llatulot par Odeur
le coût marginal augmente parallèlement au Plus est
nombre d’étudiants. Étant donné que les services
d’éducation génèrent des avantages externes, CmP = 1Q0
l’avantage marginal social donné dans la dernière
colonne est supérieur à l’avantage marginal privé. Le coût marginal social (CmS) est

TABLEAU 21.2 CmS = 3 / 2 Qo

Avantage Coût Avantage La courbe de demande de Llatulot (il n’y a pas


Quantité marginal marginal marginal d’avantages externes) est
(nombre privé (en privé (en social (en
d’étudiants) dollars) dollars) dollars) P = 12-1/20^

100 500 200 800


Q0 est la quantité offerte de Llatulot, P est le
200 400 250 700
prix de Llatulot en dollars et Qd est la quantité
300 300 300 600
demandée.
400 200 350 500
a) Dans un marché non réglementé, quelle
500 100 400 400
est la quantité d’équilibre de Llatulot ? le prix
600 0 450 300
d’équilibre ?
b) Pour que l’allocation soit efficiente, quelle
a) Représentez graphiquement les données
devrait être la quantité d’équilibre de Llatulot?
du tableau 21.2.
le prix d’équilibre ?
b) Quels seraient le prix et la quantité
c) Comparez la quantité et le prix sur le marché
d’équilibre si le marché des services
non réglementé et la quantité et le prix efficients.
d’éducation n’était pas réglementé ?
d) Le gouvernement souhaite imposer une taxe
c) Quelle est la répartition efficiente
sur Llatulot pour que le niveau de production
des étudiants à Hicksville ?
soit efficient. À combien devrait s’élever
cette taxe?
LES EFFETS EXTERNES, L'ENVIRONNEMENT ET LE SAVOIR 283

MHMNMHMMMMMHNWNMMMMNNNMNHMMMHMMMNMMMi
I5 b Les brevets permettent de régler les problèmes
RÉPONSES d’effets externes positifs.
16 d Toutes ces mesures font déplacer
la production là où AmS = CmS.
Vrai/Faux/1 ncertain
17e II s’agit du théorème de Coase.
(Justifiez votre réponse.)
18 d Pas de -l de la productivité marginale ; crée
des avantages externes ; monopole du brevet
1 F CmS = Cm P + effets externes. —> perte découlant de la restriction
2 V CmS = CmP + effets externes. d’utilisation.
3 V Des droits de propriété permettraient I9 c Un meilleur environnement est un bien
de récupérer les coûts (externes). normal, donc > 0.
& 4 F Les avantages externes touchent l’AmS 20 a Les marchés ont tendance à produire une
et non le CmS. quantité moindre que la quantité efficiente de
5 V Là où le CmP (plutôt que le CmS) croise biens accompagnés d’effets externes positifs.
la courbe de demande. 21c La courbe Am réelle est supérieure à celle
6 I Vrai si les coûts de transaction sont faibles ; qui est indiquée, donc la courbe Am réelle
autrement, faux. est située au-dessus de la courbe indiquée.
7 F La quantité à laquelle AmS = CmS 22 d Lorsque AmP = CmP = 80 $.
de la pollution. 23 c CmS — CmP est constant dans cet exemple.
8 V Les droits de propriété intellectuelle 24 c Lorsque AmS = CmS.
permettent de résoudre le problème des effets 25 c Production de trop de biens accompagnés
externes positifs du savoir. de coûts externes.
9 F Les permis permettent de contourner
le besoin de savoir des responsables
Problèmes à court développement
de la réglementation.
10 F Tous les pays ont une stratégie dominante
consistant à polluer. 1 Le gouvernement subventionne fortement les
services d’éducation parce qu’ils génèrent des
avantages externes. En particulier, lorsque les
Questions à choix multiple
individus sont instruits, l’ensemble de la société
en retire des avantages au-delà des avantages
I e Se répercute sur les tiers.
privés que reçoivent ceux qui choisissent
Ô 2 b Distance verticale entre le CmS et le CmP
d’étudier jusqu’au niveau qui leur convient.
lorsque Q = 6.
3 d À l’intersection de la courbe CmP
2 La présence de coûts externes signifie que les
et de la courbe de demande.
producteurs ne tiennent pas compte de tous les
4 b À l’intersection de la courbe CmS
coûts lorsqu’ils décident de leur volume de
et de la courbe de demande.
production. Lorsqu’il y a perception d’une
5 d Tracez le graphique. On ne sait pas si les
taxe du même montant que le coût externe,
effets externes sont positifs ou négatifs ni
ce dernier n’est plus considéré comme externe.
même s’il y en a.
De ce fait, le producteur en tiendra compte et il
6 d Le coût se répercute sur autrui.
sera donc incité à produire la quantité efficiente.
7 e Incapacité d’obtenir une allocation efficiente.
8 b À l’intersection de la courbe Cm
3 Étant donné que la courbe de coût marginal
et de la courbe de demande.
social est située en dessous de la courbe de
9 c À l’intersection de la courbe Cm
demande au niveau de production après taxe,
et de la courbe AmS.
le coût marginal social est inférieur à l’avantage
Ô 10 d Fait déplacer la Cm vers le bas de la distance
marginal. Cela signifie que la taxe a été fixée à
verticale située entre la courbe AmS
un niveau trop élevé, soit à un niveau supérieur
et la courbe de demande.
au coût externe. Par conséquent, le volume de
II c Il est dans l’intérêt public de réaliser
production d’acier après taxe sera inférieur au
une allocation efficiente.
volume efficient. Il faudrait augmenter le volume
12 e Définition.
de la production d’acier et celui de la production
Ô 13 a La courbe CmP se déplace vers le haut
de pollution en réduisant le montant de la taxe.
d’une distance verticale égale au coût externe
par unité.
14 b Lacunes du marché.
284 CHAPITRE 2 1

Nous souhaitons tous éliminer la pollution, c) Selon le théorème de Coase, si les coûts de
ceteris paribus, mais toutes nos actions, y compris transaction sont peu élevés et que des droits
la réduction de la pollution, ont un coût. Là de propriété ont été établis, il n’y a pas d’effets
encore, le concept clé à la base de l’argument externes et, que ce soit Hortense ou Jérémie
économique est celui du coût d’opportunité. qui obtienne les droits de propriété, le résultat
Quel est le coût d’opportunité de la réduction sera efficient. Du fait que les deux pionniers
de la pollution, ou, à quoi la société doit-elle peuvent résoudre amicalement leurs problèmes,
renoncer pour obtenir un environnement exempt les coûts de transaction sont faibles. L’installation
de pollution ? d’une barrière élimine l’effet externe dû
De faibles réductions de la pollution sont aux frasques de Babe et ce, peu importe qui
relativement peu coûteuses — l’élimination du possède le droit de propriété. Le résultat
plomb de l’essence et de la peinture, l’économie est efficient parce que, dans le cas de cette
d’énergie pour diminuer la production des décision, le coût social de 300 $ est inférieur
centrales à charbon, etc. Mais pour éliminer au coût de consommation de maïs qui est
toute la pollution, il faudrait éliminer toutes de 500 $.
les automobiles et tous les avions, bannir toute
source d’énergie autres que l’énergie solaire et 7 a) La figure 21.4 est une représentation graphique
l’énergie hydroélectrique, en fermant toutes les des données du tableau 21.2. La demande
usines, etc. Le coût d’élimination de toute la de services d’éducation est donnée par la
pollution est énorme, et ce coût supplémentaire courbe d’avantage marginal privé (appelée
est bien supérieur aux avantages supplémentaires AmP), la courbe de coût marginal privé est
résultant d’une plus grande réduction de la CmP et la courbe d’avantage marginal social,
pollution. Par conséquent, un certain niveau AmS. Ne tenez pas compte des autres courbes
de pollution est efficient. La pollution fait partie pour le moment.
du coût d’opportunité des avantages que nous
retirons du fait de nous déplacer en voiture FIGURE 21.4
plutôt qu’à pied, du bien-être que nous apporte
l’air climatisé par temps chaud, ainsi que les
biens usinés. Le niveau de production efficient
équilibre le coût marginal social de la pollution
par rapport à l’avantage marginal social de
la production et de la consommation associé
à ce niveau de pollution.

Les droits de propriété intellectuelle incitent


les individus à innover en octroyant à l’inventeur
d’une innovation un monopole limité qui lui
permet de retirer des profits de l’application
de sa découverte. Sans droits de propriété, tout
le monde pourrait utiliser gratuitement l’idée,
et l’inventeur n’en retirerait aucun profit.
Ce monopole limité, bien que nécessaire pour b) Dans un marché non réglementé, le prix
motiver la production de connaissances, et la quantité d’équilibre sont déterminés par
a un coût social - la restriction de la production l’intersection des courbes AmP et CmP. De
fondée sur les connaissances à un niveau ce fait, le prix d’équilibre serait de 300 $,
inférieur au niveau concurrentiel efficient. et la quantité d’équilibre, de 300 étudiants.
c) Étant donné qu’il n’y a pas de coûts externes,
a) Hortense installera une barrière. Bien que la quantité efficiente est déterminée par
le coût de cette barrière soit de 300 $ par an, l’intersection des courbes CmP et AmS.
cela lui permet quand même d’économiser Cela suppose que l’on a réalisé une allocation
500 $ de maïs que Babe mangerait en efficiente avec 500 étudiants.
l’absence de barrière.
b) Jérémie installera une barrière. Les frais 8 a) L’avantage marginal privé de chaque étudiant
annuels de barrière de 300 $ sont inférieurs augmente du montant de la subvention, soit
au dédommagement annuel de 500 $ qu’il de 200 $. La figure 21.4 présente la nouvelle
devrait payer à Hortense pour les frasques courbe AmP, nommée AmPx.
de Babe.
LES EFFETS EXTERNES, L’ENVIRONNEMENT ET LE SAVOIR 285

b) Le nouvel équilibre après l’octroi de la b) Pour réaliser une allocation efficiente,


subvention de 200 $ est situé à l’intersection le CmS doit être égal à l’avantage marginal
des courbes CmP et AmPx. Le prix d’une unité social (AmS). Étant donné qu’il n’y a pas
de services d’éducation sera approximativement d’effets externes, la courbe de demande
de 370 $ et il y aura environ 430 étudiants. se confond avec la courbe AmS. En situation
d’équilibre, Qo = Qn = Q*- P°ur trouver la
9 a) Avec une subvention de 400 $ par étudiant, quantité d’équilibre, supposez que l’équation
la courbe AmP se déplacera vers AmP2, dans CmS est égale à la demande d’équation.
la figure 21.4.
b) Avec cette subvention, l’équilibre sera à 3/2Q* = 12-l/2Q*
l’intersection des courbes CmP et AmP2. 2Q* = 12
Le prix d’une unité de services d’éducation Q* = 6
correspondant sera environ de 430 $ et il y
aura environ 570 étudiants. Pour trouver le prix d’équilibre (P*),
c) Pour que le résultat soit efficient, la subvention remplacez Q* dans l’équation de demande
doit être telle que la courbe AmP coïncide (ou de CmS).
avec la courbe AmS. Pour cela, il faut quelle
s’élève à 300 $ par étudiant. P* = 12-1/2Q*
P* = 12-1/2 (6)
d><3> 10 a) Dans un marché non réglementé, la quantité P* = 9
d’équilibre serait déterminée par l’intersection
des courbes d’offre {CmP) et de demande. c) En comparaison des résultats d’allocation
En situation d’équilibre, Q0 = QD = Q*. efficiente, le marché non réglementé produit
En supposant que l’équation CmP soit égale trop de Flatulot (8 unités par rapport à 6),
à l’équation de demande, on obtient : et le produit se vend à un prix trop bas
(8 $ par rapport à 9 $).
1Q* = 12 - 1/2Q * d) La taxe devrait être égale à la différence
3/2Q* = 12 de coût (CmS — CmP) avec une production
Q* = 8 optimale de 6 unités. Étant donné que la
courbe CmS = 1,5 X courbe CmP, la taxe
Pour trouver le prix d’équilibre (P*), devrait être de 50%.
remplacez Q* dans l’équation de demande
(ou du CmP).

P* = 12 -1/2Q *
P* = 12-1/2 (8)
P* -8
Révision

PARTIE

Chapitres 18 à 21

c) Les groupes de consommateurs font pression


PROBLÈME pour faire établir une règle permettant de
maximiser le surplus du consommateur.
Le Conseil de la radiodiffusion et des Indiquez cette règle et expliquez le résultat
télécommunicadons canadiennes (CRTC) (prix, quantité, profit économique, surplus
tient des séances publiques sur le renouvellement du consommateur) sur la figure R4.1. Si
du permis de la compagnie de câblodistribution le CRTC adopte cette règle, quelles options
Choix de Débiles. Vous avez été engagé pourriez-vous proposer pour permettre
pour aider le CRTC à évaluer les demandes à la compagnie de câblodistribution de rester
contradictoires des différents groupes intéressés en activité?
qui témoignent devant le Conseil. La figure d) Toute règle que suit le CRTC doit sembler
R4.1 indique la meilleure information répondre à l’objectif d’intérêt public de
disponible sur les coûts et revenus pour l’allocation efficiente. Déterminez la règle
Choix de Débiles. (autre que la tarification au coût marginal)
qui permet le mieux d’atteindre ce but
FIGURE R4.I et expliquez le résultat (prix, quantité, profit
économique, surplus du consommateur,
perte sèche) sur la figure R4.1.
e) Alors quelle souhaite réaliser des profits
maximum, la compagnie Choix de Débiles
reconnaît qu’il est nécessaire que le CRTC
ait l’air d’utiliser la règle indiquée en d. Quel
argument défendra Choix de Débiles ?
Utilisez la figure R4.1 pour l’expliquer.
f) Si, malgré votre avis défavorable, le CRTC
accepte l’argument de la compagnie défini
en e, quelle théorie de réglementation
sera ainsi vérifiée ?
g) Après des années de réglementation comme
celle mentionnée en f, le CRTC déréglemente
le marché de Choix de Débiles. Si les taux
a) En vous fondant sur la figure R4.1, expliquez de rendement augmentent, quelle théorie de
pourquoi il y a lacune du marché et la déréglementation sera ainsi vérifiée ?
possibilité d’intervention gouvernementale.
b) Choix de Débiles aimerait maximiser ses
profits. Déterminez la règle de maximisation
des profits de l’entreprise et expliquez les
résultats (prix, quantité, profit économique,
surplus du consommateur, perte sèche)
sur la figure R4.1.
CHAPITRES I 8 À 2 I 287

[7 XAMEN DE MI-ÉTAPE
Considérez le monopole naturel de la figure
R4.3. Le surplus du producteur est maximal
lorsque la quantité est
Allouez 32 minutes pour cet examen (16 questions, a) Qo et le prix, P3.
2 minutes par question). Pour chaque question, b) Qj et le prix, Px.
choisissez la meilleure réponse. c) et le prix, P4.
d) Q2 et Ie prix, P2-
I Reportez-vous à la figure R4.2. Le surplus e) Q3 et le prix, P0.
du producteur perdu à cause de la taxe
(ne comprenant pas la taxe payée) est égal FIGURE R4.3
au triangle
a) abc.
b) dbc.
c) dac.
d) dij.
e) gdi.

FIGURE R4.2

L’inégalité de la répartition du patrimoine est


a) moins grande que l’inégalité de la répartition
du revenu.
b) réduite par l’existence de transferts
intragénérationnels.
c) une meilleure mesure de l’inégalité de
la répartition des ressources économiques
que l’inégalité de la répartition du revenu.
2 Reportez-vous à la figure R4.2. Le surplus du d) même plus grande si l’on considère
consommateur perdu en raison de la taxe (sans la répartition du patrimoine entre le 1 %
comprendre la taxe payée) est égal au triangle des familles les plus riches.
a) abc. e) Toutes ces réponses.
b) dbc.
c) dac. La demande d’intervention gouvernementale
d) dij. dans un marché dépend
e) gdi. a) du surplus du consommateur par acheteur.
b) du nombre d’acheteurs.
3 On obtient le surplus total en additionnant c) du surplus du producteur par entreprise.
a) le gain provenant des échanges revenant d) du nombre d’entreprises.
aux consommateurs et le gain provenant des e) Toutes ces réponses.
échanges revenant aux producteurs.
b) le gain provenant de la réglementation et Reportez-vous aux courbes de Lorenz de la
le gain attribuable aux lois anti-monopoles. figure R4.4. Quel point indique que le quintile
c) les revenus des entreprises et les subventions des familles les plus riches gagne 60 % du
gouvernementales. revenu ?
d) Les dépenses des consommateurs et le profit a) a
des producteurs. b) b
e) Aucune de ces réponses. c) c
d )d
e) Aucune de ces réponses.
288 RÉVISION DE LA 4E PARTIE

10 Selon la théorie de l’intérêt public, le processus


FIGURE R4.4 politique cherche à réduire au minimum
a) le surplus du producteur.
b) le surplus du consommateur.
c) le surplus total.
d) la perte sèche.
e) l’allocation efficiente.

I I Le facteur de production le moins susceptible


de faire preuve d’une productivité marginale
décroissante est
a) la terre.
b) le travail.
c) le capital.
0 20 40 60 80 100 d) l’esprit d’entreprise.
Pourcentage cumulé des familles e) les connaissances.

I2 D’après le théorème de Coase, si les coûts de


transactions sont faibles et qu’il existe des droits
8 Reportez-vous à la figure R4.5. Le résultat de propriété,
non réglementé du marché du papier est a) les effets externes négatifs entraînent
a) quantité = 40, prix =11. des pertes sèches.
b) quantité = 40, prix =13. b) les effets externes positifs entraînent
c) quantité = 50, prix =12. des pertes sèches.
d) quantité = 50, prix = 14. c) les transactions privées sont efficientes.
e) quantité = 60, prix =13. d) les transactions publiques sont efficientes.
e) le niveau efficient de pollution sera de zéro.
FIGURE R4.5
13 Si le taux d’imposition marginal diminue
à mesure qu’augmente le revenu, le système
d’imposition est
a) progressif.
b) proportionnel.
c) négatif.
d) régressif.
e) redistributif.

14 Lequel des facteurs suivants ne fait pas partie


du patrimoine réel d’un individu ?
a) le stock de richesse.
b) le flux des gains.
c) le capital humain.
d) les biens matériels.
e) les avoirs financiers.

I5 On peut obtenir l'avantage total d’un certain


niveau de fourniture d’un bien public en
a) additionnant l’avantage marginal de chaque
niveau de fourniture jusqu’au niveau
en question.
9 Reportez-vous à la figure R4.5. Une taxe b) additionnant l’avantage marginal de chaque
de _ par tonne est nécessaire pour niveau de fourniture et en soustrayant ensuite
réaliser la production efficiente de _ le coût marginal de chaque niveau de
tonnes de papier. fourniture.
a) 14$; 50 c) additionnant l’avantage net de chaque niveau
b) 14$;30 de fourniture jusqu’au niveau en question.
c) 13$; 40 d) en multipliant l’avantage net par la quantité
d) 2$; 50 du bien public fourni.
e) 2$; 40 e) Aucune de ces réponses.
CHAPITRES 18 À 21 289

16 On demande aux électeurs de voter pour e) Choix de Débiles soutiendra que ses coûts
la proposition A ou pour la proposition B. sont beaucoup plus élevés que ceux que
La proposition A l’emportera si reflète la courbe CTM de la figure R4.1. La
a) elle se rapproche le plus de l’allocation compagnie gonflera ses coûts et prétendra
efficiente. que sa courbe CTM est plus élevée et quelle
b) elle est soutenue par les fonctionnaires. est juste tangente à la courbe de demande au
c) elle est celle que préfère l’électeur médian. point f. Si le CRTC accepte cet argument et
d) elle entraîne des avantages sociaux supérieurs applique la règle de tarification au coût moyen,
aux coûts sociaux. le prix réglementé et le résultat seront
e) elle entraîne moins d’effets externes négatifs. les mêmes que ceux obtenus avec la règle
de maximisation des profits du monopoleur
mentionnée en b.
f) Si le CRTC accepte l’argument de l’industrie
RÉPONSES présenté en e, la théorie de la capture est
vérifiée.
g) Si, après la déréglementation, les taux de
Problème rendement augmentent, la théorie d’intérêt
a) Une lacune du marché est l’incapacité public de la réglementation est vérifiée.
d’un marché déréglementé de parvenir
à une allocation efficiente. La compagnie Examen de mi-étape
Choix de Débiles est un monopole naturel
qui réalise des économies d’échelle. Même 1 a Voir le texte du manuel.
s’il est le seul à pouvoir réaliser des économies 2 b Voir le texte du manuel.
d’échelle, en l’absence de réglementation, le 3 a Surplus du consommateur plus surplus
monopole limitera la production, fera monter du producteur.
les prix et créera une perte sèche. 4 c Résultat de monopole privé.
b) La règle de maximisation des profits consiste 5 d L’inverse de a et b est vrai. La répartition
à choisir le volume de production auquel du patrimoine ne comprend pas les ressources
Cm = Rm et à fixer le prix le plus haut en capital humain, donc c est faux.
possible. Sur la figure R4.1, le prix est b, la 6 e Voir le texte du manuel.
quantité est k, le profit économique est bcgf, 7 d En passant de 80 % à 100 % de familles
le surplus du consommateur est abf et la (le quitile de familles les plus riches),

perte sèche est fhp. on déplace le revenu de 40% à 100%


c) La règle de tarification au coût marginal (fixer du total (60%).

le prix égal à Cm) maximise le surplus du 8 c Là où D = AmS croise CmP.


consommateur. Sur la figure R4.1, le prix est 9 e Une taxe de 2 $ par tonne (distance verticale
e, la quantité est q, la perte par unité est op, entre CmP et CmS) Cm de
permet au

et le surplus du consommateur est aep. l’entreprise d’être égal auCmS. Production


L’entreprise subit une perte et elle ne au point où D - AmS croise CmS.
poursuivra pas volontairement ses activités. 10 d Ou bien maximise l’allocation efficiente.
Vous pourriez suggérer que l’entreprise puisse Ile En raison d’effets externes positifs.

pratiquer la discrimination par les prix 12c II n’y a pas d’effets externes. Niveau

ou recevoir une subvention du gouvernement de pollution auquel AmS = CmS.


(financée par les taxes qui créeraient une perte I 3 d Définition.

sèche ailleurs).
14b Le flux des gains est le revenu provenant

d) Selon la règle de tarification au coût moyen, du stock de richesse individuelle.

le prix est égal au CTM. Sur la figure R4.1, 15 a b-d se rapportent à des coûts non pertinents.

le prix est dla quantité est n, le surplus


I 6 c 50% des votes plus 1.

du consommateur est adl et la perte sèche


est lmp. Si cette perte sèche est inferieure a
celle du subventionnement de la tarification
au coût moyen mentionnée en c, alors la réglé
de tarification au coût moyen est celle qui
permet le mieux de réaliser une allocation
efficiente.
Le commerce international

Les gains à l’échange


CONCEPTS CLÉS
Un pays réalise des gains en achetant à d’autres pays
des biens pour lesquels ces derniers ont un coût
La structure du commerce international
d’opportunité moindre et en vendant aux autres
pays des biens pour lesquels il a le coût d’opportunité
♦ Les importations sont les biens et services achetés
le plus bas.
aux pays étrangers, et les exportations, les biens
et services vendus à l’étranger. ♦ Le prix après échanges se situe entre les coûts
d’opportunité initiaux avant échanges des deux
• La balance commerciale = la valeur des
pays —> l’acheteur réalise des gains parce que le
exportations - la valeur des importations.
prix des biens est inférieur à leur coût d’opportunité ;
• Si la balance commerciale est
> 0 —> exportateur net.
le vendeur réalise des gains parce que le prix des
• Si la balance commerciale est biens est supérieur à leur coût d’opportunité.
< 0 —> importateur net. ♦ Les pays paient leurs importations grâce à leurs
♦ Les véhicules automobiles constituent exportations —» la valeur des exportations = la
les principales importations et exportations valeur des importations (en l’absence d’emprunts
du Canada. et de prêts internationaux).

♦ Les deux pays réalisent des gains en se spécialisant


♦ Le commerce comprend les échanges de services
comme le tourisme —> les vacances des Américains dans la production de biens pour lesquels ils
à Banff sont un produit d’exportation canadien. détiennent un avantage comparatif et en se livrant
au commerce d’autres biens —> peuvent donc
♦ Les États-Unis sont notre principal partenaire consommer à l’extérieur de la CPP.
commercial.
• Étant donné que les prix après échanges
sont différents des coûts d’opportunité avant
Le coût d’opportunité échanges, les volumes de production s’équilibrent
et l’avantage comparatif —> les pays produisent davantage de biens
destinés à l’exportation, moins de biens importés.
Les pays peuvent produire n’importe où sur la courbe
des possibilités de production ou à l’intérieur de cette ♦ Le pays possède un avantage absolu pour tous
courbe (CPP). les biens si son niveau de productivité est plus
élevé —> peut parvenir à un certain volume
♦ La pente de la CPP = A de la variable mesurée de production de tous les biens en employant
en ordonnée/ A de la variable mesurée en abscisse moins de travailleurs.
= coût d’opportunité de la production d’une unité
supplémentaire de la variable mesurée en abscisse. ♦ Même avec un avantage absolu, si les coûts
d’opportunité diffèrent d’un pays à l’autre
♦ Un pays possède un avantage comparatif dans —» tout le monde possède un avantage pour
la production d’un bien s’il peut produire ce bien un quelconque produit et peut retirer des gains
à un coût d’opportunité inférieur à celui d’un du commerce.
autre pays.
LE COMMERCE INTERNATIONAL 291

Les gains du commerce ♦ Les barrières commerciales non tarifaires limitent


dans le monde réel les importations.

• Les quotas spécifient la quantité maximale


♦ L’avantage comparatif explique la majorité
d’un bien qu’on peut importer, des licences
des échanges commerciaux —» quoique la plupart
d’importation tenant compte des quotas étant
de ceux-ci concernent des biens de même nature,
distribuées par le pays d’origine.
en raison :
• Les restrictions volontaires d’exportation
• de la diversité des préférences —> la demande (RVE) fixent la quantité maximale qu’on peut
de nombreux produits (semblables) ; exporter, les distributeurs étrangers détenant des
• des économies d’échelle —> le moyen de permis d’exportation.
production le moins coûteux pour de nombreux
♦ Les quotas/RVÉ —> -l offre d’importations
produits est la spécialisation et le commerce
—> T prix national —> T production nationale.
international.
• Augmentation des prix à l’importation
♦ Nous profitons tous, à l’instar du pays exportateur,
—> les quotas sont rentables pour les détenteurs.
de l’augmentation de nos importations, mais
• Les quotas et les tarifs ont des effets semblables
celle-ci peut entraîner des coûts d’ajustement
sur le prix et la quantité, ce qui les différencie
—> emplois/salaires perdus pour ceux qui
étant le bénéficiaire du profit = prix
travaillent dans les industries concurrentielles
de vente — prix des importations.
de l’importation —> intervention gouvernementale.

L’argumentation antiprotectionniste
Les pratiques commerciales restrictives
Les restrictions commerciales sont utilisées même
Les gouvernements limitent le commerce pour
si elles limitent les gains à l’échange pour trois raisons
protéger les industries nationales —» protectionnisme.
quelque peu fondées :
♦ Deux types de mesures restrictives - les tarifs
♦ La protection des industries stratégiques permet
douaniers et les barrières commerciales non
d’assurer la sécurité nationale ; toutefois, il est
tarifaires.
difficile de déterminer quelles sont les industries
♦ Le Canada a toujours imposé des tarifs mais, depuis stratégiques.
les années 1930, ceux-ci ont -l en raison de l’Accord
♦ En protégeant les industries naissantes, on favorise
général sur les tarifs douaniers et le commerce,
leur développement et on encourage l’apprentissage
un accord international visant à limiter les pratiques
par la pratique. Ces arguments sont peu crédibles.
restrictives des gouvernements.
♦ Les compagnies étrangères peuvent avoir
• L’Accord de libre-échange nord-américain
recours au dumping en vendant leur production
avec les États-Unis et le Mexique a entraîné
à un prix plus bas que le coût de production afin
X des barrières tarifaires.
de se placer en situation de monopole mondial.
♦ Les tarifs douaniers sont des taxes imposées
• Toutefois il est difficile de détecter le dumping,
sur les importations —> T prix des biens importés
car il est difficile de déterminer les coûts.
—> si importations —>T production nationale
• Les droits de douane compensateurs sont des
—» pertes nettes pour le pays importateur du
tarifs douaniers imposés par un gouvernement
fait que le coût d’opportunité de la production
pour concurrencer les producteurs étrangers
nationale > prix à l’importation initial.
subventionnés.
♦ Les tarifs douaniers —> réduction du volume
♦ Un pays peut limiter le commerce pour les raisons
des importations —> exportateurs des pays
suivantes, moins bien fondées :
étrangers vendent moins de biens —> -l revenu
—» si importations des pays étrangers = exportations • pour sauvegarder les emplois dans les industries
du pays d’origine —> aucun changement dans en concurrence avec les importations —» mais
la balance commerciale (en l’absence d emprunts le libre-échange suscite également la création
et de prêts internationaux). d’emplois;
• pour concurrencer la main-d’œuvre étrangère
♦ Lorsque les tarifs ont considérablement T dans
bon marché —> mais, ce qui compte, ce sont les
les années 1930 —> le commerce international
salaires et la productivité (avantage comparatif) ;
a presque été anéanti.
292 CHAPITRE 22

De nombreuses personnes engagées dans


• pour produire la diversité et la stabilité —> mais
des débats sur le commerce semblent désorientées
les grandes économies le permettent tout aussi
par le concept d’avantage comparatif, en partie
bien;
parce qu’elles considèrent implicitement
• pour pénaliser les entreprises qui ont des
l’avantage absolu comme la seule raison du
normes environnementales laxistes -A mais
commerce. Un pays possède un avantage absolu
le libre-échange et la croissance économique
sur un autre pays s’il peut produire tous les biens
permettent mieux d’augmenter la qualité
en utilisant moins de facteurs de production
de l’environnement;
que l’autre pays. Toutefois, ce pays peut tirer des
• pour empêcher les pays riches d’exploiter les
avantages du commerce. Prenons l’exemple de la
pays en développement —» mais cet argument
Californie et de la Saskatchewan. La Californie
ne tient pas compte de l’avantage comparatif.
a un meilleur climat et, grâce à l’irrigation
♦ La restriction du commerce vise principalement généralisée, elle possède un avantage absolu
la protection des groupes/industries qui souffrent pour la production de tous les produits agricoles.
de manière disproportionnée du libre-échange. En fait, la Californie produit fréquemment plus
d’une récolte par an ! On pourrait donc croire
• Le commerce entraîne des avantages nets
que la Californie n’a pas besoin de faire du
positifs, mais certaines industries concurrençant
commerce avec la Saskatchewan. Toutefois, la
les importations en ressortent perdantes.
Saskatchewan a un avantage comparatif pour la
• Les perdants sont en partie dédommagés
production de blé. La Californie se spécialisera
par le biais de l’assurance-emploi.
donc en fruits quelle échangera contre du blé.
La Californie pourrait facilement cultiver son
Les effets de l’Accord de libre-échange propre blé, mais le coût d’opportunité - les
nord-américain récoltes de fruits auxquelles elle devrait renoncer —
serait trop élevé. En se spécialisant et en faisant
♦ En dépit de mesures protectionnistes, le Canada des échanges, la Californie et la Saskatchewan
a conclu un accord commercial avec les Etats-Unis y gagnent toutes les deux.
et le Mexique qui a trait :
Dans ce chapitre, il est crucial de comprendre
• à la réduction progressive des tarifs communs,
que deux partenaires commerciaux peuvent
et ce jusqu’à leur élimination;
retirer, l’un comme l’autre, des avantages
• à la réduction des barrières commerciales
de leurs échanges. Cela est dû au fait que le
non tarifaires ;
prix après échanges se situe entre les deux coûts
• à une plus grande libéralisation des échanges
d’opportunité avant échanges des deux pays.
dans les domaines de l’énergie et des services ;
C’est ce que nous montre la figure 22.6 du
• aux futures négociations sur les subventions ;
manuel où le prix d’équilibre d’une automobile
• à l’établissement d’un mécanisme de règlement
correspond à 3 tonnes de céréales, prix
des différends.
qui se situe entre les coûts d’opportunité avant
♦ À l’heure actuelle, les effets de cet accord échanges qui sont de 1 tonne et de 9 tonnes.
ne sont pas connus, mais ils comprennent : Il arrive souvent que les étudiants ne savent
pas très bien où placer le prix lorsqu’ils résolvent
• une importante? du volume total des échanges ;
des problèmes avec ces types d’exemples.
• d’importants changements dans certains
Comment a-t-on obtenu la valeur de 3 ? Il s’agit,
secteurs —> des coûts d’ajustements élevés.
en fait, d’une valeur arbitraire, une valeur prise
au hasard. En considérant logiquement les
échanges dont il est question, on doit situer
cette valeur entre les deux valeurs avant échanges
RAPPELS qui sont 1 et 9. Toutefois, en retraçant la courbe
de demande d’importations et la courbe d’offre
I Ce chapitre nous permet d’appliquer les
d’exportations avec des pentes différentes,
concepts fondamentaux de coût d’opportunité
on aurait pu obtenir une valeur de 6 tonnes. Un
et d’avantage comparatif vus au chapitre 3
résultat qui aurait été tout aussi logique et valable.
à la question du commerce international. Les
Dans le monde réel, l’importance de la
principes de base sont immuables, qu’il s’agisse
demande de produits précis des consommateurs
du commerce entre individus d’un même pays
de chaque pays déterminera les pentes de la
ou de différents pays.
courbe de demande d’importations et la courbe
d’offre d’exportations ainsi que la position du
LE COMMERCE INTERNATIONAL 293

prix d’équilibre. Dans vos exemples, vous considérable. De ce fait, la minorité sera très
pourrez choisir où placer le prix d’équilibre motivée à faire établir une restriction, alors que
(à condition qu’il se trouve entre les deux coûts la majorité sera peu motivée à consacrer du
d’opportunité avant échanges) ou vous devrez temps et de l’énergie à s’y opposer.
suivre les indications précises qui vous seront
fournies. 5 Pour comprendre d’où viennent les pressions
favorables aux restrictions au commerce, résumons
Nous avons appris une chose importante sur un peu pour comprendre qui profite de ces
les conséquences économiques des restrictions restrictions et qui y perd.
commerciales : un tarif et un quota ont Pour les trois types de restrictions
les mêmes effets. Une restriction volontaire (tarifs douaniers, quotas et RVE), ce sont les
d’exportation (RVE) est également un quota, consommateurs qui y perdent, parce que le prix
mais il s’agit d’un quota imposé par le pays du bien importé augmente. Les producteurs
exportateur plutôt que par le pays importateur. nationaux du bien importé et leurs facteurs
Toutes les restrictions commerciales font de production y gagnent dans les trois cas,
monter le prix intérieur des biens importés et parce que le prix du bien importé augmente.
baisser le volume et la valeur des importations. Les producteurs étrangers et leurs facteurs de
Elles réduisent également la valeur des exportations production y perdent dans les trois cas, parce
d’un montant égal à la réduction de la valeur des que leurs ventes à l’exportation diminuent
importations. L’augmentation du prix résultant considérablement. En présence de quotas et
de chaque restriction produit un écart entre de RVE, les détenteurs de licences d’importation
le prix sur le marché national du bien importé réalisent des profits en achetant des biens à bas
et le prix d’offre du bien du pays étranger. prix et en les vendant à prix élevés (étrangers ou
Pour distinguer les autres restrictions intérieurs). Le gouvernement perçoit des recettes
commerciales possibles, il faut considérer le douanières grâce aux tarifs douaniers,
bénéficiaire de l’excédent. Dans le cas d’un tarif, et éventuellement gagne des votes, dans
c’est le gouvernement qui perçoit les recettes le cadre d’autres projets.
douanières. Dans le cas d’un quota imposé Si l’on considère cette liste, il n’est pas
au pays exportateur, l’excédent revient surprenant que les principaux défenseurs des
aux importateurs nationaux ayant une licence restrictions au commerce soient les producteurs
d’importation par le biais de profits accrus. intérieurs et leurs facteurs de production.
Dans le cas d’une RVE, l’excédent revient aux
exportateurs étrangers ayant reçu une licence
d’exportation de leur gouvernement.
AUTOÉVALUATION
Le principal élément de ce chapitre a trait au fait
que les gains découlant du commerce peuvent
être considérables. Dans ce cas, pourquoi les
Vrai/Faux/Incertain
pays ont-ils autant tendance à imposer des
(Justifiez votre réponse.)
restrictions au commerce ? C’est parce que,
contrairement au libre-échange, qui entraîne I Les gouvernements élus ne sont pas pressés de
des avantages généraux pour l’ensemble de réduire les restrictions au commerce parce qu’il
l’économie, il y a à la fois des gagnants et des y aurait bien moins de perdants que de gagnants.
perdants. Les gagnants réalisent au total plus
de gains que les perdants, mais ces derniers ont
tendance à se concentrer dans quelques industries
seulement. 2 Le Japon pratique le dumping de l’acier s’il le
En raison de cette concentration, certains vend au Japon à un prix inférieur au prix auquel
s’opposeront au libre-échange en invoquant il le vend au Canada.
l’argument de l’intérêt personnel. Bien qu une
minorité seulement en tirera des avantages
et que la majorité sera touchée, il n’est pas
surprenant que l’on ait recours aux restrictions 3 Lorsqu’un pays possède un avantage absolu,
commerciales. Le coût d’une restriction il ne tire pas de gains à l’échange.
commerciale sera, pour chaque élément
de la majorité, assez faible alors que 1 avantage
qu’en tirera chaque élément de la minorité sera
294 CHAPITRE 22

4 Les pays peuvent s’échanger des biens de même


TABLEAU 22.11 COMMERCE
nature en raison des économies d’échelle et
INTERNATIONAL
de la diversité des préférences.
DES TROUS DE BEIGNE

Prix Offre Demande


international d’exportation d’importation
5 En imposant des tarifs douaniers, un pays
(en dollars de trous de de trous
augmente les gains qu’il retire du commerce.
par centaine Glaçage d’Excavations
de trous) (en millions) (en millions)

0,50 1 10
6 Le commerce en fonction de l’avantage comparatif 0,75 2 8
permet à tous les partenaires commerciaux 1,00 3 6
de consommer à l’extérieur de leur CPP. 1,25 4 4
1,50 5 2
1,75 6 0

7 Si un pays A doit renoncer à 3 unités de Y


pour produire 1 unité de X et si le pays B doit 2 Reportez-vous au tableau 22.1. Les producteurs
renoncer à 4 unités de Y pour produire 1 unité de trous de beigne d’Excavations réussissent
de X, on dit que le pays A détient un avantage à convaincre leur gouvernement qu’il faut
comparatif pour la production de X. protéger l’industrie nationale contre les
importations peu coûteuses de Glaçage.
(Ils soutiennent que les trous constituent un
groupe alimentaire de base.) En réponse, le
8 Un tarif douanier imposé sur un bien fera gouvernement d’Excavations fixe un quota
monter le prix de ce bien et réduire la quantité aux importations de 3 millions de trous de
échangée. beigne. Il en résulte un prix de_$ la
centaine de trous de beigne, et la production
nationale de trous de beigne_.
a) 0,75 ; baissera
9 Un quota fera baisser considérablement le prix b) 0,75 ; augmentera
du bien importé. c) 1,25 ; sera la même
d) 1,38; augmentera
e) 1,38 ; sera la même

10 Lorsqu’un citoyen canadien séjourne dans un 3 Reportez-vous au tableau 22.1. En partant


hôtel en France, le Canada exporte un service. de l’équilibre de libre-échange, les exportateurs
de trous de beigne de Glaçage convainquent leur
gouvernement que la fabrication de trous
de beigne est menacée par les cruels caprices
du marché international et quelle doit donc être
Questions à choix multiple
subventionnée. Le gouvernement de Glaçage
répond en accordant une subvention de 0,25 $
I Reportez-vous au tableau 22.1. Dans un régime par centaine de trous de beigne. Il en résultera
de libre-échange, le prix international des trous un prix international d’environ_$ la centaine
de beigne serait de_$ par centaine de trous de trous et la quantité échangée sera de_
de beigne, et_millions de trous seraient millions de trous de beigne.
échangés. a) 1 ; 4
a) 0,75; 6 b) 1; 5
b) 1 ; 3 c) 1,12; 4
c) 1,25; 4 d) 1,12; 4,5
d) 1,25; 5 e) 1,50; 5
e) 1,50; 4
LE COMMERCE INTERNATIONAL 295

4 Reportez-vous au tableau 22.1. Le gouvernement 7 Reportez-vous à la figure 22.1. Avec l’ouverture


de Glaçage a subventionné la production du commerce entre Partyland et Cowabunga,
de trous de beigne à raison de 0,25 $ la a) il y aura un grand nombre de joyeux lurons.
centaine de trous. Les producteurs nationaux b) Partyland fournira les pizzas et la bière,
d’Excavations ont déposé une plainte auprès du car elle possède un avantage comparatif
gouvernement prétextant que la subvention pour les deux produits.
injuste a porté préjudice à la production c) Cowabunga fournira les pizzas et la bière,
nationale. Le gouvernement d’Excavations car elle possède un avantage comparatif
répond en imposant un droit compensateur de pour les deux produits.
0,25 $ sur chaque centaine de trous de beigne d) Partyland fournira la pizza, et Cowabunga
de Glaçage. Le prix national d’Excavations après fournira la bière.
les effets nets de la subvention et le droit e) Partyland fournira la bière, et Cowabunga
compensateur sera de _$ par centaine fournira la pizza.
de trous, et Excavations importera_millions
de trous. 8 Reportez-vous à la figure 22.1. Avec l’ouverture
a) 1 ; 4 du commerce entre Partyland et Cowabunga,
b) 1; 5 le prix d’échange de la bière sera
c) 1,12; 4 a) de 1 bière contre 1 pizza.
d) 1,12; 5 b) de 1 bière contre 1/3 de pizza
e) 1,25; 4 c) de 1 bière contre 3 pizzas.
d) situé entre 1 bière contre 1 pizza et 1 bière
5 Reportez-vous à la figure 22.1. Le coût
contre 3 pizzas.
d’opportunité de 1 bière à Partyland est_,
e) situé entre 1 bière contre 1 pizza et 1 bière
et le coût d’opportunité de 1 bière à Cowabunga
contre 1/3 de pizza.
est_
a) variable, selon le point sur la CPP où
9 Si le Canada impose un quota d’importation
nous le mesurons ; variable, selon le point
de un million de paires de chaussures par an,
sur la CPP où nous le mesurons.
une augmentation de la demande nationale
b) de 100 pizzas; de 25 pizzas.
de chaussures
c) de 3 pizzas ; de 1 pizza.
a) n’entraînera pas de variation du prix national
d) de 1 pizza; de 1 pizza.
des chaussures, mais suscitera une augmentation
e) de 1 pizza; de 1/3 de pizza.
de la quantité de chaussures importée.
b) n’entraînera pas de variation du prix national
FIGURE 22.1 PARTYLAND ET COWABUNGA -
des chaussures ni de la quantité de chaussures
CPP POUR LA BIÈRE ET LA PIZZA
importée.
c) entraînera une augmentation du prix national
des chaussures et de la quantité de chaussures
importée.
d) entraînera une augmentation du prix national
des chaussures mais ne suscitera aucun
changement dans la quantité de chaussures
importée.
e) entraînera une augmentation du prix
des chaussures et une variation indéterminée
de la quantité de chaussures importée,
selon que l’augmentation de la demande
6 Reportez-vous à la figure 22.1. Le coût est supérieure ou non à un million de paires.
d’opportunité de 1 pizza à Partyland est-,
et le coût d’opportunité de 1 pizza à Cowabunga 10 Lorsqu’un accord sur une restriction volontaire
est_ des exportations est conclu, l’écart entre le prix
a) variable, selon le point sur la CPP où nous à l’importation national et le prix à l’exportation
le mesurons ; variable, selon le point sur revient
la CPP où nous le mesurons. a) aux consommateurs du pays importateur.
b) de 100 bières; de 25 bières. b) à la personne qui a le droit d’importer le bien.
c) de 3 bières ; de 1 bière. c) au gouvernement du pays importateur.
d) de 1 bière ; de 1 bière. d) aux exportateurs étrangers.
e) de 1 bière ; de 3 bières. e) aux producteurs nationaux du bien.
296 CHAPITRE 22

I I La mise en œuvre de l’Accord de libre-échange I5 Le commerce international fondé sur l’avantage


entre le Canada et les États-Unis a révélé que comparatif permet à chaque pays de consommer
a) l’accord a porté un grave préjudice a) une plus grande quantité des produits qu’il
au Canada. exporte, mais une moins grande quantité des
b) l’accord a considérablement aidé le Canada. produits qu’il importe sans commerce.
c) le libre-échange n’est pas efficace. b) une plus grande quantité des produits qu’il
d) tous ceux auxquels le libre-échange porte importe, mais une moins grande quantité des
préjudice seront indemnisés. produits qu’il exporte sans commerce.
e) le volume de commerce international c) une plus grande quantité des produits qu’il
a connu une forte hausse dont profitent exporte et importe sans commerce.
tous les consommateurs, mais au coût d’un d) une moins grande quantité des produits qu’il
pourcentage élevé de pertes d’emplois à la fin exporte et importe sans commerce.
des années 1980 et au début des années 1990. e) soit a ou b, selon le prix des produits.

12 Si nous importons plus que nous n’exportons, 16 Un tarif douanier imposé au bien X qu’importe
a) nous ne pourrons pas acheter autant le pays A entraînera
de produits étrangers que nous le désirons. a) le déplacement vers le haut de la courbe
b) nous consentirons des prêts aux étrangers de demande de X du pays A.
pour leur permettre d’acheter nos produits. b) le déplacement vers le bas de la courbe
c) nous devrons financer la différence de demande de X du pays A.
en empruntant aux étrangers. c) le déplacement vers le haut de la courbe
d) nos structures commerciales, y compris d’offre de X du pays A.
la direction des exportations et des d) le déplacement vers le bas de la courbe d’offre
importations, ne seront différentes que si les de X du pays A.
exportations sont égales aux importations. e) le déplacement vers le haut de la courbe de
e) c et d. demande et de la courbe d’offre de X du pays A.

13 Si le pays A possède un avantage absolu dans I7 Lorsqu’un quota est imposé, l’écart entre le prix
la production de tous les produits, intérieur et le prix à l’exportation revient
a) il n’y aura aucun échange, car le pays A a) aux consommateurs du pays importateur.
aura un avantage comparatif sur tout. b) aux producteurs nationaux du bien.
b) il n’y aura aucun échange, car aucun c) au gouvernement du pays importateur.
pays n’aura d’avantage comparatif sur d) aux exportateurs étrangers.
un quelconque produit. e) à la personne qui détient le droit d’importer
c) il y aura probablement des échanges le bien.
et tous les pays en bénéficieront.
d) il y aura probablement des échanges, 18 Lequel des exemples suivants représente
mais le pays A n’en profitera pas. un service d’exportation canadien ?
e) il y aura probablement des échanges, a) un Canadien s’offre un repas au restaurant
mais le pays A sera le seul à en bénéficier. alors qu’il voyage en Suisse.
b) un Suisse s’offre un repas au restaurant
14 Lequel des arguments suivants n’est pas alors qu’il voyage au Canada.
un argument du protectionnisme ? c) un Canadien achète un réveil fabriqué en Suisse.
a) protéger les industries stratégiques. d) un Suisse achète un ordinateur fabriqué
b) sauvegarder les emplois dans les industries au Canada.
concurrençant les importations. e) a et b.
c) obtenir un avantage comparatif.
d) permettre aux industries naissantes
de se développer.
e) éviter que les pays riches n’exploitent
les pays pauvres.
LE COMMERCE INTERNATIONAL 297

19 Lequel des énoncés suivants sur le commerce 23 L’imposition par le pays A d’un tarif douanier
international est vraP sur les biens importés du pays B fera monter
a) Les tarifs douaniers augmenteront les emplois le prix que paient les consommateurs pour
dans nos industries d’exportation. les biens importés et
b) Les quotas sont plus efficaces que les tarifs a) réduira le volume des importations
parce qu’ils ne font pas monter les prix. et des exportations.
c) Les tarifs douaniers sont nécessaires pour b) réduira le volume des importations
nous permettre de concurrencer la main- et augmentera le volume des exportations.
d’œuvre étrangère bon marché. c) réduira le volume des importations et n’aura
d) Personne ne retire d’avantages du libre- pas d’effets sur le volume des exportations.
échange entre un pays pauvre et un pays riche. d) n’aura aucun effet sur le volume des
e) Les RVE font autant monter les prix que importations ni sur celui des exportations.
des tarifs douaniers équivalents. e) fera monter le volume des importations
mais diminuer le volume des exportations.
20 Dans le pays A, pour produire 1 unité de X, il
faut 1 unité de capital et 1 unité de main-d’œuvre 24 Lequel des éléments suivants est le principal
et, pour produire 1 unité de Y, il faut 2 unités responsable d’une réduction importante des
de travail et 2 unités de capital. Quel est le coût restrictions commerciales depuis la Deuxième
d’opportunité du bien X? Guerre mondiale?
a) Le prix de 1 unité de capital plus le prix a) La Loi Smoot-Hawley.
de 1 unité de travail. b) L’Accord sur la restriction volontaire des
b) 1 unité de capital et 1 unité de travail. exportations entre les États-Unis et le Japon.
c) 2 unités de capital et 2 unités de travail. c) L’Organisation des Nations Unies.
d) 1/2 unité de Y d) L’Accord général sur les tarifs douaniers et
e) 2 unités de Y le commerce.
e) L’Accord de libre-échange entre le Canada
21 Lorsqu’un tarif douanier est imposé, l’écart entre et les États-Unis.
le prix national et le prix à l’exportation revient
a) aux consommateurs du pays importateur. 25 Le pays A importe des biens X du pays B et
b) à la personne qui a le droit d’importer le bien. exporte des biens Y vers le pays B. Le pays A
c) aux producteurs nationaux du bien. préférera une mesure de restriction volontaire
d) aux exportateurs étrangers. des exportations plutôt qu’un quota sur X
e) au gouvernement du pays importateur. a) afin de ne pas nuire aux importations du pays B.
b) pour éviter que le pays B ne se lance dans
22 Le pays A et le pays B sont actuellement en des représailles et ne limite les exportations
situation de libre-échange. Le pays A importe du pays A.
le bien X du pays B et exporte le bien Y vers le c) pour maintenir à un niveau peu élevé le prix
pays B. S’il impose un quota à X, le pays A verra national de X.
son industrie productrice de X d) pour augmenter la recette du gouvernement.
a) se développer, et son industrie productrice e) pour aider les producteurs nationaux.
de Y se réduire.
b) se développer, et son industrie productrice
Problèmes à court développement
de Y également.
c) se réduire, et son industrie productrice
de Y également. 1 Pourquoi les deux partenaires d’un échange
d) se réduire, et son industrie productrice commercial peuvent-ils l’un et l’autre en tirer
de Y se développer. profit ?
e) se développer, et son industrie productrice
de Y demeurer inchangée. 2 Comment un tarif douanier imposé sur un bien
importé particulier influe-t-il sur le prix national
du bien, le prix à l’exportation, la quantité
importée, et la quantité du bien produit
intérieurement?

3 Comment un tarif douanier sur les importations


influe-t-il sur les exportations du pays ?
298 CHAPITRE 22

4 Les chefs syndicaux prétendent souvent que 7 Poursuivez l’analyse lorsque Atlantis et Beltran
les tarifs douaniers sont nécessaires pour protéger se livrent à des échanges au taux de 1 unité
les emplois nationaux. À la lumière de ce que d’aliments contre 1 unité de vêtements.
vous avez répondu en 2 et 3, évaluez la validité a) Si Atlantis consomme 600 unités d’aliments
de cet argument. et 400 unités de vêtements, quelles quantités
d’aliments et de vêtements seront consommées
5 Imaginez un monde simple qui comprend deux par Beltran ?
pays, Atlantis et Beltran, chacun d’eux produisant b) Compte tenu des quantités consommées
des aliments et des vêtements. Le tableau 22.2 et produites indiquées au problème 6b,
nous présente la CPP de chaque pays. combien d’unités d’aliments et de vêtements
a) En supposant que le coût d’opportunité Atlantis et Beltran importeront-ils et
soit constant dans chaque pays, terminez exporteront-ils ?
le tableau.
b) Quel est le coût d’opportunité des aliments 8 La figure 22.2 indique la courbe de demande
à Atlantis ? des vêtements ? d’importation de chemises du pays A, appelée D,
c) Quel est le coût d’opportunité des aliments et la courbe d’offre d’exportation de chemises du
à Beltran ? des vêtements ? pays B, appelée O.
d) Tracez la CPP sur des graphiques séparés. a) Quel est le prix d’une chemise
dans un régime de libre-échange ?
TABLEAU 22.2 ATLANTIS ET BELTRAN - b) Combien de chemises le pays A importera-t-il ?
CPP POUR LES ALIMENTS
ET LES VÊTEMENTS FIGURE 22.2

Atlantis Beltran

Aliments Vêtements Aliments Vêtements


(en unités) (en unités) (en unités) (en unités)

0 500 0 800
200 400 100 600
400 200
600 300
800 400
1 000 - -

6 Supposez qu’Atlantis et Beltran


sont des partenaires commerciaux.
a) Dans la fabrication de quel produit chaque
pays se spécialisera-t-il ?
b) Si 1 unité d’aliments s’échange contre 9 Supposons que les fabricants de chemises
1 unité de vêtements, comment évoluera la du pays A dont il est question au problème 8
production de chaque bien dans chaque pays ? s’inquiètent de la concurrence étrangère et que,
c) Sachant que 1 unité d’aliments s’échange en conséquence, le gouvernement du pays A
contre 1 unité de vêtements, tracez les courbes impose un tarif douanier de 9 $ par chemise.
de possibilités de consommation pour chaque a) Comment variera le prix d’une chemise
pays sur le graphique correspondant du dans le pays A ?
problème 5. b) Quel prix l’exportateur recevra-t-il réellement ?
d) Si, avant les échanges, Atlantis consommait c) Combien de chemises le pays A importera-t-il ?
600 unités d’aliments, il pouvait consommer d) À combien s’élèveront les recettes douanières ?
un maximum de 200 unités de vêtements. À qui reviendront-elles ?
Après les échanges, combien d’unités de
vêtements peut-il consommer, pour la même
quantité d’aliments ?
LE COMMERCE INTERNATIONAL 299

10 Supposez que, au lieu d’imposer un tarif <S> 4 e En plus du changement indiqué au n° 3, les
douanier, le pays A impose un quota tarifs douaniers —» si offre des exportations
de 4 millions de chemises par an. de 1 million, ce qui contrebalance exactement
a) Quel sera le prix d’une chemise dans le pays A ? le montant de la subvention.
b) Quel prix l’exportateur recevra-t-il réellement ? 5 e Le coût d’opportunité de 1 bière
c) Combien de chemises le pays A importera-t-il ? supplémentaire = pizza perdue. Pour
d) Quelle est la différence entre le montant total Partyland, de passer de 0 bière à 25 bières
payé par les consommateurs et le montant coûte 25 pizzas —> une contre une. Faites
total reçu par les exportateurs - le «profit les mêmes calculs pour Cowabunga.
excédentaire» ? À qui reviendra-t-il ? ^ e Même type de calcul que pour le n° 5.
7 d Chacun se spécialise dans le domaine où il
a un avantage comparatif (coût d’opportunité
le plus bas).
RÉPONSES
Le prix d’échange se situe entre les coûts
d’opportunité de chaque pays au point
Vrai/Faux/Incertain d’équilibre avant échanges.
(Justifiez votre réponse.) 9 j Quota —> quantité d’importations fixe
maximum. Par conséquent, si la demande
1 F Ils seront lents parce que les pertes des T —>î prix.
perdants sont individuellement bien plus 10 d Parce qu’ils ont le droit d’exporter.
grandes que les gains des gagnants. 11 e Voir le manuel.
2 F Le dumping serait de vendre au Canada 12 c Nécessaire afin d’attirer le commerce étranger.
à un prix plus bas qu’au Japon. En général, les profils de commerce restent
3 I S’il existe un avantage comparatif inchangés lorsque nous disposons d’un plus
—> il y a des gains à l’échange. grand montant de dollars.
& 4 V Les différences dans les préférences 13 c L’avantage absolu pour tous les biens
—» demande de nombreux produits. —> avantage comparatif pour quelques
Économies d’échelle —> méthode biens seulement.
de production moins coûteuse, mais 14 c Voir le manuel.
requiert la spécialisation et le commerce. 15 c La courbe des possibilités de consommation
5 F Les restrictions commerciales réduisent se trouve à l’extérieur de la CPP.
les gains à l’échange —> coût d’achat des 16 c Les tarifs douaniers —> T prix national = prix
exportations < profit courant —> perte des à l’exportation + tarif douanier —> déplacement
gains à l’échange. de la courbe d’offre vers le haut.
6 V Les pays se spécialiseront et feront des 17 e Avec l’imposition de quotas, le gouvernement
échanges pour consommer à l’extérieur national accorde le droit d’importer,
de la CPP. 18 b a et c —> importation, d—> exportation d’un bien.
Ô 7 V daun coût d’opportunité inférieur (3L< 4 Y) 19 e Les tarifs -1 exportations et les emplois dans
= perte de Y par unité de gain de X. les industries exportatrices, les quotas limitent
8 V Les tarifs douaniers —> déplacement vers l’offre —> T prix, la main-d’œuvre étrangère
le haut de la courbe d’offre des exportations bon marché ne nous porte pas nécessairement
—> T prix et 1 quantité échangée. préjudice, les pays pauvres comme les pays
9 F Quota offre —> T prix. riches peuvent avoir un avantage comparatif
10 F Le Canada importe (utilise) un service. et obtenir des avantages des échanges. La
RVE limite l’offre et fait monter les prix tout
comme les tarifs douaniers.
Questions à choix multiple <Ô 20 d
Les facteurs de production requis pour
1 c L’équilibre se situe au point où l’offre fabriquer un X pourraient fabriquer 1/2 Y.
d’exportations = la demande d importations. 21 e Ils perçoivent les recettes provenant des tarifs
2 d L’équilibre se situe au point où le quota = douaniers = prix à l’importation - prix à
la demande d’importations —>T prix. Le prix l’exportation.
intérieur plus élevé —> T production intérieure. 22 a Quotas —» f importations —> T prix
@ 3 d La subvention T l’offre d’exportation de national de X —» T production nationale.
Glaçage de 1 million d’unités pour chaque 1 importations = >1 exportations de B
prix —> nouvel équilibre au point où la nouvelle —> -f son revenu —> f ses importations
offre = ancienne demande. —> 1 exportations de A —» 1 production
de Y dans A.
300 CHAPITRE 22

23 a Tarif douanier —> T prix national des et la quantité exportée diminuent lorsqu’un tarif
importations —> i importations —> T douanier est imposé. Cette baisse du prix et
production nationale. X importations = T de la quantité exportée signifie que le revenu du
exportations de B —> ^ son revenu —> t ses pays B a diminué. Cela implique que la quantité
importations —» -l exportations de A —> 1 de Y (exportation du pays A) demandée par le
production des exportations dans A. pays B diminuera et que les exportations du pays
24 d Voir le manuel. A diminueront également.
25 b Avec une RVE —» le pays exportateur obtient
le revenu excédentaire —> moins susceptible 4 Cet argument est partiellement vrai. Comme
de lancer des représailles, étant donné que les le montre le problème 2, le tarif douanier fera
préjudices sont moins grands. augmenter la production nationale du bien
protégé, ce qui entraînera une augmentation
des emplois dans cette industrie. Toutefois,
Problèmes à court développement
comme le montre le problème 3, le même tarif
réduira le revenu étranger ainsi que les achats
1 Deux partenaires commerciaux potentiels qui étrangers de nos biens, ce qui entraînera la
souhaitent faire du commerce ensemble doivent réduction de notre volume d’exportations et de
avoir des avantages comparatifs différents ; notre production de produits exportés, et aussi
autrement dit, des coûts d’opportunité différents. des emplois dans l’industrie des exportations.
Ensuite, ils feront des échanges, et l’un et l’autre L’effet net sur les emplois n’est pas clair, mais
réaliseront des gains. S’ils ne font pas d’échanges, il n’est certainement pas important.
chacun fera face à ses propres coûts d’opportunité.
Un prix d’échange doit se situer quelque part 5 a) Le tableau 22.2 Solution nous présente
entre les coûts d’opportunité des deux partenaires. le tableau 22.2 dûment rempli. On a obtenu
Autrement dit, celui qui a le coût d’opportunité les valeurs du tableau en utilisant le coût
du bien en question le plus bas aura un avantage d’opportunité de chaque bien de chaque pays.
parce qu’il recevra un prix supérieur à son coût Voir b et c ci-dessous.
d’opportunité. De même, celui qui a le coût
d’opportunité le plus élevé aura un avantage TABLEAU 22.2 SOLUTION
parce qu’il paiera un prix inférieur à son coût ATLANTIS ET BELTRAN - CPP POUR
d’opportunité. LES ALIMENTS ET LES VÊTEMENTS

2 Un tarif douanier imposé sur un bien importé Atlantis Beltran


augmentera le prix que paient les consommateurs
Aliments Vêtements Aliments Vêtements
nationaux pour ce bien à mesure que la courbe
(en unités) (en unités) (en unités) (en unités)
d’offre des exportations se déplace vers le haut.
Le prix des exportations est déterminé par la 0 500 0 800
courbe d’offre des exportations initiale. A mesure 200 400 100 600
que le prix national du bien s’élève, la quantité 400 300 200 400
du bien demandée diminue, et donc le point 600 200 300 200
pertinent sur la courbe d’offre des exportations 800 100 400 0
initiale est situé à l’endroit correspondant à une 1 000 0 — —

quantité inférieure et à un prix des exportations


moins élevé. Cette quantité moins grande signifie b) Pour augmenter la production (consommation)
que la quantité importée diminue. La hausse du d’aliments de 200 unités, on doit diminuer
prix national entraînera également une la production de vêtements (consommation)
augmentation de la quantité du bien offerte de 100 unités en Atlantis. De ce fait, le coût
intérieurement. d’opportunité de 1 unité d’aliments est de
1/2 unité de vêtements. Ce coût d’opportunité
3 Lorsque le pays A impose un tarif douanier est constant, comme le sont ici tous les
sur ses importations du bien X, non seulement autres, pour faciliter les choses. De même, le
le volume des importations diminuera, mais coût d’opportunité des vêtements à Atlantis
le volume des exportations de Y au pays B correspond à 2 unités d’aliments.
diminuera du même montant. De ce fait, la c) À Beltran, une augmentation de 100 unités
balance commerciale reste en équilibre. Comme de la production (consommation) d’aliments
l’indique la réponse au problème 2, le prix requiert une réduction de la production
à l’exportation du bien X reçu par le pays B (consommation) de vêtements de 200 unités.
LE COMMERCE INTERNATIONAL 301

De ce fait, le coût d’opportunité des aliments c) Les courbes des possibilités de consommation
est de 2 unités de vêtements. De même, le d’Atlantis et Beltran, nommées CPCA
coût d’opportunité des vêtements à Beltran et CPCB, sont illustrées à la figure 22.3(a)
est de 1/2 unité d’aliments, et (b), respectivement. Ces courbes sont
d) La figure 22.3 (a) et (b) illustre les courbes des lignes droites qui indiquent toutes les
des possibilités de production pour Atlantis combinaisons d’aliments et de vêtements qui
et Beltran, respectivement nommées CPPA peuvent être consommées grâce aux échanges.
et CPPB. Les solutions aux problèmes 7 et 8 La position et la pente de la courbe
traitent du reste du diagramme. des possibilités de consommation pour une
économie dépendent des conditions
FIGURE 22.3 d’échanges entre les biens (un contre un dans

(a) (b) cet exemple) et le point de production de


l’économie.
On obtient, par exemple, la courbe des
possibilités de consommation pour Atlantis
(CPCA) en partant du point b sur la CPPA,
le point de production, et en examinant
les possibilités d’échanges. Par exemple,
si Atlantis échangeait 400 unités des aliments
quelle produit contre 400 unités de vêtements,
elle pourrait consommer 600 unités d’aliments
(1 000 unités produites moins 400 unités
échangées) et 400 unités de vêtements, ce qui
est représenté par le point c.
d) Si Atlantis consomme 600 unités d’aliments,
6 a) Nous voyons à la solution des problèmes sa consommation de vêtements passera, grâce
5b et c que Atlantis a un coût d’opportunité au commerce, à 400 unités, soit 200 unités
moins élevé (1/2 unité de vêtements) dans de plus que sans commerce. Le montant
la production d’aliments. Par conséquent, maximum de vêtements qu’il est possible de
Atlantis se spécialisera dans la production consommer sans commerce nous est donné par
d’aliments. Beltran, qui a le coût d’opportunité la courbe des possibilités de production. Si la
pour les vêtements le moins élevé (1/2 unité consommation d’aliments est de 600 unités,
d’aliments) se spécialisera dans la production cela est indiqué par le point a sur la CPPA. La
de vêtements. consommation maximale de vêtements pour
b) Chaque pays souhaitera produire toutes les tout niveau de consommation de vêtements
unités du bien dans lequel il se spécialise aussi avec commerce nous est donnée par la courbe
longtemps que le montant qu’il reçoit en des possibilités de consommation. Si la
échange est supérieur à leur coût d’opportunité. consommation d’aliments est de 600 unités,
Pour Atlantis, le coût d’opportunité de 1 unité cela est indiqué par le point c sur la CPCA.
d’aliments est de 1/2 unité de vêtements,
mais Atlantis peut obtenir 1 unité a) Puisque Atlantis produit 1 000 unités
de vêtements en échange. Du fait que le d’aliments par an (point b sur la CPP
coût d’opportunité est constant, Atlantis se pour consommer 600 unités d’aliments
spécialisera entièrement dans la production et 400 unités de vêtements (point c sur la
de tous les aliments possibles : 1 000 unités CPCA), Atlantis doit échanger 400 unités
par an, point b à la figure 22.3(a). De même, d’aliments contre 400 unités de vêtements.
à Beltran, le coût d’opportunité de 1 unité de Autrement dit, Beltran a échangé 400 unités
vêtements est de 1/2 unité d aliments, mais de vêtements contre 400 unités d’aliments.
1 unité de vêtements s’échangera contre Puisque Beltran produit 800 unités de
1 unité d’aliments. Le coût d’opportunité vêtements, cela signifie que Beltran doit
étant constant, Beltran se spécialisera consommer 400 unités d’aliments et 400
entièrement dans la production de vetements unités de vêtements (point c' sur la CPCB).
et produira 800 unités par an, le point b de b) Atlantis exporte 400 unités d’aliments par an
la figure 22.3(b). et importe 400 unités de vêtements. Beltran
exporte 400 unités de vêtements par an
et importe 400 unités d’aliments.
302 CHAPITRE 22

8 a) Le prix d’une chemise dans un régime 10 a) Le quota limite la quantité pouvant être
de libre-échange se situera à l’intersection importée à 4 millions de chemises par an,
de la courbe de demande d’importation indépendamment du prix, et il est représenté
de chemises du pays A et de la courbe par une ligne verticale à la figure 22.4 (qui
d’offre d’exportation de chemises du pays B. correspond à la figure 22.2). Le marché des
Cela se produit au prix de 12 $ la chemise, chemises fixera donc le prix de la chemise à 18 $
b) Le pays A importera 6 millions de chemises
par an. FIGURE 22.4

9 a) Le tarif douanier de 9 $ par chemise a pour


effet de déplacer la courbe d’offre d’exportation
(O) vers le haut d’un montant équivalent à
9 $, ce qu’illustre un déplacement de O à O’
à la figure 22.2 Solution. Le prix est maintenant
déterminé par l’intersection de la courbe D,
qui n’est pas touchée par le tarif, et la courbe
O’. Le nouveau prix d’une chemise est 18 $.

FIGURE 22.2 SOLUTION

Quantité (en millions de chemises par an)

b) Ce prix de 18 $ est reçu par ceux qui


ont le droit d’importer des chemises dans
les limites du quota. L’exportateur reçoit
le montant de 9$ ; il nous est donné par
la hauteur de la courbe O à la quantité
de 4 millions de chemises par an.
c) Le pays A importera 4 millions de chemises
par an, la limite du quota.
d) Le « profit excédentaire » est de 9 $ par
Quantité (en millions de chemises par an)
chemise (les 18 $ reçus par l’importateur)
moins les 9 $ reçus par l’exportateur)
b) Sur ces 18 $, la part du tarif est de 9$; X 4 millions de chemises, soit 36 millions de
l’exportateur ne recevra donc que les 9 $ dollars. Ce montant revient aux importateurs
restants. qui ont reçu du gouvernement du pays A
c) Le pays A n’importera maintenant que le droit d’importer dans les limites du quota.
4 millions de chemises par an. Il s’agit essentiellement d’un droit de réaliser
d) Les recettes douanières s’élèvent à 9 $ des «profits excédentaires».
(le tarif par chemise) X 4 millions (le nombre
de chemises importées), soit 36 millions de
dollars. C’est le gouvernement du pays A qui
reçoit cette somme.
ISBN 2-7613-1105-1

9782761311052
08/09/2018 14:18-3
ÉDITIONS DU RENOUVEAU RÊDAGOGIOUE INC.

Vous aimerez peut-être aussi